Echocardiography: A Case-Based Review [Har/Psc ed.] 145110961X, 9781451109610

Echocardiography: A Case-Based Review complements The Echo Manual, 3e, with real time echocardiographic examples that il

861 136 20MB

English Pages 346 [365] Year 2012

Report DMCA / Copyright

DOWNLOAD FILE

Polecaj historie

Echocardiography: A Case-Based Review [Har/Psc ed.]
 145110961X, 9781451109610

Table of contents :
Front matter
Cover
Title Page
Copyright
Dedication
Preface
Contents
CONTENTS BY SUBJECT
Cardiac Masses and Aortic Diseases
11 Severe Chest Pain
21 Systemic Sclerosis
26 Hematuria with Large Renal Mass
41 Progressive Fatigue and Marked Lower Extremity Edema
43 Monomorphic Ventricular Tachycardia after Cardiac Arrest
59 Transient Loss of Vision
60 Sharp Chest Pain
62 Fever and Back Pain
65 Murmur with Systemic Hypertension
67 Transient Loss of Motor Function in Right Upper Extremity
74 Exertional Shortness of Breath, Systolic Murmur and Lower Extremity Edema
76 Atypical Chest Pain
79 Exertional Dyspnea Following Bypass Grafting
82 Systolic and Diastolic Murmurs
87 Exertional Shortness of Breath in a 68-year-old Woman
89 Fever and Dyspnea with History of Severe Pancreatitis
91 Ventricular Tachycardia with Rapid Palpitations
92 Chest Pain in a 52-year-old Woman
Cardiomyopathies
2 Syncopal Episode
3 Progressive Exertional Shortness of Breath
34 Exertional Dyspnea
37 Three Month History of Progressive Exertional Shortness of Breath
38 Progressive Exertional Dyspnea
46 Severe Ischemic Cardiomyopathy and Functional Class IV Left Heart Failure
48 Progressive NYHA Class II-III Exertional Dyspnea and Palpitations
55 Exertional Shortness of Breath
66 Palpitations and Dyspnea
77 Dilated Cardiomyopathy
93 Soft Systolic Murmur after Fainting Episode
100 Exertional Dyspnea with Systemic Hypertension
Congenital Heart Disease
5 Systemic Hypertension
8 Exertional Shortness of Breath and Lower Extremity edema
57 Progressive Exertional Shortness of Breath after Onset of Dyspnea
64 Functional Class III Exertional Dyspnea
68 Cold left Foot
70 Exertional Shortness of Breath with Systolic Blood Pressure of 110 mm Hg
75 Murmur on Routine Physical Examination
84 Exertional Fatigue, Lower Extremity Edema, and New Murmur
88 History of Symptomatic Paroxysmal Atrial Fibrillation
90 Laterally Displaced Apical Impulse
94 Murmur in a 38-year-old Patient
95 Systolic Murmur in a Female Phlebotomist
96 Exertional Shortness of Breath with Episode of Atrial Flutter
97 Primary Billiary Cirrhosis
99 Exertional Shortness of Breath and Chest Pain in a 30-year-old woman
Coronary Artery Disease, including Stress Echocardiography
6 Myocardial Infarction
10 Increasing Dyspnea and Orthopnea
28 Class II Exertional SHortness of Breath without Angina
30 Acute Severe Retrosternal Chest Discomfort
31 Lightheadedness and Severe Fatigue
32 Retrosternal Chest Pain
35 Severe Angina
44 Exertional Chest Pain
51 Dyspnea with Modest Exertion
63 Diffuse ST-Segment Depression
69 Chest Pain and ECG Changes
73 Intermittent Dyspnea with Modest Exertion
81 Severe Chronic Obstructive Pulmonary Disease
Native and Prothetis Valvular Disease, including infective endocarditis
4 Heart Murmur
7 Acute Severe Dyspnea
9 Systolic Murmur
12 Systolic Murmur, Asymptomatic
17 Right Heart Failure
18 Harsh Systolic Murmur
20 Progressive NYHA Class II Dyspnea
23 Exertional Shortness of Breath, Bilateral Lower Extremity Edema, and Systolic Murmur after Coronary Artery Grafting
33 Dyspnea and Chest Tightness
47 Progressive Exertional Dyspnea, Weight Gain, and Peripheral Edema
53 Dyspnea and Assymetrical Ground-glass Pulmonary Infiltrates
56 Hyperlipidemia, Hypertension and Carotid Disease
54 Congenital Aortic Valve Stenosis, Status Post Replacement
61 Transient Loss of Vision with Modest Exertional Shortness of Breath
72 Asymptomatic Murmur
71 Class III Shortness of Breath
78 Three Months of Progressive Exertional Dyspnea
83 Slowly Progressive Exertional Shortness of Breath
86 Recurrent Episodes of Left Heart Failure
98 Exertional Dyspnea following Mitral and Aortic Valve Replacements
Pericardial and Right-sided Disease, including Pulmonary Hypertension
14 Cardiac Murmur During Pregnancy
16 Liver Function Test Abnormalities, Ascites, and Dyspnea
19 Harsh Systolic Murmur in Nursery Examination
25 Heart Failure
27 Transthoracic Echocardiography on a Sonography Student
29 Acute Pleuritic Chest Pain and Lightheadedness
40 Exertional Shortness of Breath with No Medical History
45 Dyspnea and Presyncope
49 Progressive Exertional Fatigue and Lower Extremity Edema
50 Progressive Exertional Shortness of Breath and Lower Extrimity Edema
Systemic Diseases
15 Progressive Dyspnea with Asthma, Rhinosinusitis, Weight Loss, and Peripheral Neuropathy
24 Symptomatic Paroxysmal Atrial Fibrillation
36 Exertional Fatigue, Dyspnea and Two Pillow Orthopnea
42 Progressive Exertional Dyspnea, Atypical Chest Discomfort, Abdominal and Lower Extremity Swelling
52 Fevers, Malaise, and Fatigue
58 Progressive Exertional Shortness of Breath with Diabetes and Systemic Hypertension
80 Exertional Shortness of Breath with History of Lymphoma
CASES
1 Normal Images and Values
2 Syncopal Episode
3 Progressive Exertional Shortness of Breath
4 Heart Murmur
5 Systemic Hypertension
6 Myocardial Infarction
7 Acute Severe Dyspnea
8 Exertional Shortness of Breath and Lower Extremity edema
9 Systolic Murmur
10 Increasing Dyspnea and Orthopnea
11 Severe Chest Pain
12 Systolic Murmur, Asymptomatic
13 Exertional Shortness of Breath and Systolic Murmur
14 Cardiac Murmur During Pregnancy
15 Progressive Dyspnea with Asthma, Rhinosinusitis, Weight Loss, and Peripheral Neuropathy
16 Liver Function Test Abnormalities, Ascites, and Dyspnea
17 Right Heart Failure
18 Harsh Systolic Murmur
19 Harsh Systolic Murmur in Nursery Examination
20 Progressive NYHA Class II Dyspnea
21 Systemic Sclerosis
22 Progressive Dyspnea
23 Exertional Shortness of Breath, Bilateral Lower Extremity Edema, and Systolic Murmur after Coronary Artery Grafting
24 Symptomatic Paroxysmal Atrial Fibrillation
25 Heart Failure
26 Hematuria with Large Renal Mass
27 Transthoracic Echocardiography on a Sonography Student
28 Class II Exertional SHortness of Breath without Angina
29 Acute Pleuritic Chest Pain and Lightheadedness
30 Acute Severe Retrosternal Chest Discomfort
31 Lightheadedness and Severe Fatigue
32 Retrosternal Chest Pain
33 Dyspnea and Chest Tightness
34 Exertional Dyspnea
35 Severe Angina
36 Exertional Fatigue, Dyspnea and Two Pillow Orthopnea
37 Three Month History of Progressive Exertional Shortness of Breath
38 Progressive Exertional Dyspnea
39 Loud Systolic Murmur
40 Exertional Shortness of Breath with No Medical History
41 Progressive Fatigue and Marked Lower Extremity Edema
42 Progressive Exertional Dyspnea, Atypical Chest Discomfort, Abdominal and Lower Extremity Swelling
43 Monomorphic Ventricular Tachycardia after Cardiac Arrest
44 Exertional Chest Pain
45 Dyspnea and Presyncope
46 Severe Ischemic Cardiomyopathy and Functional Class IV Left Heart Failure
47 Progressive Exertional Dyspnea, Weight Gain, and Peripheral Edema
48 Progressive NYHA Class II-III Exertional Dyspnea and Palpitations
49 Progressive Exertional Fatigue and Lower Extremity Edema
50 Progressive Exertional Shortness of Breath and Lower Extrimity Edema
51 Dyspnea with Modest Exertion
52 Fevers, Malaise, and Fatigue
53 Dyspnea and Assymetrical Ground-glass Pulmonary Infiltrates
54 Congenital Aortic Valve Stenosis, Status Post Replacement
55 Exertional Shortness of Breath
56 Hyperlipidemia, Hypertension and Carotid Disease
57 Progressive Exertional Shortness of Breath after Onset of Dyspnea
58 Progressive Exertional Shortness of Breath with Diabetes and Systemic Hypertension
59 Transient Loss of Vision
60 Sharp Chest Pain
61 Transient Loss of Vision with Modest Exertional Shortness of Breath
62 Fever and Back Pain
63 Diffuse ST-Segment Depression
64 Functional Class III Exertional Dyspnea
65 Murmur with Systemic Hypertension
66 Palpitations and Dyspnea
67 Transient Loss of Motor Function in Right Upper Extremity
68 Cold left Foot
69 Chest Pain and ECG Changes
70 Exertional Shortness of Breath with Systolic Blood Pressure of 110 mm Hg
71 Class III Shortness of Breath
72 Asymptomatic Murmur
73 Intermittent Dyspnea with Modest Exertion
74 Exertional Shortness of Breath, Systolic Murmur and Lower Extremity Edema
75 Murmur on Routine Physical Examination
76 Atypical Chest Pain
77 Dilated Cardiomyopathy
78 Three Months of Progressive Exertional Dyspnea
79 Exertional Dyspnea Following Bypass Grafting
80 Exertional Shortness of Breath with History of Lymphoma
81 Severe Chronic Obstructive Pulmonary Disease
82 Systolic and Diastolic Murmurs
83 Slowly Progressive Exertional Shortness of Breath
84 Exertional Fatigue, Lower Extremity Edema, and New Murmur
85 Diabetic Nephropathy
86 Recurrent Episodes of Left Heart Failure
87 Exertional Shortness of Breath in a 68-year-old Woman
88 History of Symptomatic Paroxysmal Atrial Fibrillation
89 Fever and Dyspnea with History of Severe Pancreatitis
90 Laterally Displaced Apical Impulse
91 Ventricular Tachycardia with Rapid Palpitations
92 Chest Pain in a 52-year-old Woman
93 Soft Systolic Murmur after Fainting Episode
94 Murmur in a 38-year-old Patient
95 Systolic Murmur in a Female Phlebotomist
96 Exertional Shortness of Breath with Episode of Atrial Flutter
97 Primary Billiary Cirrhosis
98 Exertional Dyspnea following Mitral and Aortic Valve Replacements
99 Exertional Shortness of Breath and Chest Pain in a 30-year-old woman
100 Exertional Dyspnea with Systemic Hypertension

Citation preview

Echocardiography A Case-Based Review

Garvan C. Kane, MD, PhD Co-Director, Echocardiography Laboratory Consultant , Division of Cardiovascular Diseases Assistant Professor of Medicine Mayo Clinic Rochester, Minnesota

Jae K. Oh, MD Cardiology Co-Director, Integrated Cardiac Imaging Consultant , Division of Cardiovascular Diseases Professor of Medicine Mayo Clinic Rochester, Minnesota Co-Director, Cardiac and Vascular Center Samsung Medical Center Seoul , South Korea

I

. Wolters Kluwer Lippincott Williams & Wilkins Health

Philadelphia • Baltimore • New York • London Buenos Aires • Hong Kong· Sydney • Tokyo

Acquisitions Editor: Frances DeStefano Product Manager: Leanne Vandetty Production Manager: Alicia Jackson Senior Manufacturing Manager: Benjamin Rivera Marketing Manager: Kimberly Schonberger Design Coordinator: Holly Reid McLaughlin Production Service: S4Carlisle © 2013 Mayo Foundation for Medical Education and Research

All rights reserved. This book is protected by copyright. No part of this book may be reproduced in any form by any means, including photocopying, or utilized by any information storage and retrieval system without writ­ ten permission from the copyright owner, except for brief quotations embodied in critical articles and reviews. Materials appearing in this book prepared by individuals as part of their official duties as U.S. government employees are not covered by the above-mentioned copyright. Printed in China 978-1-4511-0961-0 1-4511-0961-X Library of Congress Cataloging-in-Publication Data

Kane, Garvan C. Echocardiography : a case-based review I authors, Garvan C. Kane, Jae K. Oh. p. ; cm. Complement to: Echo manual I Jae K. Oh, James B. Seward, A. Jamil Tajik. 3rd ed. c2006. Includes bibliographical references and index. ISBN 978 - 1 -45 1 1 -096 1 -0 (alk. paper)-ISBN 1 -45 1 1 -096 1 -X (alk. paper) I. Oh, Jae K. II. Oh, Jae K. Echo manual. III. Title. [DNLM: 1 . Heart Diseases-ultrasonography-Case Reports. 2. Heart Diseases-ultrasonography­ Examination Questions. 3. Echocardiography-methods-Case Reports. 4. Echocardiography­ methods-Examination Questions. WG 1 8 .2] 6 1 6. 1 '207543-dc23 20 1 20 1 5 884 Care has been taken to confirm the accuracy of the information presented and to describe generally accepted practices. However, the authors, editors, and publisher are not responsible for errors or omissions or for any consequences from application of the information in this book and make no warranty, expressed or implied, with respect to the currency, completeness, or accuracy of the contents of the publication. Application of the information in a particular situation remains the professional responsibility of the practitioner. The authors, editors, and publisher have exerted every effort to ensure that drug selection and dosage set forth in this text are in accordance with current recommendations and practice at the time of publication. However, in view of ongoing research, changes in government regulations, and the constant flow of information relating to drug therapy and drug reactions, the reader is urged to check the package insert for each drug for any change in indications and dosage and for added warnings and precautions. This is particularly important when the recommended agent is a new or infrequently employed drug. Some drugs and medical devices presented in the publication have Food and Drug Administration (FDA) clearance for limited use in restricted research settings. It is the responsibility of the health care provider to as­ certain the FDA status of each drug or device planned for use in their clinical practice. To purchase additional copies of this book, call our customer service department at (800) 638-3030 or fax orders to (30 1 ) 223-2320. International customers should call (30 1 ) 223-2300. Visit Lippincott Williams & Wilkins on the Internet: at LWWcom. Lippincott Williams & Wilkins cus­ tomer service representatives are available from 8:30 am to 6 pm, EST. 10 9 8 7 6 5 4 3 2

1

In memory of

Mark J. Callahan, MD

Pref ace Echocardiography has become an essential diagnostic tool across the spectrum of cardiovascular disease. In the era of rapid technology development and the increasing use of other imaging modalities, echocar­ diography remains the single most useful imaging technique which couples comprehensive hemody­ namic data with information on cardiac structure and systolic and diastolic function. Strain-based imaging now provides sensitive incremental quantitation of myocardial function with 3-dimensional echocardiog­ raphy allowing real-time views of detailed true cardiac anatomy. We have been very gratified by the success of The Echo Manual, currently in its 3rd edition, which has aided the education of physicians and sonographers since its first publication in 1 994. As its name indi­ cates, the book is first and foremost a manual to pro­ vide instruction to the learner on the various aspects of echocardiography and their clinical applications, highlighting the strengths and limitations of the mo­ dality and providing the reader the necessary steps to accomplish a comprehensive, quantitative diagnostic examination. Echocardiography is a dynamic modality and The Echo Manual has not had real-time moving images or actual clinical cases to utilize comprehensive echo­ cardiographic data including hemodynamic calcula­ tion. In this new case-based echocardiography book, we hope to fill this void, highlighting the importance of interpreting echocardiographic images in the set­ ting of the clinical scenario. This book is designed as a guide for learners in the use of echocardiography data in the evaluation of patients through review of 1 00 selected cases from the Mayo Clinic with a variety of clinical conditions, both commonly and uncommonly encountered. We have placed a particular emphasis on the assessment of systolic and diastolic function and

quantitative hemodynamics throughout. This book needs to be used in tandem with review of the mov­ ing images, available through the on-line site. We rec­ ommend each case be reviewed in isolation from start to finish following the order of the case questions. In many cases, further echocardiographic images play an important role as the clinical cases develops, whether transesophageal, intracardiac or supplemental or sub­ sequent transthoracic imaging. While we have selected cases that hopefully are useful to the learner, whether new or old to the practice of clinical echocardiogra­ phy, we have tried to keep the cases true to life. Images at times are 'presentation quality' while at other times challenging and we have chosen to portray the cases in a predominantly random order. The focus of this book is on the interpretation of the echocardiographic data to provide guidance in the management of the patient. While acting as a stand-alone educational tool, this case-book also serves as a companion to The Echo Manual, 3rd Edition. Here we provide an answer and explanation to each question asked and we also include the location of a more detailed discussion of the topic in The Echo Manual 3rd Edition. We have had and continue to have the good for­ tune to learn and practice echocardiography with a wonderful team of physicians and sonographers . We thank our colleagues for sharing their expertise and interesting cases with us. We would like to thank our families, time from whom was taken to help com­ plete this proj ect. Finally we thank our patients for their educational images and we hope that this book will provide the reader with the tools to better diag­ nose and manage their patients through high quality echocardiography.

Garvan C Kane ]aeK Oh

vii

Contents CASE1

CASE16 Liver Function Test Abnormalities,

Normal Images and Values

Ascites, and Dyspnea

CASE2 Syncopal Episode

7

12

16

19

23

Acute Severe Dyspnea

25

28

Coronary Artery Grafting 31

CASE10 Increasing Dyspnea and Orthopnea

CASE12 Systolic Murmur, Asymptomatic

Transthoracic Echocardiography on a

Exertional Shortness of Breath

Sonography Student 42

CASE14 Cardiac Murmur during Pregnancy

91

CASE28 Class I l l Exertional Shortness of Breath without

46

Angina

94

CASE29

CASE15

Acute Pleuritic Chest Pain and

Progressive Dyspnea with Asthma, Rhinosinusitis, Weight Loss, and Peripheral Neuropathy

88

CASE27

CASE13 and Systolic Murmur

84

CASE26 Hematuria with Large Renal Mass

39

81

CASE25 Heart Failure

37

78

CASE24 Symptomatic Paroxysmal Atrial Fibrillation

34

CASE11 Severe Chest Pain

Exertional Shortness of Breath, Bilateral Lower Extremity Edema, and Systolic Murmur after

CASE9 Systolic Murmur

75

CASE23

Exertional Shortness of Breath and Lower Extremity Edema

71

CASE22 Progressive Dyspnea

CASES

66

CASE21 Systemic Sclerosis

CASE?

63

CASE20 Progressive NY HA Class I l l Dyspnea

CASE6 Myocardial Infarction

59

Harsh Systolic Murmur in Nursery Examination

CASE5 Systemic Hypertension

Harsh Systolic Murmur

CASE19

CASE4 Heart Murmur

56

CASE18

Progressive Exertional Shortness of Breath

CASE17 Right Heart Failure

CASE3

50

48

Lightheadedness

99

ix

X I C O NTENTS

CASE47

CASE30 Acute Severe Retrosternal Chest Discomfort

103

CASE31 Lightheadedness and Severe Fatigue

106

108

CASE33 Dyspnea and Chest Tightness

114

121

123

127

Congenital Aortic Valve Stenosis 130

133

137

182

CASE57 Progressive Exertional Shortness of Breath after

CASE42

Onset of Dyspnea

Progressive Exertional Dyspnea, Atypical Chest

184

CASE58

Discomfort, Abdominal and Lower Extremity 139

Progressive Exertional Shortness of Breath with Diabetes and Systemic Hypertension

CASE43

187

CASE59

Monomorphic Ventricular Tachycardia after 142

Transient Loss of Vision

189

CASE60

CASE44 144

Sharp Chest Pain

191

CASE61

CASE45 146

Transient Loss of Vision with Modest Exertional Shortness of Breath

CASE46

195

CASE62

Severe lschemic Cardiomyopathy and Functional Class IV Left Heart Failure

CASE56 and Carotid Disease

Progressive Fatigue and Marked Lower Extremity

Dyspnea and Presyncope

179

Hyperlipidemia, Hypertension

CASE41

Exertional Chest Pain

175

CASE55 Exertional Shortness of Breath

Exertional Shortness of Breath with No Medical

Cardiac Arrest

170

CASE54 Status Post Replacement

CASE40

Swelling

Dyspnea and Asymmetrical Ground-glass Pulmonary Infiltrates

CASE39

Edema

167

CASE53

CASE38

History

165

CASE52 Fevers, Malaise, and Fatigue

Three Month History of Progressive Exertional

Loud Systolic Murmur

163

CASE51 Dyspnea with Modest Exertion

CASE37

Progressive Exertional Dyspnea

161

Progressive Exertional Shortness Edema

119

Exertional Fatigue, Dyspnea, and Two Pillow

Shortness of Breath

Edema

of Breath and Lower Extremity

CASE36 Orthopnea

CASE49

CASE50

CASE35 Severe Angina

CASE48

Progressive Exertional Fatigue and Lower Extremity 110

CASE34 Exertional Dyspnea

152

Progressive NY HA Class 11-111 Exertional Dyspnea and 156 Palpitations

CASE32 Retrosternal Chest Pain

Progressive Exertional Dyspnea, Weight Gain, and Peripheral Edema

149

Fever and Back Pain

198

C O N T E N TS I X I

CASE63 Diffuse ST-Segment Depression

CASE80 200

CASE64 Functional Class Ill Exertional Dyspnea

202

CASE83 Shortness of Breath

210

CASE 68

CASE84 Edema, and New Murmur

214

Diabetic Nephropathy

Recurrent Episodes of Left Heart

Pressure of 110 mm Hg

Failure

218

CASE71 221

226

History of Symptomatic Paroxysmal

230

234

Laterally Displaced Apical Impulse

291

CASE91

CASE75 Murmur on Routine Physical

Ventricular Tachycardia with Rapid 237

Palpitations

294

CASE92

CASE76 240

CASE77

Chest Pain in a 52-Year-Old Woman

297

CASE93 244

Soft Systolic Murmur after Fainting Episode

CASE78

300

CASE94

Three Months of Progressive 248

CASE79

Murmur in a 38-Year-Old Patient

306

CASE95

Exertional Dyspnea following Bypass Grafting

289

CASE90

Systolic Murmur and Lower

Exertional Dyspnea

CASE89 Severe Pancreatitis

Exertional Shortness of Breath,

Dilated Cardiomyopathy

285

Fever and Dyspnea with History of

CASE74

Atypical Chest Pain

281

CASE88 Atrial Fibrillation

Intermittent Dyspnea with Modest

Examination

Exertional Shortness of Breath in a 68-Year-Old Woman

CASE 73

Extremity Edema

276

CASE87

CASE72

Exertion

27 3

CASE86

Exertional Shortness of Breath with Systolic Blood

Asymptomatic Murmur

270

CASE85

CASE70

Class Ill Shortness of Breath

266

Exertional Fatigue, Lower Extremity 212

CASE69 Chest Pain and ECG Changes

262

Slowly Progressive Exertional

Transient Loss of Motor Function in Right Upper

Cold Left Foot

259

CASE82 Systolic and Diastolic Murmurs

207

CASE67 Extremity

CASE81 Pulmonary Disease

205

CASE66 Palpitations and Dyspnea

256

Severe Chronic Obstructive

CASE 65 Murmur with Systemic Hypertension

Exertional Shortness of Breath with History of Lymphoma

Systolic Murmur in a Female 252

Phlebotomist

310

X I I I C O N T E N TS

CASE99

CASE96 Exertional Shortness of Breath with Episode of Atrial Flutter

Exertional Shortness of Breath and Chest Pain 316

319

CASE98 Exertional Dyspnea following Mitral and Aortic Valve Replacements

327

CASE100

CASE97 Primary Biliary Cirrhosis

in a 30-Year-Old Woman

323

Exertional Dyspnea with Systemic Hypertension

332

Contents by Subject CASE87

Cardiac Masses and Aortic Diseases

Exertional Shortness of Breath in

CASE 11 Severe Chest Pain

37

CASE 21 Systemic Sclerosis

71

CASE26 Hematuria with Large Renal Mass

281

CASE89 Fever and Dyspnea with History of Severe Pancreatitis

289

CASE91 88

Ventricular Tachycardia with Rapid Palpitations

CASE41

294

CASE92

Progressive Fatigue and Marked Lower Extremity Edema

a 68-Year-Old Woman

137

Chest Pain in a 52-Year-Old Woman

297

CASE43

Cardiomyopathies

Monomorphic Ventricular Tachycardia after Cardiac Arrest

142

CASE59 Transient Loss of Vision

189

CASE60 Sharp Chest Pain

191

CASE62 Fever and Back Pain

210

12

114

Three Month History of 123

CASE38 127

CASE46 234

Severe lschemic Cardiomyopathy and Functional Class IV Left

CASE76 240

CASE79 252

CASE82 Systolic and Diastolic Murmurs

Exertional Dyspnea

Progressive Exertional Dyspnea

Exertional Shortness of Breath, Systolic

Exertional Dyspnea following Bypass Grafting

Progressive Exertional Shortness of Breath

Progressive Exertional Shortness

CASE74

Atypical Chest Pain

CASE3

of Breath

Transient Loss of Motor Function in

Murmur and Lower Extremity Edema

7

CASE37 205

CASE67 Right Upper Extremity

Syncopal Episode

CASE34 198

CASE65 Murmur with Systemic Hypertension

CASE2

Heart Failure

149

CASE48 Progressive NY HA Class 11-111 Exertional Dyspnea and Palpitations

156

CASE 55 262

Exertional Shortness of Breath

179

xi i i

X I V I C ONT E NTS BY S U B J E C T

CASE66 Palpitations and Dyspnea

CASE96 207

CASE77 Dilated Cardiomyopathy

244

319

CASE99

Soft Systolic Murmur after 300

Exertional Shortness of Breath and Chest Pain in a 30-Year-Old Woman

CASE100 Exertional Dyspnea with Systemic Hypertension

316

CASE97 Primary Biliary Cirrhosis

CASE93 Fainting Episode

Exertional Shortness of Breath with Episode of Atrial Flutter

327

332

Coronary Artery Disease, including Stress Congenital Heart Disease

Echocardiography

CASES

CASE6

Systemic Hypertension

19

Increasing Dyspnea and Orthopnea

Exertional Shortness of Breath 28

CASE57

218

237

Exertional Fatigue, Lower Extremity

Dyspnea with Modest Exertion 270

CASE88 285

291

200

214

CASE73 Intermittent Dyspnea with

CASE94

Modest Exertion 306

CASE95 Systolic Murmur in a Female Phlebotomist

165

CASE69 Chest Pain and ECG Changes

CASE90

Murmur in a 38-Year-Old Patient

144

CASE63 Diffuse ST-Segment Depression

History of Symptomatic Paroxysmal

Laterally Displaced Apical Impulse

Exertional Chest Pain

CASE 51

CASE84

Atrial Fibrillation

119

CASE44

Murmur on Routine Physical

Edema, and New Murmur

108

CASE35 Severe Angina

CASE75

106

CASE32 Retrosternal Chest Pain

Exertional Shortness of Breath with

103

CASE31 Lightheadedness and Severe Fatigue

212

CASE70

Examination

CASE30 Discomfort

202

CASE68

Systolic Blood Pressure of 110 mm Hg

94

Acute Severe Retrosternal Chest

Functional Class I l l Exertional

Cold Left Foot

CASE28 Angina

184

CASE64 Dyspnea

34

Class Il l Exertional Shortness of Breath without

Progressive Exertional Shortness of Breath after Onset of Dyspnea

23

CASE10

CASES and Lower Extremity Edema

Myocardial Infarction

230

CASE81 Severe Chronic Obstructive Pulmonary

310

Disease

259

C O N T E N TS BY S U B J E C T I XV

Native and Prosthetic Valvular Disease,

CASE71

including Infective Endocarditis

Class I l l Shortness of Breath

CASE4

CASE72

Heart Murmur

16

25

31

42

66

CASE22 75

CASE23

78

110

Pericardial and Right-sided Disease, including Pulmonary Hypertension CASE14 Cardiac Murmur during Pregnancy

and Dyspnea

Harsh Systolic Murmur in Nursery Examination

Heart Failure

Progressive Exertional Dyspnea, Weight Gain, 152

Transthoracic Echocardiography on a

Dyspnea and Asymmetrical Ground-glass

Sonography Student 170

CASE54 175

84

91

99

CASE39 Loud Systolic Murmur

CASE56

130

CASE40

Hyperlipidemia, Hypertension and 182

Exertional Shortness of Breath with No Medical History

CASE61

133

CASE45

Transient Loss of Vision with Modest Exertional Shortness of Breath

63

CASE29 Acute Pleuritic Chest Pain and Lightheadedness

Congenital Aortic Valve Stenosis Status Post

Carotid Disease

50

CASE27

CASE53

Replacement

46

CASE25

CASE47

Pulmonary Infiltrates

323

CASE19

CASE33

and Peripheral Edema

Valve Replacements

Liver Function Test Abnormalities, Ascites,

Extremity Edema, and Systolic Murmur after

Dyspnea and Chest Tightness

CASE98

CASE16

Exertional Shortness of Breath, Bilateral Lower Coronary Artery Grafting

276

Exertional Dyspnea following Mitral and Aortic 59

CASE20

Progressive Dyspnea

CASE86 Heart Failure

56

CASE18

Progressive NYHA Class I l l Dyspnea

27 3

Recurrent Episodes of Left

CASE17

Harsh Systolic Murmur

266

CASE85 Diabetic Nephropathy

Exertional Shortness of Breath and

Right Heart Failure

CASE83 of Breath

39

CASE 13 Systolic Murmur

248

Slowly Progressive Exertional Shortness

CASE 12 Systolic Murmur, Asymptomatic

Three Months of Progressive Exertional Dyspnea

CASE9 Systolic Murmur

226

CASE78

CASE7 Acute Severe Dyspnea

Asymptomatic Murmur

221

195

Dyspnea and Presyncope

146

XVI I C ONT E NTS BY S U B J E C T

CASE49

CASE36 Exertional Fatigue, Dyspnea, and

Progressive Exertional Fatigue and Lower Extremity Edema

161

121

CASE42

CASE50 Progressive Exertional Shortness of Breath and Lower Extremity Edema

Two Pillow Orthopnea

Progressive Exertional Dyspnea, Atypical Chest 163

Discomfort, Abdominal and Lower Extremity Swelling

139

CASE52

Systemic Diseases

Fevers, Malaise, and Fatigue

CASE15

CASE58

Progressive Dyspnea with Asthma,

Progressive Exertional Shortness of

Rhinosinusitis, Weight Loss, and

Breath with Diabetes and Systemic

Peripheral Neuropathy

48

CASE24

187

CASE80 Exertional Shortness of Breath with

Symptomatic Paroxysmal Atrial Fibrillation

Hypertension

167

81

History of Lymphoma

256

Echocardiography A Case-Based Review

CAS E

1

Normal I mages a n d Va l ues

B values from a comprehensive echocardiographic examination. Please review all mov­

efore looking at abnormal studies, we need to be familiar with normal images and

ing (Videos 1 - 1 to 1-23) and still images (Figs. 1 - 1 to 1 - 1 8) obtained from an otherwise healthy normal individual who was referred for a transthoracic echocardiogram in the set­ ting of palpitations. A comprehensive echocardiography study demonstrated no abnormal findings. Also shown are animations demonstrating four standard transthoracic echocar­ diographic views and representative images obtained from each view (Animations 1 - 1 to 1 -4) . After your review of all images, please answer the following questions. You should be able to answer and understand all the questions to be able to provide diagnostically helpful data to clinicians ordering an echocardiogram.

Figure

1-1

Figure

1 -3

Figure

1 -2

Figure

1 -4

2 I E C H O C A R D I O G RA P H Y: A C A S E - B A S E D R E V I EW

Figure

1 -5

Figure

1 -8

Figure

1 -6

Figure

1 -9

Figure

1 -7

Figure

1-10

CASE 1 I 3

V•� PG

Ill

13 �, 1-.'P 0 .-.-.;'4fJ

12&182

tlrutG

....

:: :.·

··�

:J

f'YI �.� ... '""'''-'



.. ,_...

It> �..;,·, : 4\.•·-.·

.-

••

r

........,:,._...,. ' ...

I

"

.� :

,. 0

'\

'\

'

. u ::i

'

v1



a,,,,..,..,

Figure

1-1 1

Figure

1-14

Figure

1-12

Figure

1-15

Figure

1-13

Figure

1-16

IU •OD

4 I E C H O C A R D I O G RA P H Y: A C A S E - B A S E D R E V I EW

Figure

QUESTION 1 . Please estimate left ventricular ejection fraction (LVEF) by your visual subjective reading. Write that ejection fraction (EF) down somewhere. Then, calculate LVEF from left ventricular (LV) end­ diastolic dimension of 45 mm and end-systolic dimen­ sion of 28 mm. (This is a Quinones, simple method for calculating EF, which we still use in our clinical prac­ tice.) If you are not familiar with this equation, please see pages 1 1 5 to 1 1 6, section on ejection fraction, in The Echo Manual, 3rd Edition. Please compare your visual EF with the EF from Quinones method. You need to understand the apical factor.

QUESTION2. Which of the following is the modified Bernoulli equation?

A. B. C. D.

(2 x velocity)2 4 x velocity 4 + velocity2 42 x velocity

QUESTION 3. From this case, calculate right ventric­

ular systolic pressure (RVSP) . How would you report this patient's RVSP? A. B. C. D.

Cannot calculate 1 3 mm Hg 1 8 mm Hg Normal

Figure

1-17

1-18

QUESTION 4. Calculate cardiac output.

A. B. C. D. E.

4.4 5.5 6.6 7.7 8.8

L per minute L per minute L per minute L per minute L per minute

Q UESTION 5. Which of the following parameters

represents normal LV myocardial relaxation property? A. E/A > 1 B . Isovolumic relaxation time (IVRT) IVRT 70 to 1 00 milliseconds C. Deceleration time 1 60 to 240 milliseconds D. Mitral medial annulus e' 1 0 cm per second

QUESTION 6. Which of the following statements is correct regarding left atrial (LA) volume?

A. Enlarged LA volume does not always indicate increased LV filling pressure B. Healthy individuals cannot have large LA volume C. LA volume is always increased in patients with atrial fibrillation

CASE 1 I 5

Q UESTION 7. Which of the following conditions

QUESTION JO. Pulmonary vein Doppler recording is

can be correctly diagnosed from abdominal aortic pulsed wave Doppler examination?

useful in all of the following situations excep t.

A. B. C. D.

Mild to moderate aortic regurgitation (AR) Constrictive pericarditis Aortic coarctation Hypertrophic cardiomyopathy (HCM)

Q UESTION 8. What value of diastolic reversal flow

time velocity integral (TVI) in the proximal descend­ ing aorta indicates severe AR? A. B. C. D.

5 cm 1 0 cm 1 5 cm Depends on heart rate

Q UESTION 9. Which of the following segments is not seen from apical three- or five-chamber (long­ axis) view?

A. B. C. D.

Inferobasal septum Inferolateral wall Anteroseptum Apical septum

A. B. C. D.

Diastolic function assessment Aortic stenosis Mitral valve regurgitation After atrial fibrillation ablation procedure

QUESTION 11. Which of the following situations is best to use Valsalva maneuver?

A. To differentiate Grade 2 from Grade 1 diastolic dysfunction B. To differentiate aortic stenosis from HCM C. To evaluate patent foramen ovale D. To differentiate constriction from restriction

6 I E C H O C A R D I O G RA P H Y: A C A S E - B A S E D R E V I EW

ANS WER 1 : (4 5 2 - 2 82)/4 52 g ives 0 . 6 2 . The refo re, LVE F i s 62 % u n corrected . As a p i ca l contract i l ity is n o r m a l , C o rrected LVE F = 6 2 % = 60% = 66 %

+ +

[( 1 00 - 62) 6%

x

1 5%]

See The Echo Manual, 3rd Edition, d iscussion of ejec­ tion fraction on pages 1 1 5 to 1 1 6, and Figure 7-1 on page 1 1 0.

ANSWER 2: A. The Mod ified Bern o u l l i eq uation is 4 x velocity2, which ca n be f u rther s i m p l ified as (2 x ve­ locity)2 . The s i m p l ified method is m o re p ractica l for ve locities such as 3 . 5 m per second . Rather than s q u a r i n g 3 . 5 m per second fi rst before bei n g m u lti p l i ed by 4, (2 x 3 . 5) is 7 , which ca n be sq u a red to provi de the va l ue of 49 . See The Echo Manual, 3rd Edition, d i scussion of tra n sva lvu l a r g ra d i e nts on pages 63 to 66, a n d Figure 4-8 on page 6 5 .

ANSWER 3: D. U s i n g t h e mod ified Bernou l l i equation, the tricuspid reg u rg itati on (TR) velocity of 1 . 8 mis g ives a tra nstricuspid va lve g rad ient of 1 3 mm H g. If yo u add a n o r m a l r i g h t atri a l p ressu re est i m ate of 5 m m H g , RVS P is 2 3 m m H g . H oweve r, t h e TR ve l ocity p rofi l e is i n co m p l ete a n d l i kely u n d e rest i m ated , a n d w e may j u st say that p u l m o n a ry a rtery systol i c p ress u re is n o r­ m a l rathe r than g iv i n g a n act u a l va l u e . R i g h t ventri c u l a r outflow tract velocity a l so shows ra p i d onset o f systo l i c velocity com pati b l e with a n o r m a l RVS P. ANSWER 4: D. Stroke vol ume (SV) is ca lcu lated from the left ventricular outflow tract (LVOT) as a prod uct of the LVOT a rea (LVOT d i ameter x 0. 785) and the LVOT TVI . Cardiac output is the prod uct of SV and hea rt rate. Here SV = (2 . 3 cm)2 x 2 1 cm = 1 1 1 m l at a heart rate of 69 beats per m i n ute = 7 . 7 L per m i n ute See The Echo Manual, 3rd Edition, Fig u re 4- 1 6 on page 7 1 .

ANSWER 5: D. A l l oth e r p a r a m eters co u l d a l so be co nsistent with normal myoca rd i a l rel axati o n but n ot specific s i n ce a co m b i nation of h i g h fi l l i n g p ressu re a n d a b n o r m a l re laxation ca n g ive a s i m i l a r va l u e f o r E/A, IVRT, a n d dece l e ration t i m e . Early d i asto l i c velocity of the m itra l a n n u l u s has been fou n d to h ave a good cor­ relation with ta u w h i c h is the g o l d -sta n d a rd meas u re of myoca rd i a l relaxation by ca rd iac cath eterizat i o n . Al­ most if n ot a l l of myoca rd i a l d i seases h ave a b n o r m a l myoca rd i a l relaxatio n .

ANS WER 6: A. Any chron i c elevation i n LA p ress u re wi l l lead to LA d i latation over t i m e . H oweve r, i n the a bsence of elevation i n LA pressu re, modest deg rees of LA d i l atation may occur in the setting of atria l fibril lation . C h ronic d i astolic dysfunction a l so p roduces LA e n l a rge­ ment without a n increase in fi l l i ng p ressure . Wel l tra i ned hea lthy i n d ividuals ca n have increased LA vol ume. In that situation, you wou l d expect a n increased stroke vol u m e . ANSWER 7: C. Abdo m i n a l aortic vel ocity shows d ia­ stolic flow reversa l i n severe AR. Asce n d i n g aorta shows a notched velocity i n patients with hypertrophic obstructive ca rd i omyopathy, but it is not usually seen i n abdom i n a l aorta . Nonobstructive H C M does not have characteristic flow velocity pattern in the aorta . Freq uently, character­ istic abdom i n a l aorta pu lse wave Doppler velocity g ives an i n itia l d iag nostic c l u e for coa rctatio n . ANS WER 8 : C. Desce n d i n g aorta pu lse wave Doppler is very h e l pf u l i n dete rm i n i n g the severity of aortic re­ g u rgitation especia l ly when AR jet is eccentric. Althou g h there a re not many p u b l i cations regard i n g t h e param­ eter, time velocity i nteg ra l of d iasto l i c reversa l flow ve­ locity from the desce n d i n g aorta (by placing the sa m p l e vol u m e away from t h e i n n e r wa l l o f t h e aorta a t t h e level of the left su bclavia n a rtery) of 1 5 cm or g reater i n d icates severe AR. There a re however severa l other co nd itions which ca n g ive a s i m i la r d iasto l i c reversa l flow velocities. ANS WER 9: A. The i nferior sept u m is seen from the a p i ca l fou r-cha m ber view. I nferior wa l l is seen from a p i ­ cal 2 c h a m ber view. ANSWER 10: B. The p u l m o n a ry vei n Doppler p rofi le g ives i nsig hts i nto the d iastolic function (systolic b l u nting suggests elevated LA pressu re), presence of severe m itra l valve reg u rg itation (systolic flow reversals), or the presence of p u l monary vei n stenosis (a compl ication after a p u l mo­ nary vei n isolation proced u re). There is no specific p u l mo­ nary vei n Doppler finding that suggests aortic stenosis. ANSWER 11: C. The Va lsalva maneuver red uces venous return by i ncreasi n g i ntrathoractic press u re . It is often helpful (but not a lways) to differentiate pseudo-normal­ ized m itra l i nflow (grade 2) from true normal m itra l in­ flow. However, the disti nction ca n be done now easi ly by tissue Doppler i m a g i n g with the early diastolic velocity of the m itra l a n n u l us (e') . One of most va l u a b l e i n d i cations for the Va lsa lva maneuver is to assess right to left s h u nt via the patent foramen ova l e . Upon release of the ma­ neuver, venous return i ncreases to the right atri i u m and a u g ment or demonstrate right to left atrial s h u nt.

CAS E 2

Syncopal Episode

Mafter a syncopal episode. She was at college basketball practice and felt briefly light­

s. NE is a 1 9-year-old woman who is referred for a transthoracic echocardiogram

headed and then passed out, striking her head and sustaining a scalp laceration. She has no known cardiac history and is on no medications. On physical examination, her blood pressure is 1 00/62 mm Hg and heart rate 50 beats per minute with a regular rhythm. Carotid pulses and j ugular venous pulse were normal. Precordial examination was normal apart from a 1 16 systolic ejection murmur.

QUESTION 1. Concerning measurement of left

ventricular (LV) dimensions (see Video 2-1 and Figs. 2- 1 and 2-2) , which of the following statements is correct?

A. LV end-diastolic and end-systolic dimensions are measured at the level of the mitral valves tips B. The timing of the LV end-diastolic dimension measurement is at the time of the largest dimension, typically at the onset of the QRS complex C. Using the apical contractility correction factor (assuming it is normal) , the calculated LV ejec­ tion fraction is 5 8 % D . All of the choices

Figure 2-1

Figure 2-2

7

8 I E C H O C A R D I O G RA P H Y: A C A S E - B A S E D R E V I EW

QUESTION2. The estimated pulmonary artery systolic

QUESTION 3. The grade of diastolic (dys) function

(PASP) and diastolic pressures (PADP) (see Video 2-2 and Figs. 2-3 and 2-4) are:

(see Video 2-3 and Figs. 2-5 to 2-7) is:

Figure 2-5 Figure 2-3

Figure 2-6 Figure 2-4 A. B. C. D. E.

1 8/3 mm Hg 23/ 1 4 mm Hg 23/8 mm Hg 23/ 1 1 mm Hg 1 8/8 mm Hg

Figure 2-7

CASE 2 I 9

A. Normal diastolic function B. Grade 1 (impaired diastolic dysfunction) C. Grade 2 (delayed relaxation diastolic dysfunc­ tion) D. Grade 3 (restrictive diastolic dysfunction)

QUESTION 4. This pulsed wave flow pattern taken in

the abdominal aorta (see Fig. 2-8) would be compat­ ible with:

Figure 2-8 A. Coarctation of the descending thoracic aorta B. What is seen in the majority of patients with a bicuspid aortic valve C. Patent ductus arteriosus D. Severe congenital aortic stenosis

QUES TION 5. Transthoracic echocardiography can

exclude the following potential causes for syncope in a young women except: A. Critical congenital aortic stenosis B. Dilated cardiomyopathy with reduced ejection fraction C. Pulmonary arterial hypertension D. Aortic dissection E. Hypertrophic cardiomyopathy

1 0 I E C H O C A R D I O G RA P H Y: A C A S E - B AS E D R E V I EW

ANSWER 1: D. As reco m m e n ded by the America n Society o f Echoca rd iogra p hy, t h e long- a n d short-axis d i mensions ca n be obta i ned d i rectly from the end-systolic and end-d iastolic d i mensions (ESd and E Dd), measu red at the level of the m itra l tips as the smal lest a n d l a rgest d i a m eters, respectively. If there a re no reg ional wa l l mo­ tion abnormal ities, the LV d i mensions measu red from the level of the pa p i l l a ry m uscles ca n be used to ca lculate the left ventricular ejection fraction (LVEF) as fol lows: U n corrected LVE F = [(E D d)2 - (ESd)2] I (EDd)2] C o rrected LVE F = uLVEF + [( 1 00 - uLVEF) x 1 5%] uLVE F = u ncorrected LVE F

x

So h e re R V systol i c p ress u re = = And PAD P = =

PAS P = 1 8 + 5 m m H g 23 mm Hg 3 + 5 mm Hg 8 mm Hg

See The Echo Manual, 3rd Edition, pages 1 43 t o 1 47.

1 00

H e re, U n corrected LVE F = [(5 0)2 - (35)2] I (50)2] x 1 00 = 5 1 % C o r rected LVE F = 5 1 + [( 1 00 - 5 1 ) x 1 5 % ] = 5 8 % See The Echo Manual, 3rd Edition, page 1 09.

ANSWER 2: C. I n the a bsence of p u l m o n a ry stenosis, d e m o n strated h e re by a p u l m o n a ry va lve that opens n o rm a l ly o n two-d i m e n s i o n a l ( 2 D ) i m a g i n g without turbu lence on color flow i m a g i n g and a pea k ve locity of o n l y 1 . 2 m per seco n d , the rig ht ventric u l a r (RV) sys­ tol i c p ress u re is eq u iva l ent to the PAS P. RV systo l i c (a n d therefore P A systol ic) p ressu re ca n rel ia b ly be est i m ated on the basis of D o p p l e r i nterrogati o n of tricuspid va lve reg u rg itati o n . RV systol i c p ressu re is ca l c u l ated by add­ ing a n esti mate of rig ht atri a l p ressu re to the pea k g ra­ d i ent between the RV and the RA ( i . e . , fou r t i m es [the pea k tricusp i d reg u rg ita nt velocity]2) . H e re RV systol i c p ress u re = 4 (2 . 1 )2 + right atri a l p ress u re = 1 8 + RA p ress u re See The Echo Manual, 3rd Edition, F i g u re 9-2 on page 1 45, and pages 1 44 to 1 46.

The d i asto l i c PA p ressu re can be est i m ated by a d d ­ i n g a n estimate o f r i g h t atri a l p ress u re t o the g ra d i e n t betwee n the P A a n d the R V i n e n d d i astole (i . e . , fo u r t i m es [the e n d p u l m o n a ry reg u rg ita nt velocity]2) . See The Echo Manual, 3rd Edition, text and Figu re 9-7 on page 1 47.

H e re PAD P = 4 (0. 8)2 + right atr i a l p ressu re = 3 + RA p ressu re

When the d i a m eter of the i nfe r i o r ve n a cava is sma l l (< 1 7 to 2 0 m m ) a n d d ecreases b y 5 0 % o r m o re o n i n s p i ration, the RA p ress u re i s n o r m a l (a p p roxi mately 5 mm Hg).

ANSWER 3: A. T h e m i tra l i nfl ow patte r n ( F i g . 2 - 5) d e m o n strates that m ost of LV fi l l i n g occu rs e a r l y i n d i asto l e with a n E velocity o f 0 . 7 m p e r seco n d with a re l atively s h o rt dece l e ra t i o n t i m e of 1 6 1 m i l l isec­ o n d and a d i m i n utive A velocity of 0 . 3 m p e r seco n d ( E/A rat i o o f 2 . 5) . D o p p l e r i nte r rog a t i o n o f t h e p u l ­ m o n a ry ve i n s (Fi g . 2 - 6) d e m o n strates a d i a sto l i c p re­ dom i n a nt pattern . This co m b i nation of fi n d i n g s wo u l d p o i n t towa rd a rest r i ctive, G ra d e 3 , seve re d i a sto l i c dysf u n ction pattern . H owever, one a lso notes that 2 D i mages demonstrate n o r m a l LV size a n d ejection fracti on a n d the left atri u m is a l so o f n o r m a l s i z e . F i g u re 2 - 7 d e m o n strates t h e med i a l m itra l a n n u l us tissue Dop p l er with evi dence of exce l l ent myoca rd i a l relaxation with an e p r i m e (e ' ) ve­ locity of 1 3 cm per seco n d . I n n o r m a l yo u n g peo p l e , L V rel axati o n is s o v i g o ro u s t h a t the neg ative p ressu res g e n e rated i n the l eft ve n ­ tricle lead t o a l m ost a l l fi l l i n g t o occ u r early i n d i asto le, leavi n g l ittl e for atr i a l contract i o n to contri bute. It i s n o t u ncom m o n t o s e e d o m i n a nt d iasto l i c forwa rd flow ve locity i n p u l m o n a ry vei n i n hea lthy you n g i n d ivid u a l . Aga i n , the most i m porta nt p a ra m ete r for assess i n g d i a­ sto l i c f u n ction is the status of myoca rd i a l rel axation as­ sessed by e'. If e ' ve locity is normal, d i asto l i c fu nction is u s u a l l y norm a l . An i m porta nt d i sease condition with n o r m a l e' ve locity (from med i a l m itra l a n n u l us) is con­ stri ctive perica rditis. See The Echo Manual, 3rd Edition, F i g u re 8-22 on page 1 33, and text on pages 1 32 to 1 36.

ANSWER 4: B. H e re s h own i s an exa m p l e of a n o r­ m a l p u lse wave D o p p l e r a ssess m e n t of t h e a b d o m i n a l a o rta taken from t h e su bcosta l l o n g itu d i n a l p l a n e . Th is s h o u l d be p a rt of t h e sta n d a rd ech oca rd i o g ra p h i c ex­ a m i n a t i o n espec i a l ly in yo u n g patie nts with ca rd i ovas-

CASE 2 I 1 1

c u l a r sym ptoms a n d/o r hyperte n s i o n . S h own h e re a re a b r i s k systo l i c u pstro ke, b r i s k systo l i c d own stro ke, and a sm a l l early d i a sto l i c fl ow reve rsa l (seen b e l ow t h e base l i n e) . N ote a l so t h e l a c k of forwa rd fl ow. The typ i c a l fi n d i n g s i n coa rcta t i o n of t h e a o rta a re a p ro­ l o n g e d t i m e to peak ve locity a n d p e rs i stent fo rwa rd d i a sto l i c fl ow. La rge d i a sto l i c fl ow reve rsa l s a re a fea­ t u re of a pate nt d u ct u s a rt e r i o s u s o r seve re a o rt i c reg u rg itat i o n . Seve re a o rt i c ste n os i s typ i ca l ly wi l l h ave a b l u nted pea k systo l i c ve locity with a d e c reased t i m e t o pea k . A l th o u g h a p a t i e n t w i t h a b i c u s p i d a o rt i c va lve i s at r i s k for coa rctat i o n , a o rt i c va lve ste n o s i s , a n d reg u rg itat i o n-a ny of wh i c h if seve re m a y g ive rise to a b n o r m a l ities o n p u lse wave i nte rrog ation of t h e a b d o m i n a l a o rta-t h e vast m aj o r i ty of p a t i e n ts with a b i c u s p i d va lve wi l l h ave a n o r m a l fi n d i n g a s seen h e re .

ANSWER 5: D. A l l of t h e opti o n s i n t h e q u esti o n ste m a re potenti a l ca rd i ovasc u l a r ca uses fo r syn cope. A n o r m a l tra nsthora c i c ech oca rd i o g ra m ca n re l i a b ly exc l u d e a l l of these as a cause for syncope except for a n a o rt i c d i ssecti o n . S h o u l d an a o rt i c d i ssect i o n be suspected c l i n ical ly, a tra nsesophageal echoca rd iogram o r a contrast CT a n g i o g ra m of the ao rta i s req u i red . Anoth er potential cause for syncope i n a you n g person is a n a rrhyth m i a tri g g e red by myoca rd i a l ische m i a i n the sett i n g of a n a n o m a l o u s coro n a ry a rte ry. A l t h o u g h a n o m a l o u s coro n a ry ost i a m a y b e seen on tra nstho­ racic i m a g i n g , a n a l tern ative i m a g i n g m o d a l ity i s ofte n needed to exc l u d e t h i s a l so. It is a l so often d iffi c u l t t o exc l u d e a rrhyth mog e n i c r i g h t ve ntri c u l a r dys p l a s i a wh ich may req u i re ca rd iac co m p uted to mography o r mag netic reso n a nce i m a g i n g .

See The Echo Manual, 3rd Edition, F i g u re 20-29 o n page 3 54.

See The Echo Manual, 3rd Edition, discussion of goal di rected echo exam of syncope on pages 396 and 397.

CAS E 3

Prog ressive Exertional Shortness of Breath

Mness of breath (NYHA functional class III) . He is a prior smoker, but has no other r. HM is a 42-year-old man with a 7-month history of progressive exertional short­

known cardiovascular disease. His family history is notable for sudden unexplained death of his mother at the age of 40. His only sibling who is 5 5 years of age recently underwent a normal cardiovascular comprehensive examination with echocardiography. On examination, his blood pressure is 1 25/70 mm Hg and heart rate 60 bpm. His carotid pulses have brisk upstrokes, and his central venous pressure is normal. He has a sustained and localized left ventricular (LV) apical impulse. There is a 2/6 systolic ejection murmur that increases when the patient goes from a squat position to standing. He is referred for transthoracic echocardiography (see Video 3-1) .

QUESTION I. Which of the following factors is not associated with an increased risk of sudden cardiac death in this case?

A. B. C. D.

Septal wall thickness Sudden death of his mother at the age of 40 Severe central mitral valve regurgitation Nonsustained ventricular tachycardia on a Holter

QUESTION 2. Calculate the maximal instantaneous

intracavitatory gradient (see Fig. 3- 1 ) .

Figure 3-1 12

A. B. C. D. E.

1 6 mm Hg 49 mm Hg 64 mm Hg 1 36 mm Hg 1 96 mm Hg

QUESTION3. In this setting, the overall LV hemody­

namics in this case would be consistent with which of the following aortic systolic blood pressures, assuming the left atrial pressure is 20 mm Hg (see Fig. 3- 1 ) ? A. B. C. D.

90 mm Hg 1 20 mm Hg 1 5 0 mm Hg 200 mm Hg

CASE 3 I 1 3

Q UES TION 4. In hypertrophic cardiomyopathy

QUESTION 6. Following surgical myectomy, what ab­

(HCM) , what happens to the left ventricular outflow tract (LVOT) gradient after a PVC (see Fig. 3-2) ?

normal finding is present (see Video 3-2 and Fig. 3-3) ?

Figure 3-2 A. It goes up B. It goes down

QUESTION 5. With regard to the management of this patient, which of the following is true?

A. The overall complication rate is greater with surgical myectomy than with alcohol septal ab­ lation B. The risk of sudden cardiac death would be lower after septal alcohol ablation than after surgical myectomy C. Complete heart block is more likely with alco­ hol ablation than with surgical myectomy D. Alcohol septal ablation would obviate the need for defibrillator to reduce the risk of sudden cardiac death

Figure 3-3 A. B. C. D.

Ventricular septal defect Significant aortic valve regurgitation Coronary-ventricular fistula Residual systolic anterior motion of the mitral valve with mitral valve regurgitation

1 4 I E C H O C A R D I O G RA P H Y: A C A S E - B AS E D R E V I EW

ANS WER 1 : C. Ech oca rd iog ra p hy in this case dem­ o n st rates seve re i n crease in ci rcu mfe renti a l LV wa l l thickness with massive t h i c ke n i n g of the i ntraventricu­ lar septu m (septa l d i m e n s i o n of 43 mm). LV ej ection fraction i s norma l . I n H C M , seve ra l c l i n i ca l a n d echo­ card i o g ra p h i c factors have been associ ated with a n i n c reased risk o f s u d d e n ca rd iac death . The five most freq uently cited factors a re as fol l ows : S u d d e n C a rd i a c Death Risk Stratification i n H C M I . LV wa l l t h i c k n ess � 3 0 m m (typ i ca l ly septum-but a ny wa l l ) I I . A b n o r m a l b l ood p ressu re response t o exercise I l l . Nonsusta i ned ventri c u l a r tachyca rd i a I V . Fa m i ly h i sto ry o f S C D V. Recu rrent syn cope1 -3 See The Echo Manual, 3rd Edition, page 257, last sentence i n the right col u m n .

ANSWER 2: C. 6 4 m m H g . Th is conti n uous wave Dop­ pler obta i ned from the a pex i l l u strates two s i g n a l s : the LVOT (righ t) obstruction a n d superim posed i nco m p l ete s i g n a l of m itra l reg u rg itati o n (left) . It ca n be someti mes d i ffi cu lt to d i sti n g u i sh between th e m . In H C M , m itra l reg u rg itation usua l l y beg i n s at m i dsysto le when there is systo l i c a nte r i o r motion of the m itra l va lve . There­ fore, the D o p p l e r spect r u m of m itra l reg u rg itation is often i n co m p l ete a n d may s u perfi c i a l ly rese m b l e the LVOT fl ow velocity spectru m . H owever, the r i s i n g slope at m i dsysto l e is u s u a l ly perpe n d i c u l a r to the base l i n e i n m itra l reg u rg itation, whereas i t is c u rvi l i n e a r u n t i l it reaches the h i g h est velocity in the LVOT s i g n a l . F u r­ t h e r m o re , t h e m i tra l reg u rg ita t i o n ve l o c i ty s i g n a l exten d s beyo n d ej ection a n d c u l m i n ates i n m itra l for­ wa rd flow d u ri n g the onset of d i a stol e . Remember that the m itra l reg u rg itat i o n ve locity w i l l a l ways be m o re (7 m p e r second) t h a n that of the LVOT jet ve locity (4 m per seco n d ) . U s i n g the mod ified Bernou l l i equa­ tion, the pea k p ressu re g rad ient may be ca l c u l ated from the late-pea k i n g dagger-sha ped LVOT Doppler s i g n a l as fo l l ows 4v2 = 4(4)2 = 64 m m H g . See The Echo Manual, 3rd Edition, pages 259 to 26 1 , and Figures 1 5- 1 4 to 1 5-1 7 o n page 261 .

ANSWER 3: C. 1 5 0 m m H g . With t h e ava i l a b l e data, LV systo l i c p ress u re c a n b e ca l c u l ated i n two ways . I n t h e h e m odyn a m i c echoca rd i o g ra p h i c as­ sess m e n t of a patient with H C M , it i s good p rac­ tice to d e rive both m e a s u res to e n s u re i n tern a l

co nsiste n cy . LV systo l i c p ressu re e q u a l s t h e pea k g ra­ d i ent across the m itra l va lve (4v2; where v is the pea k m itra l reg u rg ita nt velocity) p l us a n estimate of l eft atri a l p ress u re . I n t h i s case, 4(7)2 + 2 0 = 2 1 6 m m H g . LV p ressu re = systo l i c b l oo d p ress u re + i ntracavitatory g ra d i ent Systo l i c b l ood p ress u re = LV p ress u re - i ntracavitatory g rad ient Systo l i c b l ood p ress u re = 2 1 6 - 64 = 1 5 2 m m Hg Altern atively (in the a bsence of a o rtic va lve d i sease), LV p ress u re a lso e q u a l s systo l i c b l ood p ressu re p l us the i ntracavitatory g rad ient. Hence, u s i n g this strategy, one can estimate the LVOT g rad ient i n two sepa rate ways i n H C M patients i n the echo laboratory: the fi rst t h ro u g h d i rectly measu ri n g t h e pea k velocity o f t h e LVOT conti n uous wave Doppler signal and the second using the pea k velocity of the m itra l reg u rg ita nt conti n uous wave Doppler signal and the bra­ chia! systol i c blood pressure . It is advisa b l e to ensure the consistency of these two strategies wherever possi ble. See The Echo Manual, 3rd Edition, fi rst paragraph on page 26 1 . See Circulation.

ANSWER 4: A. It goes u p . D u ri n g the prolonged phase of LV fi l l i n g associated with a prematu re ventri c u l a r com p l ex, there is a n i ncrease i n LV vol u m e that i n turn potentiates a n i ncrease i n LV p ressu re . I n this setting, there is a d ifferential physiologic response i n the setting of fixed (e . g . , aortic stenosis) or dynamic (e . g . , hypertrophic obstructive ca rd iomyopathy) LV obstructio n . The i ntensity of the systol i c flow increases i n aortic stenosis a n d H C M . The aortic pressure increases i n fixed obstruction, but de­ creases o r rem a i n s uncha nged in dyna m i c obstruction . These f i n d i ngs i n patients with H C M reflect the B rocken­ broug h-B ra u nwa ld-Morrow sign where postextrasystolic potentiation resu lts i n a n i ncreased LVOT g ra d ient with decreased or u nchan ged aortic pu lse pressure . S e e The Echo Manual, 3rd Edition, F i g u re 1 5-1 5 on page 261 .

ANSWER 5: C. Pati e nts with hypert ro p h i c obstruc­ tive ca rd iomyopathy, who rem a i n sym ptomatic (NYHA class Ill to IV) desp ite medica l thera py, a re ca n d idates for i nvasive thera py with either s u rgical myectomy or septa l a b lati o n . The goal of either thera py is to rel i ef the out­ flow tract obstruction t h ro u g h p hysica l remova l (myec­ tomy) or t h ro u g h therapeutic i n fa rcti o n of the excess septa l m uscle mass. In experie nced centers, the i n h os­ pita l morta l ity rates with s u rgical myecto my a re l ow a n d

CASE 3 I 1 5

the overa l l success rates h i g h . Patients with conco m i ­ ta nt orga n i c m itra l va lve d isease or obstructive coronary d i sease a re ca n d i dates for com b i ned s u rg ica l co rrective p roce d u res. M itra l reg u rg itation re lated to systol i c a n ­ terior motion o f the m itra l va lve is usua l ly co rrected by myectomy without a m itra l va lve p roced u re . Both s u rg i ­ ca l se pta l myecto my a n d septa l a l cohol a b lation red uce LVOT obstruction a n d i m p rove sym ptom g rade. To date, there has been no ra ndomized com pa rison tria l of my­ ectomy versus a b l atio n . A recent meta-a na lysis i n d i cates a si m i la r in hosp ita l morta l i ty (0 . 6 % for myectomy a n d 1 . 6 % f o r a b lation). Septa l myectomy a p pea rs t o have a lower rate of permanent pace m a ker i m p l a ntation for com p l ete heart b l ock (3 . 3 % vs . 1 8 . 4 % ), a h i g h e r suc­ cess rate (req u i red repeat p roced u re 0 . 6 % i n myectomy patients vs . 5 . 5 % in a b lation patients) . P u b l i shed data point to a poss i b l e red ucti on i n sudden ca rd iac death

a n d rates of appropriate defi bri l l ator d i scharges fol l ow­ i n g myectomy. This does not appear to be the case fol­ lowi ng septa l a blation .4-9

ANS WER 6: C. A co m p l i ca t i o n t h a t i s re l atively u n i q u e to s u rg i c a l myecto my is the u n roof i n g of a n in­ tra m u ra l septa l coro n a ry a rtery at the myectomy s ite, t h u s creati n g a coro n a ry to LVOT fistu l a . Seen h e re is a D o p p l e r flow s i g n a l , below the aortic va lve, a ris­ ing from with i n the septa l myoca rd i u m i nto the LVOT. Two d i sti n ct c h a ra cte r i st i cs s e p a rati n g a coro n a ry fist u l a from a ve ntri c u l a r se pta l d efect a re d i recti o n (i nto t h e left ve ntricle as op posed t o typ ica l ly i nto t h e r i g ht ventri cle) a n d ti m i n g (d iasto l i c as opposed t o sys­ tol i c). Postmyectomy se pta l coro n a ry to LV fistu la is very rarely of any c l i n i ca l s i g n ifi cance. 1 0

References 1. Spirito P, Bellone P, Harris KM, et al. Magnitude of left ventricular hypertrophy and risk of sudden death in hypertrophic cardiomyopathy. New EnglJ Med. 2000;342: 1 778- 1 7 8 5 . 2. McKenna WJ, Behr ER. Hypertrophic cardiomyopathy: manage­ ment, risk stratification, and prevention of sudden death. Heart. 2002;87: 1 69. 3. Maron BJ, McKenna WJ, Danielson GK, et al. American College of Cardiology/European Society of Cardiology clinical expert consensus document on hypertrophic cardiomyopathy. A report of the American College of Cardiology Foundation Task Force on Clinical Expert Consensus Documents and the European Society of Cardiology Committee for Practice Guidelines. J Am Coll Cardiol. 2003;42: 1 687. 4. McLeod CJ, Ommen SR, Ackerman MJ, et al. Surgical septal my­ ectomy decreases the risk for appropriate implantable cardioverter defibrillator discharge in obstructive hypertrophic cardiomyopa­ thy. Eur Heart]. 2007;28:25 83-2 5 8 8 . 5. Ommen SR, Maron BJ, Olivotto I, e t al. Long-term effects o f surgical septal myectomy on survival i n patients with obstructive hypertrophic cardiomyopathy.jAm Coll Cardiol. 2005;46:470-476.

6. Qin JX, Shiota T, Lever HM, et al. Outcome of patients with hypertrophic obstructive cardiomyopathy after percutaneous transluminal septal myocardial ablation and septa! myectomy surgery. J Am Coll Cardiol. 200 1 ;38: 1 994-2000. 7. Sorajj a P, Valeri U, Nishimura RA, et al. Outcome of alcohol septa! ablation for obstructive hypertrophic cardiomyopathy. Circulation. 2008; 1 1 8 : 1 3 1 - 1 39. 8 . Talrej a D R, Nishimura RA, Edwards WD , et al. Alcohol septa! ablation versus surgical septa! myectomy: comparison of effects on atrioventricular conduction tissue. j Am Coll Cardiol. 2004;44:2329-2332. 9. Woo A, Williams WG, Choi R, et al. Clinical and echocardio­ graphic determinants of long-term survival after surgical myec­ tomy in obstructive hypertrophic cardiomyopathy. Circulation. 2005 ; 1 1 1 :2033-203 8 . 1 0 . Bax ]] , Raphael D, Bernard X, et a l . Echocardiographic detection and long-term outcome of coronary artery-left ventricle fistula after septal myectomy in hypertrophic obstruc­ tive cardiomyopathy. J Am Soc Echocardiogr. 200 1 ; 1 4:308-3 1 0 .

CAS E 4

Heart M urmur

A ejection murmur for the past 1 5 years. Over the past 2 years, she describes mildly 70-year-old woman presents with a heart murmur. She has a history of a systolic

increasing exertional shortness of breath. She now gets shortness of breath walking a 500 yards. On examination, her blood pressure is 1 30/80 mm Hg and her heart rate 65 beats per minute. Her central venous pressure is normal. Her carotid upstroke is delayed. She has a 3/6 ejection systolic murmur and a single second heart sound. She is referred for transthoracic echocardiography (Videos 4- 1 to 4-6 and Figs. 4- 1 to 4-5) .

16

Figure 4-1

Figure 4-3

Figure 4-2

Figure 4-4

CASE 4 I 1 7

Q UES TION 4. Which of the following factors may

lead to a disproportionately elevated aortic mean gra­ dient for a given aortic valve area? A. B. C. D.

An increase in left ventricular (LV) contractility Anemia Aortic valve regurgitation All of the choices

QUESTION 5. Which of the following is the next best management step?

Figure 4-5

QUESTION 1. What is the calculated aortic valve area?

A. B. C. D.

0.65 cm2 0.80 cm2 0 . 8 5 cm2 1 .0 cm2

QUESTION 2. What is the peak transaortic pressure

gradient? A. B. C. D.

75 mm Hg 9 5 mm Hg 1 00 mm Hg 1 2 1 mm Hg

QUESTION 3. Doppler-derived aortic valve pressure

gradients are typically slightly lower than that of the catheter-derived aortic valve pressure gradients. A. True B. False

A. Aortic valve replacement B. Coronary angiography and then aortic valve re­ placement C. Treadmill exercise testing D. Hemodynamic catheterization to assess the aor­ tic valve area

Q UESTION 6. Which of the following statements

is not correct regarding left ventricular outflow tract (LVOT) time velocity integral (TVl) and aortic valve TVl ratio? A. It is inversely proportional to aortic valve area and LVOT area ratio B. When multiplied by LVOT area, aortic valve area is calculated C. Aortic valve area is larger when the ratio is smaller D. It is useful when LVOT diameter cannot be de­ termined

1 8 I E C H O C A R D I O G RA P H Y: A C A S E - B AS E D R E V I EW

ANS WER 1 : A. Aort i c va lve a rea (LVOT a rea)]/Ao TVI = = = =

=

[(LVOT TVI)

x

[(LVOT TVI) x (0 . 7 8 5 (LVOT D)2]/Ao TVI [(2 5) x (3 . 1 4)]/1 2 0 7 8 . 5/1 2 0 0 . 6 5 cm2

H ere two conti n uous wave Doppler s i g n a l s a re shown, o n e from the a pex (F i g . 4-4) and one from the r i g h t pa rasternal a rea (F i g . 4-5) . I n a co m p rehensive exa m i ­ nation o f a patient with aortic stenosis, it is critica l to perform a Doppler eva l uation from a l l ava i la b l e tra ns­ d ucer w i n d ows . Fifteen to twenty percent of the time, the pea k signal wi l l be obta i ned from a win dow other t h a n the a p ex. See The Echo Manual, 3rd Edition, d i scussion of Doppler echoca rd iography i n aortic stenosis on pages 1 90 and 1 9 1 .

ANS WER 2: D . B l ood flow ve locity (v) measu red with D o p p l e r ech oca rd i o g ra p hy re l i a b l y reflects the p ressu re g ra d ient accord i n g to the mod ified Bernou l l i equation . Accord i n g to t h e equation, p ressu re g ra d i ­ ent 4v2 . H ere the pea k tra n saortic flow ve l ocity is 5 . 5 m per seco n d , w h i c h corresponds to a pea k tra ns­ aortic p ressu re g ra d i ent of 1 2 1 mm H g . =

ANS WER 3 : B . False. There typ ica l ly is a sma l l d if­ fere n ce betwee n the D o p p l e r-de rived a n d cath eter­ derived aortic va lve p ress u re g ra d i e nts beca u se of the p ressu re recovery phenomeno n . Pa rt of the ki netic en­ e rgy lost d u r i n g flow passag e t h ro u g h a sma l l orifice i s recovered . The refore, this p ressu re recovery resu lts i n a h i g her a bsol ute p ressu re i n the asce n d i n g a o rta away from the ste n otic a o rtic va lve, exp l a i n i n g why the cath ete r-derived p ressu re g rad ient i s l ower t h a n the Doppler-de rived p ress u re g rad ient ( D o p p l e r echo­ ca rd i o g ra p hy measu res the h i g h est va l ue). Pressu re re­ covery i s s m a l l e r when the aorta is d i lated . H owever,

Reference 1. Omran H, Schmidt H, Hackenbroch M, et al. Silent and appar­ ent cerebral embolism after retrograde catheterization of aortic valve in valvular stenosis: a prospective, randomized study. Lan­ cet. 2003;36 1 : 1 24 1 - 1 246.

p ress u re recove ry may be an i m porta nt facto r in caus­ ing a d iscre p a n cy between echo-de rived aortic va lve a rea a n d cath ete r-derived aortic va lve a rea . ANS WER 4: D. When LV systol i c f u n ction a n d ca r­ d i a c output a re a b n o r m a l l y h i g h , the fo l l owi n g poi nt s h o u l d be con s i d e red : pea k ve locity a n d mean aortic g ra d ient va ry with c h a n ges i n stroke vo l u m e . In pa­ tients with i n c reased ca rd iac output across the aortic va lve (as i n aortic reg u rg itation o r a n e m ia), aortic ste­ nosis may n ot be severe even when the pea k velocity is 4 . 5 m per secon d or g reater a n d the mean g ra d ient is 50 m m H g o r h i g h e r . Aortic va lve a rea s h o u l d be more h e l pf u l i n dete rm i n i n g the severity of aortic stenosis i n those situatio n s . ANS WER 5 : B . This wom a n has sym ptoms a n d s i g n s o f severe aortic va lve ste n o s i s . The ech oca rd i o g r a m demonstrates a mea n systol i c g rad ient over 50 m m H g a n d a va lve a rea 8 cm per secon d has a h i g h d i a g n ostic accu racy for sepa rati n g constriction from restricti o n . In restriction, the med i a l e ' is red uced, typ i ca l ly s i g n ifica ntly, beca use of a ma rked red uction in myoca rd i a l re l axation (often in the 3 to 6 cm per seco n d ra n g e ) . In contrast, i n constriction, myoca rd i a l relaxation is normal a n d e ' i s normal or even i n creased . The exp l a nation fo r t h i s i s that ventricu l a r fi l l i n g i s l i m ited by the l atera l expa nsion of the heart beca use of the constrictive perica rd i u m , a n d most ventri c u l a r fi l l i n g is accom p l ished b y a n exag­ gerated l o n g i tu d i n a l motion of the heart. Myoca rd i a l rel axation is relatively wel l p reserved i n constriction u n ­ less the myoca rd i u m is a l so i nvolved, as i n ra d i ation i nj u ry to the h e a rt . Sti l l e ' in patie nts with constric­ tion beca use of ra d i ation or p revious ca rd i a c s u rgery is lowe r than e' i n pati ents with constriction beca use of p revious pericarditis or col l agen vasc u l a r d i seases .3 ANS WER 6: A. As d i scussed i n the a n swer to Ques­ tion 4, myoca rd i a l rel axation is normal in constriction . I n fact, beca use ra d i a l motion of the myoca rd i u m maybe restri cted b y the scarred perica rd i u m , the me­ d i a l e ' velocity is often h i g h e r than wou l d be expected fo r the pati ent's a g e . In a l m ost a l l ca rd i a c conditions, the latera l e ' is 20% g reater t h a n the med i a l e' . H ow­ eve r, i n constrict i o n , the l atera l e ' , a lth o u g h typ ica l ly of normal ve locity, is often lower t h a n the med i a l e ' i n 8 0 % o f cases l ea d i n g t o a n " a n n u l us revers u s . " Th is reve rsed latera l a n d med i a l e' ratio returns to normal after pericardi ectomy.4-6

C AS E 1 6 I 5 5

ANSWER 7: D. Although most patients with constric­ tive pericarditis have evidence of pericardia! thicken i n g , approxi m ately 2 0 % wi l l have n o r m a l thickness. Useful measu res of pericardia! thickness i nc l u d e ca rd iac M R, TEE, and card iac CT. However, it may be d ifficult to dis­ ti n g u ish sma l l amou nts of perica rdia! fluid from th icken­ ing by CT. The presence of pericard i a ! ca lcification, best appreciated by CT, is h i g h ly specific for constriction, al­ though it is present i nfreq uently.7·8

ANSWER 8: A. All c l i ps demonstrate b l u nted systol i c forwa rd flow consistent with a n elevation i n right atria l pressure. There is evidence o f diastolic flow reversa ls that i ncrease with expi ration i n (Fig . 1 6- 1 1 ) that is consistent with constrictio n . I n contrast, d iastolic flow reversa ls that i ncrease with inspiration (Fig. 1 6- 1 3) a re consistent with restrictive card iomyopathy. M i d to late pea k i n g systol i c flow reversa ls (Fig. 1 6- 1 2) are characteristic o f severe tri­ cuspid va lve reg u rg itatio n .

References 1. Oh JK, Hade LK, Seward JB, et al. Diagnostic role of Doppler echocardiography in constrictive pericarditis. J Am Coll Cardiol. 1 994;23 ( 1 ) : 1 54-1 62.

5. Veress G , Ling LH, Kim KH , et al. Mitra! and tricuspid annular veloci­ ties before and afi:er pericardiectomy in patients with constrictive pericar­ ditis. Circ Cardiovasc Imagi,ng. 20 1 1 ;4(4) :399-407. Epub 20 1 1 May 4.

2. Oh JK, Tajik AJ, Appleton CP, et al. Preload reduction to unmask the characteristic Doppler features of constrictive pericarditis. A new observation. Circulation. 1 997; 9 5 (4) :796-799 .

6. Choi JH, Choi JO, Ryu DR, et al. Mitra! and tricuspid annular velocities in constrictive pericarditis and restrictive cardiomyopa­ thy: correlation with pericardia! thickness on computed tomogra­ phy. ]A CC Cardiovasc Imaging. 20 1 1 ;4(6) : 5 67-57 5 .

3 . Ha JW, Ommen SR, Tajik AJ, e t al. Differentiation of constric­ tive pericarditis from restrictive cardiomyopathy using mitral an­ nular velocity by tissue Doppler echocardiography. Am J Cardiol. 2004;94 (3) :3 1 6-3 1 9.

7. Ling L H , Oh JK, Tei C , e t al. Pericardia! thickness measured with transesophageal echocardiography: feasibility and potential clini­ cal usefulness. ] Am Coll Cardiol. 1 997;29: 1 3 1 7 .

4. Reuss CS, Wilansky SM, Lester SJ, et al. Using mitral "annulus reversus" to diagnose constrictive pericarditis. Eur J Echocardiogr. 2009; 1 0 (30) :372-375.

8 . Talreja DR, Edwards WD, Danielson GK, e t al. Constrictive peri­ carditis in 26 patients with histologically normal pericardia! thick­ ness. Circulation. 2003; 1 08 : 1 8 52- 1 857.

CAS E

1 7

l

R i g ht Heart Fa i l u re

Apericarditis. She was well until 2 years ago when she began having fluid retention with

7 4-year-old woman was referred for a possible pericardiectomy to manage constrictive

leg edema and abdominal bloating. She was treated initially with a diuretic and had some improvement. However, her symptoms became gradually worse and underwent compre­ hensive evaluation. Cardiac catheterization showed normal coronaries, but equalization of diastolic pressures with right atrial pressure of 20 mm Hg, right ventricular (RV) end­ diastolic pressure of 2 1 mm Hg, and left ventricular end-diastolic pressure of 1 9 mm Hg. A presumptive diagnosis of constrictive pericarditis was made, and the patient was recom­ mended to have pericardiectomy. The following echocardiogram images were obtained (Figs. 1 7- 1 to 1 7-6 and Videos 17- 1 to 17-7) .

56

Figure

1 7- 1

Figure

1 7-3

Figure

1 7-2

Figure

1 7-4

C AS E 1 7 I 5 7

QUESTION 3. Figure 1 7-2 shows pulmonary regur­

gitation (PR) velocity obtained by continuous wave Doppler echocardiography. Which of the following is related to the Doppler finding? A. Normal RV diastolic pressure because PR dia­ stolic velocity is < 1 m per second B. This is a normal PR pattern with respiratory variation C. Patient has severe PR D. Pulmonary artery end-diastolic pressure is high

Figure 1 7-5

;1

9 02 c.r� • O �H� • Go u,.. • O ,.....M g

V•I PG V•l PG

( ,,,_

I

,,

Doppler with respirometer indicate (Fig. 1 7-3) ?

�.-v·· ';rT--



TD! f�'\. : .c w.-.:

QUESTION 4. What does the following hepatic vein

ffl !0:'1. � ""' \/

.. ·



.� � ;-1, HHI

-ID

rl' ·' ',,_ ' ,,

� ,; �

,,

..

'

(t

I

t� ·,

'V

1•

n,.....,, ,

o

Severe tricuspid regurgitation Constrictive pericarditis RV dysfunction RV cardiomyopathy

QUESTION 5. Mitral inflow (Fig. 1 7-4) and mitral - •u - ·HJ 0

Figure 1 7-6

QUESTION 1. What is the diagnosis of this patient?

A. B. C. D. E.

A. B. C. D.

Constrictive pericarditis Restrictive cardiomyopathy Severe tricuspid regurgitation RV cardiomyopathy B and C

QUESTION 2. Which of the following statements is correct regarding pulmonary artery systolic pressure based on the shown tricuspid regurgitant velocity (Fig. 1 7- 1 ) in this patient?

A. Pulmonary artery systolic pressure is normal B . Pulmonary artery systolic pressure cannot be reliably obtained C. Pulmonary artery systolic pressure is severely elevated

annulus velocity (Figs. 1 7-5 and 1 7-6) recordings are shown. Which of the following is correct regarding these recordings? A. Typical for constrictive pericarditis B. Left ventricular cardiomyopathy with increased filling pressure C. Not able to assess diastolic function because of atrial fibrillation

5 8 I E C H O C A R D I O G RA P H Y: A C A S E - B AS E D R E V I EW

ANSWER 1: C. The p r i m a ry fi n d i n g is one of to rren­ t i a l t r i c u s p i d va lve reg u rg i tati o n s . 2D ech oca rd i og­ ra p h y d e m o n strates an e n l a rged RV with t r i c u s p i d a n n u l a r d i l at i o n , i n co m p l ete t r i c u s p i d va lve leaflet coaptati o n , a n d severe tri c u s p i d va lve reg u rg itat i o n , d e m o n strated o n co l o r D o p p l e r a n d c h a ra cte rized on conti n uous wave Doppler by a dense, early peak­ ing, " D a g g e r-sha ped " s i g n a l . There i s a l so evi d e n ce of a left ve ntri c u l a r myopat h i c p rocess with restrictive physiology. Alth o u g h in the sett i n g of atr i a l f i b r i l lation, there is a n elevated early l eft ventri c u l a r d i asto l i c fi l l i n g velocity (E) with red u ced d iasto l i c tissue Doppler veloci­ ties, with a med i a l E/e ' ratio of 1 7 . This correl ates with the el evated l eft ve ntri c u l a r fi l l i n g pressu res fo u n d at hemodyn a m i c catheterization .

ANSWER 2: B. There a re th ree n ecessa ry com po­ n e nts to the estimation of p u l m o n a ry a rtery systo l i c p ress u re . T h e fi rst i s exc l u s i o n o f R V o utfl ow t ract (RVOT) obstruction, the secon d a properly a l i g ned tri­ cuspid reg u rg itant veloc ity, and the t h i rd esti mation of ri g ht atr i a l p ress u re . I n this case, a pea k systo l i c ve locity t h ro u g h t h e RVOT o f 0 . 6 m p e r seco n d ex­ c l u des s i g n ificant outflow tract obstruction i n d i cati n g that the d e rived R V systol i c p ressu re s h o u l d equate to the p u l m o n a ry a rtery systol i c p ress u re . On the basis of on the mod ified B e r n o u l l i e q u a t i o n , the tra n s-tricus­ p i d g rad ient is equal to fo u r t i m es the (pea k tricusp i d reg u rg itant velocity), 2 w h i c h i n t h i s case is 4 ( 2 . 2)2 1 8 m m H g . O n the basis of the d i lated coro n a ry s i n us o n the paraste r n a l lon g-axis view, the d i lated i nfe rior vena cava a n d hepatic vei ns, and the ma rked systo l i c flow reversa l s i n the hepatic ve i ns, the right atr i a l p ressu re l i ke l y is m a rkedly e l evated, � 2 0 mm H g . Ofte n ri g ht atr i a l p ress u re wi l l exceed 2 0 m m H g , which is, gener­ a l ly, the recom m e n ded u p per l i m it of n o n i nvasive ly es­ t i m ated r i g ht atri a l p ress u re . The refo re, the RV systo l i c p ressu re m a y be u n d e rest i m ated i n t h i s setti n g . F i n a l ly, in severe tricuspid va lve reg u rg itation there ca n be ven ­ tri c u l a rization o f the atrial p ress u res. Th is re n d e rs t h e s i m p l ified B e r n o u l l i eq uation u n su ita b l e beca use the p roxi m a l ve locity i s n o l o n g e r s i g n ificantly l ess than the d i sta l velocity a n d therefore ca n n ot be d i sco u nted . Therefo re, i n torrenti a l tri c u s p i d va lve reg u rg itati o n , =

the classic esti mation o f R V systo l i c p ress u re ca n n ot be rel i ed on a n d a n a lternative method s h o u l d be con­ sidered . ANS WER 3: D. The e n d - d i asto l i c ve locity of PR re­ fl ects the end-d iasto l i c p ressu re g rad ient between the p u l m o n a ry a rtery and the RV. At e n d d iastole, RV p res­ s u re s h o u l d be eq u a l to RA p ress u re . The refo re, PAE D P

=

4

x

E n d - P Rvel2

+

RAP,

where PAE D P is p u l m o n a ry a rtery e n d - d i asto l i c p res­ s u re, E n d - PRvel is PR e n d - d i asto l i c velocity a n d RAP i s r i g ht atri a l p ress u re . A n o r m a l PAE D P is :s; 1 S m m H g . S e e The Echo Manual, 3rd Edition, F i g u re 9 - 7 o n page 1 47 .

ANS WER 4 : A. T h ro u g h o u t the res p i ratory cyc l e , there is evi d e n ce of consistent systo l i c flow revers a l s i n the h e patic ve i n s, wh ich ten d t o be l a t e pea k i n g i n severe t r i c u s p i d va lve reg u rg itation a n d e a r l y pea k i n g if re lated t o severe R V dysf u n ctio n . T h e typ ical fi n d i n g s o f constriction that a re p resent o n i nterrogation o f the hepatic ve i n s a re d i asto l i c flow reversa ls that i n c rease with exp i rati o n . ANSWER 5: B. T h e typ i c a l fi n d i n g s i n const r i ctive perica rd itis a re n o r m a l o r e l evated myoca rd i a l rel ax­ ation ve locities ofte n with reversa l of the pea k ve loci­ ties between the med i a l and latera l a n n u l uses. I n the normal sett i n g , the latera l m itra l a n n u l us has a n early rel axati o n ve locity of 1 to 5 cm per seco n d h i g he r than that of the med i a l a n n u l us . I n constriction where there the late ra l a n n u l us has some constra i nt a n d a g reater proportion of myoca rd i a l re laxation is d riven t h ro u g h the sept u m , the m ed i a l a n n u l u s te n d s t o be h i g h e r . Alth o u g h the m itra l i nflow p rofi le m a y a p pea r s i m i l a rly restrictive (ea rly pea k i n g , s h o rt dece l e ration t i m es) i n both constrictive a n d restrictive phys i o l og i es, i nteg ra­ tion with the myoca rd i a l rel axat i o n velocities a l l ows easy sepa rati o n of th ese two p h e n o m e n a . H e re the high ratio of early fi l l i n g b lood ve locity (E) to the early myoca rd i a l rel axati o n ve locity (e') i n d icates the p res­ e n ce of e l evated l eft ventri c u l a r fi l l i n g p ressu res as op­ posed to constrictive phys i o l ogy.

CAS E

1 8

Harsh Systo l i c M urm u r

Aevaluate a harsh systolic murmur.

5 5-year-old asymptomatic woman i s referred for transthoracic echocardiography to

On examination, her blood pressure is 1 1 0/60 mm Hg. Heart rate regular at 68 beats per minute. Carotid pulses are reduced in intensity. The apex beat is forceful and nondis­ placed. There is a 3/6 systolic ejection murmur throughout the precordium with a single second heart sound.

QUESTION I. On the basis of the echocardiogram

(Videos 18-1 to 1 8-4 and Figs. 1 8- 1 to 1 8-5) , which of the following is true?

Figure 1 8-3

Figure 1 8-1

Figure 1 8-4

Figure 1 8-2

59

6 0 I E C H O C A R D I O G RA P H Y: A C A S E - B AS E D R E V I EW

3 days offever (temperature, 40°C) , fatigue, and nausea. Renal function is normal. Blood cultures were drawn and grew out Strep tococcus viridans relatively resistant to penicillin G. She is referred for transesophageal echo­ cardiography (Videos 18-12 to 1 8-18) . On the basis of the images, which is the appropriate management?

Figure

1 8-5

A. The echocardiographic findings are discordant from the physical examination B. There is severe aortic valve (AV) stenosis with an aortic valve area (AVA) 25% of cases

QUESTION 7. The correct appropriate next step is:

A. Complete the course of antibiotics, then prob­ able elective AV replacement, based on the re­ sults of a repeat TEE B. Urgent AV replacement without further testing C. Diagnostic coronary angiography followed by urgent AV replacement D. Permanent pacemaker placement followed by urgent AV replacement

C AS E 1 8 I 6 1

ANS WER 1: C. T h e re i s c l i n i c a l a n d ech oca rd i o­ g ra p h i c evi dence of severe AV stenosis. The ca lcu lated AVA (see below) i s 0 . 66 c m 2 • The LVOT/AV TVI ratio (d i mension l ess i n d ex) is 0 . 2 1 . Both of these parameters a re in the severe ra n g e . AVA

= = = = =

[(LVOT TVI) x (LVOT a rea)]/Ao TVI [(LVOT TVI) x (0 . 7 8 5 (LVOT D)2]/Ao TVI [(2 1 ) x (3 . 1 4)]/1 00 66/1 00 0.66 cm2

ANSWER 2: D. The d i a g nosis of seve re AV stenosis is c l e a r, a n d n o f u rther a n atom ical o r h e m odyn a m i c eva l uation i s req u i red . A T E E o f h e m odyn a m i c cath­ eterization s h o u l d o n l y be considered i n the sett i n g if there is d i scord a n ce between the cl i n i ca l and tra nstho­ racic data . The only treatment option of sym ptomatic a o rtic ste nosis is valve re p l a ce m e nt, and this s h o u l d b e co n s i d e red beca use t h e outco m e o f sym pto m atic patients oth e rwise i s poo r. Even i n the sett i n g of truly asym ptomatic AV stenosis, most patients wi l l develop sym pto ms with i n a few yea rs . A tread m i l l exe rcise stress test, to ascerta i n f u n cti o n a l l i m itation o r a n ab­ normal b l ood p ressu re response to exercise, is a usefu l test to g u i d e ma nagement i n the patient with a p p a r­ ent asym ptomatic AV stenosis beca u se m a ny of these pati ents when tested have f u n ctional l i m itation . At o u r i nstitution, w e typ ica l ly perform a n oxyg en consu m p­ tion tread m i l l exercise stress test for this p u rpose . If a patient has severe truly asym ptomatic AV stenosis with an excel lent f u n cti o n a l ca pacity, a strategy of caref u l observation is reaso n a b l e a lthough a repeat tra nstho­ racic echocard i o g ra p h i c eva l uation at least on an a n ­ n u a l basis is req u i red . Here the patient went o n ly 6 5 % of pred icted functional ca pacity a n d was refe rred for va lve replacement. 1 • 2 The patient u n d e rwent el ective AV replacement with a 2 1 -m m b i o p rosthesis. ANSWER 3: A. An i n it i a l postoperative tra n sthoracic echoca rd i o g ra m s h o u l d be performed i n all patients fol l owi n g va lve rep lacement to serve as a n i n d iv i d u a l " f i n g erpri n t " t o esta b l ish a base l i n e f o r futu re assess­ m e n t . Th i s s h o u l d be p e rfo rmed i m m e d i ately before hospita l d i sc h a rge o r with i n the fi rst few months afte r s u rgery . After th at, a n y f u rt h e r ech oca rd i o g r a m i n the n ext 3 yea rs to assess p rosthetic va lve f u n ction i s i n a p p ro p r i ate i n the a bsence of known o r suspected va lve dysfu ncti o n . Rout i n e s u rve i l l a nce after 3 yea rs is reaso n a b l e . 3

ANS WER 4: B. T E E demonstrates a s m a l l m o b i l e le­ sion o n the a nterior AV p rosthetic l eafl et without va l­ vu l a r dysf u n ction . I n it i a l ma nagement i n t h i s sett i n g of Streptococcus p rosthetic va lve endoca rd itis req u i res 6 wee ks of either co m b i nation of a /J- l acta m a n d a m i ­ nog lycoside a nti b iotic or 6 weeks of va ncomyci n . If the Streptococcus was sensitive to pe n i ci l l i n G , 6 weeks of pe n i ci l l i n ± 2 weeks of a n a m i nog lycos i d e wo u l d be a p p rop riate. S u rgery i s n ot i n d icated at this t i m e . 4 ANS WER 5 : C. The development o f a n y deg ree of heart block i n a patient with known or suspected en­ doca rd itis is very concern i n g for the development of a n aortic root a bscess i nvo lvi n g t h e u p per i ntraventricu l a r sept u m , wh ich houses t h e con d u ction syste m . I n t h i s sett i n g , a tra n seso p h a g e a l , rather t h a n tra n sthoracic, echoca rd i o g ra m i s i n d i cated . ANS WER 6: E. Despite the a bsence of sign ifica nt va l­ vu l a r vegetation, the TEE confirms the suspicion of i n­ terim abscess development i n the aortic root exte n d i n g down i nto t h e m itra l-aortic i ntervalvu l a r fibrosa with a ru ptu re of the a n e u rysm of the m itra l-aortic i nterva lvu­ l a r fi b rosa i nto the left atri u m . This is a ma rked differ­ ence to the study from 9 days earlier. Prosthetic aortic reg u rg itation is now a l so present. This is a recog n ized com p l i cation of AV endoca rd itis (native a n d prosthetic) occurring i n 3 0 % to 5 0 % of patients with AV endo­ ca rd itis. 5 · 6 ANSWER 7: B. The patient i s at ve ry h i g h risk a n d s h o u l d be considered for u rgent AV rep l acement. S u r­ gery wi l l include debridement of the interva lvu l a r fibrosa a n d repa i r or rep lacement of the m itra l va lve . C oron a ry a n g iography is contra i n d i cated i n a patient with known or suspected aortic root abscess . Alth o u g h the patient is at very h i g h risk for prog ressive hea rt b l ock, there is no i m mediate i n d ication fo r pacemake r placement a n d t h e determi nation wou l d b e m a d e after s u rgery.

62 I E C H O C A R D I O G RA P H Y: A C A S E - B AS E D R E V I EW

References 1. Pellikka PA, Sarano ME, Nishimura RA, et al. Outcome of 622 adults with asymptomatic, hemodynamically significant aortic ste­ nosis during prolonged follow-up. Circulation. 2005; 1 1 1 :32903295. 2. Rosenhek R, Binder T, Porenta G, er al. Predictors of outcome in severe asymptomatic aortic valve stenosis. N Engl j Med. 2000;343 : 6 1 1-6 1 7. 3. Douglas P, Garcia MJ , Haines DE, er al. ACCF/ASE/AHA/ ASNC/HFSA/HRS/SCAl/SCCM/SCCT/SCMR 20 1 1 appro­ priate use criteria for echocardiography. A report of the American College of Cardiology Foundation Appropriate Use Criteria Task Force, American Society of Echocardiography, American Heart Association, American Society of Nuclear Cardiology, Heart Fail­ ure Society of America, Heart Rhythm Society, Society for Cardio­ vascular Angiography and Interventions, Society of Critical Care Medicine, Society of Cardiovascular Computed Tomography, So­ ciety for Cardiovascular Magnetic Resonance American College of Chest Physicians. J Am Soc Echocardiogr. 20 1 1 ;24:229-267.

4. Baddour LM, Wilsom WR, Bayer AS, er al. AHA Scientific State­ ment. Infective endocardiris: diagnosis, antimicrobial therapy, and management of complications: a statement for healthcare profes­ sionals from the Committee on Rheumatic Fever, Endocardiris, and Kawasaki Disease, Council on Cardiovascular Disease in the Young, and the Councils on Clinical Cardiology, Stroke, and Cardiovascular Surgery and Anesthesia, American Heart Associa­ tion-Executive Summary: Endorsed by the Infectious Diseases Society of America. Circulation. 2005 ; 1 1 1 :3 1 67. 5. Karalis D G , B ansal RC, Hauck AJ , er al. Transesophageal echocardiographic recognition of subaortic complications in aortic valve endocardiris. Clinical and surgical implications. Circulation. 1 9 92;86;3 53-362. 6. Daniel WG, Miigge A, Martin RP, et al. Improvement in the diagnosis of abscesses associated with endocardiris by transesopha­ geal echocardiography. N Engl] Med. 1 9 9 1 ;324:795-800.

CAS E

1 9

Harsh Systo l i c M urm u r i n N u rsery Exa m i nation

A mur. She is feeding well and of normal weight, and her mother had an uncomplicated 1 -day-old newborn undergoes a nursery examination. She has a harsh systolic mur­

40-week pregnancy. She is referred for transthoracic echocardiography (Videos 19-1 to 1 9- 1 3 and Figs. 1 9- 1 to 1 9-4) .

Figure

Figure

1 9- 1

1 9-2a

Figure

1 9-2b

Figure 1 9-3

63

64 I E C H O C A R D I O G RA P H Y: A C A S E - B AS E D R E V I EW

QUESTION 3. There is evidence of concomitant:

A. B. C. D.

Coarctation of the aorta Patent ductus arteriosus Atrial septal defect None of the choices

QUESTION 4. Acute management might include:

Figure

1 9-4

A. B. C. D. E.

Oxygen Prostaglandin infusion Referral for pulmonary balloon valvuloplasty Oxygen and prostaglandin infusion Oxygen, prostaglandin infusion, and referral for pulmonary balloon valvuloplasty

QUESTION 1. What is the defect?

A. B. C. D. E.

Secundum atrial septal defect Tetralogy of Fallot Congenital pulmonary valve stenosis Membranous ventricular septal defect Congenital aortic valve stenosis

QUESTION 2. The maximum instantaneous systolic

gradient across the pulmonary valve is: A. B. C. D.

17 36 64 81

mm mm mm mm

Hg Hg Hg Hg

QUESTION 5. Signs of severe pulmonary valve steno­ sis may include all except:

A. B. C. D. E.

A pan-systolic murmur A widened split 52 Marked right ventricular hypertrophy Cyanosis Ejection click maximal on inspiration

C AS E 1 9 I 6 5

ANS WER 1: C. There is evi d e n ce of a dysplastic p u l ­ m o n a ry va lve with seve re stenos i s . S e e The Echo Manual, 3rd Edition, pages 353 t o 3 5 6 and Figure 20-32 o n page 3 56.

ANS WER 2: D. The pea k ve locity across the p u l mo­ n a ry va lve is 4 . 5 m per seco n d . We can ca l c u l ate the peak i n stanta n eo u s g rad ient as 4(v)2 4(4 . 5)2 81 mm Hg. =

u n d ergo bal loon va lvu loplasty, which h a s a very h i g h success rate a n d a low risk (Video 1 9-1 3) . I n t h e setting of syste m i c right sided p ressu res a n d severe p u l m o n i c stenosis, p u l m o n a ry b l ood flow is be i n g m a i nta i ned i n t h e early neonata l period t h ro u g h t h e d u ctus a rterio­ sus. M a i ntena nce of d u cta l pate n cy as a bridge to de­ fi n itive thera py is critica l and can be obta i ned thro u g h peri pheral i nfusion o f p rosta g l a n d i n s . Oxyg e n shou l d a l s o be g iven t o promote p u l mo n a ry a rteri a l d i latio n .

=

ANS WER 3 : B . Seen o n Video 1 9- 1 0 a n d F i g u re 1 9-3 , there is conti n uous flow seen com i n g from the a o rta towa rd t h e p u l m o n a ry a rtery consistent with flow t h ro u g h the d u ctus a rteriosus. See The Echo Manual, 3rd Edition, page 347, and Fig u re 20-25 on page 3 5 1 .

ANSWER 4: E. The treatment of valvu l a r p u l m o n i c stenosis is stra i g h tfo rwa rd . If t h e ri g ht ventri c u l a r sys­ to l i c p ressu re is > 5 0 to 60 m m H g , patie nts sh o u l d

ANSWER 5: E. Va lvu l a r p u l m o n i c ste n o s i s i s s u g ­ gested b y a p u l monary ej ection c l i c k that is maxi m a l on expi ration, fo l l owed by a n ejection systo l i c m u rm u r and a widely s p l i t secon d heart sou n d . The more seve re the stenosis, the longer the m u r m u r and the wider the sec­ o n d heart so u n d is s p l it . The right ve ntri c u l a r response is hypertroph ic, a lthou g h r i g ht ventri c u l a r fa i l u re may occ u r in critica l stenosis. In so me severe cases of p u l ­ m o n a ry stenosis, t h e e l evated r i g h t atr i a l p ress u re wi l l lead to pate ncy of t h e fora men ova le l ea d i n g t o a sig­ n ificant rig ht- l eft s h u nt a n d cya nosis.

CAS E 2 0

Prog ressive NYHA Class I l l Dyspnea

A class III dyspnea. He denies chest pain, presyncope, or syncope. His medical history is 75-year-old man presents for the evaluation of progressive, New York Heart Association

significant for paroxysmal atrial fibrillation and chronic kidney disease secondary to hyper­ tensive nephrosclerosis, with estimated glomerular filtration rate of 1 5 to 20 cc per minute. On examination, his systemic blood pressure is 1 00/60 mm Hg. He has a diminished pulse volume, a third heart sound, and a 2/6 systolic ejection murmur. His ECG is shown in Figure 20- 1 . He is referred for transthoracic echocardiogram (Video 20-1 and Figs. 20-2 to 20-7) .

Figure 20- 1 Figure 20-3

Figure 20-2 Figure 20-4

66

C AS E 2 0 I 6 7

QUESTION 1. On the basis o f the available data,

calculate the Doppler-derived left ventricular stroke volume (SY) . A. B. C. D.

Figure 20-5

25cc 3 5 cc 50cc 70cc

Q UESTION 2. On the basis of the available data (Figs. 20-2 to 20-7) , calculate the Doppler-derived aortic valve area.

A. B. C. D. E.

0.6 0.7 0.8 0.9 1 .0

cm2 cm2 cm2 cm2 cm2

Q UESTION 3. How would you grade this patient's diastolic function?

A. B. C. D.

Figure 20-6

Normal Grade 1 Grade 2 Grade 3

QUESTION 4. His left ventricular diastolic filling

pressure is: A. B. C. D.

Normal Increased Reduced Not able to be determined

Q UES TION 5. His lateral mitral e' velociry is most

likely:

Figure 20-7

A. B. C. D.

4 cm per second 5 cm per second 6 cm per second 1 5 cm per second

6 8 I E C H O C A R D I O G RA P H Y: A C A S E - B AS E D R E V I EW

QUESTION 6. A reasonable next step may include:

A. Aortic valve replacement after diagnostic coro­ nary angiography B. Contrast cardiac CT to assess aortic valve and left ventricular systolic function C. Cardiac hemodynamic catheterization with do­ butamine infusion D. Transesophageal echocardiography

A. Aortic valve replacement after diagnostic coro­ nary angiography B. Cardiac hemodynamic catheterization with do­ butamine infusion C. Transesophageal echocardiography D . Initiation of medical management of dilated cardiomyopathy with angiotensin-converting inhibitors and /J-blockers

QUESTION 8. Contractile reserve is defined as: Q UES TION 7. The patient was referred for stress

echocardiogram to assess aortic valve hemodynam­ ics and contractile reserve in response to dobutamine (Videos 20-2 and 20-3 and Fig. 20-8) . (In both the videos, image orientation is as follows: top left para­ sternal long axis, top right parasternal short axis, bot­ tom left apical four-chamber view with left ventricle on the left, and bottom right apical two-chamber view with inferior wall on the left and anterior wall on the right.) The next appropriate step is:

Figure 20-8

A. B. C. D.

An increase in SV of > 1 0% with dobutamine An increase in SV of >20% with dobutamine An increase in SV of >30% with dobutamine An increase in SV of >50% with dobutamine

C AS E 20 I 69

ANS WER 1: B. SV is ca l c u l ated as the p rod uct of the cross-sectional a rea of the l eft ventri c u l a r o utfl ow tract (LVOT) a n d the t i m e ve locity i nteg ral (TVI) of a p u lse wave sa m p l e from the LVOT: SV ( m l ) = [(D/2)2 x TT] = [(2/2)2 X TT ] = 34. 5 m l

x

X

[ LVOT TVI ] [1 1 ]

See The Echo Manual, 3rd Edition, F i g u re 4-1 6 o n page 7 1 .

ANS WER 2: B. Aortic va lve a rea = = = =

[(LVOT TVI) x (LVOT a rea)]/Ao TVI [ ( 1 1 ) X (TT ( 1 )2)]/50 34. 5/5 0 0 . 7 cm2

See The Echo Manual, 3rd Edition, d iscussi o n of Doppler echoca rd i o g raphy (in a o rt ic ste nos i s) on pages 1 90 and 1 9 1 .

ANS WER 3: D. M itra l early fi l l i n g pea k velocity is el­ evated at 1 . 4 m per secon d with a d i m i n utive A wave a n d visu a l ly short dece l e ration ti m e . A s i m i l a r pattern ca n be seen in a hea lthy you n g we l l cond itioned i n d i ­ v i d u a ls, it is n ot a normal pattern i n a n e l d e rly patient. Mo reover, med i a l m itra l a n n u l us ve locity of 5 cm/sec shows that the u n derlyi ng myoca rd i u m has a profo u n d d i asto l i c dysfu ncti o n . This descri bes a restrictive fi l l i n g pattern i n kee p i n g with G rade 3 or severe d i asto l i c dys­ function . ANS WER 4: B. Left ve ntri c u l a r fi l l i n g p ressu re ca n be est i m ated by t h e ratio of t h e e a r l y m itra l pea k ve locity (E) to the early d i asto l i c re l axation tissue ve­ locity of the m itra l a n n u l us (e'), with a rati o > 1 5 cor­ re lati n g with an e l evation in l eft ve ntri c u l a r d i asto l i c fi l l i n g p ress u re . H e re the E/e' ratio is 1 . 4/0 . 0 5 that is very h i g h at 2 8 .

i s p roba b l y severe if ( 1 ) the a o rtic valve a rea by the conti n u ity e q u at i o n i s 0 . 7 5 cm2 o r l ess and (2) the LVOT:AoV TVI (or ve locity) is 0 . 2 5 or l ess. I n th is situ­ ati o n , however, there a re two d i a g nostic poss i b i l ities: one, true anatom i ca l ly severe aortic stenosis and, two, f u n cti o n a l ly severe aortic ste n osis, beca use a n aortic va lve with mild or moderate ly severe stenosis may n ot open fu l ly if the SV is l ow. G ra d u a l i nfusion of dobu­ ta m i n e ( u p to 2 0 µg/kg/m i n ute) to i ncrease SV may be h e l pf u l i n d i fferentiati n g m o r p h o l o g i ca l ly severe aortic ste nosis from a decreased effective stenotic ori­ fice a rea ca used by low ca rd i a c output (pse u do-severe aortic stenosis) . This ca n be accom pl ished by ech oca r­ d i o g ra p hy or cath eterizati o n . Dobuta m i n e stress echo­ ca rd iography affords the d ia g n osti c capa b i l ities wh i l e avo i d i n g the potenti a l e m b o l i c risks o f tra n sverses a potenti a l ly seve rely stenotic aortic va lve with a hemo­ dyn a m i c catheter . 1 -3 ANS WER 7: A. I n the setting of low-outp ut, low-gra­ d i e nt aortic stenosis, dobuta m i n e stress echoca rd i og­ ra p h y has two d isti n ct ro les. The fi rst i s with reg a rd to d i sti n g u i s h i n g seve re stenosis from pse udo-severe (fu ncti o n a l ) stenosis of the va lve . The seco n d a ry ro l e is with rega rd to assess i n g for the p resence o r a bsence of i n otro p i c left ventri c u l a r rese rve . H e re g ra d u a l i nfusion of dobuta m i n e led to a p rog res­ sive n o r m a l i zation of LVOT ve locity a n d TVI . At t h i s leve l , there was a para l l e l i ncrease i n tra nsva lvu l a r ve­ locity a n d TVI with a p reservation of LVOT/aortic va lve rati os, t h u s confi rm i n g true severe aortic va lve stenosis (F i g . 2 0-9) . Aortic ste nosis s h o u l d be co n s i d e red se­ vere, wa rra nti n g aortic va lve rep l a cem ent, if the aortic va lve a rea is 1 . 0 cm2 or l ess and the m itra l g ra d i ent 30 m m H g o r more with dobuta m i n e .

ANS WER 5: C. The m itra l l ate ra l a n n u l u s e ' ve locity is typ ical ly 1 0 % to 2 0 % h i g he r t h a n that of the me­ d i a l a n n u l u s . C o n d itions w h i c h p ro d u ce lower latera l e ' than med i a l e ' a re constrictive pericarditis a n d latera l wa l l myoca rd i a l i nfa rcti on . ANS WER 6: C. W h e n LV systo l i c f u n ct i o n i s a b ­ normal a n d ca rd iac output is red uced, aortic ste nosis

Figure 20-9

7 0 I E C H O C A R D I O G RA P H Y: A C A S E - B AS E D R E V I EW

ANSWER 8: B. Beyo n d d ia g nosis, dobuta m i n e p l ays a critica l ro l e in a ssess i n g t h e p resence or a bsence of i n otrop i c reserve, defi ned as a n i n crease i n SV (or LVOT TVI) of >2 0 % with dobuta m i n e . Alth o u g h va lve re p l acement re m a i n s a better option t h a n conserva­ tive t h e ra py, the a bsence of i n otro p i c rese rve with d o b u ta m i n e porte n d s a poor perioperative morta l ity ( 5 0 % vs . 7 % ) if aortic valve rep l acement is atte m pted .

C o n s i d e ration of percuta n e o u s a o rt i c va lve rep l a ce­ ment in this sett i n g i s p r u d e n t . I n patients w i t h aortic va lve a reas < 1 cm2 a n d tricu s p i d reg u rg ita nt velocities > 4 m per seco n d , operative mor­ ta l ity is h i g h , but sti l l better t h a n conservative (nonop­ erative) ma nagement.4-7 See The Echo Manual, 3rd Edition, pages 1 86 and 1 92 t o 1 96.

References 1. deFilippi CR, Willet DL, Brickner ME, et al. Usefulness of do­ butamine echocardiography in distinguishing severe from non­ severe valvular aortic stenosis in patients with depressed left ventricular function and low transvalvular gradients. Am] Cardiol. 1 995;75 : 1 9 1 - 1 94. 2. Nishimura RA, Grantham JA, Connolly HM, Schaff HV, Hi­ gano ST, Holmes DR Jr. Low-output, low-gradient aortic stenosis in patients with depressed left ventricular systolic function: the clinical utility of the dobutamine challenge in the catheterization laboratory. Circulation. 2002; 1 06:809-8 1 3 . 3 . Omran H, Schmidt H, Hackenbroch M, e t al. Silent and appar­ ent cerebral embolism after retrograde catheterisation of the aortic valve in valvular stenosis: a prospective, randomized study. Lancet. 2003;36 1 : 1 24 1 - 1 246. 4. Monin JL, Quere JP, Monchi M, et al. Low-gradient aortic ste­ nosis: operative risk stratification and predictors for long-term outcome: a multicenter study using dobutamine stress hemody­ namics. Circulation. 2003; 1 08:3 1 9-324. Epub 2003 Jun 30.

5 . Schwammenthal E, Vered Z, Moshkowitz Y, et al. Dobutamine echocardiography in patients with aortic stenosis and left ventricu­ lar dysfunction: predicting outcome as a function of management strategy. Chest. 200 1 ; 1 1 9: 1 766- 1 777. 6. Pereira JJ, Lauer MS, Bashir M, et al. Survival after aortic valve replacement for severe aortic stenosis with low transvalvular gra­ dients and severe left ventricular dysfunction. J Am Coll Cardiol. 2002;39: 1 3 56-1 363. 7. Malouf JF, Enriquez-Sarano M, Pellikka PA, et al. Severe pul­ monary hypertension in patients with severe aortic valve stenosis: clinical profile and prognostic implications. J Am Coll Cardiol. 2002;40:789-795 . 8. Hachicha Z , Dumesnil J G , Bogaty P , Pibarot P. Paradoxical low­ flow, low-gradient severe aortic stenosis despite preserved ejection fraction is associated with higher afterload and reduced survival. Circulation. 2007 Jun 5 ; 1 1 5 (22) :2856-64.

CAS E 2 1

System i c Scl erosis

A pressures (Videos 21-1 to 2 1 -6 and Figs. 2 1 - 1 to 2 1 - 1 1 ) .

60-year-old man with a history of systemic sclerosis is referred to estimate pulmonary

Figure 2 1 - 1

Figure 2 1 -3

Figure 2 1 -2

Figure 2 1 -4

71

7 2 I E C H O C A R D I O G RA P H Y: A C A S E - B AS E D R E V I EW

""" ! t.I S rr-.,. PG 0 r-r-.HIJ \,'el 1 9 6 Ul"'o'I 0 �-.--.,..; 9 ..� v.: ts r u..... 0 r....-.K� PG " �..;- , '. (\I•·-·

'

RV TO I

�\

\.;·· 1)I ' !! �

,, .

OG ""

,.,....

; .. y

• '·

l'Vt ·.: ...

'

,

''\

,

..:.

·� \-

I

'

\

·. �



\

''\ ..

. .. I

I " ..

.......,

VT I

40

- •s n .I0

'- c nv• -00 - HJ O - ·

H0



I.



"' "

'

..

"

' '

:'.)

� '

"'"" '

- -E O _,, 0

Figure 21 -1 0

,

.

. .. �o

\. • .._,.

...



.

��.1

I

-

"'" I

..

Later a l TOI

,

I

,.

};�

2 1 cm

.

"Yf 4 .""I. " "'"• ... . ...,..._.. ·.-�. :-r . ..

••

r

·.-..... :....... io '.:"l "'

"

Figure 21 -7

\

"

• ". '.l

rw :-:--.L '· �..,.I.I

"

·v

'IA/

:

i

\

Figure 21 -9

'



. ..

',:� .,1

V m a x 90 c m/s

Figure 21 -6

..

·.

..,

LVOT T OI

-•J O

, ,

"""' 1 ! 7 r r� ' PG 0 1-r- H11 v. : 9 8J 1 rl"'• 'I PG Q ,.._., ,..; ,J v.. : Q Bl!i u... ,. O r....-.H� PG ,.. �-;-, � ( ti,....,

t

, •_ 'lo

>

� ,, � I

. •I D

Med i al TDI .

r

� V · .-

"



I

I'

""1f""I.: '

... .

:·.• �--

..

_.

. ( .

r

-.-..·.i :--r\to �"'

Figure 21 -8

• .m •

¢ ,.-wr-"49 1 &4 un • 0 ,..........J G 4 1 , ...\ • c �r��

' ,. ,,

'

Figure 21 -5

.,.,, : PG \.'•! �G

PVI . ..... \ �ll·1.1

0

.

..

- -• DO

,f

'

,,,......,....

. , ,'J

C AS E 2 1 / 7 3

QUESTION 4. The right index of myocardial perfor­

mance (Tei index) is: A. B. c. D.

0.34 0.4 1 0.52 0.60

QUESTION 5. Which o f the following contrast agents would be useful to identify the structure marked by a circle in Video 21-6 and Figure 2 1 - 1 2?

Figure 21 -1 1 QUESTION 1 . What are the estimated pulmonary ar­ tery systolic and diastolic pressures?

A. B. C. D.

4 1 1 1 7 mm Hg 36/ 1 2 mm Hg 1 3/8 mm Hg 1 7/ 1 2 mm Hg

QUESTION 2. Pulmonary artery capacitance is:

A. B. c. D.

3.6 4.6 4.9 5.5

QUESTION 3. Diastolic function is:

A. B. C. D.

Normal Grade 1 (mildly abnormal) Grade 2 (moderately abnormal) Grade 3 (severely abnormal)

Figure 21 -1 2 A. B. C. D.

Agitated saline Echo contrast Carbonated beverage Iodinated contrast

74 I E C H O C A R D I O G RA P H Y: A C A S E - B AS E D R E V I EW

ANS WER 1: A. P u l m o n a ry a rte ry systol i c press u re is est i m ated as fou r t i m es the pea k (tricuspid reg u rg ita nt velocity)2 p l u s an est i m ate of r i g ht atri a l p ress u re . P u l ­ m o n a ry a rte ry d i asto l i c p ress u re i s esti mated as fo u r ti mes the (en d p u l m o n a ry reg u rg ita nt velocity)2 p l u s a n est i m ate o f rig ht atri a l p ress u re . H e re t h e i nferior ve na cava is of normal size a n d co l l a pses norm a l ly with res­ p i ration, a n d h ence, the r i g ht atri a l p ressu re is n o r m a l (5 m m H g ) . H e re p u l mo n a ry a rtery systo l i c a n d d i asto l i c p ressu res a re est i mated as 4(3)2 + 5 4 1 m m H g a n d 4( 1 . 7)2 + 5 1 7 m m H g , respective ly. =

=

ANS WER 2: B. I n corporati n g the p u lsati l e compo­ nent of right ventri c u l a r afterload, the p u l mo n a ry a rtery ca pacita n ce (PA CAP) as measu red by catheterizati on or ech oca rd i o g raphy1 has been fo u n d to be h i g h ly p re­ d i ctive of outcome in patients with p u l m o n a ry hyper­ tens i o n . It is exp ressed as the ratio of stroke vol u m e (SV) t o p u l mo n a ry a rtery p u lse p ressu re (PA PP) . PA CAP. =

5.3

x

=

SV/PA P P

2 1 /24

=

1 1 1 cc/24 m m H g

=

4 . 6 cc/m m Hgt

ANS WER 3: C. The m itra l i nflow patte rn d e m o n ­ strates a pseudonormal pattern with e a r l y (E) a n d atri a l (A) ve locities o f s i m i l a r mag n it u d e . Med i a l a n d l atera l

Reference 1. Mahapatra S, Nishimura RA, Oh JK, et al. The prognostic value of pulmonary vascular capacitance determined by Doppler echo­ cardiography in patients with pulmonary arterial hypertension. J Am Soc Echocardiogr. 2006; 1 9: 1 045-1 050.

m itra l a n n u la r ea rly ve locities a re d e p ressed consistent with myoca rd i a l rel axation a b normal ities. With a d rop i n preload by Va lsa lva, there was a change i n the mi­ tra l i nflow patte rn to a d e l ayed relaxation (G ra d e 1 ) pattern . ANS WER 4: A. The r i g h t ventricu l a r i n dex of myo­ ca rd i a l performa nce (RI M P) is a relatively load i n depen­ dent p a ra m eter of global fu nction, which i s a ratio of the s u m of the isovo l u m i c rel axation and contraction t i m es to the r i g h t ventri c u l a r ej ection time. It ca n be easily ca lcu lated by the fo l l owi ng eq uatio n : RI M P

=

(Tricuspid va lve clos u re to open i n g t i m e) - (RV ej ection t i me)/(RV ej ection t i m e) 475 - 3 5 5/3 5 5 0 . 34 =

=

ANS WER 5: C. Pa raste r n a l l o n g-axis i m a g i n g dem­ onstrates a l a rge esophageal structu re (Video 2 1 -6 and F i g . 2 1 - 1 2) outside the perica rd i a I space a nte rior to the desce n d i n g thoracic aorta . Ad m i n istration of a ca rbo n ­ ated beve ra ge (Videos 2 1 -6 and 2 1 -7 a n d F i g . 2 1 - 1 2) i d entifies the structu re as p a rt of the gastroi ntest i n a l tract .

CAS E 22

Progressive Dyspnea

A dyspnea.

n 8 5-year-old man presents to the emergency department with 4 days of progressive

He does not report any chest pain, but has cough and shortness of breath. He was treated for pneumonia with an antibiotic that did not help his dyspnea and cough. Exami­ nation showed heart rate of 1 00 beats per minute and blood pressure of 90/50 mm Hg. He has diffuse crackles on his lung fields. There was no audible murmur. Chest x-ray showed pulmonary venous congestions, and ECG showed nonspecific ST-T changes and mild ST segment depression in Vl and V2. Transthoracic echocardio­ gram showed the following (see Videos 22- 1 to 22-9) .

QUESTION I. On the basis of mitral inflow (Fig. 22- 1 ) ,

which o f the following statements i s correct?

Figure 22-2

Figure 22-1 A. Most likely normal for this age B . E velocity of 1 . 3 m per second indicates in­ creased flow across the mitral valve C. Mitra! valve obstruction D. Normal filling pressure

A. Grade 1 B . Grade 2 C. Grade 3

QUESTION 3. From this pulmonary vein Doppler tracing (Fig. 22-3) , we obtained atrial flow reversal ve­ locity duration of 1 08 milliseconds. What is the most likely duration of mitral inflow A wave duration?

QUESTION 2. On the basis of the mitral inflow pat­ tern and the tissue Doppler velocity of medial mitral annulus (Fig. 22-2) , diastolic function is graded as which of the following? 75

7 6 I E C H O C A R D I O G RA P H Y: A C A S E - B AS E D R E V I EW

A. B. C. D.

1 . 1 cm2 and 8 8 cc 0.75 cm2 and 64 cc 0 . 6 cm2 and 57 cc 0.4 cm2 and 45 cc

QUESTION 5. How would you grade the severity of mitral regurgitation?

A. B. C. D.

Figure 22-3 A. 70 milliseconds B. 1 08 milliseconds C. 1 5 0 milliseconds

Q UESTION 4. Calculate effective regurgitant ori­ fice (ERO) and regurgitant volume in this patient (Figs. 22-4 and 22-5) .

Mild Moderate Severe Falsely high because of poor quality proximal isovelocity surface area

QUESTION 6. What is the etiology for this patient's

dyspnea? A. B. C. D.

Papillary muscle rupture Flail mitral valve Mitral valve endocarditis Rheumatic mitral valve stenosis/regurgitation

QUESTION 7. What is your next step for this patient's

management? A. B. C. D.

Figure 22-4

Figure 22-5

Triple antibiotic treatment Surgery after intra-aortic balloon pump Optimize medical therapy Percutaneous valve procedure

C AS E 22 I 7 7

ANS WER 1: B. Pulsed wave Doppler demonstrates a " restrictive " fi l l i n g pattern with a n early d i asto l i c fi l l i n g ( E ) ve locity that is el evated with a shortened dece l e ra­ tion time and a d i m i n utive l ate or atri a l fi l l i n g (A) wave . Th is pattern is seen i n sett i n g s of i n creased tra n sva l ­ v u l a r flow with a s i g n ificant left atrial (LA) t o l eft ven­ tricu l a r (LV) g ra d i ent, typ ica l ly when the LA p ressu re is h i g h a n d a h i g h vo l u m e of b l ood rushes i nto the LV early in d i asto l e . Occasi o n a l ly, t h i s ca n be observed in a yo u n g i n d ivi d u a l in the sett i n g of a n o r m a l LA p ressu re with " s u pe r " n o rm a l re laxati o n of the heart. Th is generates a negative LV p ressu re at early d i asto l e . B l ood i s " su c ked " i n by the l ow L V p ressu re rath e r than " p ushed " i n by the h i g h L A p ressure. ANS WER 2: C. Pu lsed wave tissue Doppler of the m ed i a l m itra l a n n u l u s d e m o n strates a very l ow early d i a sto l i c re laxat i o n t i s s u e (e') ve l o c i ty i n d icati n g a ma rked LV rel axati o n a b n o r m a l ity. C o u p led with the m itra l i nflow pattern, t h i s wou l d be co m pati b l e with a severe degree of d i a sto l i c dysfu nctio n (G ra de 3 ) . ANS WER 3 : A. Beca use LV p ress u re i n creases ve ry ra p i d ly with atri a l contraction, there is a shorte n i n g of atri a l fi l l i n g t i m e (m itra l i nflow A wave d u ration) that is less than p u l m o n a ry ve i n atri a l flow reversa l s . ANS WER 4: A . T h e reg u rg ita nt flow ca n be ca l c u l ated as fol l ows : Flow rate = (r)2 x 6 . 2 8 x a l iasi n g ve locity = ( 1 . 5 cm)2 x 6 . 2 8 x 3 1 cm per seco nd = 438 cc per secon d E R O = Flow rate/Pea k m itra l reg u rg ita nt ve locity E RO = 438 cc per secon d/400 cm per seco n d = 1 . 1 cm2 Reg u rg ita nt vol u m e = E RO x reg u rg ita nt t i m e velocity i nteg ra l = 1 . 1 cm2 x 80 cm = 88 cc

ANS WER 5: C. T h e re i s torre n t i a l m itra l va lve reg u rg itation . ANS WER 6: A. Best seen i n Videos 22-2 , 22-4 a n d 22-8, the l atera l pa p i l l a ry m u scle i s d eta c h e d f ro m the ventricu l a r s u rface a n d freely m o b i l e . Th is leads to ecce ntric seve re poste r i o r l y d i rected m itra l va lve reg u rg itati on . The p res u med mecha n i s m i s a rece nt rel atively asym pto matic myoca rd i a l i nfarction with sec­ o n d a ry pa p i l l a ry m uscle ru ptu re . As is often the case, patie nts a re hypote n sive with p u l mo n a ry edema a n d l ittle if a n y a u d i b l e systol i c m u rm u r. ANS WER 7: B. The treatment of c h o i ce is u rg e n t s u rg i ca l repa i r . Vasod i l ator t h e ra py a n d i ntra-aort i c b a l loon p u m p ca n be h e l pf u l te m porizi n g meas u res; howeve r, morta l ity i s h i g h short of corrective s u rg i ca l p roced u re . There is no ro l e for co nservative m e d i c a l ma nagement or at this t i m e a percuta neous repa i r . An i ntra o p e rative tra n seso p h a g e a l echocard i o g r a m (Videos 22- 1 0 a n d 22- 1 1 ) confi rms a pa p i l l a ry m uscle ru ptu re, and the patient u n derwent successf u l repa i r .

CAS E 2 3

Exertional Shortness of Breath, B i l atera l Lower Extremity Edema, a n d Systo l i c M u rm u r after Coronary Artery G raft i n g

A exenional shortness of breath, bilateral lower extremiry edema, and a systolic murmur.

68-year-old man with a history o f coronary artery grafting 1 0 years previously presents with

QUESTION I. On the basis of his transthoracic images

(Videos 23-1 and 23-2 and Figs. 23- 1 and 23-2) , his cal­ culated mitral effective regurgitant orifice (ERO) area by the proximal isovelocity surface area (PISA) method is:

A. B. C. D.

0 . 3 6 cm2 0.42 cm2 0 . 5 8 cm2 0 . 60 cm2

QUES TION 2. Which of the following provides a simplified method to calculate ERO when aliasing velocity is close to 40 cm per second?

A. Radius2/2 B. Radius/2 C. Radius x 4

QUESTION 3. On the basis of his transthoracic im­ ages (Videos 23-3 and 23-4 and Figs. 23-3 and 234) , his calculated tricuspid regurgitant volume by the PISA method is:

Figure 23-1

Figure 23-3 Figure 23-2 78

C AS E 2 3 I 7 9

Figure 23-4 A. B. c. D. E.

40 45 52 57 63

Figure 23-6 A. B. C. D. E.

cc cc cc cc cc

QUESTION 4. For a similar ERO area, tricuspid re­ gurgitation is associated with a smaller regurgitant volume than mitral valve regurgitation (MR) .

A. True B. False

1 .6 cm2 1 .75 cm2 1 . 9 cm2 2. 1 cm2 2 . 5 cm2

QUESTION 6. The patient returns 3 months later after his physician heard a continuous murmur on examination. On the basis of the transthoracic (Videos 23-6 to 23-9) and transesophageal (Videos 23-10 to 23-13, Fig. 23-7) images, there is an abnormal communication between the right atrium and:

QUESTION 5. The patient proceeds with surgical mitral valve replacement with a 3 1 -mm St. Jude tissue prosthe­ sis and tricuspid valve annuloplasty. On the basis of the predismissal echocardiogram (Video 23-5 and Figs. 23-5 and 23-6) , calculate the effective mitral valve prosthetic orifice area by the continuity method (assume that the left ventricular outflow tract [LVOT] diameter is 22 mm) .

Figure 23-7 A. B. C. D.

Figure 23-5

Right coronary cusp to right ventricle Noncoronary cusp to right atrium Noncoronary cusp to right ventricle Right coronary cusp to right atrium

8 0 I E C H O C A R D I O G RA P H Y: A C A S E - B AS E D R E V I EW

ANS WER 1: B. 0 . 42 cm2

The reg u rg ita nt flow can be ca lcu lated as fo l l ows : Flow rate = (r)2 x 6 . 2 8 x a l i as i n g velocity = (0 . 90 cm)2 x 6 . 2 8 x 43 cm per secon d = 2 1 9 c c p e r secon d ERO = Flow rate/Pea k M R ve locity E RO = 2 1 9 cc per secon d/52 0 cm per seco n d = 0 . 42 cm2 ANS WER 2: A. When a l i a s i n g velocity is 40 cm per seco n d , 6 . 2 8 x 40 = 2 5 1 . Because MR velocity is close to 500 cm per seco n d , E RO is rad i us2/2 . See The Echo Manual, 3rd Edition, PISA method on page 2 1 5. ANS WER 3: C. 5 2 cc

The reg u rg ita nt flow ca n be ca lcu l ated as fo l l ows : Flow rate = = = ERO = E RO = =

(r)2 x 6 . 2 8 x a l iasi n g ve locity (0 . 8 cm)2 x 6 . 2 8 x 3 9 cm per secon d 1 5 7 c c p e r secon d Flow rate/Pea k M R velocity 1 57 cc per secon d/3 00 cm per secon d 0 . 52 cm2

Reg u rg itant vol ume = E RO x reg u rg itant t i m e veloc­ ity i nteg ra l (TVI) = 0 . 52 cm2 x 1 00 cm = 5 2 cc See The Echo Manual, 3rd Edition, PISA method on page 2 1 5. ANS WER 4: A. T r u e . T h e E R O a n d reg u rg i ta n t vo l u m e of t h e t r i c u s p i d va lve a re ca l c u l ated w i t h t h e P I SA meth o d , a s for t h e m i tra l va lve . F o r a s i m i ­ l a r E R O a re a , h oweve r, t r i c u s p i d reg u rg i ta t i o n i s

associated w i t h a smaller reg u rg i ta n t vo l u m e t h a n M R . Pati e nts w i t h systo l i c ve n o u s f l ow reve rsa l (t h e h e p a t i c ve i n f o r t r i c u s p i d re g u rg i ta t i o n a n d t h e p u l ­ m o n a ry ve i n fo r M R) h a d a s m a l l e r re g u rg i ta n t vo l ­ u m e b u t s i m i l a r E RO a rea i n t r i c u s p i d reg u rg i tat i o n t h a n t h o s e w i t h M R . T h e refo re, t h e o p t i m a l d i a g ­ n ost i c t h re s h o l d s f o r severe t r i c u s p i d reg u rg i tat i o n a n d M R a re d i ffe r e n t f o r reg u rg i ta n t vo l u m e (45 a n d 6 0 m l , respect i v e l y) but s i m i l a r f o r an E RO of 40 m m 2 o r m o r e . ANS WER 5: D . T h e p ressu re ha lf-t i m e method over­ est i m ates the a rea of the m itra l va lve p rosthes i s . In the a bsence of s i g n ifica n t a o rtic and M R, the conti n u ity method is a bette r way to determ i n e p rostheti c va lve a reas.

M itra l va lve p rosthesis a rea = LVOT a rea x (LVOT TVI/ M itra l p rosthesis TVI) M itra l va lve p rosthesis a rea = (LVOT 02 x 0 . 785) x (LVOT TVl/M itra l p ros­ thesis TVI) M itra l va lve p rosthesis a rea = (2 . 2 2 x 0 . 785) x (30/5 5) M itra l va lve p rosth esis a rea = 2 . 1 cm2 See The Echo Manual, 3rd Edition, PISA method on page 2 1 5. ANS WER 6: B. Tra n st h o r a c i c i m a g i n g d e m o n ­ strates conti n u o u s f l ow betwee n t h e p roxi m a l a s­ cen d i n g aorta a n d a n a rea either j u st p roxi m a l or j ust d i sta l to the tri c u s p i d va lve . Tra nseso p h a g e a l i m a g i n g is req u i red t o bette r loca l ize t h e d efect a s a ri s i n g i n t h e n o ncoro n a ry cusp a n d ente r i n g t h e r i g ht atri u m (F i g . 2 3 -7) .

T h e pati ent p roceeded t o s u ccessf u l u n co m p l i cated percuta n e o u s device c l os u re g u i d ed by i ntraca rd i a c i m a g i n g (Videos 23- 1 4 t o 23-1 7) .

CAS E 24

Sym ptomatic Paroxysmal Atrial F i b r i l lation

A fibrillation. She has failed treatment trials of propafenone and sotalol. She is considered 60-year-old woman is referred for the management of symptomatic paroxysmal atrial

for a left atrial ablation procedure. She is referred for transesophageal echocardiography (Videos 24-1 to 24-5 and Fig. 24- 1 ) .

Figure 24-2 Figure 24-1 QUESTION 1. On the basis of the pulsed wave Dop­ pler of the right upper pulmonary vein (Fig. 24- 1 ) , the left atrial pressure is likely:

A. B. C. D.

Repeat left atrial ablation procedure Amiodarone antiarrhythmic CT scan of the chest Coronary angiography

A. High B . Normal

QUESTION 2. The patient returns 3 months after ab­ lation. The palpitations have resolved; however, she has persistent fatigue and now exertional shortness of breath and occasionally chest pain. She undergoes transthoracic echocardiogram (Videos 24-6 to 24-8 and Fig. 24-2) . On the basis of the findings, appropri­ ate next step should include:

81

82 I E C H O C A R D I O G RA P H Y: A C A S E - B AS E D R E V I EW

QUESTION 3. The patient undergoes transesopha­

geal echocardiography (Videos 24-9 to 24- 14 and Figs. 24-3 and 24-4) . Doppler findings are consistent with:

Figure 24-4

Figure 24-3

A. Severe mitral valve regurgitation B. Severe elevation in left atrial pressure in the absence of severe mitral regurgitation C. Pulmonary vein stenosis D. Normal Doppler findings E. Coronary to left atrial fistula

C AS E 24 I 8 3

ANS WER 1 : B. Tra n seso p h a g e a l e c h oca rd i o g ra ­ p h y s h ows n o r m a l l eft ve n t r i c u l a r systo l i c f u n ct i o n (Video 24- 1 ) , a n o r m a l l eft atri a l a p p e n d a g e w i t h ­ out t h ro m b u s (V i d eos 24-2 a n d 24-3) , a n d n o r m a l l eft (Video 24-4) a n d r i g h t (Video 24-5) p u l m o n a ry ve i n s . N o r m a l p u l m o n a ry ve n o u s flow co m p ri ses two forwa rd (positive) waves (one systol i c [wh ich may h ave two co m po n e nts] a n d o n e d i a sto l ic) a n d o n e retro­ g ra d e ( n e g a t ive) wave with atri a l contract i l ity. T h e n o r m a l patte r n i s for t h e systo l i c wave t o be g reater t h a n the d i asto l i c wave . In the sett i n g of h i g h l eft atria l pressure, there is d i m i n ition of the systol i c fl ow ve­ l o c i ty with an e l evat i o n in the d i a sto l i c-fi l l i n g wave . ANS WER 2: C. I n pati ents who have u n dergone left atri a l a b lation p roce d u res, o n e com p l i cation to con­ sider is p u l m o n a ry ve i n ste nosis, which may occ u r in 1 % to 3% of cases. The average onset of sym ptoms is with i n 2 to 6 months after the a b l ation p roced u re . Typ ical sym ptoms o f p u l m o n a ry ve i n ste n osis i n c l u d e dyspnea, co u g h , chest pa i n , a n d fati g u e .

H e re tra n st h o ra c i c echoca rd i o g r a p h y d e m o n strates a n o r m a l l eft atri a l size. P u l se-wave i nterrog ati o n of the r i g h t u p p e r p u l mo n a ry ve i n d e m o n strates h i g h e r

Reference 1. Packer DL, Keelan P, Munger TM, et al. Clinical presenta­ tion, investigation, and management of pulmonary vein ste­ nosis complicating ablation for atrial fibrillation. Circulation. 2005 ; 1 1 1 : 546-5 54.

t h a n expected fo rwa rd flow ve locities. T h e rem a i n i n g p u l m o n a ry ve i n s ca n be d i ffi c u lt t o v i s u a l ize with a tra nsthoracic exa m i nation . P u l m o n a ry vei n stenosis s h o u l d be con s i d e red i n every pati ent with n ew p u l m o n a ry sym pto ms afte r a l eft atri a l a b l ation p roced u re . Alth o u g h T E E may be d i a g ­ nostic, the reco m m ended stu d ies t o assess the p u l mo­ n a ry ve i n s a re CT or M R I . ANS WER 3 : C. A s s u g g ested b y t h e s u rface echo­ ca rd iogram and confi rmed on the TEE, the flow ve loci­ ties com i n g t h ro u g h the right u p pe r p u l mo n a ry ve i n (Fig . 24-4) a re h i g h e r t h a n expected with early systol i c velocity o f 1 . 5 m p e r seco nd a n d a m e a n g rad ient of 9 mm H g . Th i s is stro n g ly suggestive of p u l m o n a ry ve i n ste n o s i s . I n contrast, i nterrog ation o f t h e l eft u p pe r p u l m o n a ry vei n (Fig . 24-3) i l l ustrates a normal p rofi l e w i t h e a r l y systo l i c velocity o f 0 . 6 m per seco n d .

I n a d d ition to CT sca n n i n g of the p u l m o n a ry ve i ns, q u a ntitative rad i o n u c l i d e ve nti l ation p e rfus i o n sca n­ ning of the l u n gs is h e l pfu l . Percuta neous b a l loon d i l a­ tion and stenti n g p rovi d es i m med i ate sym ptom re l i ef, althou g h recu rre n ce a n d the need of repeat i nterven­ tion is co m m o n . 1

CAS E 2 5

Heart Fa i l u re

A tracings were obtained.

60-year-old man presents with heart failure. Echocardiography images and Doppler

QUESTION 1. On the basis of the 2D echocardiog­ raphy clips (Videos 25- 1 to 25-3) , which of the fol­ lowing is the most likely etiology for patient's heart failure?

A. B. C. D.

Systolic heart failure Diastolic heart failure Valvular heart disease Rhythm related heart failure

QUESTION 2. On the basis of Figures 25- 1 to 25-6 (mitral inflow and pulmonary vein Doppler tracings) , which of the following statements is correct? • ,,

-,

1

ID "' 1: 0.,:

f- �

-

·"':.r

'

. ,. .

lJ L. . . ..,, ,.

t

, , I� f4



,, 1

,



, •

.l , \

.\

.

"'"

..,, ,

l

1' �

.

Figure 25-1

84

..

,, .,



., .

rw � ': "· �'11 · ..: '•' ...' ·�· � · �---



-!t> "

I

/i

' .{� ' • . ., I ·w .

/�"-'

Figure 25-2

I - , ,. - ! PO

· nn - 110

- ,. - )0

- c� &

. ·10

Figure 25-3

C AS E 2 5 I 8 5

A. Left ventricular (LV) filling pressure is definitely elevated B . Diastolic filling pressure cannot estimated be­ cause of atrial fibrillation C. Mitral inflow deceleration time (DT) is still prognostic even in atrial fibrillation D. E/e' does not work at all in atrial fibrillation for the estimation of filling pressure

QUESTION 3. The M-mode of LV (Fig. 25-7) indi­ cates which of the following?

Figure 25-4

Figure 25-7 Figure 25-5

A. B. C. D.

Atrial fibrillation Translation Interventricular dependence Conduction delay

QUESTION 4. What would you expect to see in he­ patic vein if the patient has constrictive pericarditis (Figs. 25-8 to 25- 1 0) ?

Figure 25-6

8 6 I E C H O C A R D I O G RA P H Y: A C A S E - B AS E D R E V I EW

A. B. C. D.

Diastolic flow reversals with expiration Diastolic flow reversals with inspiration Systolic flow reversals with expiration Diastolic flow reversals with inspiration

QUESTION 5. Which of the following statements is usually correct regarding mitral annulus velocity in patients with constriction?

Figure 25-8

A. Medial, but not lateral, mitral annulus early diastolic velocity is reduced ( 1 5, a lthough e' is borderl i n e at 7 cm per second and the E/e' of 1 1 falls in the i n determi nate gray zone. Of note, the left atri u m wh ile enlarged is only m i ldly so. Although the picture is not enti rely norm a l , there a re insufficient data t o classify the d i asto l i c dys­ function as moderately a b normal a n d therefore a l i kely expla nation of the patient's shortness of breath . ANS WER 4: C. I n add ition to a decreased e' a n d a n

3 . A pseudonorm a l pattern is demonstrated b y show­ i n g a shorte n i n g of m itra l A d u ratio n i n the ab­ sence of a short PR i nterva l or by demonstrati n g prolonged p u l monary vei n ' a ' d u ration exceed i n g m itra l A d u ration . 4 . Color M-mode of m itra l i nflow ca n determ i n e the rate of flow propagation i n the LV. With worse n i n g d i asto l i c f u n ction, myoca rd i a l relaxation is always i m pa i red a n d flow propagation is slow, even when left atria l pressu re and m itra l E velocity a re i ncreased . See The Echo Manual, 3rd Edition, pages 1 34 to 1 35 and Figure 8-8 on page 1 25. ANS WER 5: A. I n this case, repeat measu re of data

i mmediately after exercise demonstrated a clear change from preexercise fi n d i ngs. The normal fi n d i ngs after exercise a re a modest increase i n m itra l i nflow velocities associated with a modest i n crease in early myocard ial relaxation (e'). These are re lated to i ncreased flow i nto the LV seen with the i ncreased ca rd iac output of exercise and an increased rate of relaxation seen in the normal myocard i u m . This proportional rise i n E and e' results i n a si m i lar or someti mes lower E/e' ratio. Assessment of E/e' d u ring or i m mediately after exercise has a lso been dem­ onstrated to correlate with an elevation i n left ventricular fi l l i n g pressu res. Here the m itra l i nflow pattern becomes " restrictive " with an E/A ratio of 2 . 5 and a deceleration time of 1 30 m i l l iseconds. The e' does not change, so the E/e' ratio i ncreases from 1 1 to 2 1 . There is a lso evidence of increasing pulmonary hypertension with an estimated right ventricular systolic pressu re more than 1 00 mm Hg, a finding that i ncreases the specificity for the left-sided echo Doppler data . The hemodyn a m i cs of preserved ejection fraction heart fa i l u re was confi rmed at ca rdiac catheterization (Fig. 28-8).2-6 See The Echo Manual, 3rd Edition, discussion of esti­ mation of fi l l i ng pressu res at rest and with exercise on page 1 38, and Ta bles 8-4 and 8-5 on page 1 39.

E/e' ratio > 1 5, t h e fol l owi ng factors h e l p d i sti n g u ish a pseudonormal from a true normal pattern : 1 . M i d-diasto l i c flow beca use of ma rked i m p a i rment of myoca rd i a l relaxatio n . 2 . A decrease i n preload, b y havi n g the patient sit, perform the Va lsalva m a n euver, or ta ke s u b l i n g u a l n itrog lyce r i n , m a y be a b l e t o u n mask the u n der­ lyi n g i m pa i red re laxation of the left ventricle (LV), ca u s i n g the E/A ratio to decrease by 0 . 5 or more a n d reversa l of the E/A ratio. I n normal people, both the E a n d A velocities decrease more proportiona l ly with a decrease i n fi l l i n g .

Figure 28-8

9 8 I E C H O C A R D I O G RA P H Y: A C A S E - B AS E D R E V I EW

References 1. Bergeron S, Ommen SR, Bailey KR, et al. Exercise echocardio­ graphic findings and outcome of patients referred for evaluation of dyspnea. ] Am Coll Cardiol. 2004;43:2242-2246. 2. Ha JW, Lulic F, Bailey K, et al. Effects of treadmill exercise on mirral inflow and annular velocities in healthy adults. Am j Car­ diol. 2003;9 1 : 1 1 4- 1 1 5 . 3 . H a JW, O h JK, Pellikka PA, e t al. Diastolic stress echocardiogra­ phy: a novel noninvasive diagnostic test for diastolic dysfunction using supine bicycle exercise Doppler echocardiography. jAm Soc Echocardiogr. 2005 ; 1 8 :63-68. 4. Burgess MI, Jenkins C, Sharman JE, et al. Diastolic stress echo­ cardiography: hemodynamic validation and clinical significance

of estimation of ventricular filling pressure with exercise. J Am Coll Cardiol. 2006;47: 1 89 1- 1 900.

5 . Talreja DR, Nishimura RA, Oh JK. Estimation of left ventricu­ lar filling pressure with exercise by Doppler echocardiography in patients with normal systolic function: a simultaneous echocar­ diographic-cardiac catheterization study. j Am Soc Echocardiogr. 2007;20:477-479. 6. Holland DJ, Prasad SB, Marwick TH. Prognostic implications of left ventricular filling pressure with exercise. Circ Cardiovasc Imaging. 20 1 0;3(2) : 1 49- 1 56.

CAS E 2 9

l

Acute Pleuritic Ch est Pa i n a n d Lightheadedness

Asystolic blood pressure varies between 90 and 1 00 mm Hg. He undergoes a CT scan 65-year-old man presents with acute pleuritic chest pain and lightheadedness. His

of the chest with contrast (Figs. 29- 1 and 29-2) . On the basis of the findings, he undergoes transthoracic echocardiography (Videos 29- 1 to 29-8 and Figs. 29-3 to 29-5) .

Figure 29-3

Figure 29-1

Figure 29-4

99

Figure 29-2

1 0 0 I E C H O C A R D I O G RA P H Y: A C A S E - B A S E D R E V I E W

QUESTION 4. With regard to the use of echocardiog­

raphy in a patient with suspected or confirmed pul­ monary embolism, which of the following statements is true?

Figure 29-5

QUESTION 1. The RIMP (Tei index) is:

A. B. c. D.

0.22 0.28 0 . 34 0.56

QUESTION 2. Which o f the following i s true with regard to right ventricular (RV) function?

A. The RIMP (Tei index) likely underestimates the severity of the RV dysfunction because of the acute high afterload B. The RIMP (Tei index) likely underestimates the severity of the RV dysfunction because of the acute high preload C. The RIMP (Tei index) and lateral tricuspid an­ nulus tissue Doppler systolic velocity are con­ cordant D. The lateral tricuspid tissue Doppler systolic ve­ locity likely overestimates the RV dysfunction

QUESTION 3. What do the right atrial mobile masses

represent? A. B. C. D.

Renal tumor Venous cast Thrombi in situ Atrial myxoma

A. Transthoracic echocardiography cannot stratify the risk of shock and death in patients present­ ing with pulmonary embolism who are normo­ tensive B. Transthoracic echocardiography will demon­ strate the evidence of proximal pulmonary ar­ terial thrombus or thrombus-in-transit in 20% to 25% of patients presenting with pulmonary embolism C. Routine transthoracic echocardiography per­ formed 6 weeks after acute pulmonary embolus should be considered to identify patients at risk for chronic thromboembolic pulmonary hyper­ tension D. Peak tricuspid regurgitant systolic velocity cor­ relates with the severity of the pulmonary em­ bolism

QUESTION 5. With regard to the use of thrombolytic

therapy in a patient with confirmed pulmonary embo­ lism, which of the following statements is true? A. Fibrinolytic therapy is indicated in all patients with acute pulmonary embolism B. Fibrinolytic therapy is indicated in patients with evidence of shock in the setting of acute pulmonary embolism C. Fibrinolytic therapy is contraindicated in pa­ tients with acute pulmonary embolism and evidence of thrombus-in-transit on echocar­ diography

CASE 29 I 1 0 1

ANS WER 1: B. The myoca rd ial performance index of

the RV is a g lobal estimate of RV systol i c a n d d iasto l i c function a n d is defi ned as t h e ratio o f isovo l u m i c t i m e (nonejection work) d ivided b y t h e ejection t i m e (ejection work) (F i g . 29-6).

Figure 29-6 Repri nted from Tei C, D ujard i n KS, H o d g e DO, et a l . D o p p l e r echocard i o ­ g r a p h i c i n dex f o r assess ment of g l o b a l r igh t ventri c u l a r f u n ct i o n . J Am Soc Echocardiogr 1 996:9;838-847, with permission from Elsevier.

It is ca lculated as the (tricuspid va lve closure to open­ ing time) - (RV ejection time)/(RV ejection time), with a n u pper reference l i m it be i n g 0.40. Here RI M P

=

(4 1 0 - 32 0)/32 0

=

0.28

ANS WER 2 : B . The RI M P (Tei i n d ex), feasible i n most

patients, is reproducib le, relatively i ndependent of after­ load a n d heart rates a n d avoids the geometric ass u m p­ tions of some other g lobal measu res of RV function . H owever, it has been demonstrated to be u n re l i a b l e w h e n t h e ri g ht atria l pressu re is h i g h partic u l a rly i n the acute setti n g . This leads to a more ra pid equal ization of right atrial and RV pressu res, shorte n i n g the isovo l u m i c relaxation time a n d resulting i n an inappropriately sma l l R I M P . T h e latera l tricusp id systol i c velocity b y pu lsed tis­ sue Doppler is a reprod ucible measu re of basa l longi­ tud i n a l systol ic fu nction with a level < 1 0 to 1 2 cm per secon d i n d i cati n g abnormal contractil ity. 1 •2 ANS WER 3: B. Seen i n the right atri u m is t h ro m b i ­

i n -transit. These t h ro m b i a re h i g h ly m o b i l e a n d h ave the a p pearance of a s n a ke or popcorn . The m o b i l e m ass comes from en b l o c e m bo l ization o f ve n o u s th rom b i cast. ANSWER 4: C. Although there has been a n assort­

ment of criteria used that is a large body of data that

demonstrates echoca rd iography identifies patients at i n ­ creased risk for shock and death i n t h e setti ng o f acute p u l monary embolism a n d is partic u l a rly helpful in pa­ tients who are normotensive. Prospectively performed echoca rd iogra p hy stu d i es in 209 consecutive patients who had acute p u l monary embolism demonstrated that 3 1 % of normotensive patients did have RV dysfunction (RV d i l atatio n , pa radoxical septa l motio n , or tricuspid reg u rg itation velocity >2 . 8 m per second) and 1 0 % of them developed shock a n d 5 % d ied in hospita l . 3 Occas i o n a l ly, t h ro m b i - i n -tra nsit a re d etected i n t h e cham bers on t h e rig ht side o f the heart. I n the i nter­ nati o n a l cooperative p u l m o n a ry e m b o l i s m reg i stry, this fi n d i n g was observed in 42 (4%) of 1 , 1 3 5 patients with p u l monary e m bolism who had echoca rd iography stu d ies. These th rombi a re h i g h ly mobile a n d have the a p peara n ce of popcorn or a snake. The m o b i l e mass comes from en b l oc embol ization of venous t h rom b i cast. T h e y i e l d o f visua l izing proxi mal p u l monary a rterial throm bus, although occasiona l ly seen on tra nsthoracic, is h i g her with tra nsesophageal echoca rd iography. C h ro n i c thromboe m b o l i c p u l mo n a ry hypertension i s a syn d rome of p u l m o n a ry hyperte n s i o n a n d right heart fa i l u re that occu rs after o n e or more e p i sodes of p u l ­ m o n a ry e m b o l i fa i l t o resorb a n d tri g gers a c h ro n i c d i sta l p u l m o n a ry vasc u l a r remode l i n g p rocess . A l ­ t h o u g h m o re t h a n h a l f o f patients p resenti n g with c h ro n i c t h ro m boem bol ic d i sease were not previously awa re of p u l m o n a ry e m bo l i , scree n i n g echoca rd iog­ ra p h y a p p roxi m ately 6 weeks afte r a c l i n i c a l event of p u l m o n a ry e m b o l i c eve nt wi l l i d e ntify th ose pa­ tients who a re at h i g h risk for persistent p u l m o n a ry hyperte n si o n . I n the acute setti n g , the RV is u n a b l e t o g e n e rate a R V systol i c pressu re m uch h i g h e r t h a n 50 t o 60 m m H g . H e n ce, ma rkers o f severity i n acute p u l m o n a ry e m b o l ism i n c l u d e d e g ree of RV d i l a t i o n or dysf u n ction, ma rkers of e levated ri g ht atria l p res­ s u re, a n d i n d i cators of poor l eft ventri c u l a r fi l l i n g . H owever, est i m ates o f p u l m o n a ry a rtery p ress u re, a l ­ thou g h often mostly a b n ormal overa l l , refl ect d i sease severity less wel l .3-6 ANS WER 5: B. F i b r i n o lysis, i n a d d ition to system i c

anticoa g u lation with hepa r i n -based thera py, i s i n d i ­ cated i n patie nts w i t h massive p u l m o n a ry e m bo l i s m (i . e . , associated with s hock) or submassive ( i . e . , cl i n ica l evidence o f adverse prog nosis, for exa m p l e, severe RV dysfunction on ech ocard iogra p hy, new hemodyn a m i c i n sta b i l ity, o r maj o r myoca rd i a l n ecrosis) . F i b r i n oly­ sis is not recom mended for patients with p u l monary

1 0 2 I E C H O C A R D I O G RA P H Y: A C A S E - B A S E D R E V I E W

e m b o l u s that is of low risk. I n patients with a l a rg e b u rd e n o f th rom b u s- i n -tra nsit l i ke i n t h i s case, con­ s i d e ration s h o u l d be g iven to thera py in a d d ition to h e p a r i n -based a nt i coa g u l a t i o n a l o n e-s pecifi ca l ly t h r o m b o lyti c t h e ra py o r percuta n e o u s o r s u r g i c a l

e m bol ectomy. Decisions s h o u l d be based on patient characteristics i n c l u d i n g bleed i n g risk a n d expert ava i l ­ a b i l ity. O u r patient p roceeded w i t h successf u l emer­ gent s u rg ica l e m bolectomy a n d d i d wel l .

References 1. Tei C, Duj ardin KS , Hodge DO, er al. Doppler echocardio­ graphic index for assessment of global righr ventricular funcrion. j Am Soc Echocardiogr. 1 996:9;838-847. 2 . Rudski LG, Lai WW, Afilalo J , er al. Guidelines for rhe echo­ cardiographic assessment of rhe righr hearr in adulrs: a reporr from rhe American Sociery of Echocardiography: endorsed by rhe European Associarion of Echocardiography, a registered branch of rhe European Sociery of Cardiology, and rhe Ca­ nadian Sociery of Echocardiography. J Am Soc Echocardiogr. 20 1 0:23;685-7 1 3 . 3 . Grifoni S , Olivorro I , Cecchini P , e r al. Shorr-rerm clinical our­ come of parients wirh acure pulmonary embolism, normal blood pressure, and echocardiographic righr ventricular dysfuncrion. Circulation. 2000; 1 0 1 :28 1 7-2822.

4 . Jaff MR, McMurrry MS, Archer SL, er al. Management o f mas­ sive and submassive pulmonary embolism, iliofemoral deep vein rhrombosis and chronic rhromboembolic pulmonary hyperren­ sion. Circulation. 20 1 1 ; 1 23: 1 788- 1 830. 5 . Sanchez 0 , Trinquarr L, Colomber I, er al. Prognostic value of righr ventricular dysfunction in patients wirh haemodynamically srable pulmonary embolism: a sysremaric review. Eur Heart ]. 2008;29: 1 569-1 577. 6. Pen go V, Lensing AW, Prins MH, er al. Incidence of chronic rhromboembolic pulmonary hyperrension afrer pulmonary em­ bolism. N Engl] Med. 2004;350:2257-2264.

CAS E 3 0

Acute Severe Retrosternal Chest D iscomfort

A chest discomfort. She no longer has chest pain; however, she feels weak. Her blood n 80-year-old woman presents 24 hours after an episode of acute severe retrosternal

pressure is 90/60 mm Hg. Her ECG is shown in Figure 30- 1 . She undergoes an emergent transthoracic echocardiogram (see Videos 30-1 to 30-8 and Figs. 30-2 to 30-4) .

Figure 30-1

Figure 30-3

Figure 30-2

Figure 30-4

1 03

1 0 4 I E C H O C A R D I O G RA P H Y: A C A S E - B A S E D R E V I E W

QUESTION 1. The calculated cardiac index is (body surface area 1 . 6 cm 2) : =

A. B. c. D.

1.8 2.2 2.5 2.9

QUESTION 2. On the basis of the echocardiographic findings, the appropriate management steps include:

A. Heparin intravenous anticoagulation and CT chest scan B. Intravenous fluid and inotrope administration C. Urgent surgical consultation D . Intravenous diuresis and intra-aortic balloon pump E. Intravenous nitroglycerin and intra-aortic bal­ loon pump

QUESTION 3. The expected tricuspid annular systolic plane excursion (TAPSE) in this case would be:

A. B. C. D.

4 mm 1 4 mm 24 mm 34 mm

QUESTION 4. The next day the patient became pro­

gressively more short of breath with low oxygen satu­ rations. Oxygen saturations remained 8 5 % despite high-flow supplemental oxygen. An emergent echocar­ diogram is ordered. Which of the following components of the echocardiogram is most likely to be diagnostic? A. 2D and color Doppler assessment of the mitral valve B . 2D and color Doppler assessment of the inter­ ventricular septum C. 2D assessment of the pericardium D. 2D regional wall motion assessment of the left ventricle E. Agitated saline contrast study

QUESTION 5. On reviewing

Video 30-9, the likely

pathology is: A. B. C. D.

Normal echo contrast study Evidence of an intracardiac left-right shunt Evidence of an intracardiac right-left shunt Evidence of an intrapulmonary shunt

CASE 30 I 1 05

ANS WER 1: A. Stroke vol u m e (SV) is ca lculated from

the left ventri c u l a r outflow tract (LVOT) as a prod u ct of the LVOT a rea ([LVOT d i a m eter/2]2 x 3 . 1 4) a n d the LVOT time velocity i nteg ra l . Here SV 3 . 1 4 x (212 cm)2 x 1 3 c m 4 1 m l a t a heart rate of 7 0 beats per m i n ute ca rd i a c output of 2 . 9 L per m i n ute a n d a ca rd iac i n dex of 1 . 8 Um i n/m2 =

the absol ute dista nce of excursion of the latera l tricus­ pid a n n u l us toward the a pex d u ri n g systole. A normal va l u e is bei n g �2 0 mm. A red uction i n TAPSE (< 1 5 mm) has been shown to strongly associate with a high risk of adverse events fol l owi ng acute myocard ial i nfa rction . 1

=

=

See The Echo Manual, 3rd Edition, F i g u re 4- 1 6 on pages 7 1 and 72. ANS WER 2: B. There is evi dence of right ventri c u l a r

e n l a rgement a n d dysf u n ction i n the sett i n g o f i nferior wa l l hypo k i n e s i s . These fi n d i n g s a re con s i stent with i nfa rction in the d i stri b u t i o n of the r i g h t coro n a ry a rtery. Hypotension is m e d i ated th rou g h poor rig ht­ sided ca rd iac output a n d d ecreased l eft ventric u l a r fi l l ­ i n g . M a nagement s h o u l d i nvolve avo i d a nce o f age nts that d rop left ventri c u l a r preload f u rther (e . g . , d i u ret­ ics a n d n itrog lycerin); i n stead , the use of i ntravenous fl u i d s and i n otropes is i n d i cated . H e p a r i n and ch est CT wou l d be the a p p ropriate l i n e of m a nagement if an acute p u l mo n a ry e m b o l u s was the co nsideratio n , w h i c h is n o t the case here. ANS WER 3: A. An easy to obta i n meas u re that re­

flects right ventric u l a r longitu d i n a l systol i c fu nction is

Reference 1. Antoni ML, Scherptong RW, Atary JZ, et al. Prognostic value of right ventricular function in patients after acute myocardial in­ farction treated with primary percutaneous coronary intervention. Circ Cardiovasc Imaging. 20 1 0;3:264-27 1 .

ANS WER 4: E. Ech oca rd iogra phy i s a key test i n the

eva l uation of a new com p l i cation fol l owi ng myoca rd i a l i nfa rction . T h e presence o f hypoxe m i a that persists de­ spite oxygen s u p p l e mentation is h i g h ly suggestive of shunt p hysiology. An ag itated sal i ne study to assess for rig ht-left s h u nt is needed . ANS WER 5: C. O n a g i tated sa l i n e a d m i n istration

and fi l l i ng of the right atri u m , there is a l m ost i m med i­ ate presence of contrast bu bbles i n the left-sided cham­ bers, i n itia l ly the left atri u m a n d the left ventricle. This is diag nostic of a rig ht to left s h u nt at the atria l leve l . The degree of s h u nt i n g and hence peri pheral hypoxe m i a is exacerbated b y the h i g h right atrial pressu res seen in the setting of acute right ventricular i nfa rction that open u p a patent fora men ova l e . This cond ition was d ifficult to manage before the a p p l ication of a percuta­ neous closure devise.

CAS E

Lightheaded ness a n d Severe Fat i g u e

Mpresents to the emergency department after the onset of lightheadedness and severe r. VD is a 46-year-old man with a history of diabetes mellitus and hypertension who

fatigue. Three days before, he had spent the day in bed with a bad case of indigestion. On examination, he has a systolic blood pressure of75 mm Hg and an audible systolic murmur. He undergoes a bedside transthoracic echocardiogram (see Video 31-1 , Fig. 3 1 - 1) .

QUESTION 3. The findings on coronary angiography

are likely: A. Occluded left circumflex coronary artery B. Occluded left anterior descending coronary artery (LAD) C. Occluded right coronary artery D . High-grade obstructive three-vessel coronary disease

QUESTION 4. The right ventricular (RV) systolic

Figure 3 1 - 1

QUES TION 1. Which o f the following mechanisms

likely underlies the findings? A. Anterolateral papillary muscle rupture B. Postmyocardial infarction rupture of the ven­ tricular septum C. Left ventricular (LV) outflow tract obstruction D. Free wall rupture

QUESTION 2. Diagnostic findings on invasive cath­

eterization include: A. Oxygen saturation samples B. Pulmonary capillary wedge tracing C. LV end-diastolic pressure change following a premature ventricular contraction D. Thermodilution cardiac output measurements

1 06

pressure is: A. B. C. D.

50 25 25 60

mm mm mm mm

Hg Hg Hg plus right atrial pressure Hg

QUESTION 5. The appropriate management is:

A. Intra-aortic balloon pump and surgical consul­ tation B. Intra-aortic balloon pump and percutaneous coronary intervention C. Percutaneous coronary intervention and percu­ taneous defect closure

CASE 3 1 I 1 07

ANSWER I: B. Ventricular septa I ruptu re occu rs i n 1 %

to 3 % of patients after myocard i a l i nfa rction, a n d it oc­ curs d u ri n g the early phase of acute i nfa rction (with i n t h e fi rst week) . A s i n free wa l l ru ptu re, ventricular septa I ru ptu re is more com mon i n elderly women who have not had a previous myoca rd i a l i nfa rcti o n . The typical c l i n ical presentation is a n ew systol i c m u rm u r, with abrupt a n d prog ressive hemodyn a m i c deterioratio n . The d ifferential diagnosis of a new systolic murmur i n patients with acute myocard ial i nfarction i ncl udes infarct-related ventricular septal ruptu re, papillary muscle dysfunction or rupture, pericardia! rub, acute LV outflow tract obstruction, and free wall rupture. After physical ex­ amination, echocardiography is the next logical noninvasive diagnostic procedure for all patients with a new murmur, especially for those who are hemodynamica l ly unstable. Infarct-related ventricular septal defect is diagnosed by the demonstration of a disru pted ventricular septu m with a left-to-right shunt, as in this case. It is interesting that the patient's main symptom was fatigue and light-headedness, but no shortness of breath which is a predominant symp­ tom of severe acute mitral reg urg itation due to papillary muscle rupture. Another important condition to keep in mind for patients with a new systolic murmur after acute myocardial infarction is acute LVOT obstruction. The defect is located in the reg ion of th i n ned myoca r­ d i u m with dyskinetic motion . Here echoca rd iogra p hy demonstrates a m i ldly e n l a rged LV with depressed sys­ tol i c fu ncti on, a n d LV ejection fraction is 3 0 % to 3 5 % . Th ere i s a nterior a n d a p i ca l a k i nesis. The LV a ntero­ septu m is dyskinetic, a n d there is a serp i g i nous defect thro u g h the basa l portion with l eft-to-right s h u nti n g .

artery). The prevalence is higher in the setting of fi rst myo­ cardial infarction . Septal rupture may occur in the setting of acute infarction with a large wrap around left anterior descending artery or an acute infarction involving the right coronary artery. When the rupture is in the inferoseptum, the myocardial infarction usual ly involves the RV, which portends a poor prognosis. An inferoseptal ventricular sep­ tal rupture can be a serpiginous septal tear, and an antero­ apical septal ventricular septal rupture may evolve into an LV free wall rupture. Seen here the extensive wall motion abnormalities i nvolving the septum, anterior wall, and apex point toward the LAD as being the infarct-related artery. See The Echo Manual, 3rd Edition, page 1 62. ANSWER 4: A. The pea k velocity from the conti n uous

wave Doppler i nterrogation across the ventricular septal defect corresponds to the peak g rad ient between the LV and RV. Seen here the conti n uous wave Doppler trac­ ing i n d i cates a conti n uous shunt th rough the ventricular septa l defect except d u ri n g early d iastole. The pea k sys­ tol i c flow velocity is 2 . 5 m per second (F i g . 3 1 -2), cor­ respo n d i n g to a 25 mm Hg pressure g rad ient between the LV and the RV. Systol i c blood pressu re was 75 mm Hg, hence RV systol i c pressu re = 50 m m H g . See The Echo Manual, 3rd Edition, page 1 62 a n d Fig u re 1 0-1 1 o n page 1 64.

See The Echo Manual, 3rd Edition, page 1 62. ANS WER 2: A . Oxyg e n sat u ration sa m p l e s . T h e

cha racte ristic fi n d i n gs a t rig ht heart catheterization of a patient with an acute ventri c u l a r septa l defect a re of an oxygen satu ration " setu p " with a p u l monary a rte­ rial oxygen satu ration 1 0 % g reater than the oxygen satu ration i n the ri g ht atri u m .

Figure 3 1 -2

See The Echo Manual, 3rd Edition, Ta ble 1 0-2 o n page 1 62.

ANS WER 5: B. I ntra-aortic bal loon p u m p a n d surgi­

ANS WER 3: B. An i ncreased risk of ventricular septal

ruptu re is seen i n patients with hig h-grade single-vessel coronary disease (especia l ly the left anterior descending Reference 1. Reeder GS. Identification and treatment of complications of myocardial infarction. Mayo Clin Proc. 1 995;70:880-8 84.

ca l consu ltation . The cu rrent recom mended approach to an i nfarct-related ventricular septa l ru ptu re is afterload red uction, i ntra-aortic bal loon p u m p cou nterpu lsation, and u rgent surgical intervention . 1

CAS E 3 2

Retrosternal Chest Pa i n

A chest pain. She is observed for 6 hours, and serial ECG and troponin T measurements

65-year-old woman presents t o the emergency department after 1 hour o f retrosternal

are normal. Her exercise capacity is limited secondary to severe arthritis in her knee. She is referred for dobutamine stress echocardiography (see Videos 32- 1 to 32-4) (Please note that the apical 4 chamber view is in the Mayo Clinic format with the left ventricle displayed on the left) .

QUESTION 1. Stress echocardiography demonstrates

QUESTION 3. Decreasing the mechanical index to

evidence of:

optimize contrast images can be accomplished by:

A. B. C. D. E.

Normal study Anterior ischemia Anterior infarction and inferior ischemia Inferior and lateral ischemia Lateral infarction

A. B. C. D. E. F.

Increasing the ultrasound frequency Lowering the ultrasound frequency Decreasing the output power of the transducer Decreasing the gain settings of the transducer A and C B and D

QUESTION 2. Which of the following findings at

dobutamine stress echocardiography is suggestive of myocardial ischemia? A. Normal resting wall motion, an initial hyper­ dynamic response with low-dose dobutamine followed by a decline in function at higher doses B. A dobutamine-induced deterioration in normal wall motion without a change at low-dose do­ butamine C. A hypokinetic apex at rest that becomes akinetic at higher doses of dobutamine D. A hypotensive response to peak doses of dobu­ tamine E. A, B, and C

1 08

QUESTION 4. Appropriate indications for dobuta­ mine stress echocardiography might include all of the following except?

A. P reoperative cardiac risk assessment in a 5 5 -year-old diabetic man being considered for aortobifemoral bypass for symptomatic peripheral arterial disease B. A 70-year-old woman complaining of atypical angina that occurs with brisk activity with 1 . 5 mm of downsloping lateral ST depression on her resting ECG C. A 68-year-old man with typical angina occur­ ring after climbing three flights of stairs D. A 5 8-year-old woman with an exertional chest pain syndrome who had a negative stress ECG test, stopping at a peak heart rate of 1 1 0 beats per minute because of limiting knee pain E. A and D

C A S E 32 I 1 0 9

ANS WER 1: D. Reg ional contracti l i ty was normal at

ANSWER 2: E. The typ ical fi n d i n g of ischemia d u r i n g

rest; however, with i n crementa l dobuta m i n e i nfusion, the patient developed i nferior a n d latera l hypok i n esis with l eft ventri c u l a r d i lation and a red uction i n ej ec­ tion fraction from 6 5 % to 5 0 % at peak stress. F i n d i n gs that a re h i g h ly suggestive of l eft c i rcu mflex +/- right coronary a rtery d i sease. C oron a ry a n g iography demon­ strated 9 0 % left c i rcu mflex (Fig . 3 2 - 1 ) a n d 7 0 % right coro n a ry a rtery (F i g . 32-2) stenoses .

a dobuta m i n e stress echoca rd iog ra m is the deve lop­ ment of new/worsen i n g wa l l motion a b n o r m a l ities with dobuta m i n e i nfusio n . These seg ments may or may not i n iti a l ly d isplay hypercontractil ity at lower doses of dobuta m i ne. A hypotensive response to dobuta m i n e is typica l ly rel ated to the vasod i latory effects of peri phera l ,8-receptor sti m u lation i n t h e setting o f a sma l l hyperdy­ n a m i c left ventricle a n d does not necessa rily represent the presence of coro n a ry a rtery d i sease . Hypotension may a lso occur i n the setting of severe dynamic outflow tract obstruction i n d uced by dobuta m i n e . Patients with ch ron ica l ly ischemic myoca rd i a l seg ments may demon­ strate myoca rd i a l h i bernatio n . Th is is cha racterized by a bi phasic myoca rd ial response to dobuta m i n e i nfusion with a n i m p rovement i n resting wa l l motion a b norma l i ­ ties a t low doses o f dobuta m i n e with a su bseq uent de­ c l i n e in myoca rd i a l contract i l ity at h i g her doses. ANS WER 3: E. M e ch a n i c a l i n d ex is p roport i o n a l

t o the pea k power a n d i nversely proporti o n a l t o t h e sq u a re root o f t h e freq uency. ANS WER 4: E. Dobuta m i n e stress echocardiog ra p hy

Figure 32-1

is a p p ropriate i n patients who have va l i d i n d ications for stress echoca rd i o g ra p hy but a re u n a b l e to exer­ cise. Patient A l i kely has a l i m ited exercise ca pacity, is at more than m i l d c l i n i ca l risk of isch e m i c heart d is­ ease, a n d wi l l u n dergo a h i g h-risk nonemergent s u r­ g ical p roced u re . Patient B req u i res i m a g i n g beca use her stress ECG wi l l be no ndiag nostic, but a n exercise stress shou l d be co nsidered . Patient C has exertional sym ptoms a n d a high pretest proba b i l ity for coro n a ry a rtery d isease. Patient D has a n eq u ivoca l stress E C G test beca use o f nonca rd iac l i m itation a n d is a p p ropri­ ate to u n dergo a p h a rmacolog ic stress test to eva l u ate her chest pa i n syn d rome . 1

Figure 32-2 References 1 . Douglas PS, Khandheria B , Stainback RF , et al. ACCF/ASE/ACEP/AHNASNC/SCAI/SCCT/SCMR 2008 appropriateness criteria for stress echocardiography. J Am Coll Cardiol. 2008; 1 8: 1 1 27- 1 1 47.

CAS E 3 3

Dyspnea and Chest Ti ghtness

Apresents for evaluation. She has had two recent hospitalizations for acute respira­

72-year-old woman with dyspnea and chest tightness on mild to moderate exertion

tory failure. Her medical history is significant for systemic hypertension, cardiomyopathy, atrial fibrillation, and defibrillator placement.

QUESTION I. On the basis of the 1 2-lead ECG (Fig. 33- 1 ) , what is the likely type of cardiomyopathy?

Figure 33-2 Figure 33- 1 A. B. C. D.

Septal hypertrophic cardiomyopathy Apical variant of hypertrophic cardiomyopathy lschemic cardiomyopathy Right ventricular dysplasia

QUES TION 2. On the basis of the transthoracic echocardiographic images (Videos 33- 1 to 33-5 and Figs . 33-2 to 33-6) , the findings are consistent with:

1 10

Figure 33-3

CASE 33 I 1 1 1

A. B. C. D.

Normal left ventricular regional wall contractility Apical infarct with aneurysmal formation Apical hypertrophic cardiomyopathy Contained apical rupture

QUESTION 3. Diastolic function is:

A. B. C. D.

Figure 33-4

Normal Mildly abnormal Moderately abnormal Severely abnormal

QUESTION 4. Given these findings, a limited exercise capacity, and significant symptoms, which of the fol­ lowing is a likely beneficial management option?

A. B. C. D.

Left ventricular assist device Alcohol ablation Surgical myectomy None of the choices

QUESTION 5. The patient underwent successful

treatment and left ventriculography 2 weeks later

(Video 33-6) . Figure 33-5

Figure 33-6

After 3 months, the patient represents with acute left ventricular failure. She undergoes repeat echocardiog­ raphy (Videos 33-7 to 33-9 and Figs. 33-7 to 33-9) . The likely mechanism for the patient's acute pulmonary edema is:

Figure 33-7

1 1 2 I E C H O C A R D I O G RA P H Y: A C A S E - B A S E D R E V I E W

A. B. C. D.

Severe mitral valve regurgitation Apical ballooning syndrome Acute anterior wall myocardial infarction Severe diastolic dysfunction

QUESTION 6. The patient is referred back to the oper­ ating room for mitral valve replacement. On the basis of the intraoperative study (Videos 33-10 to 33-13), the next step should be:

Figure 33-8

Figure 33-9

A. B. C. D.

Cancel the case Obtain a confirmatory contrast left ventriculogram Intravenous dobutamine Intravenous phenylephrine

C A S E 33 I 1 1 3

ANS WER I: A. The E C G demonstrates deep sym ­

metrical precord i a l T-waves typica l for the pattern seen with a p ica l hypertro p h i c ca rd i o myopathy. ANSWER 2: C. 2D i mages d e m o n strate seve re hy­

pertrophy of the m i d a n d a p ical two-th i rds of the left ve ntricle with systo l i c o b l iteration at t h i s l evel a n d n e a r d iasto l i c o b l iterati o n o f t h e neck i nto t h e a p ical pouc h . ANSWER 3: D. T h e functional ventric u l a r c h a m ber i s

essentia l ly t h e basa l one-th i rd that resu lts i n restrictive fi l l i n g typ ified by the m itra l i nflow pattern with a h i g h early ( E ) velocity that is m u c h g reater t h a n a d i m i n utive A velocity, d i asto l i c predo m i n a nt p u l monary vei n fl ow, a n d a m a rked ly red uced early d i asto l i c med i a l a n n u l us tissue velocity (e').

Figure 33-1 0

ANSWER 4: C. In patients with symptomatic a pical

hypertro p h i c ca rd i omyopathy refra ctory to m e d i ca l management, a s u rg i ca l a pica l (a n d here m i dventricu­ l a r) myectomy has been performed with a reaso n a b l e success . There is no role f o r a l cohol a blation i n the a p i ­ ca l va riant o f hypertro p h i c ca rd iomyopathy. 1 ANS WER 5: A. Repeat tra nsthoracic ech oca rd iog­

ra phy demonstrates m itra l va lve leafl et tetheri n g with resu lta nt severe m itra l va lve reg u rg itation . ANSWER 6: D. Tra nsesophageal i mages demonstrate

that there a p pears to be only m i l d m itra l va lve reg u r­ g itation (Fi g . 3 3 - 1 O). This stri k i n g difference com p a red with the tra nsth oracic echoca rd i o g ra m i s l i kely be­ ca use of d ifferen ces i n the loa d i n g con d itions brought a bout by a n esthes i a . I nfusion of p henyl e p h r i n e that i ncreased the systol ic blood pressu re from 90 m m H g t o 1 5 0 m m H g resu lted i n the u n m a s k i n g o f severe m itra l va lve reg u rg itation (Fig . 3 3 - 1 1 ) associated with a rise i n the mean p u l monary a rtery pressu re from 2 5 to

Figure 33-1 1 5 0 m m H g . The patient u n derwent s u ccessf u l m itra l va lve repl acement a n d d i d very wel l . Differences i n load i n g con d itions with a n esthesia may lead to u n deresti mation of m itra l va lve reg u rg itation by red uction i n preload that leads to a decrease i n mi­ tra l a n n u l us d i a meter a n d i m p roved leaflet coaptation a n d/or a red uction i n afterload that decreases the pres­ s u re g rad ient across the m itra l va lve . 2•3

References 1. Schaff HV, Brown ML, Dearani JA, et al. Apical myectomy: a new surgical technique for management of severely symptomatic patients with apical hypertrophic cardiomyopathy. j 7horac Car­ diovasc Surg. 20 1 0; 1 39:634-640. 2. Grewal KS, Malkowski MJ, Piracha AR, et al. Effect of general an­ esthesia on the severity of mitral regurgitation by transesophageal echocardiography. Am J Cardiol. 2000 ; 8 5 : 1 99-203.

3. Mihalatos DG, Gopal AS, Kates R, et al. Intraoperative assessment of mitral regurgitation: role of phenylephrine challenge. j Am Soc Echocardiogr. 2006; 1 9: 1 1 5 8- 1 1 64.

CAS E 3 4

Exertional Dyspnea

Adelivery of a healthy baby boy at 4 1 weeks of gestation. This was her first pregnancy 36-year-old woman presents with 4 days of exertional dyspnea. She is 6 days after

and was uncomplicated. She has no medical history. On examination, her heart rate is 1 30 beats per minute. There is evidence of neck vein distension and peripheral edema. Her body surface area is 1 . 8 and her body mass index 27 (Videos 34- 1 to 34-5 and Figs. 34- 1 to 34-6) .

1 14

Figure 34-1

Figure 34-3

Figure 34-2

Figure 34-4

CASE 3 4 I 1 1 5

QUESTION 2. The Doppler-derived left ventricular

cardiac index is: A. B. C. D. E.

1 .9 2.2 2.6 3.5 4.0

L/min/m2 L/min/m2 L/min/m2 L/min/m2 L/min/m2

QUESTION 3. The calculated left atrial (LA) volume

index by the biplane area length method is:

Figure 34-5

A. B. C. D. E.

18 21 24 28 32

mL/m2 mL/m2 mL/m2 mL/m2 mL/m2

QUESTION 4. The likely etiology for the clinical and

echocardiographic findings is: A. Chronic idiopathic dilated cardiomyopathy exacerbated by the postpartum state B. Amniotic fluid embolus C. Peripartum cardiomyopathy D. Spontaneous coronary artery dissection

Figure 34-6

QUESTION 1. What is the calculated left ventricular ej ection fraction (LVEF) ?

A. B. c. D. E.

1 1% 1 6% 21% 26% 31%

QUESTION 5. Appropriate steps in this patient's management might include all of the following except:

A. Angiotensin-converting enzyme inhibition B. Counseling to avoid lactation C. fS-Adrenoceptor antagonism D. Bromocriptine E. Immunosuppression

(ACE)

1 1 6 I E C H O C A R D I O G RA P H Y: A C A S E - B A S E D R E V I E W

QUESTION 6. The patient returns 9 months later after

medical therapy. On the basis of repeat transthoracic echocardiography (Video 34-6 and Fig. 34-7) , recom­ mendations include:

Figure 34-7

A. Further pregnancies are contraindicated B. ACE inhibition may be stopped, but further pregnancies should be avoided C. ACE inhibition may be stopped, and it is likely safe to consider a future pregnancy

CASE 34 I 1 1 7

ANS WER 1: B. As reco m m e n d ed by the American

Society of Ech ocard iogra p hy, the long- a n d short-axis d i mensions ca n be obta i ned d i rectly from the ESd and EDd, meas u red at the l evel of the m itra l ti ps as the sma l l est and l a rg est d ia m eters, respectively. If there a re no reg i o n a l wa l l motion a b n o r m a l ities, the l eft ventric u l a r d i mensions measured from the level of the papi l l a ry m uscles can be used to ca lcu late the LVE F as fol l ows : U ncorrected LVE F ( u LVEF) = [(E Dd)2 - (ESd)2]/ (EDd)2] x 1 00 Corrected LVE F = u LVEF + [( 1 00 - u LVEF) x correction facto r % ] T h e correction factor is 1 5 % if a pica l function is nor­ m a l , 5 % if a pical function is hypok i n etic, or 0 % if a p i ­ ca l fu nction is severely hypokinetic or a k i n etic. Here, u LVEF = [(60)2 - (55)2]/(60)2] x 1 00 = 1 6 % C orrected LVE F = 1 6 + [( 1 00 - 1 6) x 0 % ] = 1 6 % ANS WER 2: B. Stroke vol u m e (SV) is ca l c u l ated as

the prod uct of the cross-sectional a rea of the left ven­ tric u l a r outflow tract (LVOT) and the time velocity i nte­ g ra l (TVI) of a p u l se-wave sa m p l e from the LVOT. SV (ml) = [(D/2)2 x JT] x [ LVOT TVI] = [(2/2)2 x JT] x [1 O] =3.14x 10 = 3 1 ml C a rd i a c o u t p u t i s t h e p rod u ct o f S V a n d h e a rt rate = 3 1 x 1 3 0 = 4,030 m l per m i n ute C a rd iac i n d ex = ca rd i a c outpuVbody su rface a rea = 4/1 . 8 = 2 . 2 Um i n/m2 See The Echo Manual, 3rd Edition, F i g u re 4- 1 6 on page 7 1 . ANS WER 3 : A . The b i p l a n e a rea-l ength method re­

q u i res measu r i n g LA a rea from two orthog o n a l a p i c a l vi ews (a p i ca l fou r c h a m be r a n d a p ical two c h a m ber) a n d LA length . The average of the two lengths is used . LA vol u me = (0 . 8 5 x a rea , x a rea)/Average length LA vol u me = (0 . 8 5 x 1 3 x 1 3)/4 . 5 * * It is i m porta nt t o use consistent u n its: cm2 for a rea a n d cm for length .

LA vol u m e = 3 2 cc When i n dexed to body su rface a rea = 1 8 cc per m2 ANS WER 4: C. The echoca rd iogram demonstrates

m i l d to moderate left ventri c u l a r d i l atat i o n , seve re global red u ction in left ventricu l a r contracti l ity with de­ pressed ejection fraction a n d SV. The LA size is norma l . The most l i kely etiology i s peri part u m ca rd i o myopathy that ca n present a nyti m e between the last tri mester of preg n a n cy a n d 6 months postpart u m , most com ­ monly becom i n g c l i n i ca l ly a p p a rent i n the fi rst month after del ivery. It is more common i n patients older than 30 yea rs a n d/or who a re Africa n America n . A c h ron ic ca rd i o myopathy is l ess l i kely i n the sett i n g of a n en­ tirely normal LA size, a n d if it were to have g iven rise to c l i n ical heart fa i l u re, wou l d more l i kely have done so either i n the seco n d to th i rd tri mester or i m medi­ ately postpartu m . An a m n i otic fl u i d embolus is a ra re co m p l i cation occu rri n g i n the i m med iate postpa rt u m period, a n d a lthough left ventric u l a r dysfunction m a y occu r, the pri m a ry insult is seen with the rig ht ventricle (d i l ation and dysfunction). A sponta neous coronary a r­ tery d i ssection wou l d be expected to have a n associ­ ated c l i n ical chest pain syn d rome and l eft ventri c u l a r reg i o n a l dysf u n ctio n . ANS WER 5: E. Sta ndard medical thera py for systol i c

heart fa i l u re i n c l u d i n g AC E i n h i bition a n d B-blockade i s critica l . It is i m porta nt t o recog n ize that AC E i n h i b itors a re contra i n d i cated if the d i a g nosis is made prior to delivery as they a re h i g h ly teratoge n i c . Recent stud ies have i dentified a potenti a l ly pathophysiologic rol e by a defective antioxidant defense mecha n ism triggeri ng pathog e n i c prolactin fra g m ents, a n d hence, s u p p res­ sion prolacti n p rod uction with bromocri pti ne is i n d i ­ cated (wa rfarin a nticoa g u lation a lso needed). For the sa me reasons, cou n se l i n g a g a i nst lactation is reason­ a b l e . l m m u n os u p p ression is only i n d icated in the set­ ti n g of cases of b i opsy-proven g iant cel l myoca rd itis or other acute i nflam matory ca rd iomyopath ies but not id iopath ic peri partum ca rd iomyopathy. 1 ANS WER 6: A. Repeat echocard iog ra p hy d e m o n ­

strates t h a t the l eft ventri c u l a r s i ze has n o r m a l i zed and t h e LVE F h a s i m p rove d , tho u g h rem a i n i n g de­ p ressed, with a ca l c u l ated LVE F 41 % . I n this sett i n g , the pati ent needs t o rema i n on hea rt fa i l u re therapy, a n d futu re p reg n a n cy s h o u l d be avo i d ed beca use it wou l d co m e with a h i g h risk to the moth e r .

1 1 8 I E C H O C A R D I O G RA P H Y: A C A S E - B A S E D R E V I E W

Reference 1. Sliwa K, Blauwet L, Tibazarwa K, et al. Evaluation of bro­ mocriptine in the treatment of acute severe peripartum car­ diomyopathy: a proof-of-concept pilot study. Circulation . 20 1 0; 1 2 1 : 1 46 5-73 .

CAS E 3 5

Severe Ang i n a

Aangina (Videos 35- 1 and 35-2) .

53-year-old woman presents t o the emergency department fo r the evaluation o f severe

QUESTION 1 . Which of the following scenarios may explain the patient's echo findings?

A. B. C. D. E.

Subarachnoid hemorrhage Pheochromocytoma Recent emotional stress Coronary artery atherosclerosis All of the scenarios

QUES TION 3. If the systemic blood pressure is 80/50 mm Hg, the peak left ventricular pressure is approximately:

A. B. C. D.

60 mm Hg 80 mm Hg 1 1 0 mm Hg 1 30 mm Hg

QUESTION 2. Appropriate treatment strategies for

QUESTION 4. Which of the following statements is

the patient's hypotension might include all of the fol­ lowing excep t-.

correct?

A. B. C. D.

Intravenous Intravenous Intravenous Intravenous

dopamine metoprolol phenylephrine fluids

A. This condition occurs more frequently in young women with stress B. LV function does not recover fully in most of patients C. Myocardial contrast echo shows a large perfusion defect D. This condition can occur repeatedly

119

1 2 0 I E C H O C A R D I O G RA P H Y: A C A S E - B A S E D R E V I E W

ANSWER 1: E. Tra n st h o r a c i c e c h oca rd i o g ra p h y

d e m o n strates hype rdyn a m i c basa l l eft ve n t ri c u l a r contract i l ity with a k i nesis o f t h e m i d a n d d ista l l eft ve ntric l e . Alth o u g h th ese fi n d i n g s may occ u r i n the sett i n g of severe coro n a ry a rtery d iease, the a p pear­ a n ce i s consistent with that w h i c h occu rs i n the set­ t i n g of an ac ute myoca rd i a l i nfa rct i o n with n o r m a l coro n a ry a rte ries. See The Echo Manual, 3rd Edition, page 1 67. ANS WER 2: A . E c h oca rd i o g ra p hy d e m o n strates

a dyna m i c l eft ventri c u l a r outflow tract obstruction seco n d a ry to t h e hyperdyn a m i c basa l contract i l ity that occ u rs i n a p p roxi m ately 1 5 % of stress- i n d u ced ca rd i omyopathy. There is systo l i c a nter i o r moti on of t h e a nterior m itra l va lve l eafl et with poster i o r­ d i rected m itra l va lve reg u rg itation . Appropriate man­ agement i n t h i s sett i n g i n c l u d es i ntrave n o u s fl u i d s,

/J-b lockers, peri phera l vasoconstrictors, and the avoid­ a nce of i n otropic agents . 1

ANS WER 3 : D . T h e pea k l eft ventri c u l a r systol i c

pressu re wi l l b e t h e s u m o f t h e system i c systol i c blood pressu re (80 mm H g) a n d the left ventri c u l a r outfl ow tract pressu re g ra d ient (4 x [3 . 5]2) 1 2 9 m m H g . =

ANS WER 4: D . Stress i n d u ced ca rd iomyopathy is a

phenomenon that most co m m o n ly affects post-meno­ pausal women . Patients typica l ly present with a chest pa i n syn d ro m e a n d E C G a b n o r m a l i t i es (fre q u e ntly ST seg ment elevation) but a re fou n d to have norma l coro n a ry a rteries at a n g iography (a n d echo contrast perfusion). Left ventri c u l a r f u n ction is depressed but recovers i n the majority to normal with i n days to a few weeks. Wh i l e most patients do we l l , up to 1 0 % have have a recu rrence. 1 ·2

Reference 1. Bybee KA, Kara T, Prasad A, et al. Systemic review: transient left ventricular apical ballooning: a syndrome that mimics ST­ segment elevation myocardial infarction. Ann Intern Med. 2004: 1 4 1 ( 1 1 ) :8 5 8-865.

2. Madh avan M, Rihal CS, Lerman A, Prasad A. Acute heart failure in apical ballooning syndrome (TakoTsubo/stress cardiomyopathy) : clinical correlates and Mayo Clinic risk score. J Am Coll Cardiol 20 1 1 ;57(12) : 1 400-0 1 .

CAS E 3 6

Exertional Fati g u e, Dyspnea, a n d Two-Pi l l ow Orthopnea

AHe is referred for transthoracic echocardiography (Video 36- 1) .

30-year-old man presents with exertional fatigue, dyspnea, and two-pillow orthopnea.

QUESTION 1 . Potential underlying etiologies fo r the findings include all of the following excep t-.

A. B. C. D. E.

Idiopathic Iron overload HIV infection Scleroderma Myocarditis

QUESTION 2. The Doppler-derived cardiac output (see Figs. 36- 1 and 36-2) is:

A. B. C. D. E.

2.9 3.5 4.2 6.5 7.4

L L L L L

per per per per per

minute minute minute minute minute

QUESTION 3. An appropriate next step may include:

A. B. C. D. E.

Iron studies Endomyocardial biopsy Coronary angiography Systemic anticoagulation All of the choices

QUESTION 4. Endomyocardial biopsy

(Video 36-2)

is associated with a risk of ventricular thrombus. A. True B. False QUESTION 5. With regard to testing for hemochro­ matosis, which of the following is true?

Figure 36-1

Figure 36-2

A. A serum ferritin level >300 ng per ml is diag­ nostic for hemochromatosis B. A negative endomyocardial biopsy excludes the diagnosis C. Shortened relaxation times of cardiac MRI signals is highly sensitive for the diagnosis

121

1 2 2 I E C H O C A R D I O G RA P H Y: A C A S E - B A S E D R E V I E W

ANS WER 1 : D. An u n expected fi n d i n g of d i lated

ANSWER 4: A. Tr u e . Desp ite t h e ra pe u t i c a nt i co­

ca rd i omyopathy in a you n g person shou l d raise the suspicion for genetic i d iopath ic d i l ated ca rd iomyopa­ thy or a seco n d a ry cause such as h e moch ro matosis ( i ron overload), H IV i n fecti on, a n d postviral myoca rd i­ tis. Scleroderma may lead to a serositis, va lve dysfu nc­ tion, a n d/or p u l monary a rterial hypertension . M a rked global left ventricu l a r systol i c dysfunction is not a fea­ ture typ ica l ly seen in patients with sa rcoidosis. 1

a g u lation with u nfractionated hepari n , the day after endomyoca rd i a l biopsy repeat tra nsthoracic ech oca r­ d iog raphy (Vi deo 36-3) demonstrates more mobi l ity to the th rom bus i n the left ventri c u l a r a pex a n d new mobile thrombus attached to the ventricu lar septa l s u r­ face of the right ventricle at the site of endomyoca rd i a l biopsy. ANS WER 5: C. A l t h o u g h a s e r u m ferrit i n l eve l

ANS WER 2: E. The ca rd iac output can be ca lcu lated

as = (7Tr2)(LVOT TVl)(H R) = 0 . 7 8 5(D)2 (LVOT TVl)(H R) = 0 . 785 x (2 . 5)2 x ( 1 5) x ( 1 00) = 7 . 4 L per m i n ute See The Echo Manual, 3rd Edition, stroke vol u m e on p a g e 1 1 6.

ANS WER 3: E. All of the choices. In patients who pres­

ent with heart fa i l u re in the setting of u nexplai ned left ventricular g lobal hypokinesis, one shou ld consider po­ tentia l primary etiologies. Th is should i nclude coronary artery disease, hemoch romatosis, a n d other primary eti­ olog ies of d i l ated ca rdiomyopathy. F u rthermore, owi ng to the presence of mobile masses, h i g h ly suspicious for thrombus, the patients should also be a nticoag ulated .

> 3 0 0 n g per m l (>200 n g per m l i n wo m e n ) i s t h e s u g g ested cutoff for scree n i n g i n patie nts; it m u st be recog n ized that se r u m ferrit i n i s a n a cute p h ase rea cta nt and may i n crease with stress. E n d o myoca r­ d i a l b i o psy is t h e tra d i ti o n a l g o l d sta n d a rd for t h e d i a g n o s i s of ca rd i a c h e m oc h ro m a tos i s ; h oweve r, the i ro n d e position in the h e a rt i s n ot u n iform a n d m a y be m i ssed o n b i opsy. I n i ro n - l oaded tissue, the p a ra m a g n et i c effect of i ro n s h o rte ns the re laxat i o n t i m e of tissue s i g n a l s . C a rd i a c m a g n et i c reso n a nce rel axation t i m es correl ate n ot o n ly with the p resence o r a bsence of i ro n ove r l o a d b u t a l so with t h e r i s k of c l i n ica l deco m pensat i o n i n patie nts w i t h card i a c h e moch rom atos i s . 2 With m e d i c a l thera py ( l i s i n o p ri l , carved i l o l , wa rfa r i n , a n d chelation thera py), t h e patient g rad u a l ly i m p roved left ventri c u l a r contracti l ity (see Video 36-4) .

References 1. Varess G, Bruce CJ, Andreen K, et al. Acute thrombus formation as a complication of right ventricular biopsy. J Am Soc Echocar­ diogr. 20 1 0;23: 1 039-1 044.

2. Gujjs P, Rosing DR, Tripodi DJ , e t al. Iron overload cardio­ myopathy. ] Am Coll Cardiol. 20 1 0 ; 5 6 : 1 - 1 2 .

CAS E 3 7

Three- M o nth H i story of Progressive Exertional Shortness of Breath

Aof breath. He is referred for transthoracic echocardiogram (Videos 37- 1 to 37- 1 1 and 60-year-old man presents with a 3-month history of progressive exertional shortness

Figs. 37- 1 to 37-3) .

Figure 37-1

Figure 37-3

QUESTION 1. The left ventricular filling pressures are:

A. B. C. D.

Normal Mildly elevated Moderately elevated Severely elevated

QUESTION 2. Which of the following blood tests is

most likely to aid in the diagnosis in this patient?

Figure 37-2

A. B. C. D. E.

Immunoassay for serum free light chains Iron studies Serum brain natriuretic peptide level Serum angiotensin converting enzyme level Serum protein electrophoresis

1 23

1 2 4 I E C H O C A R D I O G RA P H Y: A C A S E - B A S E D R E V I E W

QUESTION 3. Which of the following systolic longi­

tudinal strain patterns would be most consistent with the 2D findings and diagnosis? A. B. C. D.

Figure Figure Figure Figure

37-4A 37-4B 3 7-4C 37-4D

Figure 37-4C

Figure 37-4A Figure 37-40 QUESTION 4. Factor or factors that are helpful in determining the appropriate treatment strategies and prognosis in this disorder include:

A. Cardiac troponin T B . N terminal pro-brain natriuretic peptide (NT-pro BNP) C. Serum uric acid D. Serum free light chains E. All of the choices QUESTION 5. What is the likely incidence of intra­ cardiac thrombus in this patient?

Figure 37-48

A. B. c. D.

0% to 1 0% 1 1 % to 25% 26% to 40% >40%

C A S E 37 I 1 2 5

ANS WER 1: D. M itra l i nflow pea k early (E) velocity

is h i g h at 1 .4 m per second with a ma rked red uction i n early d i asto l i c med i a l tissue velocity (e') at 3 cm per second . An E/e' ratio >40 is com pati b l e with a ma rked e l evation in left ve ntri c l e fi l l i n g p ressu re a n a logous to g rade 3 diasto l i c dysfu nction seen i n patients with s i n u s rhyth m . ANSWER 2: A. The 2 0 fi n d i n g s of i ncreased t h i ck­

ness of a l l ventri c u l a r a n d atr i a l wa l ls, a l l fou r heart va lve leaflets, a n d a perica rd i a ! effusion a re very suspi­ cious for a system i c i nfi ltrative d isorder such as a my­ loidosis. Serum i m m u n oglob u l i ns wi l l a l most certa i n ly be a bnormal with a n a bnormal i n crease i n serum free l i ght c h a i n s characteriz i n g a clonal excess of c i rcu lat­ i n g plasma ce l ls . Alth o u g h serum a n d u r i n e p rotei n electro p h o resis a re often a b n orma l , they lack sen ­ sitivity a n d i f negative do not exc l u d e t h e d i a g nosis. Seru m i ron stu d i es (a bnorma l i n hemoch romatosis­ wh ich ca n be associated with a d i l ated ca rd i o myopa­ thy) a n d seru m a n g iote n s i n convert i n g enzyme l evel (often elevated in sa rcoidosis) wou l d both be expected to be norma l . The deg ree of e l evation in bra i n natri­ u retic pepti de levels provides p rog nostic i nformation i n system i c amyloidosis; however, it is nonspecific and wou l d not help in cha racterizi n g the card iomyopathy. Evi dence of a myloid depositi on in tissue wi l l a lso be req u i red to confi rm the presence of a myloidosis. Com­ mon sites of biopsy with high yields (often obviati n g t h e need for ca rdiac biopsy) include abdom i n a l fat pad, rectu m, and bone ma rrow. See The Echo Manual, 3rd Edition, pages 274 to 276. ANSWER3: B. The typ ica l stra i n pattern seen with AL

a myloidosis is a m a rked red u ction in g l oba l l eft ven ­ tri c u l a r l o n g itu d i n a l systol i c stra i n with preservation of a p i cal f u n ction (B). The g l o b a l average of - 1 0 % is m a rked ly red uced . For co m pa rison, F i g u re 3 7 -4A

demonstrates a n o rm a l l eft ve ntri c u l a r stra i n pattern (norm a l more negative than - 1 8 % ) . F i g u re 37-4( with overa l l normal pattern a p a rt from a foca l ma rked re­ d u ction in stra i n in the septum and F i g u re 3 7-40 with ma rked red uction in a p ica l stra i n wou l d be the typi ca l patterns o f reg ional dysfunction seen with hypertrophic ca rd iomyopathy of the septa I (C) or a p i ca l (0) va ria nts . Systol i c l o n g itu d i n a l stra i n b y speckle tracking provides a sensitive, reprod ucible assessment of left ventri c u l a r systo l i c fu nction identifyi ng g loba l a n d reg ional a b nor­ m a l ity patterns that a re h e l pf u l i n eva l uati n g patients with suspected card i o myopathies. 1 •2 ANSWER 4: E. The presence a n d d e g ree of ca rd iac

i nvolvement d o m i n ates prog n osis i n patients with sys­ tem i c amyloidosis. Va rious ma rkers a re very h e l pful i n assess i n g both prog nosis a s wel l a s suita b i l ity for tol­ erati n g stem cel l transpla ntatio n . These include serum u r i c acid a n d seru m free l i g ht chains. A simple prog­ nostic model is based on th ree va riables: a serum u ric acid >8 mg per di, a n NT-pro-B N P >332 ng per L, and a ca rd i a c tropo n i n T >0.03 5 µg per L. 1 -4 ANSWER 5: D. T h e re a re a h ost of factors that

s u g gest t h i s patient i s at a pa rti c u l a rly h i g h risk of i ntraca rd i a c t h ro m bosi s . T h e stu d y by F e n g et a l . 5 eva l u ated t h e c h a ra cte ristics associ ated with i ntra ­ ca rd i a c th ro m bosis seen i n patients with a myloi dosi s . Factors that were associ ated with i ntraca r d i a c t h rom­ bos i s i n c l u ded AL a myloidosis, atria l fi bri l lation, and worse d i asto l i c f u n cti o n . Patients with restrictive fi l l ­ i n g pattern (g rade 3 or 4 d i asto l i c dysf u n ction) were 1 7-fo l d m o re l i kely to h ave i n tra ca rd i a c t h ro m bos i s com p a red w i t h th ose w i t h a non restrictive fi l l i n g pat­ tern (grade 1 or 2) even in the sett i n g of s i n u s rhyth m . l ntraca rd i a c t h ro m b u s i s p resent i n ventricles a s we l l as i n atri a . T E E shou l d b e con s i d e red i n patients with c a rd i a c a my l o i d os i s and a restrictive d i asto l i c fi l l i n g pattern .

1 2 6 I E C H O C A R D I O G RA P H Y: A C A S E - B A S E D R E V I E W

References 1. Bellavia D, Pellikka PA, Al-Zahrani GB, et al. Independent pre­ dictors of survival in primary systemic (Al) amyloidosis, including cardiac biomarkers and left ventricular strain imaging: an obser­ vational cohort study. ] Am Soc Echocardiogr. 20 1 0;23 :643-652. 2. Dispenzieri A, Gertz MA, Kyle RA, et al. Prognostication of survival using cardiac troponins and N-terminal pro-brain natriuretic peptide in patients with primary systemic amyloidosis undergoing peripheral blood stem cell transplantation. Blood. 2004; 1 04: 1 8 8 1-1 887. 3. Dispenzieri A, Lacy MQ, Katzmann JA, et al. Absolute values of immunoglobulin free light chains are prognostic in patients with

primary systemic amyloidosis undergoing peripheral blood stem cell transplantation. Blood. 2006; 1 07:3378-3383. 4. Gertz M, Lacy M, Dispenzieri A, et al. Troponin T level as an exclusion criterion for stem cell transplantation in light-chain amyloidosis. Leuk Lymphoma. 2008;49:36-4 1 . 5 . Feng D , Syed IS, Martinez M , e t al. Intracardiac thrombosis and anticoagulation therapy in cardiac amyloidosis. Circulation. 2009; 1 1 9:2490-2497.

CAS E 3 8

Progressive Exert i o n a l Dyspnea

Msive exertional dyspnea, now functional class III. He denies angina or orthopnea.

r. SA is an 8 1 -year-old man who presents with a 6-month history of progres­

He has modest peripheral edema controlled by a low-dose loop diuretic. He has long­ standing systemic hypertension and chronic kidney disease with a baseline creatinine of 2 mg per dl. On examination, his blood pressure was 1 1 0/ 5 0 mm Hg and pulse rate 80 beats per minute and regular. His central venous pressure was elevated to 1 5 cm. Carotid pulses are of normal intensity. His chest is clear. He had a 3/6 pansystolic mur­ mur at the lower left sternal border and an audible third heart sound. He was referred for transthoracic echocardiography (see Video 38- 1) .

QUESTION 1 . Diastolic function (see

Video 38- 1

and Figs. 38- 1 through 38-3) is:

Figure 38-3

Figure 38-1

A. Normal B. Mildly abnormal (delayed relaxation) C. Moderate to severely abnormal with restrictive filling

Figure 38-2 1 27

1 2 8 I E C H O C A R D I O G RA P H Y: A C A S E - B A S E D R E V I E W

QUESTION 2. Doppler profile across the patent fora­

QUESTION 3. The likely diagnosis (see Video

men ovale (see Video 38-1 and Fig. 3 8-4) suggests:

is: A. B. C. D. E.

38- 1 )

Eosinophilic heart disease Cardiac amyloidosis Idiopathic (primary) pulmonary hypertension Hypertrophic obstructive cardiomyopathy Dilated cardiomyopathy

QUES TION 4. Which of the following statements re­ garding senile cardiac amyloidosis compared with pri­ mary (AL) amyloidosis is false?

Figure 38-4 A. B. C. D.

Severe right-left shunt Left atrial hypertension Severe tricuspid valve regurgitation Severe mitral valve regurgitation

A. Patients with senile cardiac amyloidosis tend to be older B. Patients with senile cardiac amyloidosis almost always are male C. Left ventricular (LV) wall thicknesses tend to be less in senile amyloidosis D. Survival in senile amyloidosis may be up to five times greater

C A S E 38 I 1 2 9

ANS WER I: C. Diastolic function is moderate to se­

ANS WER 3: B. The l i kely d i a g nosis is card iac a my­

verely a bnorma l . M itra l i nflow pattern demonstrates restrictive fi l l i n g pattern with E>>A a n d a short decel­ eration time, a lthough it can be normal by itself espe­ cia lly in a you n g i n d ivid u a l who ca n have a short DT as wel l . However, the main d ifference between normal and pseudonormal m itra l i nflow is that pseudonormal i nflow is a result of delayed myocard ial relaxation that decreases E velocity and prolongs deceleration time (DT), and elevated fi l l i n g pressu re that increases E velocity and shortens DT. Pea k early d i astolic velocity (e') on tissue Doppler of med i a l m itra l a n n u l us i n d i cates the status of myoca rd ial relaxation and is red uced (e' 3 cm per second) resu lti ng in a ratio of E/e' of 30, consistent with a severe elevation in LV fi l l i n g pressures. E/e' > 1 5 usu­ a l ly i n d i cates elevated d iastolic LV fi l l i n g press u re . Pul­ monary vei n Doppler profi le of d i asto l i c predom i n a n t forwa rd flow is a lso consistent with an elevation i n left atrial pressu re, although s i m i l a r p u l monary vei n Doppler velocity pattern ca n be seen i n a you n g hea lthy i n d i ­ vid u a l . I t s h o u l d a lso b e noted that p u l monary vei n atrial flow reversa l velocity is not pro m i nent as expected in a patient with increased fi l l i ng pressure. It is related to prolonged PR i nterva l on the electroca rd iogra m so that the onset of atrial contraction takes place before the dia­ stolic forward flow is completed . Pu l monary vei n atrial flow reversa l pattern of prolonged flow d u ration and i ncreased velocity is specific, but not sensitive for detect­ ing i ncreased left ventricular end diastolic pressure.

l o i d o s i s . Two-d i m e n s i o n a l i m a g i n g d e m o n strates increased left and right ventricular wa l l thickness, thick­ ened va lve leaflets, a n d a perica rd i a ! effusio n . The con­ ste l l ation of fi n d i ngs cou p led with the severe d iasto l i c fu nction a b n ormal ities poi nts towa rd a d i a g nosis of ca rdiac a myloidosis. Although many d i sorders may g ive rise to an i n crease in LV wa l l th ickness, i nvolvement of a l l heart cham bers, the va lves, a n d perica rd i u m is char­ acteristic of a n i nfi ltrative d isorder. C a rdiac a myloidosis ca used by the deposition of a myloid fi b r i l s p redom i ­ na ntly i n t h e myoca rd i a l i nterstiti u m g ives rise t o a n i ncrease i n LV wa l l thickness a n d profo u n d relaxation abnormal ities but preservation i n LV chamber size and ejection fraction u ntil l ate in the d i sease . ECG a b nor­ m a l ities a re common a n d classica l ly demonstrate low voltage with a pseudoi nfa rct pattern (see Fi g . 38-5).

=

I n t h i s patient, e n d o myoca rd i a l b i o psy confi rmed the presence of the sen i l e-ca rd iac restricted su btype (tra nsthyret i n positive) on a my l o i d osis (positive s u l ­ fated a l c i a n b l u e sta i n i ng).

S e e The Echo Manual, 3rd Edition, page 1 20. ANS WER 2: B. Doppler profile across the patent fora­

men ovale suggests left atrial hypertension . Doppler pro­ file across patent foramen ovale demonstrates flow from the left to right atri um, i mplyi ng an LA pressu re hig her than right atrial pressure. On the basis of the mod ified Bernou l l i equation (P 4v2), the pressu re g rad ient be­ tween these cham bers equals 4(1 .9)2, that is, 1 4 mm Hg. Subcostal imaging demonstrating sign ificant enlargement of the i nferior vena cava sign ifies that right atrial pressure is high ( 1 0 to 1 5 mm Hg). Therefore, left atrial pressu re at the end of systole is i n the range of 24 to 29 mm H g . =

S e e The Echo Manual, 3rd Edition, F i g u re 4- 1 4 o n page 69.

Figure 38-5 See The Echo Manual, 3rd Edition, a myloidosis on pages 274 to 279. ANS WER 4: C. Patients with sen ile card iac amyloidosis

tend to be male, be older, and have a greater deg ree of LV wal l thickening than do patients with primary (AL) ca r­ diac amyloidosis. Survival estimates are much longer with senile amyloidosis despite the older ages of the patients.1 ·2

References 1. Kyle RA, Spittell PC, Gertz MA, et al. The premortem recogni­ tion of systemic senile amyloidosis with cardiac involvement. Am j Med. 1 996; 1 0 1 :395-400.

2. Ng B, Connors LH, Davidoff R, et al. Senile systemic amyloido­ sis presenting with heart failure: a comparison with light chain­ associated amyloidosis. Arch Intern Med. 2005 ; 1 6 5 : 1 425-1429.

CAS E 3 9

Loud Systol ic M u rm u r

A a preemployment medical examination. She has n o symptoms. H e r heart rate is

30-year-old woman presents fo r the evaluation o f a loud systolic murmur heard o n

70 beats per minute and her blood pressure 1 28/64 mm Hg.

QUESTION 1. On the basis of the lesion

(Videos 39- 1 to 39-3, Fig. 39- 1 , Videos 39-4 to 39-6, and Figs. 39-2 and 39-3) , what lesion is present?

Figure 3 9-3

Figure 3 9-1

A. B. C. D.

Membranous ventricular septal defect Muscular ventricular septal defect Inlet ventricular septal defect Primum atrial septal defect

QUESTION 2. What is the calculated right ventricular systolic pressure (RVSP) ?

A. B. C. D.

1 00 mm Hg plus right atrial pressure 28 mm Hg 28 mm Hg plus right atrial pressure 1 00 mm Hg

QUESTION 3. The most common form of ventricular

septal defect in young children is:

Figure 39-2 1 30

A. B. C. D.

Membranous ventricular septal defect Muscular ventricular septal defect Inlet ventricular septal defect Supracristal ventricular septal defect

CASE 39 / 1 3 1

QUES TION 4. In the parasternal short-axis view

QUESTION 5. Complications of ventricular septal

(Fig. 39-4) , a membranous septal defect will typically be located in which of the following locations (illus­ trated in the figure as A, B or C) ?

defect include:

Figure 39-4 AoV, aortic valve; LA, left atrium; RA, right atrium; TV, tricuspid valve; RVOT, right ventricular out­ flow tract; PV, pulmonary valve; MPA, main pul­ monary artery. A. Location A B . Location B C. Location C

A. B. C. D.

Pulmonary hypertension Aortic valve regurgitation Right ventricular outflow tract obstruction All of the choices

1 3 2 I E C H O C A R D I O G RA P H Y: A C A S E - B A S E D R E V I E W

ANS WER I: B. Best seen i n the a p ica l fou r-cha m ber

that by a d u lthood the most co mmon form of ventricu­ lar septa l defect is the mem branous type.

view, there is a sma l l m uscu l a r ventric u l a r septa l defect measu ri ng a p p roxi mately 4 mm i n width . The l eft a n d r i g h t ventricles a re o f normal size.

ANS WER 4 : A. M e m b ra n o u s ventri c u l a r septa l de­

ANS WER 2: B. The pea k velocity across the ventric­

u l a r septa l defect is h i g h (5 m per second) consistent with a restrictive defect and a normal RVS P. There is a l ow-ve locity left-r i g h t d i asto l i c g ra d ient i n d i cati n g that r i g h t ventric u l a r d i asto l i c pressu res a re a lso low. The pea k velocity from the conti n uous wave Doppler i nterrog ation across the ventri c u l a r septa l defect cor­ responds to the pea k g ra d ient between the l eft a n d rig ht ventricles. T h e pea k systol i c flow velocity o f 5 m per second corresponds to a 1 00 m m H g pressu re g ra­ d ient between the l eft ventricle and the right ventri cle. G iven the systol i c bl ood press u re is 1 28 m m H g , then the RVS P 2 8 m m H g . =

See The Echo Manual, 3rd Edition, page 1 62 a n d Fig­ u re 1 0-1 on page 1 64.

ANS WER 3: B. The most com mon form of ventricu­ l a r septa I defect i n c h i l d ren is the m uscu l a r type . These

defects typ ica l ly a re s m a l l and su rro u n ded enti rely by ca rd iac m uscl e . U s u a l ly, t h ey a re l ocated in the d i s ­ ta l two-th i rds o f the l eft ventri c u l a r septu m . T h ey freq uently close d u ri n g the cou rse of c h i l d hood such

fects a re located at the base of the heart, at the j u nc­ tion of the m uscu l a r, AV atri a-ventricu l a r, and outlet portions of the septu m . The m e m b ra n o u s sept u m is the th i n nest portion of the normal septum, a n d ven­ tricu l a r septa l defects i n this l ocati on a re i m mediately adjacent to the aortic and tricusp i d va lves, which can easi ly be a p p reciated in the short-axis view. The prox­ i m ity of m e m b ra n o u s d efects to the tricusp i d a n d aortic va lves l eaves t h e adjacent leafl ets somewhat u n s u pported . C onseq uently, the patients req u i re pro­ longed s u rve i l l a nce for the development of prog ressive reg u rg itatio n , even if the defect is sma l l . See The Echo Manual, 3rd Edition, Figure 20- 1 3 on page 343.

ANS WER 5: D. R i g ht ventri c u l a r outflow tract ob­

struction occ u rs seco n d a ry to m uscu l a r i n fu n d i b u l a r obstruction . A s u p racrista l ve ntric u l a r septa l defect leads to a lack of s u p port of the right coro n a ry cusp resu lti ng i n aortic va lve reg u rg itation . A l a rge ventricu­ lar septa I d efect wi l l lead to a l a rge deg ree of s h u nt i n g thro u g h the p u l monary vasc u l atu re, the d evelopment of severe p u l m o n a ry hypertensio n, and a reversa l of the s h u nt.

CAS E 4 0

Exertional Shortn ess of Breath with N o Medical H istory

Ano medical history of note. She is referred for transthoracic echocardiogram (Videos 26-year-old woman comes for the evaluation of exertional shortness of breath. She has

40- 1 to 40- 1 1 and Figs. 40- 1 to 40- 1 0) .

Figure 40-1

Figure 40-3

Figure 40-2

Figure 40-4

1 33

1 3 4 I E C H O C A R D I O G RA P H Y: A C A S E - B A S E D R E V I E W

Figure 40-5

Figure 40-8

Figure 40-6

Figure 40-9

Figure 40-7

Figure 40-1 0

C A S E 40 I 1 3 5

Q UESTION 1. What are the estimated pulmonary

artery pressures? A. B. C. D. E.

2 1 /9 mm Hg 2 1 / 1 3 mm Hg 36/ 1 8 mm Hg 60/ 1 5 mm Hg 65/20 mm Hg

QUESTION 2. What is the Doppler-derived cardiac

output? A. B. C. D.

4 5 6 7

L per minute L per minute L per minute L per minute

A. B. C. D. E.

Pulmonary valve stenosis Pulmonary valve regurgitation Pulmonary hypertension Bicuspid pulmonary valve Normal finding

QUESTION 7. What is the cause for right ventricular

enlargement? A. B. C. D.

Primum atrial septal defect Sinus venosus atrial septal defect Secundum atrial septal defect Intact atrial septum and primary severe tricus­ pid valve regurgitation

QUESTION 8. After transesophageal echocardiogra­

phy (Video 40- 14 and Fig. 40- 1 1 ) , the appropriate next step includes: QUESTION 3. The right ventricular index of myocar­ dial performance (RIMP) (Tei index) is:

A. B. c. D. E.

0 .40 0.45 0.50 0.55 0 . 60

QUESTION 4. Measures of right ventricular longitu­

dinal systolic function are: A. Normal B. Mild to moderately reduced C. Severely reduced

Figure 40- 1 1 QUESTION 5. What are the structure and cause of

blood Bow indicated with the arrow in Videos 40- 12 and 40- 1 3? A. Coronary sinus B . Left inferior pulmonary vein C. Left circumflex coronary artery to left atrial fistula D. Artifact

Q UES TION 6. The M-mode tracing (Fig. 40-4)

through the pulmonary valve is associated with which finding?

A. Surgical repair or replacement of the tricuspid valve B . Percutaneous closure of the atrial septal defect C. Surgical closure of the atrial septal defect D. Hemodynamic catheterization

1 3 6 I E C H O C A R D I O G RA P H Y: A C A S E - B A S E D R E V I E W

ANS WER 1: E. Pulmona ry artery systolic pressure is es­

timated as fou r times the peak (tricuspid reg urg itant ve­ locity)2 plus an esti mate of right atrial pressure. Pul monary a rtery d iastolic pressu re is estimated as fou r times the (end pul monary reg u rg ita nt velocity)2 p l us a n estimate of right atrial pressure. Here the i nferior vena cava is of normal size and collapses normal ly with respiration, and hence, the rig ht atrial pressure is normal (5 mm Hg). Here pul monary a rtery systolic pressure is estimated as 4(3 .9)2 plus 5 = 65 mm Hg and 4( 1 .9)2 plus 5 = 20 mm H g . ANS WER 2 : B . Stroke vol u me (SV) is ca l c u l ated as

the product of the cross-sectional a rea of the left ven­ tric u l a r outfl ow tract (LVOT) a n d the time velocity i nte­ g ra l (TVI) of a p u l se-wave sa m p l e from the LVOT. SV (ml) = [(D/1 . 8)2 x lT] x [ LVOT TVI] = (2/1 . 8)2 x 7T] x [ 1 7] = 3 . 88 x 1 7 = 66 m l C a rd i a c o u t p u t i s t h e p rod u ct o f SV a n d h e a rt rate = 66 x 87 = 5 . 7 L per m i n ute See The Echo Manual, 3rd Edition, F i g u re 4- 1 6 on page 7 1 .

ANSWER3: E. The RI M P is a relatively load-independent

parameter of global function that is a ratio of the sum of the isovolumic relaxation and contraction times to the right ventricular ejection time. It can be easily ca lculated by the fol lowi ng equation: RI M P = [Tricuspid va lve closu re to ope n i n g ti me] - [right ventri cu l a r ej ect i o n t i m e]/[ R i g ht ventr i c u l a r ej ection time] = 445 - 280/2 80 = 0 . 6 0 ANS WER 4: A. U n l i ke the l eft ve ntricle, the p re­

d o m i n a nt orientation of myoca rd i a l fibers i n the right ventricle runs in the l o n g i tu d i n a l p l a n e with a m u ch g reater p roportion of right ventri c u l a r contraction d etermi ned by the base movi n g u p towa rd the a p ex rather than ra d i a l contracti l i ty. This study has two mea­ su res of long itud i n a l motion of the tricus pid a n n u l us as measu res of r i g h t ventri c u l a r l o n g itu d i n a l systo l i c motion . T h e fi rst is a meas u re o f t h e d i sta nce o f a n ­ n u l a r excu rs ion d u ri n g systole, the tri cuspid a n n u l a r p l a n e systol i c exc u rs i o n . A n o r m a l va l u e i n a you n g patient s h o u l d be >20 t o 2 2 m m (here 2 7 m m ) . A l ­ tern ative ly, o n e ca n measu re t h e pea k systo l i c tis­ sue velocity of the tricuspid latera l a n n u l us (S'), here

1 3 cm per second is a lso norm a l . The American Society of Ech oca rd iogra p hy recommends at least one of these two measu res is performed to assess rig ht ventric u l a r systol i c function . ANS WER 5: B. Echoca rd iography demonstrates nor­

m a l a natomy of the left i nferior p u l monary vei n d ra i n ­ i n g i nto t h e left atri u m with n o r m a l l a m i n a r flow. ANS WER 6: C. An M-mode record i n gs from the pul­

monary va lve shows the cha racte ristic a p peara n ce of p u l monary hypertension with a " W " shape with m i d ­ systol i c i nte rruption o f va lve open i n g p rod u c i n g the notc h i n g a n d " W " shape. This is beca use of pressu res waves refl ected back to the proxi m a l p u l monary a rtery i n the sett i n g of h i g h p u l mo n a ry vasc u l a r resista nce. See The Echo Manual, 3rd Edition, F i g u re 9-88 on page 1 48.

ANS WER 7: C. Atr i a l septa l d efects com p rise a p­

proxi mately 1 0 % of congen ita l hea rt d i sease. The most com mon form is the secu n d u m defect accounting for 60% to 7 5 % of atria l septa l defects. La rge atria l septa l defects res u lt i n a l a rge left-to-rig ht s h u nt that ca uses right heart vol u m e overload a n d cha m ber e n l a rgement as i n this case. Secu n d u m atria l septa l defects a re typi­ ca l ly fou n d in the centra l portion of the sept u m , s u r­ rou n d i n g the fora men ova le, a n d a re frequently oblong or e l l i ptica l . See The Echo Manual, 3rd Edition, atria l septal de­ fect discussion on pages 334 to 335.

ANS WER 8: D. C l os u re of a n atri a l septa l d efect

(whether percuta neously or s u rg ica l ly) is i n d i cated i n the setting of rig ht-sided cham ber e n l a rgement rega rd­ less of the presence of sym pto m s . G iven the associ­ ated severe tricuspid valve reg u rg itation, consideration wou l d be g iven to s u rg i ca l clos u re with concom itant tricuspid va lve repa i r rather than percuta neous clos u re a lone, desp ite the adequate r i m surro u n d i n g the de­ fect. H owever, the presence of s i g n ificant p u l monary hypertension wou l d wa rra nt h emodyn a m i c catheter­ ization to d i scern the m a g n itude of fixed i ntri nsic p u l ­ m o n a ry a rterial hypertension rather t h a n t h a t s i m ply beca use of i n creased tra n s p u l m o n a ry flow. I n deed, at ca rd iac catheterizatio n , the patient had s i g n ificant p u l mo n a ry hypertension and ma rked i n crease i n p u l­ monary vasc u l a r resista nce a n d was reco mmen d ed to rece ive p u l m o n a ry vasc u l a r ta rgeted thera py rather than ASD closure.

CAS E 4 1

Progressive Fat i g u e a n d M a rked Lower Extremity Edema

Asive fatigue and marked lower extremity edema. His medical history is significant for 45-year-old man presents to the emergency department for the evaluation of progres­

right innominate vein stenosis secondary to a clotted dialysis catheter. He has undergone uncomplicated percutaneous revascularization for 2 years before. He continues to dialyze three times a week through a left forearm fistula. He has diabetes mellitus. On examination, his heart rate is 70 beats per minute and his blood pressure 1 70/70 mm Hg. His central venous pressure is elevated. He has a 2/6 pansystolic murmur that increases on inspiration. He has pitting edema of the lower extremities to midthigh.

QUESTION 1. What is the most likely explanation

Q UES TION 3. On the basis of the hepatic vein

for the abnormal finding seen on parasternal long-axis imaging (Videos 41-1 to 41-3 and Fig. 4 1 - 1 ) ?

Doppler (Fig. 4 1 -2) , which of the following is false?

Figure

4 1 -2

Figure 4 1 - 1 A. Metastatic carcinoma to pericardium B. Pericardia! fat C. Myxoma QUESTION 2. The findings demonstrated at the level of the tricuspid valve (Videos 41-4 to 41-6) likely represent:

A. B. C. D.

Thrombus Myxoma Tricuspid prosthesis Foreign material

A. Systolic Row reversals secondary to tricuspid valve regurgitation tend to peak later in systole B. Forward Row decreases during expiration C. An increase in systolic Row reversals with expiration suggests the presence of constrictive hemodynamics D. The sum of the peak systolic forward Bow velocity and the peak velocity of atrial reversal is associated with the right atrial pressure

1 37

1 3 8 I E C H O C A R D I O G RA P H Y: A C A S E - B A S E D R E V I E W

ANS WER I: B. Fat may acc u m u late a ro u n d the epi­

ANS WER 3: D. See F i g u re 4 1 -4 . Alt h o u g h there is

card i a l su rface of the heart or between the perica rd i a ! layers. I n the setti n g o f a perica rd i a ! effusion, fat may a p pear as a n i rreg u l a r, hyperechoic mass m o b i l e in the perica rd i a ! space or on the epicard i a l su rface and may be m i sta ken as a metastatic tumor. Li pomatous tissue is easily d isti n g u ished by CT as hypodense appea r i n g tissue with atten uation o f between -20 t o 50 H o u n ­ sfi e l d u n its a n d on M RI , where its tissue wi l l d is p l ay h i g h s i g n a l i ntensity on T , - a n d T -we i g hted i m a g i n g 2 a n d s i g n a l d ropout on fat-satu ration i m a g i n g . 1 • 2

no s i m u lta n eous resp i rometer record i n g , forwa rd flow (below the base l i ne) i ncreases with i n s p i ration ( I N S P) a n d decreases with exp i ration (EXP) . Th ro u g hout the resp i ratory cycle, there is d iasto l i c p redo m i n a nt for­ wa rd flow (D FW) . Systol i c flow is pred o m i n a ntly rever­ sal flow (SR) i n d i cative of elevated rig ht atrial pressu re a n d i l l ustrat i n g a s i g n a l that peaks i n later systole, i n kee p i n g with systol i c flow reversa ls seen with sig n ifi­ ca nt tricusp i d va lve reg u rg itation . H e re systol i c flow reversa ls tend to fa l l i n expi ration . A ma rked decrease in d i asto l i c forwa rd flow a n d an i ncrease in d i asto l i c flow reversa ls observed with exp i ration (not seen here, see case 1 6) a re h i g h ly suggestive of constrictive he­ modyn a m ics . 3

ANS WER 2: D. Ech oca rd i o g ra p h y d e m o n strates

two large tubular structu res crossing the tricuspid va lve ca using sign ificant va lvu lar reg u rg itation a n d probably i nterferi n g with the fi l l i n g of the ri g ht ventricle. They have a d i a m eter of 1 cm each, have the appea ra n ce of meta l l ic stents, and represent the m i g ration of the previ­ ously placed venous vascular stents. They were success­ fu l ly removed percuta neously with a snare (F i g . 4 1 -3) .

Figure 4 1 -4 See The Echo Manual, 3rd Edition, discussion of echo findings in constrictive pericarditis on pages 294 to 299.

Figure 4 1 -3

References 1. Ahn YK, Park JC, Park WS, et al. A case of prominent epicardial fat mimicking a tumor on echocardiography. J Korean Med Sci. 1 999; 1 4 : 5 7 1 -574. 2. Pressman G, Verma N. Pericardial fat masquerading as tumor. Echocardiography. 201 0;27:E1 8-E20.

3. Ommen SR, Nishimura RA, Hurrell DG, et al. Assessment of right atrial pressure with 2-dimensional and Doppler echocar­ diography: a simultaneous catheterization and echocardiographic study. Mayo Clin Proc. 2000;75:24-29.

CAS E 42

Progressive Exert i o n a l Dyspnea, Atypical Chest D iscomfort, Abd o m i n a l a n d Lower Extrem ity Swe l l i n g

Mmigraines, asthma, and gastroesophageal reflux disease. She presents with a 1 -year s. HA i s a 3 8-year-old woman with a medical history o f bipolar affective disorder,

history of progressive exertional dyspnea, atypical chest discomfort, abdominal and lower extremity swelling. On examination, her blood pressure is 1 30/60 mm Hg and heart rate 98 beats per minute. Her apical impulse is displaced laterally, and she has systolic and diastolic murmurs. There is evidence of ascites and significant lower extremity edema. She is referred for transthoracic echocardiography (see Video 42- 1) .

QUESTION 1 . Echo two-dimensional and Doppler findings are consistent with:

A. Mild aortic valve regurgitation B . Mild-moderate aortic valve regurgitation C. Moderate-severe aortic valve regurgitation

QUESTION 5. Which of the following is correct with regard to the "reverse-dagger" shape of the tricuspid regurgitation (TR) velocity (Fig. 42- 1 ) ?

QUESTION2. In addition to aortic valve regurgitation, what other cardiovascular pathology can cause diastolic flow reversal in the descending aorta?

A. B. C. D.

Coarctation of the aorta Patent ductus arteriosus (PDA) Aortic dissection Supravalvular aortic stenosis

QUESTION3. Which of the following chronic medical

therapies is likely responsible for the patient's findings? A. B. C. D.

Omeprazole for gastroesophageal reflux disease Montelukast for asthma Lithium for bipolar affective disorder Ergotamine for migraine headaches

Figure 42-1 A. The shape is because of high right atrial pressure B. The shape is characteristic for all TR velocities C. The shape is because of rapid rise of right atrial pressure D. The shape is because of pulmonary hypertension

QUESTION 4. Which of the following settings could also likely explain the echocardiographic findings?

A. Primary bronchial carcinoid B. Hepatic carcinoid metastases with patent foramen ovale C. Primary ovarian carcinoid 1 39

1 4 0 I E C H O C A R D I O G RA P H Y: A C A S E - B A S E D R E V I E W

ANS WER 1: C. There is moderate-severe aortic va lve

reg u rg itation . See The Echo Manual, 3rd Edition, pages 207 to 2 1 0 a n d Appen d i x 20 on page 4 1 3 .

I n assessment o f aortic va lve reg u rg itation, the width of the reg u rg ita nt jet at its ori g i n relative to the d i men­ sion of the l eft ve ntricu l a r o utfl ow tract (LVOT) is a good p red i ctor of the a n g iog ra p h i c severity of aortic reg u rg itation . Othe r Doppler featu res that should be used in determ i n i n g the severity of reg u rg itation i n ­ clude pressu re ha lftime o f the conti n u ous wave Dop­ pler s i g n a l of the reg u rg ita nt jet, d i asto l i c reversa l flow i n the d esce n d i n g aorta, a n d the deceleration time of the m itra l flow E velocity. Ve na contracta is the s m a l l ­ est n e c k o f the fl ow reg ion at the level o f the aortic va lve, i m med iately below the flow convergence re­ g i o n . This is smaller than the width of the jet of aortic reg u rg itation i n the LVOT. The vena contracta is usua lly measu red from the pa rasternal long-axis view, a n d a ven a contracta width l a rger than 6 m m is specific for severe aortic reg u rg itatio n . I n severe aortic reg u rg itatio n , system i c d i asto l i c pres­ s u re decreases q u ickly, so that the aortic reg u rg itation s i g n a l (correspond i n g to the p ressu re d i fference be­ tween the aorta a n d the l eft ventricle) has a shorten ed dece l e ration t i m e a n d t h u s shortened p ress u re h a lf­ t i m e . A pressu re h a lftime between 200 a n d 500 m i l ­ l iseconds wou l d be com pati b l e with moderate aortic va lve reg u rg i tation . D iasto l i c retrograde flow ca n be demonstrated in the desce n d i n g thoracic aorta a n d even i n t h e a b d o m i n a l aorta . H o l o d iasto l i c reve rsa l of fl ow is a l so com pati b l e with severe a o rtic va lve reg u rg itatio n . ANSWER 2: B. Patent d u ctus a rteriosus (PDA) . Persis­

tent pate ncy of the d u ctus a rteriosus is a n a rterial com­ m u n i cation between the u p per desce n d i n g aorta a n d t h e p u l monary a rtery a n d usua l ly a rises from t h e upper desce n d i n g aorta a n d co n nects to the d i sta l main p u l ­ monary a rtery, n e a r t h e ori g i n o f the left p u l monary a r­ tery. This results i n a d i asto l i c ru n off of aortic pressu re a n d may be associated with d i asto l i c fl ow reversa ls seen i n the p roxi m a l desce n d i n g thoracic aorta (see Case 1 9, Fig . 1 9-3) . Most persistent PDAs a re sma l l but a re associated with a somewhat i ncreased risk for i n ­ fective endarteritis (rel ative t o t h e n o r m a l heart), even when the vol u me load is i ns i g n ifica nt. I n the a bsence of other ca rd iac abnormal ities, clos u re of the PDA de­ creases the risk of e n d a rteritis to that of the genera l

population, a n d therefore, clos u re b y su rgery or cath­ eter device is genera l ly recommended for any PDA that has a n a u d i b l e m u rm u r . A l a rge PDA ca n lead to p u l ­ monary hypertension a n d left atria l a n d left ventric u l a r vol u m e overload . ANS WER 3: D. E rgota m i n e for m i g ra i n e headaches.

Omeprazole is not noted for cardiac toxicity. Monte l u ­ kast, l i ke a l l leu kotriene i n h i b itors, h a s been l i n ked with tri g g e r i n g or u n maski n g C h u rg-Stra uss syn d rome i n patients with severe asth m a . C ha racterized b y asthma, peri phera l a n d tissue hypereosinoph i l ia , a n d sma l l ves­ sel vasc u l itis, C h u rg-Stra uss syn d rome can affect the heart i n u p to 2 5 % of cases . C a rd iac m a n ifestations i n c l u d e endomyoca rd i a l th icken i n g a n d throm bus for­ mation, l eft ventri c u l a r dysfunction, a n d perica rd i a ! i n ­ flam mation . Lith i u m is most notable for ca using ca rdiac rhythm abnorm a l ities i nc l u d i n g s i n u s node dysfunction a n d bradyca rd i a . The ergot derivatives ergota m i ne and methyserg ide were previously used widely for the pro­ phylaxis a n d treatment of m i g ra i nes but a re used less freq uently beca use of ava i lable alternatives with better adverse risk p rofi les. The i r nota b l e card iac toxicity is the i n d uction of va lvu l a r a b normal ities s i m i l a r to those seen with carc i n o i d synd ro m e . Other agents l i n ked with s i m i l a r valvu lar findi ngs include the a ppetite suppressa nt com b i nation of fenfl u ra m i n e a n d dexfenfl u ra m i n e ( " Fen-Phen " ) a n d t h e dopa m i n e agonists, pergo l i d e a n d cabergoli ne, used for t h e treat­ ment of Pa rki nson d isease. The l i n k between these agents a n d d i sease states a ppea rs to be sti m u l ation of the seroto n i n 2B rece ptors that in turn sti m u l ates fi­ broblast prol iferation a n d va lvu lar thicke n i n g . I n severe cases, va lve l eaflets become retracted a n d prog ressive i m mobile with sign ificant va lvu lar reg u rg itation . ANS WER 4: B. H e patic ca rci n o i d m etastases with

patent fora m e n ova l e . Because the su bsta nces that a re secreted by a carci n o i d tumor a re i nactivated by the l u n g , it is predom i n a ntly the ri g ht side of the heart that is i nvolved . H owever, the left-sided ca rd iac va lves a re i nvolved in 5 % to 7 % of cases. This is most l i kely beca use of the p resence of a patent fora m e n ova l e t h a t a l lows the t u m o r su bsta nces t o p a s s from the right atri u m to the left atri u m or beca use of p u l mo­ n a ry metastasis. Although bronch i a l ca rci noid if it is se­ cretora ry may g ive rise to left-sided va lve i nvolvement these tumors typ ica l ly do not secrete . The nota b l e fact a bout ca rc i n o i d i nvolvement of the testes o r ova ries is that they a re d isti n ct from other abdom i n a l tumor

C A S E 42 I 1 4 1

sites beca use they may g ive rise to rig ht-sided ca rd iac i nvolvement even in the a bsence of hepatic d isease . Th is is beca use of exceptional venous s u p p ly of the gonads that bypass the l iver. ANS WER 5: C. The shape is beca use of a ra p i d rise

of right atr i a l p ress u re . Typ i ca l ly, the shape of the

t r i c u s p i d reg u rg ita nt jet i s a sym metri ca l p a ra bo l i c c u rve . H owever, i n t h e setti n g o f torrenti a l tricusp i d va lve reg u rg itat i o n , t h e l a rge vol u m e o f b l ood ree n ­ teri n g the r i g ht atri u m d u ri n g systo l e ca uses a ra p i d rise i n r i g h t atria l pressu re t h a t eq u i l i b rates with the rig ht ventric u l a r p ressu re and term i n ates the reg u rg i ­ ta nt wavefo rm t o be more tria n g u la r .

CAS E 4 3

Monomorphic Ventricular Tachyca rd ia after Card iac Arrest

Aher teacher with an automated external defibrillator. She has a further run of mono­

9-year-old girl suffers a cardiac arrest at school. She is appropriately resuscitated by

morphic ventricular tachycardia in hospital and undergoes implantation of a defibrillator. She is referred for further evaluation (Video 43- 1) .

QUESTION 1 . The likely etiology of her dysrhythmia

is: A. B. C. D. E.

1 42

Long QT interval syndrome Cardiac fibroma Cardiac rhabdomyosarcoma Metastatic carcinoma Myxoma

QUES TION 2. The most appropriate management of further dysrhythmia is:

A. B. C. D.

Antiarrhythmic therapy Tumor resection Cardiac transplantation Chemotherapy

C A S E 43 I 1 4 3

ANS WER I : B. Tra n st h o r a c i c e c h oca rd i o g ra p h y

d e m o n strates a l a rge, d iscrete, h o m o g e n o u s mass present i n the latera l wa l l of the left ventricle. The ap­ pea ra n ce i n l i g ht of the h i story is most suggestive of a ca rd i a c f i b roma . Cardiac fi bromas a re ben i g n neoplasms a r i s i n g from myoca rd ial fibroblasts . Arisi n g most com mo n ly i n the left ventricle (typica l ly the free wa l l), they a re the sec­ ond most com mon tumor in the ped iatric popu lation; however, they a re ra rely a lso seen i n a d u lts. The m a n i ­ festations o f the tumor va ry depen d i n g on t h e i r size and location . Typ ical presentations i n c l u d e m a l i g nant ventricular dysrhyth m i a a n d heart fa i l u re .

resection is i n d i cated whenever the tumor is associated with sign ificant c l i n ical man ifestations. Typica l ly, fibro­ mas ca n be " peeled away from the u nderlyi n g myo­ card i u m after hypothermic card io p u l monary a rrest and ca rdioplegic myoca rd i a l protection . " Altho u g h a few patients may req u i re ca rd iac transpla ntation for very extensive tumors, most fibromas may be resected with­ out com p l ication . The long-term outcome after resec­ tion, as in this patient (Fig . 43- 1 ), is genera l ly excellent. 1 •2

C a rd iac fi bromas a re typ ica l ly ech oca rd i o g ra p h i ca l ly wel l c i rcu mscribed a n d conta i n ca lcification i n u p to 5 0 % of cases . See The Echo Manual, 3rd Edition, page 3 1 1 .

ANSWER 2: B. C a rd i a c fi bromas a re ben i g n tumors,

for which there is no role for chemotherapy. S u rg ica l

F i g u re 43- 1 . Resected ca rd iac fi broma.

References 1. E!Bardissi AW, Dearani JA, Daly RC, et al. Analysis of benign ventricular tumors: long-term outcome after resection. J Thorac Cardiovasc Surg. 2008; 1 3 5 : 1 0 6 1 - 1 068.

2. E!Bardissi AW, Dearani JA, Daly RC, et al. Survival after resec­ tion of primary cardiac tumors: a 48-year experience. Circulation. 2008; 1 1 8 ( 1 4) (suppl) :S7-S l 5 .

CAS E 44

Exertional Ch est Pa i n

Agram for the assessment of new onset exertional chest pain.

77-year-old man with a history o f hyperlipidemia i s referred fo r exercise echocardio­

His resting ECG demonstrates non specific ST-T changes. Prior to stress, his rest echocardiogram demonstrates normal left ventricular size with an ejection fraction of 60% . He has grade 1 diastolic dysfunction, a left atrial volume index of 30 cc per m2, and normal cardiac valves. Images were suboptimal, and echo contrast was given for left ventricular opacification.

QUESTION 1. What was the intervention that likely improved the image with regard to the use of echo contrast for left ventricular opacification (see Video

44-1) : A. More contrast was given B . The mechanical index was decreased C. The dose of contrast was decreased QUESTION 2. Which of the fo llowing statements concerning mechanical index is false?

A. Mechanical index is a measure of acoustic power B. Mechanical index is proportional to the acous­ tic pressure C. Mechanical index is proportional to the square root of the ultrasound frequency D . Mechanical index greater than 0.7 is likely to destroy microbubbles QUESTION 3. The patient exercised on a standard Bruce treadmill protocol for 6 minutes (98% of pre­ dicted) , developing chest pain. The stress ECG was nondiagnostic secondary to resting ST-T segment change. Exercise stress echocardiography demonstrates

1 44

evidence of (see Videos 44-2 and 44-3. Orientation in both clips is as follows: top left, apical 4 chamber in the Mayo format with left ventricle displayed on the left; top right, apical 3 chamber; bottom left, apical short axis; bottom right, apical 2 chamber) : A. B. C. D. E.

Anterior infarction Lateral wall ischemia Anterior ischemia Inferior infarction No evidence of ischemia

QUESTION 4. The following is/are true with respect to findings suggesting evidence of severe ischemia on exercise stress echocardiography:

A. The development of multiple wall motion ab­ normalities with stress B. Dilation of the left ventricular cavity and/or a reduction in left ventricular ejection fraction are more common in exercise rather than dobuta­ mine stress C. Hypotension is more specific for severe coro­ nary disease with exercise rather than dobuta­ mine stress D. All of the choices

C A S E 44 I 1 4 5

ANS WER 1 : B. The fi rst p a rt of the cl i p d e m o n ­

strates a bsence o f contrast i n the d ista l two-th i rds of the l eft ventric l e . Alth o u g h t h i s may be rel ated to the i nsuffi cient contrast, the fi rst step is to e n s u re that the myoca rd i a l i n dex is low. I n deed here the Ml was red u ced from 0 . 6 to 0 . 2 5 resu lti n g in the avo i d a nce of b u b b l e destructi on without the need for f u rther contrast a d m i n istration . If excessive contrast is present in the left ventricle, there wi l l be atten uation effects a n d rel ative d rop-out of contrast seen in the d i sta l p a rt of the i ma g e . H e re a re lative red ucti on i n the concentration of contrast wi l l a i d i n a l lowi n g u n iform opacification . See The Echo Manual, 3rd Edition, page 1 0 1 .

ANS WER 2: C. The u ltraso u n d acoustic power is ex­

pressed as the mech a n ical i n dex, which is proportional to the acoustic p ressu re a n d i nversely proportional to the sq u a re root of the u ltraso u n d freq uency. A me­ cha n i ca l i n d ex h i g h e r than 0 . 7 is l i kely to destroy the m i crob u bbles: Acoustic pressu re Mech a n ica l i n d ex =

{fQ

See The Echo Manual, 3rd Edition, gas-fi l led m icro­ bubbles discussion on pages 1 00 to 1 0 1 .

ANS WER 3 : C. At rest, left ventri c u l a r wa l l motion i s

norma l . Followi ng exercise stress there is hypoki nesis/ seve re hypo k i n e s i s of t h e a n te r i o r wa l l (base to a p ex), sept u m ( m i d a n d a p ex), a n d a p ical latera l a n d i nfe r i o r seg m e nts, s u g gest i n g i sc h e m i a i n t h e terri­ tory of the proxi m a l l eft a nterior desce n d i n g coro n a ry a rtery. S u bse q u e n t coro n a ry a n g i o g r a p h y d e m o n ­ strated h i g h - g ra d e ste notic l e s i o n s i n the l eft a nte­ rior desce n d i n g coro n a ry a rtery a n d a d i a g o n a l a rtery (see F i g . 44- 1 ) .

F i g u re 44-1

See The Echo Manual, 3rd Edition, pages 1 76 to 1 80.

ANS WER 4: D. In a d d ition to the deve l o p m e n t of

m u l t i p l e reg i o n a l wa l l motion a b n o r m a l i t i es, oth e r factors t h a t a re specific f o r the p rese nce o f severe coro n a ry a rtery d i sease i n c l u d e l eft ventricu l a r d i l a ­ ti o n , a n d a decl i n e i n l eft ventri c u l a r systo l i c f u n ction (both fi n d i n g s l ess co m m o n ly seen with dobuta m i ne stress, desp ite a h i g h b u rd e n of coro n a ry d i sease). Hypoten s i o n with exercise i s a lso a specific fi n d i n g for seve re d isease u n l i ke with dobuta m i n e where hy­ pote n s i o n may a l so be related to syste m i c vasod i l a­ t i o n from /J-2 sti m u l at i o n o r dyn a m i c o utfl ow tract obstruction . See The Echo Manual, 3rd Edition, Ta ble 1 1 -4 on page 1 79.

CAS E 4 5

Dyspnea and Presyncope

Apressure is 1 20/80 mm Hg and her heart rate 80 beats per minute. Her apex beat is non­

74-year-old woman presents with dyspnea and presyncope. On examination, her blood

displaced. There is a 3/6 harsh systolic murmur with a single second heart sound. She is referred for transthoracic echocardiogram (Videos 45-1 and 45-2 and Figs. 45-1 through 45-3) .

F i g u re 45-1

F i g u re 45-3

QUESTION 1. What is the calculated cardiac out­ put, given a left ventricular outflow tract diameter of 20 mm (Fig. 45- 1 ) ? A. 3 . 9 L per minute B. 4.3 L per minute C. 5 . 5 L per minute D. 6.2 L per minute

QUESTION 2. How often is the peak velocity of aor­ tic stenosis obtained from a nonapical window?

F i g u re 45-2

1 46

A. B. c. D. E.

Rarely 5% 1 0% 20% 30%

C A S E 45 I 1 4 7

QUESTION 3. What is the calculated aortic valve area

(Videos 45- 1 and 45-2 and Figs. 45- 1 and 45-3) ? A. B. C. D. E.

0.65 cm2 0.7 cm2 0.75 cm2 0 . 8 cm2 0 . 8 5 cm2 F i g u re 45-4

QUESTION 4. Coronary angiography demonstrated

three-vessel coronary disease, and the patient under­ went aortic valve replacement with a tissue biopros­ thesis and a left internal mammary graft to left anterior descending coronary artery and saphenous vein grafts from the aorta to the distal right and obtuse marginal arteries. The procedure was uncomplicated. On postoperative day 6, the patient acutely becomes nauseated, complains of abdominal pain, and passes out. She had no palpable blood pressure. She requires 2 minutes of chest compressions and received vaso­ pressin, and her systolic blood pressure improved to 80 mm Hg. A 1 2-lead ECG was obtained (see Fig. 45-4) . The next appropriate step includes:

A. B. C. D.

Surgical reexploration of coronary grafts Coronary angiography CT scan of cardiac function, arteries, and grafts Intravenous thrombolytic therapy

QUESTION 5. Following this test or intervention, a transthoracic echocardiogram was performed (see Videos 45-3 through 45-7) . An appropriate next step involves:

A. Diuresis and ongoing inotropic support B . A transesophageal echocardiogram to better assess chamber size and function C. CT scan to evaluate pulmonary arteries for pul­ monary embolus

1 4 8 I E C H O C A R D I O G RA P H Y: A C A S E - B A S E D R E V I E W

ANS WER 1 : B. The ca rd iac output can be ca lcu lated

as = (nr2)(LVOT TVl)(H R) = 0 . 78 5 (D2)(LVOT TVl)(H R) = 0. 7 8 5 (22)(2 5)( 5 5) = 4 . 3 L per m i n ute See The Echo Manual, 3rd Edition, pages 69 to 7 1 a n d Figure 4-1 6 on page 7 1 .

ANS WER 2: D. Here two conti n uous wave Doppler

signals a re shown, one from the a pex a n d one from the right pa rasternal a rea . In a co m p rehensive exa m i ­ nation o f a patient with aortic stenosis, it is critica l to perform a Doppler eva l uation from a l l ava i l a b l e tra ns­ d u cer windows . Approxi m ately 2 0 % of the ti me, the pea k s i g n a l wi l l be obta i ned from a window other than the a pex. See The Echo Manual, 3rd Edition, Doppler echoca r­ d iography (in aortic stenosis) on pages 1 90 and 1 9 1 .

ANS WER 3: A.

Aortic va lve a rea = [(LVOT TVI) x (LVOT a rea)]/Ao TVI = [(LVOT TVI) x 0 . 7 8 5 (LVOT D)2]/Ao TVI = [(2 5) x (3 . 1 4)]/1 2 0 = 7 8 . 5/1 2 0 = 0 . 6 5 cm2

ANS WER 4: B. E C G demonstrates acute i nj u ry pat­

tern in the i nferior a n d a nterolatera l leads. The patient rem a i n s in shock. Angiog ra p hy a l l ows for s i m u lta neous assessment of native and g raft patency and the poten­ tial for placement of a n i ntra-aortic ba l l oon pump and

Reference 1. Price S, Prour J, J aggar SI, et al. "Tamponade" following cardiac surgery: terminology and echocardiography may both mislead. Eur] Cardiothoracic Surg. 2004;26: 1 1 56- 1 1 60.

if req u i red percuta neous coro n a ry and or g raft i nter­ vention . Thrombolyti c therapy wou l d be co ntra i n d i ­ cated i n t h e i m med i ate postoperative period . C o r o n a ry a n g i o g ra p h y d e m o n strates u n c h a n g e d severe n ative vesse l coro n a ry d i sease with patent g rafts . An i ntra-aort i c b a l loon p u m p i s p l aced . The patient rece i ves i ntrave n o u s fl u i d res uscitat i o n a n d d o pa m i n e . T h e r i g h t atri a l p ress u re i s 1 9 m m H g , p u l m o n a ry ca p i l l a ry wed g e p ress u re i s 2 3 m m H g , a n d ca rd i a c output i s 1 . 5 L p e r m i n ute .

ANS WER 5: B. Tra nsthoracic i mages a re su bopti m a l ;

however, there is a n i m p ression o f external clot i n t h e pericard i a ! space. Alth o u g h this cou l d be assessed by a CT sca n , a tra n seso p h a g e a l ech oca rd i o g ra m can be performed ra p i d ly at the bedside i n the i ntens ive ca re u n it. A tra nsesophageal echoca rd iogra m demon­ strates s i g n ifica nt perica rd i a ! fl u i d and sol i d mate r i a l (pres u m ed cl ot) i n the perica r d i a ! space (Videos 45-8 t h roug h 45-1 1 ) . The patient went to the operati n g room where t h e mediasti n a l a n d perica rd i a ! clot was evacuati n g lea d i n g to an i m med iate i m p rovement i n hemodyn a m i cs. Th is case i l l ustrates cha racteristics of ta m ponade that may occ u r in the early postoperative state . Typ ical ly, tra nsthoracic i m a g i n g is d iffi cult a n d classic fi n d i n g s o f ta m ponade, a n d/or fl u i d col lections a re s e e n i n o n ly a p p roxi mately two-th i rds of patients. Tra nsesophageal echoca rd iography is req u i red i n the rem a i n der to ad­ e q u ately visual ize the fl u i d . Effusions occu rri n g early after su rgery a re often sma l l a n d freq uently loca l ized . S u rg ical exp l o ration is preferred ove r percuta neous perica rd i ocentesis. 1

CAS E 4 6

Severe lsche m i c Card i o myopathy a n d Functional Class IV Left Heart Fa i l ure

AIV left heart failure is referred for echocardiography (see Videos 46-1 through 46-4 76-year-old man with known severe ischemic cardiomyopathy and functional class

and Figs. 46- 1 through 46-5) .

F i g u re 46-1

F i g u re 46-3

F i g u re 46-2

F i g u re 46-4

1 49

1 5 0 I E C H O C A R D I O G RA P H Y: A C A S E - B A S E D R E V I E W

and 46-7) . The status of aortic valve opening (see Video 46-5) is influenced by:

F i g ure 46-5

F i g ure 46-7

QUESTION I. The Doppler-derived left ventricular

(LV) cardiac output is: A. 3.0 L per minute B. 3 . 5 L per minute

C. 4 L per minute D. 4 . 5 L per minute

QUESTION 2. The calculated mitral effective regur­ gitant orifice (ERO) area by the proximal isovelocity surface area method is:

A. 0.28 cm2 B. 0.32 cm2

C. 0.38 cm2 D. 0.46 cm2

QUESTION 3. The patient is referred for implanta­ tion of a continuous How LV assist device (LVAD, Heart Mate II) as destination therapy for his heart failure. He is referred for followup echocardiogra­ phy (see Videos 46-6 through 46-9 and Figs. 46-6

A. B. C. D.

LVAD function LV contractility LV end-diastolic pressure All of the choices

QUESTION 4. The patient's LV filling pressures are

likely: A. Normal B. High C. Unable to assess based on available data QUESTION 5. The inflow cannula systolic velocity profile (see Fig. 46-6) is:

A. Too low, the velocity should be >2 m per second B. Normal C. Too high, the velocity should be < 1 m per second QUESTION 6. In patients with LV assist devices, the limitation to cardiac output is by the:

A. B. C. D.

F i g ure 46-6

LV Right ventricle LVAD There is no limitation to output

C A S E 46 I 1 5 1

ANS WER 1: B. Stroke vol u m e (SV) is ca l c u l ated as

ANS WER 4: A. A critical function of the LVAD is u n ­

the prod uct of the cross-sectional a rea of the LV out­ flow tract a n d the velocity time i nteg ra l of a pu lse wave sa m p l e from the LVOT.

load i n g o f the LV. Echoca rd iography is critica l i n t h e n o n i nvasive assessment o f l oa d i n g statu s . M a rkers of h i g h left atria l (LA) p ressu re i n c l u d e a n i ntra-atrial septum that bows towa rd the r i g ht (pa rti c u l a r i n the setti n g of a d i lated, nonco l l a p s i n g i nferior vena cava), a restrictive m itra l i nflow fi l l i n g pattern with a short de­ celeration t i m e . H e re the i ntra-atri a l sept u m is sh ifted towa rd the right (see Video 46-7), a n d the vena cava is s m a l l a n d col l a pses norm a l ly (see Video 46-8), i n­ d icati n g the LA p ressu re is l ess t h a n the r i g ht atr i a l (RA) pressu re a n d that t h e R A press u re is not h i g h . The m itra l i nflow deceleration time is a lso not shortened .

SV (ml) = = = =

[(D/2)2 x n] x [LVOT TVI] [(2 .4/2)2 x n] x [ 1 1 ] 4.52 x 1 1 50 m l

C a rdiac output is t h e prod uct o f S V a n d heart rate = 50 x 70 = 3 5 00 m l per m i n ute . ANS WER 2: B. 0 . 3 2 cm2

The reg u rg ita nt flow can be ca lculated as fol l ows : 6 . 2 8 x a l iasi n g velocity = (0 . 7 0 cm)2 x 6 . 2 8 x 46 cm per secon d = 1 42 c c p e r secon d

Flow rate = (r)2

x

E RO = Flow rate/Pea k M R velocity

E RO = 1 42 cc per second/439 cm per second = 0.32 cm2

ANS WER 3: D. The aortic va lve open i n g status is a

co m p l ex p h e n o m e n o n that is affected by m a n y fac­ tors a n d refl ects the p ressu re/f u n ction relations h i p of the LV to the LV assist d evice . The a o rtic va lve wi l l open i f t h e d e g ree of LV flow supersedes LVAD flow. Factors that lead to an i n crease in LV fl ow i n c l u d e h i g h LV e n d d i asto l i c p ressu re or a n i n crease i n LV contract i l ity. Factors that lead to a fa l l i n LVAD fl ow may i n c l u d e obstructi o n to the i nfl ow ca n n u la , the p u m p, o r the outflow ca n n u l a , o r a red uction i n the rate of the p u m p .

ANS WER 5: B. I n assess i n g the pati ent with a n LV

assist device, i m a g i n g of the inflow can n u la is critica l . T h e inflow can n u la m ust b e sitti ng centra l ly i n t h e LV a pex d i rected towa rd the m itra l va lve (best eva l u ated in the fou r- a n d two-ch a m ber views) . There s h o u l d be l a m i n a r color flow without any turbulence. A t t h e entra nce t o the i nflow ca n n u la , p u lse wave Doppler should demonstrate moda l p u lsat i l e flow between 1 and 2 m per second . If it is low, it is beca use the pa­ tient is vol ume depleted or there is a problem with the i nflow ca n n u l a . If it is h i g h , there is typica l ly some ob­ struction to the i nflow ca n n u l a . ANS WER 6: B . I n patients with a n LV assist device,

the l i m it i n g factor in ca rd i a c output is the right ven­ tri c l e . If the LVAD output exceeds the flow i nto the LV (determi ned by the rig ht ventricu l a r output), a l ife­ th reate n i n g " suck-down eve n t " occu rs with p hysi ca l col l a pse of the LV m a n ifested by i m med i ate d rop i n perfus i o n , acute ventri c u l a r dysrhyt h m i a , a n d r i g h t ventric u l a r d i l atation (see Video 46-9) .

CAS E 4 7

Prog ressive Exertional Dyspnea, Wei g ht G a i n, a n d Periphera l Edema

Aweight gain, and peripheral edema. Her history is notable for a mitral valve surgical 65-year-old woman presents with a 6-year history of progressive exertional dyspnea,

commissurotomy 20 years before.

QUESTION I. On the basis of her transthoracic images

(Videos 47-1 to 47-14 and Figs. 47- 1 to 47- 1 1 ) , the cal­ culated tricuspid effective regurgitant orifice (ERO) area by the proximal isovelocity surface area (PISA) method is:

F i g u re 47-3

F i g ure 47- 1

TfO

-

360 m •

v ·.

: . �..-/

j..&...

I

ti.

·'iA ,

I

' ..

,. , , , -

.

.

F i g ure 47-2

1 52

-

-..

] F i g u re 47-4

C A S E 47 I 1 5 3

F i g u re 47-5

F i g u re 47-8

F i g u re 47-6

F i g u re 47-9

F i g u re 47-7

F i g u re 47-1 0

1 5 4 I E C H O C A R D I O G RA P H Y: A C A S E - B A S E D R E V I E W

QUESTION 4. Calculate the right ventricular (Tei)

index of myocardial performance (RIMP) . A. B. c. D.

0.25 0.33 0 . 60 0.72

QUESTION 5. With regard to the following param­

eters of right ventricular function:

F i g u re 47- 1 1

A. B. C. D.

0.65 cm2 0 . 8 0 cm2 0 . 9 5 cm2 1 .2 cm2

QUESTION 2. Which of the following findings is con­ sistent here with the presence of severe tricuspid valve regurgitation?

A. B. C. D. E.

Incomplete tricuspid valve leaflet coaptation Coronary sinus dilatation Systolic flow reversals in the hepatic veins A PISA radius >0.9 cm2 All of the choices

QUESTION 3. Which of the following is the potential cause of an enlarged coronary sinus?

A. B. C. D. E.

Anomalous drainage of the left pulmonary veins Persistent left superior vena cava Severe pulmonary arterial hypertension A and B A, B, and C

A. The RIMP, TAPSE, and Lateral S' are consis­ tent with normal right ventricular systolic func­ tion B. The TASPSE and Lateral S' are falsely low be­ cause of the preload C. The RIMP is falsely low because of the preload D . The RIMP, TAPSE, and Lateral S ' are con­ sistent with reduced right ventricular systolic function (TAPSE, tricuspid annular plane systolic excursion; Lateral S ' , tricuspid annulus lateral peak systolic tissue velocity) .

QUESTION 6. Appropriate management steps follow­ ing coronary angiography include:

A. Percutaneous mitral valve balloon valvuloplasty B . Mitral valve repair and tricuspid valve replace­ ment C. Mitral valve replacement and tricuspid valve repair D. Mitral and tricuspid valve replacements

C A S E 47 I 1 5 5

ANS WER 1: D. 1 . 2 cm2

ANS WER 4: B. The RI M P incorporates both systol i c

The regu rg itant flow ca n be ca lcu lated as follows:

and d iastolic t i m e i ntervals t o give a g lobal ventricular index of myocard ial performance.

x 6 . 2 8 x a l iasing velocity a n g l e correction factor* = ( 1 . 2 cm)2 x 6 . 2 8 x 2 9 cm per second x 2 2 0/1 80 = 3 2 1 cc per secon d

Flow rate = (r)2

The i ndex ca n be easily derived from the tricuspid va lve open i n g to clos i n g time (TC O) a n d the p u l monary va lve ejection time (PVET) as RI M P TC O - PVET/PVET

x

=

Here RI M P = 360 - 2 7 0/2 7 0 = 0 . 3 3

* U n l i ke the m itra l va lve where the leaflets a re fl at, t h e tricuspid va lve leafl ets a re fu n n e l sha ped . Th is needs to be accou nted for when ca lcu lati n g PISA. The ass u m p­ tion of a hem ispheric shape of the fl ow convergence needs to be corrected for by measu ri n g the a n g l e of the leafl ets . Typical ly, the a n g l e of the tricuspid leaf­ l ets is 2 2 0° rather than the 1 80° of the m itra l valve . Without t h i s adj u stment, t h e add iti o n a l a rea o f flow convergence wou l d not be accou nted for if the a n g l e o f the leaflets were not meas u red, w h i c h wou l d lead to a n u n d e restimate of the reg u rg itation severity. ERO

Flow rate/Pea k tricuspid reg u rg itation velocity E RO = 3 2 1 cc per second/260 cm per second 1 . 2 cm2 =

See The Echo Manual, 3rd Edition, pages 1 37 and 1 38.

ANS WER 5: C. Measures of the extent (TAPSE) and

the peak velocity (Latera l S') of longitu d i n a l right ven­ tricu lar contraction may be easily obtai ned from apica l long-axis fou r-cha mber view. Here both these measu res a re sign ificantly red uced reflecting a reduction in right ventricular systolic function. The RI M P, however, is nor­ m a l . The l i kely reason for this d iscordance is " pseudonor­ mal ization " of the R I M P caused by a relative reduction i n t h e isovolumic contraction time i n the setting o f very h i g h right atria l and right ventricular end-d iastolic pressures.

=

ANS WER 6: D. Transthoracic echocardiography dem­ ANS WER 2: E. A l l of the choices. I n com p l ete tricus­

p i d va lve leaflet coaptatio n , coro n a ry s i n u s d i latation, systol i c flow reversa ls i n the hepatic vei n s a n d a P I SA rad i u s > 0 . 9 cm2 when present i n the sett i n g of tricus­ pid va lve reg u rg itation a re a l l suggestive that the re­ g u rg itation is severe . See The Echo Manual, 3rd Edition, d iscuss io n of echo fi n d i ngs in severe tricuspid va lve reg u rg itation on pages 1 1 9 to 220, and Appendix 2 1 on page 41 4.

ANS WER 3: E. All of the choices. Alth o u g h not spe­

cific for tricusp i d va lve reg u rg itation, a d i l ated coronary s i n u s is com m o n l y a s i g n of longsta n d i n g right atrial hypertension (frequently a featu re of severe tricuspid va lve reg u rg itation). See The Echo Manual, 3rd Edition, expla nation of ca uses of enlarged coronary s i n u s on page 346.

onstrates rheumatic aortic, m itra l va lve, and tricuspid valve d isease. The aortic valve is no more than m i l d ly af­ fected and does not req u i re intervention . The m itra l valve has the classic anterior leaflet " hockey-stick " deform ity with nod u l a r leaflet ca lcification typica l ly of rheumatic i nvolvement. There is m i l d-moderate stenosis (mean grad ient 5 mm Hg) with moderate mitra l valve reg u rgi­ tation (reg u rg itant vol u me 3 5 cc) . The tricuspid valve is moderately thickened (pa rticula rly the a nterior leaflet) with a d i lated a n n u l us and severe regu rg itation and is the primary sou rce of the patient's sym ptoms. The patient should be referred for tricuspid valve replacement. The mitra l valve should be add ressed at the same time and is not amenable for repa ir. The patient u nderwent success­ fu l m itral and tricuspid valve replacements and is doing well in follow-up.

CAS E 4 8

Prog ressive NYHA Class I I a n d I l l Exert i o n a l Dyspnea and Pa l p itations

AAssociation functional class II to III exertional dyspnea and palpitations. Over the

40-year-old male patient presents for the evaluation of progressive New York Heart

past month he has developed exertional presyncope. His only medication is metopro­ lol succinate 1 00 mg two times daily. On examination, his blood pressure is 1 1 5/70 mm Hg and heart rate 5 0 beats per minute (bpm) . He has a brisk carotid upstroke and normal central venous pressure. His left ventricular apical impulse is sustained and local­ ized. He has a 2/6 systolic ejection murmur at the apex that does not change with Val­ salva with little change from squat to stand. His 1 2-lead ECG is shown in Figure 48- 1 . He is referred for transthoracic echocardiogram (see Video 48- 1) .

QUESTION 3. Peak continuous wave Doppler gradi­ ents were recorded through the left ventricular outflow tract (LVOT) at rest (see Fig. 48-2) , with Valsalva (see Fig. 48-3) , and with amyl nitrite (see Fig. 48-4) . The appropriate next step for this patient would be:

l ����__.....,..._._��-..i.���---i.---���lr"'-_.,_��,..--. l

�.-1-"-���-----_,___� F i g ure 48-1

QUESTION 1. The findings in the parasternal long­ axis evaluation are consistent with:

A. Normal examination B. Mitral valve stenosis C. Hypertrophic cardiomyopathy with systolic an­ terior motion of the mitral valve D . Hypertrophic cardiomyopathy without evi­ dence of outflow tract obstruction

QUESTION 2. The expected effect on the systolic ejec­ tion murmur with inhalation of amyl nitrite would be:

A. Increase in murmur B. No change C. Decrease in murmur

1 56

F i g ure 48-2

C A S E 48 I 1 5 7

A. No evidence of ischemia B. Evidence of anterior ischemia C. Evidence of inferior ischemia QUESTION 5. On the basis of the findings from the stress echocardiogram (see Video 48-4 and Fig. 48-5) , the next step i n management should include:

F i g u re 48-3

F i g u re 48-5

A. Refer for hemodynamic left and right heart catheterization B. Refer for alcohol septal ablation C. Refer for surgical septal myectomy D. Nothing further, medical management only

F i g u re 48-4

A. Refer for Doppler stress echocardiography B . Refer for hemodynamic left and right heart catheterization C. Refer for alcohol septal ablation D . Refer for surgical septal myectomy

QUESTION 6. The following still M-mode tracing (see Fig. 48-6) demonstrates evidence of:

QUESTION 4. The patient is referred for a treadmill

stress echocardiogram to assess functional capacity, and clinical and left ventricular hemodynamics. He ex­ ercised on a Bruce protocol for a little over 9 minutes and stopped secondary to dyspnea and lightheaded­ ness. His preexercise heart rate was 58 bpm and blood pressure 1 22/70 mm Hg. His peak heart rate was 1 60 bpm and blood pressure 1 00/70 mm Hg. The regional wall motion assessment (see Videos 48-2 and 48-3) demonstrates:

F i g u re 48-6

1 5 8 I E C H O C A R D I O G RA P H Y: A C A S E - B A S E D R E V I E W

A. Systolic anterior motion of the anterior mitral valve leaflet B. Mitral stenosis C. Posterior leaflet mitral valve prolapse

QUESTION 7. Who is responsible for describing the

echo finding in Figure 48-6 first? A. B. C. D.

Dr. Harvey Feigenbaum Dr. Jamil Tajik Dr. Pravin Shah Dr. Liv Hade

C A S E 48 I 1 5 9

ANS WER 1: D. The left ventricle is of normal size

with normal contract i l ity. There is increased septa l wa l l thicke n i n g particula rly of the basal septu m , suggestive of hypertro p h i c ca rd iomyopathy. There is no evidence of rest i n g outflow tract obstruction o r systol i c a nterior motion of the m itra l va lve at rest.

myectomy patients vs. 5 . 5 % i n a b lation pati ents) 1 -4. The patient here underwent successful surgica l myec­ tomy (Fig . 48-7).

ANS WER 2: A. Amyl n itrite is a ra p i d ly active vasod i­

lator that is often used as a d iag nostic tool to reveal a dyn a m i c LVOT obstructio n . Other m a n euvers i n c l u d e a Va lsa lva or squat to sta n d . ANS WER 3 : A. I n t h i s p a t i e n t w i t h hypertro p h i c

ca rd i o myopathy, it i s l i ke l y t h a t exert i o n a l sym p ­ tom s o f dys p n e a a re re l ated t o a dyna m i c o utfl ow tract obstructio n . If o n e can d o c u m e n t a s i g n ifi c a n t i n sta nta n e o u s g ra d i e n t ( 5 0 m m H g o r g reater) o n maxi m a l m e d i c a l t h e ra py, a septa l red u ction p roce­ d u re s u c h as a l cohol se pta l a b l at i o n o r s u rg i ca l my­ ectomy i s an a p p ro p r i ate co n s i d e ratio n . H oweve r, so fa r desp ite attem pts with the Va lsa lva m a n e uver or i n h a l e d a myl n i trite, n o outfl ow tract g ra d i e nt has been i n d uced . An exercise Doppler study i s the next a p p ro p r i ate ste p . ANS WER 4: A. Echocard i o g ra p h ic i m a g es a t rest

demonstrate normal left ventricu l a r contract i l ity with normal poststress i ncremental i ncrease in contract i l ity with decrease i n left ventric u l a r size. ANSWER 5: C. The exercise stress test re prod u ced

the patient's typ ica l sym ptoms of exertional dyspnea and l i g htheaded ness associated with system i c hypo­ tensio n . Doppler echoca rd iography was able to dem­ onstrate that this occu rred i n the sett i n g of dyna m i c outflow tract obstruction with a h i g h g ra d i ent. G iven that these fi n d i ngs occu rred on h i g h-dose B-blockers, a referra l for a septa l red uction proced u re is appropri­ ate . I n a you n g person without sign ificant comorb i d i ­ ties, a surgica l septa l myectomy is the preferred option over a lcohol septa l a b lation . With a s i m i l a r i n -hospita l morta l i ty (0 . 6 % f o r myectomy a n d 1 . 6 % f o r a b l a­ tion), a lower rate of perma nent pacemaker i m p l a nta­ tion for complete hea rt block (3 . 3 % vs. 1 8 . 4 % ) a n d a h i g her success rate (req u i red repeat proced u re 0 . 6 % i n

F i g u re 48-7

There is a Doppler s i g n a l recorded at 7 m per secon d (wh ich corresponds t o a p e a k i n sta nta neous g ra d ient of 1 96 m m Hg). It is u n clear if this all represents the dyn a m i c g rad ient or a s i g n a l conta m i n ated with m itra l reg u rg itation . H owever, even if this is the pea k m itra l reg u rg ita nt velocity, this i m p l ies that the LVOT g ra d i ­ ent is h i g h regardless. Left ve ntr i c u l a r systo l i c p ress u re eq u a l s the pea k g ra d i e n t a c ross t h e m i tra l va lve (4v2 , w h e re v i s t h e p e a k m itra l reg u rg i ta nt ve l o c i ty) p l u s a n est i m ate o f l eft atr i a l p ress u re . I n t h i s case, 4(7 ) 2 + 2 0 = 2 1 5 m m Hg. Alternatively ( i n the a bsence of aortic valve d i sease), left ventri c u l a r p ress u re a lso e q u a l s systol i c b l ood p ressu re p l u s the i ntracavita ry g ra d ient. Therefore, i n t h i s case (if the D o p p l e r s i g ­ n a l is m itra l reg u rg itation), the i ntracavita ry g rad ient = 2 1 5 - 1 00 = 1 1 5 m m H g . ANS WER 6: A . M-mode o f hypertro p h i c ca rd i omy­

opathy demonstrates the evidence of systol i c a nterior motion of the a nterior m itra l va lve leaflet. See The Echo Manual, 3rd Edition, F i g u re 2-1 9 on page 23.

ANS WER 7: C. Dr. Pravi n S h a h fi rst descri bed the

echo fi n d i n g in F i g u re 48-6 .

1 6 0 I E C H O C A R D I O G RA P H Y: A C A S E - B A S E D R E V I E W

References 1. Ommen SR, Maron BJ, Olivotto I, et al. Long-term effects of sur­ gical septal myectomy on survival in patients with obstructive hy­ pemophic cardiomyopathy. ]Am Coll Cardiol. 2005;46:470-476. 2. Qin JX, Shiota T, Lever HM, et al. Outcome of patients with hy­ pertrophic obstructive cardiomyopathy after percutaneous trans­ luminal septal myocardial ablation and septal myectomy surgery. j Am Coll Cardiol. 200 1 ;3 8 : 1 994-2000. 3. Sorajja P, Valeti U, Nishimura RA, et al. Outcome of alcohol septal ablation for obstructive hypertrophic cardiomyopathy. Cir­ culation. 2008; 1 1 8 : 1 3 1 - 1 3 9 .

4. Talrej a D R , Nishimura RA , Edwards WD, e t a l . Alcohol sep­ ta! ablation versus surgical septa! myectomy: comparison of ef­ fects on atrioventricular conduction tissue. j Am Coll Cardiol. 2004;44:2329-2332. 5 . Shah PM, Gramiak R, Kramer DH. Ultrasound localization of left ventricular outflow obstruction in hypertrophic obstructive cardiomyopathy. Circulation. 1 969;40:3-l l .

CAS E 4 9

Progressive Exert i o n a l Fat i g u e and Lower Extrem ity Edema

Aedema.

42-year-old woman presents with progressive exertional fatigue and lower extremity

Q UESTION I. On the basis of your review of the

echocardiogram (Videos 49- 1 through 49-9) , the predominant pathology present is: A. Tricuspid valve atresia B. Ebstein anomaly C. Right ventricular dysplasia with annular dila­ tion and secondary tricuspid valve regurgitation D. Carcinoid syndrome

Co ncerning this right heart abnormality, 25% of patients are prone to primary ventricular tachydysrhythmia. Q UES TION 2.

A. True B. False

F i g u re 49-1

A. B. C. D.

Papillary fibroelastoma Vegetation Blood cyst Artifact

QUESTION 3. Typical examination findings may in­

clude all of the following excep t-. A. Normal j ugular venous pulsations B. Cyanosis C. A pan-systolic murmur that increases on inspi­ ration D . A narrowly split second heart sound

QUESTION 5. All of the following congenital cardiac

lesions are associated with maternal rubella infection

excep t-. A. B. C. D.

Ebstein anomaly Tetralogy of Fallot Coarctation of the aorta Patent ductus arteriosus

QUESTION 4. The likely secondary finding on this

echocardiogram (Videos 49- 10 through 49-1 3 and Fig. 49- 1 , arrow) is:

1 61

1 6 2 I E C H O C A R D I O G RA P H Y: A C A S E - B A S E D R E V I E W

ANS WER 1 : B. Seen here is a ma rked exa m p l e of

E bste i n a n o m a ly1 cha racterized by a morpholog ica l ly a n d f u n cti o n a l ly a b n o r m a l tricuspid valve . The pos­ terior and septa l l eafl ets fa i l to d e la m i n ate from the ventric u l a r wa l l lea d i n g to a pica l displacement of the tri cusp i d a n n u l u s (F i g . 49-2) . E bste i n a n o m a ly is de­ fi ned as a d i splacement of septa l tricusp i d va lve i n ser­ tion 8 m m per m2 towa rd the ventri c u l a r a pex. T h i s l e a d s t o atria l i zation o f the r i g h t ventricle.

ated and potenti a l ly may degenerate i nto seco n d a ry ventri c u l a r dysrhyt h m ias. ANS WER 3: D. U n l i ke most other cases of severe tri­

cuspid va lve reg u rg itation, patients with E bste i n anom­ aly typica l ly have normal j u g u l a r pu lsations beca use of the com p l i a nce of the h uge right atrial e n l a rgement. Patients may be cya notic beca use of right-to-left shu nt­ i n g through either an associated atria l septa l defect or a patent foramen ova l e (one or either of which is present in 50% of cases). Typica l of rig ht-sided m u r m u rs, the m u rm u r of tricuspid va lve reg u rg itation wi l l a u g ment with the i ncreased rig ht-sided fi l l i n g on inspiratio n . Pa­ tient's with E bste i n a n o m a ly wi l l have either a widely split \ (beca use of a delay i n p u l monary va lve closu re) or a fixed spl it 5 in the setting of an atria l septa I defect. 2 ANS WER 4: C. The d ifferenti a l d i a g nosis for freely

F i g ure 49-2

ANS WER 2: B. False. E bste i n a nomaly is associated

with the p resence of a ccessory pathways a n d the Wolff-Pa rkinson-Wh ite synd rome i n 2 0 % to 2 5 % of cases. Patients a re prone to s u p raventricu l a r rhyt h m a b norma l ities mediated e i t h e r t h ro u g h the accessory pathway or thro u g h the atrial dysrhythmias ca used by atria l e n l a rgement. U n l i ke right ventri c u l a r dysplastic ca rd i o myopath ies, pri m a ry ventric u l a r dysrhyth m i a is u ncom m o n , a lthoug h atrial dysrhyth m i a , partic u l a rly if con d u cted down a n accessory pathway, is poorly toler-

Reference 1. Attenhofer Jost CH, Connolly HM, Dearani JA, et al. Ebstein's anomaly. Circulation. 2007; 1 1 5 :277-2 8 5 .

m o b i l e lesions p resent on h e a rt va lves, o r h e re at­ tached to the Eustach i a n valve, i n c l udes tumors, veg­ etati o n , t h ro m b us, a n d a rtifacts . H e re, thou g h , the t h i n -wa l l and centra l echo-free space is characteristic of a be n i g n b l ood cyst. M o re com m o n in ch i l d re n , cysts a re typica l ly fou n d on cardiac va lves a n d typ ical ly reg ress with age. They a re of l ittl e c l i n i ca l i m porta nce except for the potenti a l for the echoca rd iogra p h e r to m isdiag nose them as someth i n g more s i n i ster lea d i n g t o u n necessa ry invasive d i a g n ostic or therapeutics. ANS WER 5: A. U n l i ke the other defects l i sted , Eb­

ste i n a nomaly is not associated with maternal rubella infectio n . E bste i n a n o m a ly has been l i n ked with ma­ ternal l i th i u m use. See The Echo Manual, 3rd Edition, pages 563 to 565 for discussion of E bstein's anomaly.

CAS E 5 0

Progressive Exert i o n a l Shortness of Breath a n d Lower Extrem ity Edema

Abreath and lower extremity edema. He is now New York Heart Association functional

52-year-old man presents with a 6-month history of progressive exertional shortness of

class IV. On examination, his blood pressure is 95/50 mm Hg and he has a heart rate of 1 00 beats per minute. He has distended neck veins and three-plus bilateral pedal edema. His chest is clear. He is referred for transthoracic echocardiography (see Video 50-1) .

Q UESTION I . I n this case, all o f the following are

true excep t-. A. The absence of right atrial collapse suggests against echo evidence of tamponade physiology B. The presence of respiratory variation in mitral inflow supports echo evidence of tamponade physiology C. Right ventricular (RV) end-diastolic pressure is likely higher than mean left atrial pressure D. Hepatic vein dilation suggests an elevation in right atrial pressure E. Left ventricular ejection fraction is increased (hyperdynamic)

Q UESTION 4. Which of the following is the most specific two-dimensional (2D) echocardiographic finding for cardiac tamponade?

A. B. C. D.

RV diastolic collapse Right atrial diastolic collapse Abnormal ventricular septal motion Dilated inferior vena cava

QUESTION 5. Left ventricular diastolic filling pres­

sure based on mitral inflow is: A. B. C. D.

Normal Reduced Increased Not able to be determined

QUESTION 2. In the calculation of estimated RV sys­ tolic pressure, the correct tricuspid velocity to take is:

A. In end inspiration B. In end expiration C. An average of all consecutive values

QUESTION 3. The most likely clinical finding in this

patient is: A. B. C. D. E.

Audible fourth heart sound Sclerodactyly and esophageal dilatation Lung mass on chest x-ray Acute pulmonary embolus Muffled second heart sound

1 63

1 6 4 I E C H O C A R D I O G RA P H Y: A C A S E - B A S E D R E V I E W

ANS WER I: A. This case is one of severe p u l monary

a n d right atria l hypertension with s u peri m posed ca r­ d i a c ta m ponade. Here we see 2 D evidence of l eft atrial col l a pse but not the more usual RV or right atri a l col­ l a pse, i n d icati n g that perica rd i a ! press u re exceeds left atria l but not RV or right atrial pressure. Th is is a classic echoca rd iogra p h i c fi n d i n g seen i n ta m ponade patients with severe p u l monary hypertension i n whom there is associated severe elevation i n rig ht-sided i ntraca rd iac pressu res. Although any deg ree of pericardia! fluid present i n a pa­ tient with p u l monary hypertension is associated with a worse prognosis (reflecti ng it as a marker of elevated right atria l pressure), the occu rrence of moderate or greater degrees of pericard i a ! fluid in the setting of pul monary a rterial hypertension is m uch less com mon . Assessment of the hemodynamic sign ificance of pericardia! effusions i n the setting of p u l monary hypertension is cha l leng ing and largely rel ies on the fi n d i ngs of left-sided chamber d iastolic col lapse. Doppler stud ies from m itra l i nflow and hepatic vei n a re characteristic for ca rd iac ta m ponade with respi ratory va riation of m itra l i nflow (A velocities a re i n d icated by a rrow) a n d i n c reased d iasto l i c hepatic vei n flow with exp i ration . 1 -2

tends to be lower with h i g h er p u l monary a rtery pres­ su res. With i n s p i ratio n , flow i nto rig ht-sided cham bers increases a n d pea k tricuspid reg u rg itant velocities ten d t o modestly decl i n e . T h e s e h e m odyn a m i c c h a n ges with res p i ration ca n be m a g n ified in cases where i n ­ trathoracic pressu res swi n gs a re exaggerated such a s obstructive l u n g d isease. ANS WER 3: B. A pericard i a ! effusion may develop

i n the setting of severe p u l monary a rterial hypertension beca use of right atria l hypertension a n d/or associated co n nective tissue d isease . Althou g h the presence of even s m a l l amou nts of perica rd i a ! fl u i d i n the setting of p u l monary hypertension is associated with a worse prog nosis, sufficient fl u i d does not accumulate to have hemodyna m i c sign ificance in most cases. In the smaller proportion of patients with p u l m o n a ry hypertension who develop moderate/severe amou nts of fl u i d a l most a l l have associated scleroderma/C REST synd rome. ANS WER 4: A. When present, RV d i asto l i c co l l a pse

is most specific for ta m ponade with h i g h e r i ntra peri­ ca rd i a l pressu re than RV pressu re d u r i n g early d iasto l e . Oth er 2 D fi n d i ngs a re freq uently present n o t o n ly i n ta m ponade but a lso i n other con d itions. ANS WER 5 : B . E ve locity is m uch lower than that

ANS WER 2: B. Hemodyn a m i c measu res of RV end­

systol i c or p u l monary a rtery pressu res s h o u l d be made i n end exp i ration whether i n the echocard iogra p h i c or catheterization laboratory beca use this is the point at which i ntrath oracic pressu re is cl osest to atmospheric press u re . Typ ica l ly with expi ration, rig ht-sided i nflow

of A velocity. Resp i ratory va riation of m itra l i nflow ve­ locities is more pro n o u n ced than A velocity. Both the ta m ponade p hysiology a n d the RV dysfu nction i n the sett i n g of p u l mo n a ry hypertension i n this patient a re l i m iting flow to the left heart cham bers resu lti n g i n a g rade 1 d i asto l i c dysfunction pattern .

References 1. Eysmann SB, Palevsky HL, Reichek N, et al. Two-dimensional and Doppler-echocardiographic and cardiac catheterization cor­ relates of survival in primary pulmonary hypenension. Circulation. 1 989;80:353-360.

2. Plotnick GD, Rubin DC, Feliciano Z, et al. Pulmonary hyperten­ sion decreases the predictive accuracy of echocardiographic clues for cardiac tamponade. Chest. 1 995; 1 07:9 1 9 .

CAS E 5 1

Dyspnea with Modest Exertion

Apnea with modest exertion. She denies angina, orthopnea, or edema. She is referred 64-year-old woman without significant medical history presents with new-onset dys­

for exercise stress echocardiography.

QUESTION I. On the basis of her resting study, what is the diastolic function (see Video 5 1- 1) ?

A. B. C. D.

Normal Mildly abnormal (grade 1 ) Moderately abnormal (grade 2) Severely abnormal (grade 3)

QUESTION 2. After review of the rest (on left of quads) and stress (on right side of quads) images (see Video 5 1 -2) , the stress echocardiogram (apical 4 chamber images are in Mayo format with left ventricle on the left) is:

A. Negative for ischemia B. Positive for ischemia in the distribution of the left anterior descending artery C. Positive for ischemia in the distribution of the right coronary artery D. Positive for multivessel ischemia

QUESTION 4. Two weeks later, she presents to the emergency department after an episode of syncope. A repeat transthoracic echocardiogram is obtained (see Video 5 1 -3) . There is now evidence of:

A. B. C. D.

Significant pulmonary hypertension Severe tricuspid valve regurgitation Tricuspid stenosis Acute myocardial infarction because of occlu­ sion of the left anterior descending artery

QUESTION 5. Appropriate steps might include all of the following excep t-.

A. B. C. D.

Coronary angiography CT angiography Emergency cardiac surgery Thrombolytic therapy

QUESTION 3. The most likely etiology for this pa­

tient's dyspnea is: A. B. C. D. E.

Symptomatic coronary artery disease Pulmonary arterial hypertension Elevation in left ventricular filling pressures Mitral stenosis Pulmonary disease and/or deconditioning

1 65

1 6 6 I E C H O C A R D I O G RA P H Y: A C A S E - B A S E D R E V I E W

cates where early d i asto l i c fi l l i n g ( E ) is l ess t h a n l ate d iasto l i c fi l l i n g (A) related to d e l ayed left ventri c u l a r relaxatio n . This corresponds t o g rade 1 or m i l d ly ab­ normal d i asto l i c function .

the pea k velocity of early m itra l fi l l i n g (E) to that of the pea k early d iasto l i c velocity (e') on tissue Doppler of med i a l m itra l a n n u l u s . An E'e' ratio > 1 5 usually i n d i ­ cates elevated d iastolic left ventricular fi l l i n g pressu re) . I n the a bsence of these fi n d i ngs, dyspnea is com monly related to p u l monary d isease a n d/or decond ition i n g .

See The Echo Manual, 3rd Edition, g rad i n g of dia­ stolic function on pages 1 32 to 1 34.

ANS WER 4: B . Echo i mages i n d icate a ma rked inter­

ANS WER 1 : B. The m itra l i nflow pattern here i n d i ­

S h e exercised for 3 m i n utes 30 seco n d s on a B ruce protocol to a d o u b l e prod uct of 24, 700 and stopped beca use of dyspnea . She had no ch est pa i n , a n d her stress E C G was n o n d i a g n ostic beca use of rest i n g ST chan ges. ANS WER 2: A. Seen here is a normal left ventricu­

lar response to exercise with genera l ized i ncrease i n left ventri c u l a r myoca rd i a l contractil ity, a decl i n e i n left ventric u l a r size, a n d an i ncrease i n left ventricu l a r ejec­ tion fraction . No reg i o n a l wa l l motion a bn o rm a l ities a re see n . See The Echo Manual, 3rd Edition, Ta b l e 1 1 -3 o n page 1 78.

ANSWER 3: E. Exercise stress ech oca rd iogra p hy has

the capacity to eva l uate/screen all major ca rdiac ca uses of dyspnea . Beyond the assessment of coronary a rtery d i sease, echoca r d i o g ra p h i c eva l uation of l eft-sided va lve d i sease, p u l monary pressu res, a n d left-sided fi l l ­ i n g pressu res (suggests b y a n elevation i n t h e ratio of

va l change, with a D-shaped left ventricle, severe ri g ht ventricu l a r e n l a rgement, tricuspid a n n u l a r d i lation, a n d severe tricuspid va lve reg u rg itation t h a t a re new. F i n d ­ i n gs here suggestive o f severe tricuspid reg u rg itation i n c l u d e a severely d i lated a n n u l us with poor leafl et coa ptation, a dense conti n uous wave Doppler s i g n a l , a n d a dagger-s h a ped sig n a l . One wou l d expect t o fi n d systol i c fl ow reversa ls i n the hepatic vei ns. ANS WER 5 : B . The sudden i n te rva l change with

evi d e n ce of severe right ventri c u l a r overload i n the sett i n g of syncope a n d recent new onset of dys p n ea suggests a h i g h l i ke l i hood of p u l monary e m b o l i as the u n d e rlyi n g eti o l ogy, a s u s p i c i o n that co u l d be con­ fi rmed by C T a n g iography of the c h est . Options of thera py i n t h i s sett i n g i n c l u d e con s i d e ration of either e m e r g e n cy th rom bectomy or i ntrave n o u s t h ro m bo­ lytic therapy. 1 -5 See The Echo Manual, 3rd Edition, pages 1 49 to 1 5 1 .

References 1. Goldhaber SZ. Thrombolysis for pulmonary embolism. N Engl j Med. 2002;347: 1 1 3 1- 1 1 32. 2. Sadeghi HM, Kimura BJ, Raisinghani A, et al. Does lowering pulmonary arterial pressure eliminate severe functional tricuspid regurgitation? Insights from pulmonary thromboendarterectomy. j Am Coll Cardiol. 2004;44: 1 26- 1 32. 3. Konstantinides S, Geibel A, Heusel G, et al. Management strat­ egies and prognosis of pulmonary embolism-3 trial investiga-

tors. Heparin plus alteplase compared with heparin alone in patients with submassive pulmonary embolism. N Engl j Med. 2002;347: 1 1 43- 1 1 5 0. 4. Tapson VF . Acute pulmonary embolism. N Engl j Med. 2008;3 5 8 : 1 037.

CAS E 5 2

Fevers, M a l a ise, a n d Fatigue

A She has a medical history of autoimmune thyroid disease on hormone replacement 33-year-old woman presents with a 2-month history of fevers, malaise, and fatigue.

with a normal sensitive thyroid-stimulating hormone. Examination is normal. Twelve­ lead ECG demonstrates T-wave inversion in her precordial leads (Fig. 52- 1 ) , and she is referred for echocardiogram (Video 52- 1) .

A. B. c. D.

33% 40% 60% 63%

QUESTION 2. Calculate the mitral valve regurgitant

volume (Figs. 52-3 and 52-4) .

F i g u re 52-1

QUESTION 1. Calculate the left ventricular (LV) frac­

tional shortening (FS) (Fig. 52-2) .

F i g u re 52-3

F i g u re 52-2

1 67

1 6 8 I E C H O C A R D I O G RA P H Y: A C A S E - B A S E D R E V I E W

QUESTION 4. The echocardiographic findings would

be most compatible with abnormalities on which of the following tests? A. B. C. D. E.

F i g ure 52-4

A. B. c. D. E.

15 21 29 40 45

cc cc cc cc cc

QUESTION 3. When the aliasing velocity is 30 cm per second for PISA calculation, which of the follow­ ing PISA radii provides an effective regurgitant ori­ fice (ERO) of 0.4 cm2 assuming mitral regurgitation (MR) velocity of 5 m per second?

A. B. C. D.

0 . 5 cm 0.75 cm 1 . 0 cm 1 . 5 cm

Serum protein electrophoresis Serum white blood cell count Urinary 5-HIAA excretion Fasting transferrin saturation Antinuclear antibody and anticentromere anti­ bodies

CASE 52 I 1 69

ANS WER 1: B. 40 % . FS is the percentage change i n

See The Echo Manual, 3rd Edition, PISA equation on page 2 1 5.

LV d i mensions with each LV contraction . LVE D - LVES LVE D

ANS WER 3: C. When the a l iasing velocity is 30 cm

where LVE D is the LV e n d - d i asto l i c d i m e n s i o n a n d LVE S i s t h e LV e n d-systo l i c d i mension . Normal FS for men is 2 5 % to 43 % a n d for women is 2 7 % to 45 % .

per second, the flow rate ca n be ca lculated as 6 . 2 8 x 30 x rad i u s . If the flow rate is d ivided by the M R veloc­ ity of 500 cm per second, it g ives an ERO that is 0 . 4 x rad i u s . Therefore, the ra d i us of 1 cm g ives ERO of 0 . 4 cm2.

Here LVE D = 43 m m a n d LVES = 2 6 m m . Therefore, FS = (43-2 6)/43 x 1 00 % = 40 % .

ANS WER 4: B. T h e echoca rd i o g ra p h i c f i n d i n g s

Fs =

x

1 00 0110 '

See The Echo Manual, 3rd Edition, page 1 1 5.

ANS WER 2: B. 2 1 cc.

The reg u rg ita nt flow ca n be ca l c u l ated as fol l ows : Flow = (r)2 x 6 . 2 8 x a l i as i n g velocity = (0 . 6 cm)2 x 6 . 2 8 x 30 cm per second = 68 cc per secon d ERO = Fl ow/Pea k M R velocity E RO = 68 cc per secon d/48 5 cm per secon d = 0 . 1 4 cm2 x reg u rg ita nt time velocity i nteg ra l = 0 . 1 4 cm2 x 1 5 0 cm = 2 1 cc

Reg u rg ita nt vol u m e = E RO

Reference 1. Ommen SR, Seward JB, Tajik AJ . Clinical and echocardio­ graphic features of hypereosinophilic syndromes. Am J Cardiol. 2000;86: 1 1 0- 1 1 3 .

here demonstrate biventric u l a r endoca rd i a l thicke n i n g with t h rombotic-fi b rotic o b l iteration o f t h e ventricu­ l a r a p i ces, a n d myoca rd i a l contrast i m a g i n g reveals cha racteristic a pical throm bus. These fi n d i ngs a re char­ a cteristic of hypereosi n o p h i l i c heart d isease . Patients typica l ly have m a rked ly elevated eosi nop h i l cou nts for 6 or more months. 1 See The Echo Manual, 3rd Edition, hypereosinoph ilic synd rome and Figure 1 6-1 7 on page 284.

CAS E 5 3

Dyspnea and Asym metrica l Ground-g lass Pulmonary I nfi ltrates

A and Figs. 53- 1 to 5 3- 1 0) . He has been hospitalized o n three occasions over the past 72-year-old man is referred for transthoracic echocardiogram (Videos 53- 1 to 53-1 6

5 months for dyspnea and asymmetrical ground-glass pulmonary infiltrates of unclear etiology. Two months ago, he underwent an adenosine sestamibi perfusion study that was normal. His history is significant for a mitral valve bioprosthesis for mitral valve regurgita­ tion and one vessel bypass graft 9 years ago.

•c

LVOT V rrui x 0 8 n� l VOT TVI 20 cm LVOT di.1ml?'tcn 7.5 c m

. . . . .. '\

,•

..

1 70

I

..

-:ti,

,

,,

.f

'

y

•1

- 0 '

I

..

, ..

j'

,/

�·

. . � .. . ..

,

�·

- .:i '

\

I 0 .:

F i g ure 53-1

F i g u re 53-3

F i g ure 53-2

F i g u re 53-4

r· 20 15 10 5

F i g u re 66-2

207

2 0 8 I E C H O C A R D I O G RA P H Y: A C A S E - B A S E D R E V I E W

QUESTION 4. Which of the following statements re­

garding LV systolic function is/are correct (Figs. 66-3 to 66-5 and Videos 66- 1 to 66-5) ?

F i g u re 66-5

F i g ure 66-3

A. Using the apical correction factor, the calcu­ lated left ventricular ejection fraction (LVEF) is 64% B. The Doppler-derived cardiac output is 3 . 1 L per minute C. There are no discrete regional wall motion abnormalities D. All of the choices QUESTION 5. Which of the following medications

would most likely improve the patient's symptoms? A. B. C. D. F i g u re 66-4

Isosorbide mononitrate Metoprolol succinate Digoxin Lisinopril

CASE 66 I 209

where u LVEF is u ncorrected LVE F . ANS WER 1: C. The u nderlyi ng rhyth m of this patient

is atri a l fl utter. Atri a l fl utter is generating A velocity of 2 0 cm per secon d (or 0 . 2 m per second) j ust before the Q R S . It may be better to des i g n ate the velocity as F velocity, but is a n a logous to A velocity i n s i n u s rhyth m . There i s a n other velocity (0 . 5 m per second) between E and A (or F) ve locities, and it is not related to fl ut­ ter wave beca use A velocity prod uced by fl utter was m uch lower. The velocity is " L wave " that is related to delayed myoca rd i a l relaxatio n .

Here, U ncorrected LVE F = [(43)2 - (2 8)2/(43)2] x 1 00 = 5 8 % Corrected LVE F = 58 + [( 1 00 - 5 8 ) x 1 5 % ] = 64% See The Echo Manual, 3rd Edition, page 1 09.

Stroke vol u m e (SV) is ca lcu lated as the prod uct of the cross-sectional a rea of the l eft ventricular outflow tract (LVOT) a n d the t i m e velocity i nteg ra l (TVI) of a p u lse wave sa m ple from the LVOT.

delayed .

SV (ml) = [(D/2)2 x n] x [LVOT TVI] = [ ( 1 . 8/2)2 x 7t] x [22] = 5 6 ml

ANS WER 3: A. The fact that E velocity is h i g h ( 1 .4 m

C a rdiac output is eq u a l to the product of SV a n d heart rate = 56 x 56 = 3 . 1 L per m i n ute

ANS WER 2 : A. Myoca rd i a l re l axat i o n is m a rked ly

per second) with a l ittl e va ria b i l ity from beat to beat i n the setti n g of i rreg u l a r rhythm a n d deceleration time is short i n d i cates that fi l l i n g pressu re is elevated i n this patient. That is a lso su pported by the fi n d i n g that E/e' ratio is >20 beca use e' velocity va ries from 0 . 5 to 0 . 7 c m p e r seco n d . T h e velocity pea k r i g ht after e' vel oc­ ity of m itra l a n n u l us is a n a logous to L wave in m itra l i nflow fol l owed by a velocity generated by fl utter. E/e' ratio ca n be used for patients with atri a l fi b r i l lation or si n us tachyca rd ia with fusion of E a n d A velocities for estimation of LV fi l l i n g press u re . ANS WER 4: D . I n t h e a bsence o f LV reg i o n a l wa l l

motion a b n o r m a l ities, t h e LV d i mensions m ea s u red from the level of the papi l l a ry m u scles can be used to ca lculate the LVE F as fol l ows: U n corrected LVE F = [(EDd)2 - (ESd)2/(E Dd)2] x 1 00 C orrected LVE F = u LVEF + [( 1 00 - u LVEF) x 1 5 % ] ,

See The Echo Manual, 3rd Edition, F i g u re 4- 1 6 on page 7 1 .

The a p pa rent d isco n nect between a low card iac out­ put a n d a preserved LVE F is not an u ncom m o n fi n d i n g i n patients with a restrictive card iomyopathy. ANS WER 5: A. This patient has d iasto l i c heart fa i l ­

u re with preserved ejection fraction . H i s LV fi l l i n g a n d S V a re relatively fixed, s o t h a t l ower heart rate a n d afterload red u ction m a y decrease ca rd iac output a n d blood press u re, respectively. Because i n creased LV fi l l ­ i n g press u re i s m ost l i kely respo n s i b l e fo r dyspnea, the pati ent's sym ptoms wi l l i m p rove with a p reload red ucer such as isosorbide monon itrate .

CAS E 67

Transient Loss of M otor Fu nction i n R i g ht Upper Extrem ity

A bubble study, following a transient loss of motor function in his right upper extrem­ 46-year-old man is referred for a transthoracic echocardiogram with agitated saline

ity. One week previously, he underwent surgical resection of a suppurative appendicitis and is currently receiving intravenous antibiotics through a peripherally inserted central (PIC) venous catheter. Apart from the transient neurologic event, which has entirely re­ solved, he feels well without any systemic symptoms. He has no cardiovascular history. Examination is otherwise entirely normal (see Video 67- 1) .

QUES TION 1 . What is the structure identified by the arrow (Fig. 67- 1 ) ?

QUESTION 2. The appropriate next step in this pa­ tient's management is:

A. Referral for closure of patent foramen ovale or atrial septal defect B . Referral to vascular radiology C. Transesophageal echocardiogram to exclude thrombus/vegetation QUESTION 3. What is the mobile structure in the

right atrium? A. B. C. D. F i g ure 67- 1

A. B. C. D. E.

210

Descending thoracic aorta Coronary sinus Esophagus Left pulmonary artery Left inferior pulmonary vein

Thrombus Eustachian valve Ventricular pacing lead Myxoma

QUESTION 4. How would you estimate left ventricu­

lar ejection fraction? A. B. c. D.

35% 50% 65% 75%

CASE 67 I 2 1 1

ANS WER 1: A. The ro u n d structu re posterior to

the left atri u m a n d outside the perica rd i u m is the de­ scen d i n g thoracic aorta that is of n o r m a l d i mensi o n . T h e coro n a ry s i n us is a s m a l l e r structu re present i n the posterior atrioventri c u l a r g roove p resent with i n the perica rd i u m .

confirmed that the tip was located i n the m i d -ascend­ i n g thoracic aorta (F i g . 67-3) . I ntra-arterial p lacement occu rred in the right a ntec u b ita l fossa where the tip had been p l a ced t h ro u g h the b rach i a ! ve i n i nto the brac h i a ! a rtery.

See The Echo Manual, 3rd Edition, page 8 a n d Fig u re 2-4 on page 1 1 .

ANS WER 2: B. The resting tra nsthoracic ech oca rd io­

g ra m is norm a l . With a d m i n istration of ag itated sa l i n e, b u b b l es a re fi rst visu a l ized in the l eft ventricle, then after a p p roxi m ately five to seven card iac cycles b u b b l es a re seen i n the rig ht-si d ed c h a m bers. Preced i n g the presence of b u b b les i n the left ventricle, the descend­ i n g thoracic aorta (Fi g . 67-2 , arrow) is seen to opacify.

F i g u re 67-3

See The Echo Manual, 3rd Edition, the role of echo­ ca rd iog raphy i n the eva l uation of cardiac sou rce of embolus on page 396. F i g u re 67-2

The o n ly plausible exp la nation for these fi n d i ngs is that the PIC catheter has been m isp laced with the tip lyi n g i n the a rterial system . With ag itated sa l i ne i ntravenous ad m i n istrati on, the thoracic aorta opacifies fi rst a n d re­ lated to m i l d aortic va lve reg u rg itation a sma l l amount of sa l i n e refl uxes i nto the left ventric l e . G iven th ese fi n d i ngs, the a p p ro p r i ate n ext ste ps wou l d i n c l u d e cessati o n o f the u s e o f the catheter, assessment of catheter location by vasc u l a r rad i o l ogy, a n d remova l . Iod i n ated contrast i njection t h ro u g h the PIC catheter

ANS WER 3: B. In feta l l ife, the eustach i a n va lve

h e l ps d i rect the flow of oxygenated bl ood from the i n ­ ferior vena cava across t h e fora men ova le i n utero. Al­ tho u g h the eustach i a n va lve rem a i n s after b i rth, it has no specific function and reg resses to va ryi n g deg rees . By a d u lthood , its typ ica l appeara nce is a crescentic fold of tissue en doca rd i u m arising from the a nterior rim of the orifice of the i nferior vena cava . H owever, i n some patie nts (as i n this case), the eustach i a n valve persists as a h i g h ly mobile, elongated th i n structu re projecting severa l centi meters i nto the r i g ht atri a l cavity. ANS WER 4: B. 6 5 % .

CAS E 6 8

Cold Left Foot

A room where she undergoes embolectomy. You are asked to perform a transesophageal

60-year-old woman presents with a cold left foot. She is brought to the operating

echocardiogram.

QUESTION 1. Which left ventricular wall segment is highlighted (Video 68- 1 and Fig. 68- 1 ) ?

QUESTION 3. The left-sided chambers, valves, and left atrial appendage were all normal. Review Videos 68-4 and 68-5. Is the foramen ovale patent?

A. Yes B. No QUESTION 4. The next appropriate step is:

A. B. C. D.

Aspirin Clopidogrel Warfarin Referral for a percutaneous closure of the fora­ men ovale E. Further imaging

F i g ure 68-1

A. B. C. D.

Mid Mid Mid Mid

anterior wall anteroseptal wall inferior wall inferolateral wall

QUESTION 5. On the basis of the images updated

from the descending thoracic aorta (Videos 68-6 to 68-8) , what is the appropriate next step? A. Systemic anticoagulation with warfarin B. Prednisone C. Foramen ovale closure

QUESTION 2. Patients with this finding on echocar­

QUESTION 6. Which of the following is not associ­

diography (Videos 68-2 and 68-3) are also likely to have:

ated with an increased rate of atherosclerotic plaque in the thoracic aorta in patients older than 55 years?

A. B. C. D. E.

212

An atrial septal defect A patent foramen ovale and higher risk of stroke A patent ductus arteriosus A persistent left superior vena cava Coarctation of the aorta

A. B. C. D. E.

Age Smoking history Systolic blood pressure Sex Location

CASE 68 I 2 1 3

ANS WER I: C. M i d i nferior wa l l . Va rious orienta­

ANS WER 4: E. An i nteg ra l part of the tra nsesopha­

tions may be used on tra nsgastric tra n seso p h a g e a l i m a g i n g w i t h o u t a ny clear sta n d a rd . H e re the right ventricle is shown on the left of the screen with the left ventric u l a r septu m between 7 o'clock a n d 1 1 o'clock. The seg ment i l l u strated i n the fi g u re between 1 1 a n d 1 o'clock is fla n ked b y t h e l iver a n d is t h e i nferior wa l l o f the left ventricle.

geal exa m i nation for eva l uation sou rce of embolus is a ca reful u ltraso u n d exa m i nation of the thoracic aorta .

ANS WER 2 : B . An atri a l septa l a n e u rysm is defi ned

as a h i g h ly mobile atri a l septum that has a maxi m u m t o m i n i m u m exc u rsion d ista nce o f 1 5 m m . An atri a l septa l a n e u rysm is present i n u p t o 3 % t o 4 % o f pa­ tients referred for tra nsesophageal ech oca rd iogra m to assess for a n e m b o l i c sou rce. Atrial septa l a n e u rysms a re a lso associated with cerebra l embolic events . The reason for this is u n clear but may repeat to the fre­ q u ent conco m ita nt pate nt fora m e n ova l e ( 5 0 % to 7 5 % ) . The ca use a n d effect re lationsh i p of a patent fora men ova le in the setti ng of an atria l septa l a n e u ­ rysm a n d a cryptoge n i c embolic event rem a i n s u nclear. ANS WER 3: A. Yes . Color Doppler demonstrates a

b i d i rectional s h u nt across the fora men ova le. Ag itated sa l i n e demonstrated a modest a m o u nt of rig ht- l eft sh u nt i n g on Va lsa lva release.

ANS WER 5: A. The tra nsesophageal demonstrated

foca l a reas of com p l ex atherosc l e rosis with one no­ ta ble h i g h ly mobile seg ment of atherosclerotic throm­ botic plaque. There a re no echo fi n d i n g s that suggest aortitis. Appropriate thera py s h o u l d i nc l u d e consider­ ation of wa rfa rin a nticoa g u latio n . S u rg ica l remova l of the foca l aortic debris s h o u l d be considered in patients with recu rrent em bol ic events . ANS WER 6: D . I n patients o l d e r than 5 5 yea rs, sex

is n ot p a rtic u l a rly associ ated with the preva l e n ce of atheroscl e rotic p l a q u e in the thoracic aorta . Fa ctors associated with risk i n c l u d e adva n c i n g age, s m o k i n g h i story, a n d i ncreasing systol i c blood pressure. P l a q u e is m uch more p reva lent i n the a rch a n d d esce n d i n g aorta t h a n i n t h e ascend i n g aorta . See The Echo Manual, 3rd Edition, F i g u re 1 9-3 on page 324.

CAS E 6 9

Chest Pa i n a n d ECG Changes

A brought emergently to the catheterization laboratory. An echocardiogram was per­

40-year-old man nonsmoker presents with chest pain and ECG changes. He was

formed.

QUESTION I. His echocardiogram (Videos 69- 1 to 69-12 and Figs. 69- 1 to 69-5) is consistent with:

F i g ure 69-3

F i g ure 69-1

"

M 1 t ra l i nfl ow

.'



v

I

t

1,

� '. 1 0 mm Mitral valve versus aortic valve endocarditis Patients taken chronic aspirin at diagnosis

QUESTION 6. The patient makes slow and steady im­

provement from a neurologic perspective on appropri­ ate antibiotic therapy. One week later, a transthoracic echocardiogram is repeated (Video 85-1 8 and 85-19) . A further 2 weeks later, all neurologic symptoms have resolved, but now he develops worsening dyspnea, and therefore a repeat echocardiogram is performed (Videos 85-20 to 85-24 and Fig. 85-2) .There is now evidence of

F i g u re 85-2

A. Mitral valve posterior leaflet prolapse with significant valvular regurgitation B. Malcoaptation of the mitral valve leaflets secondary to the mobile vegetation C. Anterior mitral leaflet perforation D. Posterior mitral leaflet perforation E. No significant change

Q UESTION 7. Assume a mitral regurgitant peak velocity of 5 m per second. What is the calculated effective regurgitant orifice?

A. B. c. D. E.

0.25 0.34 0.39 0 .44 0.52

C A S E 85 I 2 7 5

ANS WER 1: C. There i s evi dence of general ized re­ d u ction in left ve ntri c u l a r contracti l i ty with g lo b a l hy­ poki n esis worst i n the i nferior wa l l . The est i m ated LVE F is 40 % t o 45 % . On the b a s i s o f the two-d i me n s i o n a l g u i d e d M - m ode o f the LVE F at the pa p i l l a ry m uscle l eve l , the e n d d iasto l i c left ventri c u l a r d i mension i s 60 mm and the end systo l i c d i mension 45 m m . LVE F =

602 - 452

= 44 %

602 ANS WER 2: A. With d o b uta m i n e stress, t h e re is mod est g l o ba l i m p rove m e n t in contracti l ity without evi dence of ische m i a . ANS WER 3 : C. T h e patient h a s bactere m i a with a n orga n ism that h a s a h i g h p ropensity for causi n g left­ s i d e d e n d oca rd it i s . I n a d d it i o n , t h e re is c l i n ica l a n d rad i o g ra p h i c evi dence s u g g estive o f i n fective e m b o l i c events t o the b ra i n . T h e s u s p i c i o n fo r 5 . aureus i nfec­ tive endoca rd itis is very h i g h . Alth o u g h a tra nsth oracic echoca rd iog ra m wou l d be a reaso n a b l e i n itial i nvesti­ gati o n , a neg ative tra n sth oracic study wo u l d n ot be suffi c i e n t to exc l u d e the c l i n i ca l d i a g n os i s g iven the high c l i n ical p retest p roba b i l ity. ANS WER 4: C. On tra nseso p h a g e a l ech oca rd i og­ ra p hy, there i s evi d e nce of l a rge vegetation o n the poste rior m itra l leafl et i nvo lvi n g the middle a n d l ate ra l sca l l ops a n d exte n d i n g to the atr i a l s u rface . The tri­ cuspid a n d aortic va lves (a n d p u l m o n a ry-not shown) h ave no ech oca rd i ogra p h i c featu res of endocard itis. ANS WER 5: F. E m b o l i zation ca n be a d evastati n g com p l i cation i n i nfective e n d oca rd itis a n d ca n occ u r l o n g afte r a p p ropri ate a nt i m icro b i a l therapy h a s been i n stituted . E m bo l i zation can be a featu re of both left­ a n d rig ht-si d ed e n doca rd itis; h owever; eve nts that a re

c l i n ical ly s i g n ificant a re m u c h more com m o n from l eft­ sided i nfection a n d occ u r i n 1 0 % to 40 % of patients with l eft-si d ed endoca rd itis. C l i n ical cha racte ristics that i m pa rt a h i g h e r risk of embol izati on i n c l u d e the bacte­ r i a l pathogens such as S. aureus a n d Staphylococcus bovis a n d f u n g a l pathog e n s . The risk ten d s to fa l l with the d u ration of a p p ropri ate a nti m i crob i a l thera py. Pa­ tie nts p reviously o n a s p i r i n that is conti n u ed te n d to have a lowe r risk of eve nts; howeve r, the i n stitution of asp i r i n is not adva nta geous a n d may in fact i ncrease the risk. Ech oca rd i ogra p h ical ly, the b i g g e r the risk, the h i g h e r the risk with the m itra l lesions a h i g h e r risk that aortic lesions. There is some evi dence that vegetation on the a nte rior m itra l va lve l eaflet have a h i g h e r em­ bol i c risk t h a n those on the poste rior m itra l va lve . ANS WER 6: D. There is n ew seve re m itra l reg u rg ita­ tion d i rected a nteriorly. C l ose- u p vi ews demonstrate a perforation of the poste rior l eafl et (Videos 85-2 1 and 85-22) . See The Echo Manual, 3rd Edition, d iscuss ion of com pl ications of endoca rditis on pages 246 to 248.

ANS WER 7: D. ERO =

E RO =

6.28

x

r2

x

a l i asi n g ve locity

Pea k M R ve locity 6.28

x

0 . 92 500

x

43

0 . 44 cm2

An E RO >0.4 i n d i cates m itral reg u rgitation in t h e severe ra n g e . See The Echo Manual, 3rd Edition, discussion o f PISA method on page 2 1 5.

The pati ent p roceed ed to the operati n g room where a successf u l m itra l va lve rep l a cement was pe rformed and h e did we l l .

References 1. Bayer AS, Bolger AF, Taubert KA, et al. Diagnosis and manage­ ment of infective endocarditis and its complications. Circulation. 1 998;98 :2936-2948 .

4 . Thuny F, Di Salvo G , Belliard 0 , et al. Risk of embolism and death in infective endocarditis: prognostic value of echocardiography: a prospective multicenter study. Circulation. 2005; 1 1 2:69-75.

2. Rohmann S, Erbe! R , Gorge G, e t al. Clinical relevance of vegeta­ tion localization by transoesophageal echocardiography in infec­ tive endocarditis. Eur Heart]. 1 992; 1 3 :446-452.

5 . Anavekar NS, Tleyjeh IM, Anavekar NS, et al. Impact of prior antiplatelet therapy on risk of embolism in infective endocarditis. Clin Infect Dis. 2007;44: 1 1 80- 1 1 86.

3 . Sanfilippo AJ, Picard MH, Newell JB, et al. Echocardiographic assessment of patients with infectious endocarditis: prediction of risk for complications. ] Am Coll Cardiol. 1 99 1 ; 1 8 : 1 1 9 1- 1 1 99.

CAS E 8 6

Recu rrent Episodes of Left Heart Fa i l u re

A tory of aortic and mitral valve replacements for rheumatic disease. She presents now 5 5-year-old woman is referred for a transthoracic echocardiogram. She has a prior his­

with recurrent episodes of left heart failure.

QUESTION I.

Video 86- 1 demonstrates which of the

following: A. Left pleural effusion B. Right pleural effusion C. Posterior pericardial effusion

QUESTION 2. The transmitral prosthetic valve dia­ stolic mean gradient (Videos 86-4 to 86-8 and Figs. 86- 1 to 86-3) is consistent with: F i g u re 86-2

F i g u re 86-1

F i g u re 86-3

276

C A S E 86 I 2 7 7

A. Normal mitral bioprosthetic function B. Significant prosthetic dysfunction likely related to leaflet stenosis or obstruction C. Significant prosthetic dysfunction likely related to regurgitation

QUESTION 3. What is the significance of the finding

in Figure 86-3 ? A. B. C. D.

Pulmonary hypertension Severe tricuspid valve regurgitation Tricuspid valve stenosis Normal finding F i g u re 86-4

QUESTION 4. The next step in the assessment of this patient's dyspnea should include:

A. Hemodynamic catheterization B. Cardiac magnetic resonance scan C. Transesophageal echocardiogram QUESTION 5. Following review of Videos 86-9 to 86- 13, the mitral valve prosthetic gradient appears

related to: A. Normal mitral prosthetic function B. Significant mitral valvular regurgitation C. Significant mitral valvular obstruction F i g u re 86-5

Based on Video 86- 1 4 and Figures 86-4 and 86-5 , calculate the periprosthetic ef­ fective regurgitant orifice (ERO) by the PISA method. Q UES TION 6.

A. B. C. D.

0.3 0.4 0.5 0.6

cm2 cm2 cm2 cm2

QUESTION 7. The location of the periprosthetic leak is:

A. Medial B . Lateral

2 7 8 I E C H O C A R D I O G RA P H Y: A C A S E - B A S E D R E V I E W

QUESTION 8. The lesion identified by the arrow in

QUESTION 1 0. The likely success rate of percutane­

Video 86- 1 5 is most likely:

ous repair of the perivalvular leak is:

A. B. C. D. E.

Vegetation Suture material A papillary fibroelastoma An atrial myxoma Artifact

QUESTION 9. The finding in Video

A. B. C. D.

86- 1 6 is associ­

ated with a higher risk of: A. Ventricular septa! defect B . Anomalous pulmonary venous drainage of the right upper pulmonary vein C. A patent foramen ovale (PFO) and a higher risk of stroke D. An atrial septal defect E. Patent ductus arteriosus

20% to 40% 40% to 60% 60% to 80% >90%

CASE 86 I 279

ANS WER 1: A. Left-s i d e d p l e u ra l effu s i o n s a re seen poste r i o r to t h e hea rt o n l eft pa raste r n a l l o n g -axis i m a g i n g . T h ey a p p e a r a s a n e c h o-free s p a c e t h a t ca n be d i sti n g u i s h e d f r o m p e r i ca rd i a I fl u i d by ( 1 ) t h e p resence (a lth o u g h n ot a l ways p resent) of atel ectat i c l u n g a n d ( 2 ) i t s locat i o n , t h a t i s , poste r i o r (V i d e o 86-2) rat h e r t h a n a nte r i o r (Video 86-3) to t h e de­ scen d i n g thoracic a o rta ( i n d i cated by ye l l ow tri a n g l e i n cl i ps) .

ANS WER 6: B . The reg u rg ita nt flow ca n b e ca l c u ­ lated as fol l ows : 6.28

x

a l i a s i n g ve locity

= (0 . 9 1 cm)2

x

6.28

Flow rate = (r)2

x

x

46 cm per seco n d

= 2 3 9 c c per secon d ERO = F l ow rate/pea k M R ve locity = 2 3 9 cc per secon d/580 cm per secon d = 0 . 4 1 cm2

ANS WER 2: C. There is evi d e n ce of i n c reased flow ve locities across the m itra l va lve with a n early (E) peak ve locity of 2 m per seco n d . I n creased flow ve locities may be re l ated to flow acce l e ration re l ated to va lve obstruct i o n , or i n creased q u a ntity of flow re l ated to va lvu l a r reg u rg itation . A usefu l i n d i cator to h e l p sepa­ rate these mech a n isms i s the p ressu re h a l fti m e . The p ressu re ha lfti me is prolon ged in valvu l a r obstruction but not i n reg u rg itati o n . H e re a p ressu re h a lft i m e of 8 5 m i l l iseco n d s s u g g ests there i s n o l eaflet obstruc­ tion but rather a degree of m itra l va lve reg u rg itation that is n ot a p p reciated on color Doppler i nterrog ation of the va lve . This i s l i kely re l ated to echo shadowi n g m e d i ated b y the p rosthetic material o f the aortic a n d m itra l p rostheses. See The Echo Manual, 3rd Edition, pages 2 2 9 a n d 2 30 .

See The Echo Manual, 3rd Edition, discussion of PISA method on page 2 1 5.

ANS WER 7: B. Tra n seso p h a g e a l ech oca rd i o g ra m d e m o n strates a n isol ated l atera l periva lv u l a r d efect . The defect seen at fo u r degrees i n the m i deso p h a g u s is bes i d e the i nferol atera l aspect o f the sew i n g ri n g . ANSWER 8: B. T h e lesion i dentified is i n t h e typ ical location for a s u rg i ca l sutu re pl aced at the time of va lve i n serti o n . C o m m o n l y m is i nterp reted as a path o l o g i c fi n d i n g , this is one o f a series o f i n dependent s u rg ica l sutu re kn ots p resent c i rcu mferentia l ly a ro u n d the a n ­ n u l a r ri ng (Video 86-1 7) . Th ree-d i mensional (3 D) echo­ ca rd i o g raphy h i g h l i g hts this wel l -corre l ati n g wel l with the fi n d i ngs p resent at the time of s u rgery (Fig . 86-6) .

ANS WER 3: A. The pea k tricusp i d reg u rg ita nt veloc­ ity at 3 . 8 m per second (p ressu re g ra d i ent 5 8 mm Hg) is co nsistent with s i g n ifica nt p u l mo n a ry hyperte nsion that is p res u m a b ly d u e to, a n d further u n d e rscores, the s i g n ificance of the m itra l va lvu l a r dysfu nction .

ANSWER 4: C. T h e tra nsth o r a c i c e c h oca rd i o g r a m s u g g ests m itra l p rost h e s i s dysfu n ct i o n , m o re l i ke l y re l ated t o valvu l a r reg u rg itation a n d seco n d a ry p u l ­ m o n a ry hypertensi o n . T h e best test i n t h i s sett i n g to bette r c h a ra cte rize m itra l va lvu l a r f u n ct i o n/dysf u n c­ tion is a tra nseso p h a g e a l ech oca rd i o g ra m .

ANS WER 5: B. Tra nseso p h a g e a l e c h oca rd i o g r a m d e m o n strates n o r m a l p rosthetic leaflet m o b i l ity b u t partia l d e h i scence with peri p rosthetic reg u rg itation .

F i g u re 86-6

2 8 0 I E C H O C A R D I O G RA P H Y: A C A S E - B A S E D R E V I E W

ANS WER 9: C. Approxi mately 20% of patie nts with atri a l septa l a n e u rys ms h ave a PFO. Therefore, if one i d entifies a n atr i a l septa l a n e u rysm , it is i m porta nt to ca refu l ly look for PFO, p a rti c u l a rly if the pati ent has suffered a n e u ro l o g i de m b o l i c eve nt.

Reference 1. Sorajja P, Cabalka AK, Hagler DJ, et al. Percutaneous repair of paravalvular prosthetic regurgitation: acute and 30-day outcomes in 1 1 5 patients. Gire Cardiovasc lnterv. 20 1 1 ;4(4) : 3 1 4-32 1 .

ANSWER 10: C. Tra n seso p h a g e a l echoca rd i o g r a m d e m o n strates a n isol ated l atera l periva lv u l a r d efect . Th i s is a defect that a p pears q u ite a p p roacha b l e per­ cuta neously a n d has a h i g h l i ke l i hood of successfu l re­ pa i r. 1 Tra nsesophagea l echoca rd iog raphy p l ays a critica l ro l e i n g u i d i n g percuta neous clos u re of m itra l periva l ­ v u l a r lea ks, with 3 D i m a g i n g p l ayi n g a n i m po rta nt ro le i n defi n i n g the defect size, location, a n d relationsh i p to adj o i n i n g structu res i n c l u d i n g the va lve leafl ets .

CAS E 8 7

Exertional Shortness of Breath i n a 68-Year-Old Woman

A of breath (Videos 87- 1 to 87-6 and Figs. 87- 1 to 87-4) .

68-year-old woman is referred for an echocardiogram to evaluate exertional shortness

F i g u re 87-1

F i g u re 87-3

F i g u re 87-2

F i g u re 87-4

281

2 8 2 I E C H O C A R D I O G RA P H Y: A C A S E - B A S E D R E V I E W

QUESTION I. The mechanism of the mitral valve dis­

ease is likely: A. Rheumatic disease B. Leaflet prolapse C. Functional (ischemia)

QUESTION 2. As the aliasing velocity was set about

40 cm per second, what simple equation can be used to calculate the effective regurgitant orifice (ERO) with respect to mitral valve regurgitation? A. B. C. D.

ERO ERO ERO ERO

= = = =

?

?/2 r/2 2?

F i g u re 87-6

QUESTION 3. A TEE confirmed moderate to severe mitral valve regurgitation and mild to moderate ste­ nosis. In the setting of functional class III shortness of breath, the patient was referred for mitral valve replacement and did well. Postoperatively, she was referred for transthoracic echocardiography (Videos 87-7 and 87-8, and Figs. 87-5 and 87-6) . Which of the flowing factors will least impact the interpreta­ tion of the Doppler interrogation of the mitral valve prosthesis?

F i g u re 87-5

A. B. C. D.

Heart rate Gender Systemic blood pressure Left ventricular outflow tract (LVOT) time velocity integral

C A S E 87 I 2 8 3

QUESTION 4. Two years later, she presents for repeat

QUESTION 5. Which o f the following management

echocardiographic evaluation. She now describes func­ tional class III shortness of breath. A diastolic murmur is heard. She is referred for transthoracic echocardiography (Videos 87-9 to 87-13 and Fig. 87-7) and subsequently transesophageal echocardiography (Videos 87-14 and 87-15) . What is your recommended treatment?

strategies is appropriate to prevent a bioprosthetic thrombosis?

F i g u re 87-7

A. Intravenous heparin and warfarin to a goal INR of 2 to 3 B . Intravenous thrombolytics C. Emergency surgery

A. Mitral tissue prosthesis, sinus rhythm, left ven­ tricular ejection fraction (LVEF) 3 5%-aspirin B. Mitral tissue prosthesis, atrial fibrillation, LVEF 60%-aspirin and warfarin C. Mitral tissue prosthesis, sinus rhythm, LVEF 75 %-no therapy required

2 84 I E C H O C A R D I O G RA P H Y: A C A S E - B A S E D R E V I E W

ANS WER 1 : A. Tra n s t h o r a c i c e c h oca rd i o g r a p h y d e m o n strates t h e typ i c a l echoca rd i o g ra p h i c c h a n ges of rheu matic m itra l valve d isease . The typ ica l fi n d i ngs include thicke n i n g a n d ca lcificati on of the leafl ets and su bva lvu l a r a p p a ratus with com m iss u ra l ca lcificati o n . T h e a nte r i o r m itra l va lve leaflet i n l o n g axis h a s the typ ica l " H ockey-stick" a p peara nce i n d iasto l e with i m ­ m o b i l ity o f t h e posterior l eaflet. ANS WER 2: B. E RO = 6 . 2 8 (r2)

a l i a s i n g velocity

x -----

M R ve locity If the a l i a s i n g ve locity is about 40 cm per seco n d a n d t h e m itra l va lve reg u rg itation velocity is ass u m ed t o b e 5 0 0 cm p e r seco n d , then E RO =

6.28

x

r2

x

40 cm per secon d

500 cm per secon d

251

x

500

r2

=

r2

p ro l a p s i n g from the l eft atri u m i nto the l eft ve ntri c l e w i t h e a c h hea rtbeat. G iven its s i z e a n d m o b i l ity there is a high risk of e m bo l ization . The g u i d e l i nes wo u l d rec­ o m m e n d , g iven its size a n d the deg ree of sym ptoms, the patient i s referred for emerg e n cy s u rgery. Intrave­ nous th ro m bo lytic thera py is p referred fo r rig ht-sided t h rom botic va lves, smaller clots, a n d patie nts who a re felt to be too h i g h risk for s u rg e ry. B i o p rosthetic va lves a re widely used as a n a ltern ative to mech a n i ca l p rosth eses for e n d -sta ge m itra l va lve d i sease, a n d fo l l owi n g the i n it i a l h ea l i n g p hase after va lve i m p l a ntati o n , the risks of tissue b i o p rost h et i c va lve th rom bosis o r re l ated e m b o l ism a re g e n e ra l ly considered ve ry low, thus long-te rm system i c a nticoag­ u lation ca n typ i ca l ly be avo i d ed . The potential for b io­ p rosthetic va lve t h rom bosis a n d dysfu nction, a ltho u g h low, is sti l l elevated com pa red with natu ra l va lves a n d is i n c reased b y the co ncomitant existe nce o f atri a l f i ­ b ri l l ation o r left ventri c u l a r dysf u n cti o n .

2

E RO becomes h a l f of the r2 va l u e . ANS WER 3 : B . Tra n sva lvu l a r g ra d i e n ts a re ve ry dependent on hea rt rates . A mean d i asto l i c g rad ient of 8 to 1 0 mm Hg across a tissue m itra l p rosthesis wo u l d be q u ite a b n o r m a l at a h e a rt rate o f 60 beats p e r m i n ute (bpm), b u t m i g ht be q u ite n o r m a l a t a h e a rt rate of 1 2 0 b p m . Alth o u g h syste m i c b l ood p ressu re does n ot g reatly affect d i a sto l i c g ra d i ents, m itra l valve reg u rg itati o n i s very d e p e n d e n t o n b l ood p ress u re, te n d i n g to i n c rease as the b l ood p ressu re i n c reases . S h o u l d the patient h ave a h i g h output state, for ex­ a m ple, a n e m i a , fever, a n d the tra nsva l v u l a r g ra d i ents wi l l be elevated . Th is ca n be sepa rated from a b n o r m a l ­ ity b y a s i m i l a r d e g ree o f elevation i n the LVOT fl ow. G e n d e r has no i m pact on valve g ra d i e nts.

ANS WER 5: B. B i o p rosthetic va lves a re often cho­ sen in p refe re nce to the str u ct u ra l ly m o re d u ra b l e mech a n i c a l va lves based solely o n the a bsence o f a req u i re m e n t for m a n datory wa rfa r i n a nticoa g u lation and a consequent red uced risk of b l eed i n g com p l i ca­ tions, partic u l a rly i n elderly patients. Alth o u g h a vari ety of strateg ies have been descri bed i n the ea rly postop­ e rative period, a u n iversa l a ntith ro m bot i c strategy is n ot clear. Recent g u i d e l i nes by the American H e a rt As­ sociation/American C o l lege of C a rd i ology reco m m e n d the u s e o f asp i r i n i n a l l patients receivi n g b i o p rosthetic h e a rt va lves and to co n s i d e r wa rfa ri n in the i n it i a l 3 months a n d/or the p rese nce o f r i s k factors . I n deed, e m e rg i n g data point towa rd the potenti a l benefits of a nti p l ate l et thera py i n preventi n g t h rom botic com p l i ­ cations post b i o p rosthetic va lve rep l acement. Patients with atri a l fi b ri l l at i o n a n d/o r l eft ventri c u l a r systo l i c dysfu nction s h o u l d receive both asp i r i n a n d wa rfa ri n . 1 ·3

ANS WER 4: C. TTE d e m o n strates a l a rg e m o b i l e t h ro m b u s atta ched t o o n e o f the p rosthetic l eafl ets

References 1. Stein PD, Alpert JS, Bussey HI, et al. Antithrombotic therapy in patients with mechanical and biological prosthetic heart valves. Chest. 200 1 ; 1 1 9 ( l ) (suppl) :220S-227S. 2. Bonow RO, Carabello BA, Chatterjee K, et al. ACC/AHA 2006 guidelines for the management of patients with valvular heart disease: a report of the American College of Cardiology/American Heart Association Task Force on Practice Guidelines (writing

committee to develop guidelines for the management of patients with valvular heart disease) . American College of Cardiology Web Site. http://www. acc.org/clinical/ guidelines/valvular/index. pd£ Accessed. 3. Colli A, Verhoye J-P, Leguerrier A, et al. Anticoagulation or anti platelet therapy of bioprosthetic heart valves recipients: an unresolved issue. Eur] Cardio Surg. 2007;3 1 : 573-577.

CAS E 8 8

H i story of Sym ptomatic Paroxysmal Atrial F i b r i l lation

A 88-3 and Figs. 8 8- 1 to 8 8-6) . She has a history of symptomatic paroxysmal atrial 68-year-old woman is referred for a transthoracic echocardiogram (Videos 88-1 to

fibrillation.

F i g u re 88-1

F i g u re 88-3

F i g u re 88-2

F i g u re 88-4

285

2 8 6 I E C H O C A R D I O G RA P H Y: A C A S E - B A S E D R E V I E W

A. B. C. D. E.

Normal diastolic function Grade 3 diastolic dysfunction (restrictive filling) Elevated left ventricular filling pressures Normal left ventricular filling pressures Indeterminate left ventricular filling pressures

QUESTION 3. The patient is considered for left atrial

radiofrequency ablation. Before this she is referred for transesophageal echocardiography. Based on images (Videos 88-4 and 88-5 and Fig. 88-7) , which state­ ment is correct? F i g u re 88-5

F i g u re 88-7

F i g u re 88-6

QUESTION 1. Based on a calculation of the left atrial volume by the area-length method (assume a body surface area of 2. 1 m2) , the patient's left atrial size is:

A. B. C. D.

Normal Mildly enlarged Moderately enlarged Severely enlarged

QUESTION 2. Based on the available data, which of

the following statements correctly describes an assess­ ment of diastolic function and left ventricular filling pressures?

A. Direct current cardioversion is not indicated B. The left atrial appendage (LAA) emptying ve­ locities indicate a low likelihood of successful cardioversion C. The LAA emptying velocities indicate an in­ creased risk of future LAA thrombus D. There is echocardiographic evidence of LAA thrombus

QUESTION 4. The patient undergoes successful ra­ diofrequency wide area circumferential left atrial abla­ tion. The day following the procedure, she is in sinus rhythm, but complains of mild sharp chest pains par­ ticularly when lying down. She is referred for transtho­ racic echocardiogram. Based on the available images

C A S E 88 I 2 8 7

Video 88-6 through 88-9 and Figures 88-8 and 8 8-9, which of the following statements is correct?

A . Acute pericarditis i s excluded based on the echocardiographic findings B. There is evidence of a new anterior pericardia! effusion C. The mitral inflow and inferior vena cava are highly suggestive of constrictive physiology D. A gated CT scan of the chest allows assessment of both the pericardium and pleural spaces.

QUESTION 5. Which of the following statements is

correct?

F i g u re 88-8

A. There is Doppler evidence of mildly reduced left atrial contractility B. There is evidence of delayed relaxation (grade 1 ) left ventricular filling pattern C. There is evidence of electrical but not mechani­ cal left atrial function

QUESTION 6. The patient was dismissed on a combi­ nation of colchicine and indomethacin, but returned 1 week later with persistent chest pain and new short­ ness of breath. A repeat transthoracic echocardiogram was performed. Based on the images (Videos 88- 10 to 88- 12) , which test will most likely be diagnostic?

F i g u re 88-9

A. B. C. D.

A transesophageal echocardiogram A repeat CT scan of the chest A coronary angiogram A 1 2-lead ECG

2 8 8 I E C H O C A R D I O G RA P H Y: A C A S E - B A S E D R E V I E W

ANS WER 1 : B. T h e l eft atri a l vol u m e s h o u l d be meas u red based on fo u r measu reme nts at ventri c u l a r e n d systo l e : the a rea a n d the l e n gth i n two orthog o n a l vi ews (a p i c a l fo u r-c h a m b e r a n d a p ica l two-c h a m be r vi ews) . T h e length i s meas u red from t h e rea r atri a l wa l l to t h e va lve p l a n e i n both vi ews a n d ave raged . A n o r­ m a l l eft atr i a l vol u m e i n dexed to BSA i s 2 2 ± 6 cc per m2, a m i l d l y e n l a rged LA 2 9 to 3 3 cc per m2, a mod­ erately d i l ated LA 34 to 3 9 cc per m2 , and a severely d i lated LA �40 cc per m2 . H e re LA vol u m e = [(0 . 8 5)(A )(A ) ]/(L) 1 = [(0 . 8 5)(2 3 . 4)(2 0 . 3 ) ]/(6 . 2 ) = 6 5 . 1 cc LA vol u m e i n dex = 6 5 . 1 /2 . 1 = 3 1 cc per m2 See The Echo Manual, 3rd Edition, d iscuss io n of atria l size and vol u m e on pages 1 1 0 and 1 1 2, and Appendix 8 on page 405.

ANS WER 2: E. The u s u a l assessment of l eft ve ntric­ u l a r d i a sto l i c function ca n n ot be a p p l ied in the setti n g o f atri a l fi b ri l l ation . H oweve r, i n s o m e patients with atri a l fi b r i l lation, a n est i m ate of left ve ntri c u l a r fi l l i n g p ressu res may b e made. A n u m ber o f stud ies have sug­ g ested that a n e l evated E/e ' rati o p a rti c u l a rly i n the setting of a short deceleration t i m e corre lates wel l with elevated left ventricu l a r fi l l i n g pressu res. However, here the conste l l ation of fi n d i ngs is i n determ i nate. The E/e ' ratio of 1 0 to 1 2 i s n e ither n o r m a l n o r h i g h b u t i n the i n d eterm i n ate ra n g e . The dece l e ration is a lso not particu l a rly shortened . H e n ce, o n e is u n a b l e to confi­ dently est i m ate l eft ventri c u l a r fi l l i n g p ressu res in t h i s situation . ANS WER 3: A. T h e re i s c l e a r evi d e n ce of e l ectri­ ca l atri a l activity (P waves) on ECG with evi d e n ce of normal LAA f u n ction on Doppler i m a g i n g , a n d hence ca rd i overs i o n is n ot i n d i cated . N o r m a l LAA ve locities a re g reater t h a n 50 cm per seco n d (here 70 to 80 cm per seco n d ) . See The Echo Manual, 3rd Edition, d i scussion o f left atrial a ppendage flow velocities on page 394.

ANS WER 4: D. Alth o u g h patie nts with acute peri­ ca rd itis may d is p l ay evi d e n ce of a perica rd i a ! effusion or constrictive p hysiol ogy, m a ny patie nts have a n o r­ m a l echoca rd i o g ra m . There i s a heterogeneously echo dense space a nte rior to the heart that rep resents fat rather than fl u i d . It was p resent a lso o n p rea b l at i o n i m ages. The c l a s s i c fi n d i n g s o f a restrictive m itra l i n ­ flow pattern ( h i g h E-wave ve locity t h a t i n creases with exp i ration) and d i l ated IVC a re not p resent. A CT chest does potenti a l ly a l l ow assessment of both the perica r­ d i a ! a n d p l e u ra l spaces, a lthou g h it ca n be d i ffi cult to sepa rate a sma l l a m o u n t of perica rd i a ! fl u i d from peri­ ca rd i a ! t h i c ke n i n g . As the patient h a s persistent p e r i ca rd i a ! pa i n , a C T ch est w a s p e rfo r m e d that w a s a l so u n re m a r ka b l e (Video 88-1 3) . ANS WER 5: A. Left atri a l systo l i c stra i n assessment is emerg i n g as high fidel ity meas u re of l eft atrial contrac­ ti l ity with a normal level more negative than - 1 5 % . Al­ thou g h the A-wave velocity is g reater than the E-wave ve locity, t h i s does not re p rese nt a g ra d e 1 d i asto l i c dysf u n ction pattern b u t rath er a " fa lse " elevation be­ cause of a p a rtly fused s i g n a l . Typical ly, if the E ve locity is g reater t h a n 20 cm per secon d at the on set of the A wave (E at A, here 40 cm per second), both the A velocity a n d the E/A ratio may be affected . ANS WER 6: B. U n l i ke the p revious stu d i es, there a re now ve ry poor a p ica l i m ages. G iven t h i s fi n d i n g a n d i n t h e sett i n g o f o n g o i n g ch est pa i n a n d n ew dys p n ea a n d the sense of a l so on s u bcosta l i ma g i n g of poor echo penetra n ce beyo n d the a nterior perica rd i u m , a pneu moperica rd i u m s h o u l d be considered . A CT chest wo u l d be d ia g n ostic in t h i s situati o n . I n d ee d , a C T ch est (Video 88- 1 4) confi rms a i r i n t h e perica rd i a ! space i m p l icati n g a l e a k betwee n the eso p h a g u s and the perica rd ia I space as a delayed co m ­ p l i cation o f the atri a l a b lation p roced u re . The patient was b ro u g ht to the operati n g room where a n a b n o r­ m a l com m u n i cation was i dentified a n d repa i red .

References Pappone C, Oral H, Santinelli V, et al. Atrio-esophageal fistula as a complication of percutaneous transcatheter ablation of atrial fibrilla­ tion. Circulation. 2004; 1 09 (22) :2724-2726. Antonini-Canterin F, Nicolosi GL, Mascitelli L, et al. Direct dem­ onstration of an air-fluid interface by two-dimensional echocardiog-

raphy: a new diagnostic sign of hydropneumopericardium. ] Am Soc Echocardigr. 1 996;9(2) : 1 87- 1 89.

CAS E 8 9

Fever and Dyspnea with H i story of Severe Pancreatitis

A treated with a prolonged course of total parenteral nutrition. He had made a recovery 45-year-old man with a medical history of severe pancreatitis and pancreatic pseudocyst

with normal diet resumed 3 weeks prior. He now presents with fever and dyspnea. He is referred for transesophageal echocardiography (Videos 89- 1 to 89-17 and Figure 89- 1 ) .

Q UESTION 2. What i s the incidence o f thrombus formation in patients who have an indwelling central venous catheter?

A. B. c. D.

1% 1 0% 30% 50%

F i g u re 89-1

QUESTION 1. What is the likely cause of the finding

seen? A. B. C. D.

Thrombus Atrial myxoma Tumor spread from renal cell carcinoma Papillary fibroelastoma

289

2 9 0 I E C H O C A R D I O G RA P H Y: A C A S E - B A S E D R E V I E W

ANS WER 1 : A. T E E d e m o n strates a l a rg e m o b i l e mass attached t o t h e u p per r i g ht atri a l wa l l a n d lower su perior ve na cava . Alth o u g h an atri a l myxoma is pos­ s i b le, a p p roxi m ately 9 0 % of myxomas a re attached to the i ntra-atr i a l septu m . Thro m b u s is bar fa r the most l i kely a n d p res u m a b l y is related to the centra l ve nous cath eter that was recently i n p lace for the pa rentera l n utrition . Tu mor extension from a re n a l ce l l ca rci noma

wo u l d e nter the r i g h t atri u m from the i nferior ve n a cava (Video 89-1 8) . ANS WER 2 : B . T h e i ncidence o f t h rom b u s i n either the su perior ven a cava or right atri u m i n patients with an i n dwe l l i n g centra l ve n o u s cath ete r has been re­ ported to be between 2 % a n d 1 7 % . 1 ·3

References 1. Merrer J, De Jonghe B , Golliot F, et al. French Catheter Study Group in Intensive Care. Complications of femoral and subcla­ vian venous catheterization in critically ill patients: a randomized controlled trial. JAMA. 200 1 ;286:700-707. 2. Timsit JF, Farkas JC, Boyer JM, et al. Central vein catheter-related thrombosis in intensive care patients: incidence, risks factors, and relationship with catheter-related sepsis. Chest. 1 988; 1 14:207-2 1 3 .

3. Ghani MK, Boccalandro F, Denktas AE, et al. Right atrial throm­ bus formation associated with central venous catheters utilization in hemodialysis patients. Intensive Care Med. 2003;29: 1 829-1 832.

CAS E 9 0

Latera l ly Displaced Apica l I m p u lse

A chest pain. On examination, his apical impulse is displaced laterally and a soft mid­

22-year-old man is referred for an echocardiogram to evaluate 12 months of atypical

systolic murmur is audible.

QUESTION I. Based on the echocardiographic im­

ages (Videos 90-1 to 90-7 and Figures 90- 1 to 90-3) , the most likely diagnosis is:

F i g u re 90-3

F i g u re 90- 1

A. B. C. D.

Pulmonary hypertension Aortic valve stenosis Ventricular septa! defect Congenital absence of the pericardium

QUESTION2. The pulmonary vein pattern (Fig. 90-4) is:

F i g u re 90-2

F i g u re 90-4

291

2 9 2 I E C H O C A R D I O G RA P H Y: A C A S E - B A S E D R E V I E W

A. Suggestive of high left atrial pressure B . Is to be expected in this condition C. Suggestive of associated severe mitral valve re­ gurgitation D. Normal

QUESTION 3. Which of the following conditions has

been associated with congenital absence of the peri­ cardium? A. B. C. D.

Atrial septal defect Bronchogenic cysts Bicuspid aortic valve All of the choices

C A S E 90 I 2 9 3

ANS WER I: D. The co n g e n ita l co m p l ete a bsence of the left-s ided perica rd i u m is associated with a s h i ft of the heart leftwa rd a n d l atera l l y with the r i g ht ven­ tricle a p pea r i n g b i g g e r as more is seen in the sta n d a rd pa raste r n a l i mages with a n ove ra l l u n us u a l o r i e nta­ tion of the heart . The short-axis view demonstrates a paradoxica l systol i c a nterior motion of the ve ntri c u l a r sept u m w i t h v i g oro u s l eft ve ntri c u l a r poste rior wa l l movement. The a p ica l fo u r-cha m ber view of the heart has exaggerated m o b i l ity with a " swi n g i n g heart " a n d a d isplaced a pex. See The Echo Manual, 3rd Edition, pages 289 to 290.

ANS WER 2: B. Pulsed wave Doppler i nterrogation of the p u l m o n a ry vei n i n d i cates m i n i m a l systol i c forwa rd

flow b u t rather fi l l i n g of t h e left atri u m d u ri n g d iasto l e . T h e typ ica l i n d i cation f o r this is a n elevation i n l eft atria l p ressu re . H oweve r, it m ust be re membered that one of the phys i o l o g i c compon ents of n o r m a l atri a l fi l l i n g is the contri bution of a decrease in perica rd i a ! p ress u re d u ri n g ventric u l a r ejection that is conferred to the atria and a u g ment systo l i c atri a l fi l l i n g . Th is com ponent wi l l b e lost i n a bsence of t h e perica rd i u m a n d hence is a n expected fi n d i n g i n t h i s case . ANS WER 3: D. C o n g e n ita l a bsence of the perica r­ d i u m is associ ated with a h i g h i ncidence of the l i sted a b normal ities.

CAS E 9 1

Ventricu lar Tachyca rd i a with Rapid Pa l p itations

A

3 5-year-old man with frequent episodes of ventricular tachycardia presents for his first echocardiogram. He was well until the previous 6 months when he began to develop episodes of rapid palpitations. Apart from a slow pulse rate (40 beats per minute) , cardio­ vascular examination was normal.

QUESTION 1. What is the average left ventricular

QUESTION 3. Based on

outflow tract (LVOT) diameter for men measured by echocardiography?

Figure 9 1 -2, the pulmonary artery (PA) end-diastolic pressure is:

A. B. C. D. E.

19 21 23 25 27

Videos 9 1 -2 and 9 1 -3 and

mm mm mm mm mm

QUESTION 2. Based on Video 91-1 and Figure 9 1 - 1 (note the LVOT diameter is 2 3 mm) , the calculated cardiac output is:

F i g u re 91 -2

A. Normal B . Mildly elevated C. Severely elevated

QUESTION 4. Based on the review of Videos 9 1 -4 to 91-13 and Figures 9 1 -3 to 9 1 -6, which of the fol­ F i g u re 91 -1

A. B. C. D. E. 2 94

4.2 5.3 4.8 5.9 6.3

L per minute L per minute L per minute L per minute L per minute

lowing is the most likely process related to the right ventricular (RV) free wall? A. B. C. D.

Artifact Pericardial cyst Benign primary cardiac tumor Aneurysm with thrombus

CASE 9 1 I 295

F i g u re 9 1 -3

F i g u re 9 1 -6

QUESTION5. Based on the tricuspid annular tissue peak

velocity, which of the following tricuspid annular plane systolic excursion (TAPSE) values would be expected? A. B. C. D.

F i g u re 9 1 -4

2 mm 1 2 mm 22 mm 52 mm

QUESTION 6. The patient proceeds with surgical ex­ ploration (see Videos 91-14 to 91-16 and Figure 9 1 -7) . Explain the type o f artifact commonly experienced when imaging in the operating room.

F i g u re 9 1 -7

F i g u re 9 1 -5

A. B. C. D.

Acoustic shadowing artifact Reverberation artifact Noise artifact Near-field clutter artifact

2 9 6 I E C H O C A R D I O G RA P H Y: A C A S E - B A S E D R E V I E W

ANS WER 1: B. The mean LVOT d i a mete r for men is 2 1 m m a n d for wom e n 1 9 m m . ANS WER 2: D. The ca rd i a c output ca n b e ca l c u l ated as fol l ows : = = =

0 . 78 5 (LVOT d i a meter)2(LVOT TVl)(H R) 0. 7 8 5 (2 . 3)2(3 5)(4 1 ) 5 . 9 L per m i n ute

See The Echo Manual, 3rd Edition, d iscussio n of stroke volu m e on page 1 1 6.

ANS WER 3: A. The PA e n d - d iasto l i c p ressu re ca n be est i m ated by the s u m of the peak p u l m o n a ry reg u rg i ­ ta nt j et velocity g ra d i ent a n d a n esti mate o f ri g ht atri a l p ress u re . H e re t h e e n d P R velocity is q u ite n o r m a l at 1 m per seco nd ( i . e . , a g rad ient of 4 m m Hg). H e re the i nferior vena cava is of normal size and co l l a pses with i n s p i rati o n , s u g g esti n g a normal r i g ht atri a l p ress u re . Th is is a fi n d i n g supported b y norma l physica l exa m i na­ tion fi n d i n g s .

the m itra l sewi n g ri n g in Video 9 1 -1 7 . Reverberation a rtifact occu rs when the u ltraso u n d bou n ces backwa rd a n d forwa rd (reverberates) between two h i g h ly refl ec­ tive s u rfaces . The a rtifact a p pea rs as m u lt i p l e para l l e l l i nes situated i n t h e fa r f i e l d exte n d i n g away from t h e tra n sd ucer (e . g . , see Fig . 9 1 -8) . Near-fi e l d clutter, seen when structu res parti c u l a rly close to the tra nsd ucer a re cha racterized poorly, is d u e to h i g h -a m p l itude osc i l la­ tions of the p i ezoelectric e l e m e nts i n the tra n s d u cer. An exa m p l e of a near-fi e l d cl utter a rtifact is shown i n Video 9 1 - 1 8, where t h e a pex of t h e l eft ventricle ap­ pears to have a n i n d ist i n ct en doca rd i a l border that to an u ntra i ned eye may s u g g est t h rom b u s . The use of echo contrast confi rms that, i n deed, t h i s is an a rtifact rather t h a n t h ro m b u s (Video 9 1 - 1 9) .

ANS WER 4 : C. Echoca rd i o g ra p hy d e m o n strates a l a rge RV mass with i n the RV free wa l l , which a p pears we l l d e m a rcated a n d sepa rate from the su rro u n d i n g myoca rd i u m a n d perica rd i u m . G ive n t h e d e m a rcation t h i s s u g gests a ben i g n rather than m a l i g na nt p rocess, a lthou g h f u rther test i n g is req u i red . ANS WER 5: C. Despite the RV free wa l l mass, the tricuspid a n n u la r systol i c motion rema i n s normal with a peak velocity of 1 5 to 1 6 cm per seco n d . H e n ce, a normal TAPS E wo u l d be expected (l i kely i n the ra n g e of 20 to 30 mm).

The patient p roceeded to resection of the mass. Pa­ thology confi rmed it to be a h e m a n g i o m a .

ANS WER 6: D . C o m m o n ly encou ntered i n t h e oper­ ati n g room as i l l ustrated i n F i g u re 9 1 -7 is noise a rtifact ca used by electromagnetic i nterference from the use of cautery. Shadowi n g a rtifact is when there is a p p a rent d ropout beyo n d a h i g h ly refl ective structu re, such as

C a rd i a c h e m a n g iomas1 a re ra re be n i g n p ri m a ry ca rd i a c t u m o rs a n d h ave b e e n re ported t o occ u r i n a l m ost eve ry ca rd i a c locat i o n . Ofte n asym ptomatic, c l i n i ca l p resentation wi l l depend o n size a n d l ocati o n . Myo­ ca rd i a l tumors as in t h i s case may p resent with rhyt h m d istu rbances. S u d d e n death has b e e n reported .

Reference 1. Mongal LS , Salat R, Anis A, et al. Enormous right atrial hem­ angioma in an asymptomatic patient: a case report and literature review. Echocardiography. 2009;26:973-976.

F i g u re 9 1 -8

CAS E 9 2

Chest Pa i n i n a 52-Year-Old Woman

Aan episode of chest pain (Videos 92- 1 to 92- 12 and Figs. 92- 1 to 92-5) .

52-year-old woman presents for a transthoracic echocardiogram performed because of

F i g u re 92-1

F i g u re 92-3

F i g u re 92-2

F i g u re 92-4

297

2 9 8 I E C H O C A R D I O G RA P H Y: A C A S E - B A S E D R E V I E W

QUESTION 3. Which of the following statements

concerning the use of echo contrast is false (Videos 92- 1 3 and 92- 14) ? A. Echo contrast in this case confirms prominent vascularity B. Echo contrast in this case confirms the presence of intrapericardial clot C. Echo contrast in this case suggests a benign mass

F i g u re 92-5

QUES TION 1. Findings are consistent with:

A. B. C. D. E.

Pericardia! effusion Pericardia! clot Pericardia! tumor Pericardia! cyst Loculated pleural effusion with a normal peri­ cardium

QUESTION 2. There are two-dimensional (2D) and

Doppler features of constriction/tamponade. A. True B. False

QUESTION 4. Pericardia! tumors are most commonly:

A. Benign primary tumors B . Malignant primary tumors C. Malignant secondary metastatic tumors

Q UES TION 5. The patient underwent surgical

removal. Pathology confirmed a leiomyosarcoma. Which of the following pieces of medical history is likely present? A. B. C. D.

Asbestos exposure AIDS infection Prior radiation for breast cancer Prior Epstein-Barr virus infection

CASE 92 I 299

ANS WER 1: C. 2 D i m a g i n g d e m o n strates a l a rge i rreg u l a r (9 x 3 cm) mass with i n the perica rd i a ! space adjacent to the ca rd i a c a pex (pri m a r i ly the r i g ht ven­ tricle). The pericard i u m is t h i c ke n e d , and there is an associated fl u i d col l ection . ANS WER 2: B. Desp ite the l a rge perica rd i a I p rocess, there is l ittl e to suggest hemodyn a m i c s i g n ifica nce. The m itra l i nflow is n o r m a l without resp i rophasic c h a n g e . T h e septa l motion is norma l . The i nfe rior ven a cava is normal i n size. ANS WER 3: A. Echo contrast a d m i n istration ca n be usef u l i n the assessment of i ntraca rd iac masses where opacification is consistent with a vasc u l a r s u p p ly. The re latively ra p i d and co m p l ete o p a c i fi cati o n of the mass in this case (Video 92- 1 4) i m p l icates p ro m i nent vasc u l a rity.

ANS WER 4: C. A s is the case with a l l ca rd iac tumors, metastatic t u m ors a re by fa r the most co m m o n g ro u p . O f p r i m a ry ca rd i a c t u mo rs, be n i g n t u m o rs a re a p p roxi­ mately twi ce as com m o n as m a l i g n a nt t u mors. ANS WER 5: C. Le i o myosa rco ma i s a ve ry ra re p r i ­ m a ry card i a c m a l i g n a ncy de rived from smooth m uscle cel ls . U n l i ke m a ny p r i m a ry card i a c t u m o rs, they co m­ m o n ly a rise o n t h e l eft side p a rti c u l a rly i n t h e l eft atri u m . Th is t u m o r was l i kely sti m u l ated in t h i s patient by prior rad i ati o n thera py. Oth e r ra re ca rd i ac t u m o rs i n c l u d e mesoth e l ioma (typ i ca l ly associated with p r i o r asbestos exposu re). A I D S is associated w i t h Kapos i 's sa rcoma a n d lym p h o m a . E pste i n-B a r r v i r u s is associ ­ ated with lym phomas.

CAS E 9 3

S oft Systo l i c M u rm u r after Fa inting Episode

A on a hot humid day. A soft systolic murmur is heard on physical examination. He is 1 9-year-old college athlete is referred after a fainting episode while playing basketball

referred for transthoracic echocardiography.

QUESTION I. Which of the following constellations of findings is not a contraindication to participation in competitive athletics?

A. A low normal sized left ventricle (LV) with an ejection fraction (EF) of 70%, a ventricu­ lar diastolic septal diameter of 1 9 mm, and a global average peak systolic longitudinal strain of - 1 4% B . A mildly dilated LV with an EF of 60%, LV wall thickness of 1 5 mm, medial annular peak systolic tissue Doppler imaging of 1 2 cm per second, and e' velocity of 1 5 cm per second with a stroke volume index of 70 cc per m2 C. A normal LV size, left ventricular ejection frac­ tion (LVEF) of 65%, left atrial volume index of 22 cm3 per m2, a bicuspid aortic valve without stenosis or regurgitation, and a mid ascending aortic diameter of 52 mm D. A normal LV size, LVEF 60%, cardiac output of 7. 5 L per minute, normal aortic valve leaflet mobility, and transaortic time velocity integral of 1 1 0 cm

F i g u re 93- 1

QUESTION 2. Based on review of his echocardiogram

(Videos 93- 1 to 93-8 and Figs. 93- 1 to 93- 1 0) , the likely finding on 1 2-lead ECG would be:

300

F i g u re 93-2

C A S E 93 I 3 0 1

F i g u re 93-6

F i g u re 93-3 , v

'I 8 '1R

r ·f'f'

9' JP •

, ., , ., 1 ll"' lllt

I M-

• ...

t is-

,

.

] • '

,. i.;

F i g u re 93-4

F i g u re 93-7

F i g u re 93-5

F i g u re 93-8

3 0 2 I E C H O C A R D I O G RA P H Y: A C A S E - B A S E D R E V I E W

QUESTION 4. Based on the two-dimensional (2D)

images and Doppler findings at rest (Fig. 93-6) , Valsalva (Fig. 93-7) , and following amyl nitrate (Fig. 93-8) , which of the following management strat­ egies is preferred? A. B. C. D. E.

Alcohol septal ablation Surgical myectomy Diuretic therapy ,8-Blocker therapy Dihydropyridine calcium channel blocker, for example, amlodipine

F i g u re 93-9

Q UES TION 5. The degree of septal hypertrophy (Fig. 93- 1 ) correlates with the risk of sudden cardiac death.

A. True B. False QUESTION 6. Which of the following management steps is most appropriate?

F i g u re 93- 1 0

A. Low voltage B. Deep symmetrical T wave inversions in the pre­ cordial leads C. Diffuse ST segment elevation with PR segment depression D. LV hypertrophy QUESTION 3. Possible findings on genetic testing

might include: A. A defect in the gene encoding ,8-myosin heavy chain present in the patient and possibly a sibling B. Genetic defect in cardiac troponin T in patient and in male but not in female relatives C. Defect in the SCN5A cardiac sodium channel D. Desmin gene mutation E. Genetic testing would most likely be negative

A. Stop the metoprolol and arrange outpatient follow-up B. Increase the metoprolol for presumed worsen­ ing outflow tract obstruction C. Repeat transthoracic echocardiography D. Obtain transesophageal echocardiography

QUESTION 7. He proceeds for elective placement

of an implantable cardiac defibrillator, initiation of metoprolol, and is counseled to avoid competitive athletics. One week later, he returns with increased shortness of breath and an episode of syncope. Inter­ rogation of his device demonstrates no dysrhythmia. On examination, his heart rate is 1 00 beats per min­ ute, blood pressure (BP) 90/50 mm Hg, neck veins are elevated, and no murmurs are audible (at rest or with Valsalva) . Based on the images obtained (Videos 93-9 to 93- 12 and Figs. 93- 1 1 and 93- 1 2) , the likely hepatic vein profile would demonstrate:

CASE 93 I 303

A . Systolic predominant forward flow with atrial flow reversals B. Flow reversals in diastole that increase upon inspiration C. Flow reversals in diastole that increase upon expiration

QUESTION 8. After ultrasound-guided pericardia! ac­ cess is obtained, the return is bloody. 1 0 cc of agitated saline is administered through the 1 6-gauge venous sheath (Video 93- 12) . Is it appropriate to proceed with placement of a sheath over a guidewire and a pericardia! drain.

F i g u re 93-1 1

A. True B. False

QUESTION 9. The laboratory findings (see table) on the pericardia! fluid are most likely:

F i g u re 93-1 2

Lactate Hemog l o b i n ,

Tota l N u c l eated

g per d i

Cells, per µ I

A.

0.8

B. c.

Neutro p h i ls, %

Lym p hocytes, %

1 1 , 000

90

0

1 80,000

98

0

12

4,500

21

16

24,000

68

1 60

15

D.

0.5

E.

0

G l ucose,

Dehyd rogenase,

mg per d i

U per L

69 1 0,000

2

89

650

75

1 00

700

3 0 4 I E C H O C A R D I O G RA P H Y: A C A S E - B A S E D R E V I E W

ANSWER 1: B. Afte r yea rs of i ntense tra i n i n g , a n ath­ l ete's h e a rt may deve l o p a g l o b a l i n crease i n LV wa l l thickn ess . Th is someti mes ca n b e confused with hyper­ tro p h i c ca rd i o myopathy ( H C M ) . U n l i ke pati ents with H C M , a n ath lete's heart wi l l te nd to be u pper norm a l i n size or m i l d ly e n l a rged with LV wa l l thicknesses n o th icker t h a n 1 7 m m w i t h normal d iasto l i c f u n cti o n , tis­ sue D o p p l e r, and stra i n i m a g i n g (e . g . , option B , see exa m p l e Videos 93-1 3 to 93-1 5 and F i g . 93 - 1 3) . This is i n contrast to option A, where the LV i s t h i c k with g l obal stra i n red u ction descri b i n g a case of H C M . The presence of H C M is a contra i n d i cation to com petitive ath l eti cs, a n d i n deed H C M is the p redo m i n a nt a b n or­ m a l ity that u n d erl ies sudden death i n ath l etes . Other high risk factors i n c l u d e certa i n coronary a n o m a l ies a n d proa rrhyth m i c con d itions s u c h as long Q T syn d rome or r i g ht ventri c u l a r (RV) dysplasi a . Other con d itions with wh ich com petitive ath l etes s h o u l d be avoided i nc l u d e seve re ascen d i n g a o rt i c d i l a t i o n ( p a rt i c u l a rly i n t h e sett i n g o f M a rfa n syn d rome or b i c u s p i d aortic va lve, option C). Option D is co ncern i n g for seve re s u b- or s u p ra-aortic stenosis.

ANS WER 3 : A . Septa l va riant H C M i s com m o n l y as­ sociated with a gene defect and is typ i ca l ly a utoso m a l dom i n a nt; hence, defects i n male or female fi rst-degree re l atives a re com mon . H ence, fi rst-deg ree re latives re­ q u i re p h e n otyp i c a n d/o r genetic scree n i n g . The most co m m o n defects a re those e n cod i n g ,8- myos i n h eavy ch a i n . D efects i n ca rd i a c tropo n i n T a re a l so we l l descri bed . Defects i n the SC N SA sod i u m c h a n n e l gene a re c h a racte rist i c of B r u g a d a syn d rome, and desm i n gene m utati ons a re associated with RV dysplasia . ANS WER 4: D. Desp ite the m a ss ive se pta l hyper­ trophy, there is l ittl e o utfl ow tract obstruction at rest or i n d uc i b l e . H e n ce, there is no evi d e n ce to s u g g est a septa ! red u ction p roced u re wi l l p rovide a n y c l i n ical benefit. M o reover, th ese p roced u res a re rese rved for sym pto m at i c patie nts with s i g n ifica nt o utflow tract obstruction desp ite opti m a l medical thera py. Vasod i la­ tor agents such as d i u retics or d i hyd ro pyri d i n e ca lci u m c h a n n e l b l ocker s h o u l d b e avoided . The patient s h o u l d be p rescri bed ,8- b l ocker thera py. ANS WER 5: A. Echocard i og ra p hy i n this case dem­ onstrates severe i ncrease i n LV wa l l thickness with mas­ sive t h i cken i n g of the i ntraventri c u l a r sept u m (septa ! d i mension of 43 m m) . The degree of LV hypertrophy is associated with a n i n c reased risk of s u d d e n ca rd i ac death, parti c u l a rly with a maxi m u m thickness g reate r t h a n 30 m m . Other freq ue ntly cited risk factors of s u d ­ d e n card i a c d e a t h i n patients w i t h H C M i n c l u d e a n a b normal B P response t o exercise, nons usta i ned ve n­ tric u l a r tachyca rd i a or freq uent e p i sodes of syncope, and a fa m i ly h i story of sudden ca rd i a c death .

F i g u re 93- 1 3

S e e The Echo Manual, 3rd Edition, d iscussio n of ath lete's heart versus HCM on page 267.

ANSWER 2: B. 2D ech oca rd i o g ra p hy d e m o n strates g l o b a l i n crease in LV mass with mass ive hypertrophy of the i ntraventri c u l a r septu m . Evi d e n ce of LV hyper­ tro p hy wou l d be expected on E C G . Low voltage wou l d b e expected i n t h e sett i n g o f a l a rge perica rd i a ! effu­ sion or ca rd i a c a myloidosis. ST seg ment e l evation a n d PR depression is cha racteristic o f acute pericard itis, a n d p record i a l T wave i nvers i o n i s cha racte ristic o f a p ical va riant of H C M .

The magnitude of hypertrophy is related d i rectly to the risk of sudden death . The c u m u lative risk for sudden death is near 0% for a wa l l thickness of 1 9 mm or l ess and 40 % for a thickness of 30 m m or more.

ANS WER 6: C. In a patient with a recent device pl aced who p rese nts with s h o rtn ess of b reath , syn ­ cope, ta chyca rd i a , hypote n s i o n , a n d evi d e n ce o f ve­ nous hypertension, o n e s h o u l d consider tra nsthoracic ech oca rd i o g ra m to eva l u ate for the p resence of card i a c ta m po n a d e . ANSWER 7: C. E c h oca rd i og ra p hy d e m o n strates a l a rge c i rcu mferential perica rd i a ! effusion with featu res

CASE 93 I 305

of ta m po n a d e . B est seen on Video 93-9 a n d F i g u re 9 3 - 1 1 , t h e re is m a rked early d i asto l i c RV co l l a pse, consistent with e l evated i ntra perica rd i a l p ressu re . The i nferior ve na cava is p l ethoric with m i n i m a l respi ratory c h a n ge, a lso consistent with h i g h right atri a l p ress u re . T h e h a l l mark i n t h e hepatic vei ns o f ca rd iac ta m ponade is the p resence of d iasto l i c flow reversa ls that i n crease with exp i ration (F i g . 93 - 1 4) .

F i g u re 93-1 4

ANS WER 8: A. Whenever one gets a b l oody return from pericard i a ! access, it i s critica l to confi rm that yo u a re i n the perica rd i a ! space rather t h a n i n a n i ntraca r­ d i a c c h a m ber. Ad m i n istration of a g i tated sa l i n e with echo i m a g i n g (Video 93-1 2) confi rms a perica rd i a ! lo­ catio n . The a g i tated sa l i n e fa i l s to ra p i d ly pass thro u g h ­ o u t t h e perica rd i a ! space l i kely beca use o f the b l oody content.

See The Echo Manual, 3rd Edition, d iscuss ion of echoca rd iog raph ica l ly g u ided perica rdiocentesis on page 293, and Fig u re 1 7-1 2 on page 296.

ANSWER 9: C. A l a rge pericard i a ! effusion occurring i m m e d i ately after device p l acement i s most l i kely re­ lated to lead perfo ration . In t h i s i n sistence, a hemor­ rha g i c effusion is expected . A hemog l o b i n i n perica rd i a I fl u i d > 5 0 % of the seru m hemog l o b i n is consistent with a h e m o r r h a g i c eff u s i o n ca u sed by perforat i o n . Th i s d o e s not a lways req u i re s u rg i ca l m a n a gement. If the site of perforation is sma l l and pati ent is sta ble, fo l l ow­ i n g evacuation of the bloody effusion a n d m a i ntena nce of a d ry perica rd i a ! space (th ro u g h i nterm itte nt asp i ra­ tion of the perica rd i a ! d ra i n ) wi l l freq uently a l low the perforati o n site to hea l . B l ood i n the pericard i a ! space is ve ry i rritative . Pati ents freq uently develop s i g n ificant p e r i ca rd i a ! pa i n fol l owi n g a s p i rati o n and need a g ­ g ressive treatment with i ntra perica rd i a l l i doca i ne, ora l a n d/o r i ntrave n o u s n o n stero i d a l a n t i - i nfl a m matories and in a re p ractice freq uently a l so receive colch i c i n e . A s a p reca ution a g a i nst the deve l o p m e n t o f pericar­ d i a ! constrict i o n , a cou rse of m a i nte n a nce sched u l ed anti- i nf l a m m atory thera py is used for 4 to 6 weeks fol­ low i n g centesi s . O ptions A a n d B a re exa m p les of the most com m o n patterns with reason a b ly high levels of n ucleated cells, mostly neutro p h i ls, with low l evels of hemog l o b i n , typ ica l ly of effusions occu rri n g in the set­ ting of acute perica rd itis. The pattern of a more c h ro n i c serous effusion is h i g h l i g hted i n option E w i t h low ce l l cou nts, p red o m i n a ntly lymph ocytes . O ption B with a ve ry h i g h cel l , lactate de hyd rogenase co u nt, a n d very low g l u cose occu rs i n the sett i n g of h i g h ly i nf l a m m a ­ to ry p e r i ca rd itis beca use o f a r h e u mato l o g i c i l l n ess such as rheumatoi d a rth ritis or system i c l u p u s erythe­ matosus or a n acute bacterial perica rd itis.

References Spirito P, Bellone P, Harris KM, et al. Magnitude of left ventricular hypertrophy and risk of sudden death in hypertrophic cardiomyopa­ thy. N Engl] Med. 2000;342: 1 778- 1 7 8 5 . McKenna WJ, Behr ER. Hypertrophic cardiomyopathy: manage­ ment, risk stratification, and prevention of sudden death. Heart. 2002;87: 1 69.

Maron BJ, McKenna WJ, Danielson GK, e t al. American College o f Cardiology/European Society of Cardiology clinical expert consensus document on hypertrophic cardiomyopathy. A report of the Ameri­ can College of Cardiology Foundation Task Force on Clinical Ex­ pert Consensus Documents and the European Society of Cardiology Committee for Practice Guidelines. JAm Coll Cardiol. 2003;42: 1 687.

CAS E 9 4

M urmur i n a 38-Year-Old Patient

A blood pressure is 1 40/80 mm Hg. See Videos 94-1 to 94-1 8 and Figures 94- 1 to 94-9.

38-year-old patient presents for an echocardiogram to evaluate a murmur. Systemic

306

F i g u re 94-1

F i g u re 94-3

F i g u re 94-2

F i g u re 94-4

C A S E 94 I 3 0 7

CR 1 02'1: • ;... .. \' fl ' Ill OG •_.•, Vo< :.: ...: ;• , .,... >G ! "11 ��" ' l'!.V•-.·

..

"

'

F i g u re 94-8

F i g u re 94-6

F i g u re 94-9

�· -

··: J

�i �·.'I. • :t1•·.1

' n rm.• C rNT' H\J • :' 1 irr. • 0 n•rh-q

r

:;.. .( � : "

-''

. t} 0

'II\

.Ill I

, \'

\

I

. � :::

' ,.

...,

'

F i g u re 94-5

'

{\



V' I �l'!r."I.·,,

F i g u re 94-7



·

•0

ID

- .\ ] 0 ,. 0

QUESTION 1. Which of the following defects is re­

lated to the abnormal flow? A. B. C. D.

Atrial septal defect Gerbode defect Membranous ventricular septal defect (VSD) Ruptured sinus ofValsalva aneurysm

QUES TION 2. The expected physical examination

finding is: A. B. C. D.

A harsh pansystolic murmur A soft or potentially absent systolic murmur A midpeaking systolic ejection murmur A continuous murmur

3 0 8 I E C H O C A R D I O G RA P H Y: A C A S E - B A S E D R E V I E W

QUESTION 3. The estimated right ventricular (RV)

QUESTION 5. Findings that would be attributable to

systolic pressure in this case is:

this defect that warrants consideration of intervention include all of the following excep t-.

A. B. C. D.

40 mm Hg 5 0 mm Hg 65 mm Hg 1 00 mm Hg

QUESTION 4. The likely grade of diastolic function/ dysfunction based on the available data is:

A. B. C. D.

Normal Mild (grade 1 ) diastolic dysfunction Moderate (grade 2) diastolic dysfunction Severe (grade 3) diastolic dysfunction

A. An elevated pulmonary vascular resistance B. Progressive RV dilation in the absence of pul­ monary hypertension C. Severe tricuspid valve regurgitation (TR) D. Left ventricular (LV) dilation

C A S E 94 I 3 0 9

ANS WER 1: C. Th e re is evi d e n ce of l eft- r i g h t fl ow from LV to RV th ro u g h a m e m b ra n o u s VS D . This is the most com mon type of VSD seen i n a d u lt patients. A Gerbode defect is a n u n us u a l VSD com m u n i cati n g d i ­ rectly between t h e LV a n d t h e r i g h t atri u m . T h i s is often iatrog e n i c occu rri ng at the t i m e of va lve rep l acement. See The Echo Manual, 3rd Edition, discussion o f types VSDs and i maging notes on pages 337 to 344.

(i i i) Esti mation o f p u l m o n a ry a rte ry d i asto l i c p ressu re (PA D P) by (4v2 + RAP), where v = e n d PR velocity The e n d PR velocity is q u ite normal ( 1 m per second) as i s the est i m ated RA p ress u re with a n o r m a l sized i nfe rior ve na cava and a normal p u l sed wave Do pple r p rofi l e i n the hepatic vei n . H e n ce, the est i mated PAD P is normal a t 9 t o 1 0 m m H g . T h i s suggests a g a i nst the p resence of p u l m o n a ry hypertension .

of

ANS WER 2: A. The m u rm u r of a VSD is pansysto l i c with l ittl e if a ny d iasto l i c co m ponent. It is med iated by the l a rg e p ressu re g rad ient in systol e between the LV a n d RV. U n less the patient deve lops p u l m o n a ry hyper­ te nsion a n d seco n d a ry RV hyperte n s i o n , the m u rm u r is typ ical ly l o u d a s t h e g ra d i ent is h i g h . An a u d i b l e d i a ­ sto l i c com ponent s u g g ests e i t h e r there is a n a ltern ate com m u n i cation rather than VS D, for exa m p le, aortic to atri a l com m u n i cation (e . g . , s i n us of Va lsa lva ru ptu re), o r there i s a n a d d i t i o n a l lesi o n . ANS WER 3: A. I n t h i s patient, the TR Doppler p rofi le is freq uently conta m i n ated by the Doppler signal from the VS D . H e n ce, one m ust be ca ref u l n ot to overes­ t i m ate the TR vel ocity lead i n g to an ove rest i m ate of p u l m o n a ry p ress u res. In a patient with a VS D , it is a l ­ ways prudent t o assess p u l m o n a ry a rtery p ress u res by t h ree methods: (i) Based on the fo l l owi n g equation : RV systo l i c p ressu re = LV systo l i c p ress u re - pea k VS D systol i c g ra d i ent ( i n the a bsence of LV outflow tract obstruction pea k LV systo l i c p ressu re = systo l i c b l ood p ressu re) RV systol i c p ressu re = LV systo l i c p ressu re - 4v2 = LV systo l i c p ressu re - 4(5)2 = 1 40 - 1 00 = 40 mm Hg (ii) By the sta n d a rd fas h i o n (4v2 + right atri a l p ress u re (RAP), where v = peak TR velocity

O n the basis of the est i m ated RV systol i c p ressu re by the VS D g ra d i ent a n d the fact that p u l m o n a ry a rtery d i a sto l i c RA p ressu res a re norm a l , we can work back and esti mate that the tra n stricuspid (RA-RV) g rad ient is a p p roxi mately 35 m m H g , and he nce the expected TR peak velocity is 3 m per seco nd (F i g . 94-8) . I n deed, t h i s corresponds with the one c l e a r s i g n a l that a p pears u n conta m i n ated . See The Echo Manual, 3rd Edition, page 1 62 and Fig­ u re 1 0-1 1 on page 1 64.

ANS WER 4: A. The com b i n ation of a n o r m a l m itra l i nflow patte rn (E/A ratio of 1 . 3), a normal dece l e ration time (200 m i l l iseconds), and a normal E/e ' ratio of 8 . 8 is com pati b l e with n o r m a l d i asto l i c f u n ctio n . ANS WER 5: B . A s m a l l restr i ctive VS D l i ke i n t h i s case ra rely causes cl i n ica l com p l i cation, u n l ess there i s prog ressive va lvu l a r destruction, f o r exa m ple, TR i n this case . A l a rg e r defect may lead to p rog ressive p u l mo­ n a ry hypertension or LV d i lation, either of which wou l d be a n i n d i cati o n o f i nterve n ti o n . A VS D , h oweve r, shou l d n ot cause isol ated RV d i lati o n , as the i n creased vo l u m e of b l ood e nters the RV o n l y d u r i n g systo le, where it passes d i rectly th ro u g h the open p u l m o n a ry va lve i nto the p u l mo n a ry c i rc u l atio n . The p resence of RV e n l a rgement i n the sett i n g of a VS D s h o u l d p ro m pt the ech oca rd i og ra p h e r to seek a n a lte r nate exp l a n a ­ t i o n , for exa m ple, atria l septa l defect, a n o m a l o u s p u l ­ m o n a ry vei n , or p u l m o n a ry hypertension .

CAS E 9 5

Syst o l i c M u rm u r i n a Fem a l e P h l eboto m i st

A 95- 12 and Figs. 9 5 - 1 to 95- 1 1 ) after a systolic murmur was heard on a preemploy­ 24-year-old female phlebotomist is referred for an echocardiogram (Videos 95-1 to

ment physical examination.

310

F i g u re 95-1

F i g u re 95-3

F i g u re 95-2

F i g u re 95-4

CASE 95 I 3 1 1

F i g u re 95-5

F i g u re 95-8

F i g u re 95-6

F i g u re 95-9

F i g u re 95-7

F i g u re 95- 1 0

3 1 2 I E C H O C A R D I O G RA P H Y: A C A S E - B A S E D R E V I E W

QUESTION 4. Quantify the severity of the MR.

A. Mild to moderate B . Moderate to severe C. Severe QUESTION 5. The patient was referred for trans­ esophageal echocardiography (Videos 95- 1 3 to 95- 16) that confirms the findings seen on the trans­ thoracic echocardiogram. Based on the pulmonary vein pulsed wave Doppler interrogation (Fig. 9 5- 1 2) : F i g u re 95-1 1

QUESTION 1. What condition is present?

A. B. C. D. E.

Mitral valve (MV) prolapse Rheumatic MV stenosis Parachute MV MY arcade (Hammock valve) Cleft MV

QUESTION 2. This question relates to the quantifica­ tion of mitral valve regurgitation (MR) by the volu­ metric or continuity method. What is the appropriate location of the sample volume to determine the time velocity integral (TVI) of transmitral flow (Fig. 95- 1 ) ?

A. Pulsed wave Doppler based on a sample volume in position A B . Pulsed wave Doppler based on a sample volume in position B C. Pulsed wave Doppler based on a sample volume in position C D . Depth of sample volume not important as con­ tinuous wave Doppler should be used QUESTION 3. This question also relates to the quan­

tification of MR by the volumetric or continuity method. An inaccurately over measured mitral annu­ lus diameter will lead to: A. An overestimation of the MR B. An underestimation of the MR

F i g u re 95-1 2

A. Findings are consistent with severe pulmonary hypertension B. Findings are consistent with severe MR C. Findings are consistent with elevated left atrial pressure D. Doppler profile is normal

CASE 95 / 3 1 3

QUESTION 6. Based on Figure 95- 1 3 (using a sim­

plified version of the PISA method) , what is the esti­ mated MR volume?

F i g u re 95- 1 3

A. B. c. D.

20 30 40 50

cc cc cc cc

3 1 4 I E C H O C A R D I O G RA P H Y: A C A S E - B A S E D R E V I E W

ANS WER I: D. Two-d i m e n s i o n a l i m a g es d e m o n ­ strate t h i ckened m a rg i n s o f the MV w i t h restricted motion of the a nterior MV leaflet. At fi rst g l a nce, one m i g ht suspect rheu matic MV stenosis with a l m ost a " h oc key sti c k " a p pe a ra n ce to t h e a nt e r i o r l eafl et (Video 95-1 ) . H owever, the l eafl et itse lf is n ot t h i ck­ ened, and n ota bly on short axis, there i s no com m is­ s u ra l fusion (Videos 95-4 and 95-5) . It is a p p a rent that the MV l eafl ets a p pear to d i rectly i n sert i nto the pa p i l ­ l a ry m uscle with l i ttle, if a ny, chordae (Video 95-2) . As we sca n down to the m i d left ve ntricle (Video 95-6), there a re two n o rm a l ly positi o n e d pa p i l l a ry m uscl es excl u d i n g a parach ute MV. Apica l vi ews f u rther u n der­ score that the MV l eafl ets, parti c u l a rly a nterior l eafl et, a re i nserti n g d i rectly i nto the pa p i l l a ry m uscles. The two-ch a m ber views i l l ustrate l eafl ets that d i rectly in­ se rt i nto the pa p i l l a ry m uscles, b u t a l so a re attached with a n echo brig ht, l i kely fi b rous, b a n d of tiss u e . This a rc of tissue restri cts leaflet, partic u l a rly a nte rior leaflet exc u rsi o n . Th is may i m pede l eft ve ntri c u l a r fi l l i n g or as i n t h i s case a lter leaflet geometry a n d g ive rise to reg u rg itati on .

MV a rca d e is a ra re co n g e n ita l a n o m a ly of MV de­ vel o p me nt, whereas i n this case, the MV chordae a re u n d e rdeveloped a n d either very short a n d thick or ab­ sent with the va lve leaflet t i ps i n serti n g d i rectly i nto the pa p i l l a ry m u scl e . It i s p resumed that the lesion a rises as an a rrest in the normal MV deve l o p m ent. Typ i c a l ly, the a nte rior l eafl et is affected to a g reate r extent than the posterior l eaflet. The va lvu l a r orifice, seen on short axis, is norma l . Th is s h o u l d b e reco g n ized a s bei n g d i sti n ct from rheu­ matic m itra l ste n os i s (Videos 95-1 7 to 95-1 9) or a parach ute MV (Videos 9 5-20 to 95-23) for the rea­ sons d i scussed . ANS WER 2: C. U n l i ke the positi o n i n g of the sa m p l e vol u m e f o r the assessment o f d i asto l i c fu nction f o r t h e assessment o f stroke vol u m e t h ro u g h the MV, the sam­ ple m u st be p l aced at the m itra l a n n u l us (opt i o n C ) . A k i n t o t h e stro ke vol u m e ca lcu lation a t t h e l eft ven ­ tri c u l a r outfl ow tract (LVOT) l eve l , the stroke vol u m e is ca l c u l ated afte r the i nteg ration of the m itra l a n n u l a r a rea a n d t h e TVI . ANS WER 3 : A . The M R vo l u me = the M V flow - l eft ventri c u l a r stroke vol u m e If the m itra l a n n u l us is overesti mated, the MV flow is overest i mated . As the rad i us of the m itra l va le a n n u l us

i s sq u a red, erro r i n meas u rement is m u lt i p l ied i n the ca lcu lation of fl ow. See The Echo Manual, 3rd Edition, d i scussion of vol u m etric method in the assessment of MR on page 21 3 .

ANS WER 4: A. Qua ntify i n g MR by the PISA method The reg u rg ita nt flow can be ca l c u l ated as fol l ows : Flow rate = (r)2 x 6 . 2 8 x a l iasi n g velocity = (O . 53 cm)2 x 6 . 2 8 x 5 1 cm per secon d = 90 c c per secon d Effective reg u rg itant orifice (E RO) = F l ow rate/pea k MR velocity = 90 cc per seco n d/5 7 0 cm per secon d = 0 . 1 7 cm2

M R vol u m e = E RO = 0. 1 7

x

x

M R TV 1

1 9 1 = 3 2 cc.

See The Echo Manual, 3rd Edition, d iscussion of PISA method on page 2 1 5.

Q u a ntify i n g MR by the vo l u m etric method

MR vol u me = MV flow - LVOT flow

= (An n u l us 02 x 0 . 7 8 5 x TVl) M v - (a n n u l u s 02 x 0 . 7 8 5 x TVl \voT = (3 . 8 2 X 0 . 7 8 5 X 9 ) M V - ( 1 . 92 X 0 . 78 5 X 2 6)LVOT = 1 02 - 7 4 = 2 8 cc

Summary: The degree of M R by either the PISA or vol u ­ metric method is at the l ower ra nge o f g rade 1 1/IV (m i l d t o moderate) reg u rg itatio n . ANS WER 5: D . I n seve re M R, there may b e systol i c flow reversa l i n t h e p u l m o n a ry vei n ; however, t h e ab­ sence of systo l i c f l ow reve rsa l i n the p u l mo n a ry ve i n does n ot exc l u d e severe M R . T h e systol i c forwa rd flow i nto the l eft atri u m is a lso b l u nted i n other ca uses of e levated l eft atri a l p ressu re . See The Echo Manual, 3rd Edition, Figure 1 2-34 o n page 2 1 3, a n d d i scussion o f p u l m o n a ry vei n flow velocities on pages 1 26 and 1 27 .

ANS WER 6: B . The PISA m ethod ca n be s i m p l ified, which i s especi a l ly h e l pf u l when the pea k ve locity of

CASE 95 I 3 1 5

the M R j et ca n n ot be obta i n ed . 1 I n add iti o n , s i m p l ifica­ tion is h e l pf u l in determ i n i n g the severity of MR i ntra­ operatively when a s u rg i ca l decision needs to be made without d e l ay. The PISA method ca n be s i m p l ifi ed in several d iffere nt ways, a n d all the methods a re based on the sa me form u l a .

Therefore, M R RegV =

---

M Rve I ooty .

=

6.28

x x

=

x

a l i as i n g ve locity

x

M RTV1

= 6.28

M RVe I ooty . x

r2

x

a l i a s i n g ve locity

a l i a s i n g ve locity

P I SA flow rate

------

= 2r2 x a l iasi ng velocity = 2 (0 . 6 52) x 34 = 2 9 cc O n e ca n a l so est i m ate the ERO by assu m i n g the M R ve locity i s 5 m per secon d a s fol l ows : E RO = 6 . 2 8 (r2)

-------

x

3.25

3.25

M RTVI r2

r2

=

Therefore, o n e ca n s i m p l ify the equation as fo l l ows : M itra ! RegV = E RO

x

3.25

See The Echo Manual, 3rd Edition o n page 2 1 6.

The re lation between the TVI a n d the pea k ve locity of MR has been shown to be re l atively consta nt. H e nce, usually

6.28

-------

x

a l i a s i n g velocity ------

M R Ve locity

If one ass u m es the M R velocity is 500 cm per seco n d , then E RO =

6.28

x

r2

x

3 6 cm per secon d

500 cm per secon d x

M RTYi

M RVelocity

Reference 1 . Rossi A, Dujardin KS, Bailey KR, et al. Rapid estimation of regur­ gitant volume by the proximal isovelocity surface area method in mitral regurgitation: can continuous-wave Doppler echocardiog­ raphy be omitted? j Am Soc Echocardiogr. 1 998; 1 1 : 1 38- 1 48 .

= 0 . 2 cm2

= 0 . 4 5 (r2)

CAS E 9 6

Exertional Shortness of Breath with Episode of Atrial Fl utter

A breath. Over the past 9 months, he has had three episodes of symptomatic self-limited

40-year-old man presents for the evaluation of symptoms of exertional shortness of

palpitations, one of which was documented as atrial Butter. He has no other cardiac his­ tory. On examination his blood pressure is 1 40/70 mm Hg. His j ugular venous pulse is normal. He has a mild parasternal lift. Normal heart sounds and a 1 -2/6 systolic ejection murmur along with the lower left sternal border.

QUESTION 1. Based on review of the transthoracic echo images (Videos 96-1 to 96- 1 1 and Figs. 96- 1 to 96-6) , which of the following statements is correct?

F i g u re 96-3

F i g u re 96-1

F i g u re 96-4

F i g u re 96-2

316

CASE 96 / 3 1 7

A. Normal right ventricular (RV) size B. Mild to moderate RV enlargement C. Moderate to severe RV enlargement QUESTION 2. Based on Figures 96-3 and 96-4, which of the following statements is correct?

F i g u re 96-5

A. The tricuspid annular plane excursion (TAPSE) and RV tissue Doppler imaging TDI are both compatible with normal RV contractility B. The TAPSE and RV TDI are both compatible with reduced RV contractility C. The TAPSE but not the RV TDI is compatible with normal RV contractility D. The RV TDI but not the TAPSE is compatible with normal RV contractility QUESTION3. The patient's symptoms are likely due to:

A. B. C. D.

Pulmonary hypertension Tricuspid valve prolapse Sinus venosus atrial septal defect Anomalous right-sided pulmonary venous drainage E. RV cardiomyopathy

F i g u re 96-6

3 1 8 I E C H O C A R D I O G RA P H Y: A C A S E - B A S E D R E V I E W

ANS WER 1 : C. E c h oca rd i o g ra p h i c i m a ges d e m ­ onstrate moderate t o severe e n l a rgement i n R V size. I n the a p i ca l fo u r-c h a m be r vi ew, the RV i s s i m i l a r o r s l i g htly l a rger the l eft ventric l e . ANS WER 2 : A. The m uscle fi bers respon s i b l e f o r t h e maj o rity o f R V contract i l ity a re orie ntated i n the l o n ­ g itud i n a l p l a n e with the base o f the R V movi n g down towa rd the a pex d u ri n g systo l e . Two s i m p l e meas u res of this l o n g itu d i n a l contract i l ity a re the TAPSE obta i ned by p l a c i n g a n M-mode c u rsor thro u g h the tricuspid a n ­ n u l us a n d meas u r i n g the d i sta nce o f a n n u l a r exc u r­ s i o n d u ri n g systo l e (normal exc u rs i o n i s g reater t h a n 2 0 m m) . The oth e r meas u re is a c h i eved by p laci n g a tissue Do pple r sa m p l e vol u m e at the tricusp i d a n n u l us a n d meas u r i n g the peak velocity of systo l i c a n n u la r ex­ c u rsion (normal bei n g > 1 4 to 1 5 cm per seco n d ) . Both meas u res here a re q u ite norm a l . ANS WER 3 : D. E c h oca rd i og ra p hy d e m o n strates moderate to severe RV e n l a rgement with fl atte n i n g of

Reference Vida VL, Padalino MA, Boccuzzo G, et al. Scimitar syndrome. A European Congenital Heart Surgeons Association (ECHSA) multi­ centric study. Circulation. 20 1 0; 1 22 ( 1 2) : 1 1 5 9- 1 1 66.

the i nterventri c u l a r sept u m d u ri n g d i astole com pati b l e w i t h R V vo l u m e overloa d . The t r i c u s p i d reg u rg itant ve l ocity i s u p per normal a n d not the ca use of the RV e n l a rg e m e n t but rather a conseq u e n ce of the lesion descri bed below. S u bcosta l i m a g i n g demonstrates a n a b n o r m a l vesse l d ra i n i n g i nto the i nfe rior ve n a cava op posite the hepatic vei n . C o l o r Doppler i nterrogation demonstrates flow i nto the IVC t h ro u g h both systo le a n d d i asto l e . Pu lse wave i n terrogation confi rms a pul­ m o n a ry ve n o u s flow patte r n . T h i s l e s i o n i s refe rred to as the " Sc i m ita r syn d ro m e " 1 when the r i g ht l owe r p u l monary vei n d ra i ns i nto the IVC i nferior vena cava . The syn d ro m e i s so n a m e d , as the a b n o r m a l p u l mo­ n a ry ve i n (s) g ive a c u rvi l i n e a r pattern on ch est X-ray rese m b l i n g the m i d d l e Easte rn sa b re . T h e lowe r r i g ht l u n g is co m m o n ly hypo p l asti c . S e e The Echo Manual, 3rd Edition, d i scu ssion of anomalous pulmonary vei ns on pages 343 to 345.

CAS E 9 7

Primary B i l i a ry Ci rrhosis

A (Videos 97- 1 to 97-9 and Figs. 97- 1 to 97-8) .

43-year-old woman with primary biliary cirrhosis is referred for echocardiography

F i g u re 97-1

F i g u re 97-3

F i g u re 97-2

F i g u re 97-4

319

3 2 0 I E C H O C A R D I O G RA P H Y: A C A S E - B A S E D R E V I E W

F i g u re 97-8

F i g u re 97-5

QUES TION 1. Appropriate indications for a trans­ thoracic echocardiogram for patients with cirrhosis include:

F i g u re 97-6

A. Screen for the presence of pulmonary hyperten­ sion B. Screen for the presence of intrapulmonary shunt C. Evaluate an asymptomatic 2/6 systolic ejection murmur D. Screen for the presence of pulmonary hyperten­ sion and intrapulmonary shunt E. Screen for the presence of pulmonary hyperten­ sion and intrapulmonary shunt, and evaluate an asymptomatic 2/6 systolic ejection murmur QUESTION 2. The cardiac index (body surface area =

1 . 5) in this case is: A. In the expected range for patients with cirrhosis in patients with cirrhosis B. Not in the range of what is expected in patients with cirrhosis QUESTION 3. Agitated saline administration shows

evidence of:

F i g u re 97-7

A. B. C. D.

lntracardiac shunt Persistent left superior vena cava Intrapulmonary shunt No right-left shunt

C A S E 97 I 3 2 1

QUESTION 4. The significance of a large intrapulmo­

Q UESTION 6. A peak tricuspid regurgitant (TR)

nary shunt in patients with cirrhosis is:

velocity/right ventricular outflow tract time velocity integral (RVOT TVI) greater than 0 . 1 2 in patients with cirrhosis:

A. A relative contraindication to liver transplantation

B. No impact on the timing ofliver transplantation C. A factor that may lead to an earlier transplant date QUESTION 5. The presence of a right ventricular sys­ tolic pressure greater than 60 mm Hg in patients with cirrhosis is: A. A relative contraindication to liver transplanta­

tion B. No impact on the timing of liver transplanta­ tion C. A factor that may lead to an earlier transplant date

A. Suggests portopulmonary hypertension

B. Suggests hepatopulmonary syndrome C. Is typical D. Is of little value

3 2 2 I E C H O C A R D I O G RA P H Y: A C A S E - B A S E D R E V I E W

ANS WER 1: D. Patients with l iver d i sease may com ­ m o n ly have evi d e n ce o f p u l mo n a ry hyperte n s i o n , i n ­ tra p u l m o n a ry s h u nt i n g , a n d systo l i c m u r m u rs . T h e i r s i g n ifica n ce is d i scussed i n s u bseq uent a n swe rs . ANS WER 2: A. The hyperdyn a m i c c i rcu lation asso­ ci ated with c i rrhosis is typ i ca l ly cha racterized by h i g h card iac output a n d i n d ex as i n th is case . C a rd i a c i n dex = [D2

x

0 . 78 5

x

C a rd iac i n d ex = [ 2 . 3 2 x 0 . 78 5 = 5 . 2 Um i n/m2

TVI Lvor x

30

x

x

H R]/B SA

67]/1 . 5

ANS WER 3: C. After a d m i n i stration of a g i tated sa­ l i ne, there i s a d e l ayed r i g ht-l eft s h u n t seen a p p roxi­ mately 6 to 7 beats afte r sa l i n e fi rst a p p e a rs in the rig ht-sided c h a m bers, this is stro n g ly suggestive of an i ntra p u l m o n a ry s h u nt . Fol lowi n g a g i tated sa l i n e a d m i n istration i ntravenously, if an atri a l s h u nt is p resent, b u b b les wi l l a p pear i m me­ d i ately i n the l eft atri u m after be i n g seen i n the rig ht atri u m . In case of a l eft-ri g ht s h u nt, a negative con­ trast effect is seen . If the patient has a n i ntra p u l m o n a ry s h u nt, more t h a n th ree ca rd iac cycles may be needed fo r the b u b b les to go th ro u g h the p u l mo n a ry c i rc u l a ­ tion before they a p pear i n the l eft atri u m . Another i n d ication for a g i tated sa l i ne i s the eva l uation of a persistent left s u perior ve na cava that d ra i n s i nto the coro n a ry s i n us . In this case, ag itated sa l i n e s h o u l d be i njected i nto a l eft a rm ve i n . The e n l a rged coro n a ry s i n u s i n the left atrioventricu l a r g roove wi l l be opaci­ fied fi rst. ANSWER 4: C. The eva l u ation for a n i ntra p u l mo n a ry s h u nt is an i m porta nt one as it is a n i n d icator of possible hepato p u l mo n a ry syn d rome, a p u l mo n a ry vasod i l atory d i sorder that may com p l icate adva nced l iver d isease . 1

References 1. Rodriguez-Roisin R, Krowka MJ . Hepatopulmonary syndrome. N Engl] Med. 2008;358 :2378-2387. 2. Farzaneh-Far R, McKeown BH, Dang D, et al. Accuracy of Dop­ pler-estimated pulmonary vascular resistance in patients before liver transplantation. Am ] Cardiol. 2008; 1 0 1 (2) :259-262.

I n patie nts who h ave evi d e n ce of d e l ayed s h u nti n g on contrast ech oca rd i o g ra p h y a n d have hypoxia (with norm a l ch est X-ray and p u l m o n a ry f u n ction test i n g), a d iag nosis of hepatop u l mo n a ry syn d rome is m a d e . Pa­ tie nts with h epatop u l mo n a ry syn d rome do p a rti c u l a rly poorly without l iver tra nspla ntation, and hence patients may be g iven a h i g her prio rity for tra nspla ntation . ANSWER 5: A. Anot h e r c r i t i ca l co m po n e n t of t h e ech oca rd iogra p h i c assessment o f patients w i t h ci rrho­ sis is a n accu rate assessment of p u l m o n a ry a rte ry p res­ s u res. A sig n ificant m i no rity of patie nts with ci rrhosis h ave a n e l evated p u l m o n a ry a rtery systo l i c p ress u re . T h i s i s o f maj o r s i g n ifica nce as patients with ci rrhosis and u n contro l l ed p u l m o n a ry hypertension have a n ex­ cessive perioperative tran sp l ant morta l ity. An estimated r i g h t ventric u l a r systo l i c p ressu re g reater t h a n 50 m m H g i n a patient with ci rrhosis (or u n exp l a i ned r i g ht­ s i d ed c h a m ber e n l a rg e m e nt) s h o u l d u n d ergo confi r­ matory rig ht heart cath ete rizati on to eva l uate fo r the p resence and seve rity of p u l m o n a ry hyperte n s i o n . Catheterization wi l l h e l p d isti n g u i sh a flow-med i ated p u l m o n a ry hypertension (in the sett i n g of h i g h output) from a p u l m o n a ry a rteriopathy with a h i g h p u l m o n a ry vasc u l a r resi sta nce (porto p u l m o n a ry hyperte n s i o n ) . The p resence o f porto p u l m o n a ry hyperte n s i o n i s a contra i n d i cation fo r l iver tra nspla nt, a lthoug h sel ected patie nts may s u ccessfu l ly receive a tra n s p l a nt if t h e i r p u l m o n a ry hypertension is contro l led with thera py. ANS WER 6: A. Alth o u g h t h e majority of patie nts with ci rrhosis have a h i g h card i a c output and normal/ low p u l mo n a ry vasc u l a r resi sta nce, a p p roxi m ately 5 % o f patie n ts with c i rrhosi s deve l o p p o rto p u l m o n a ry hyperte n s i o n . An e l evati o n i n t h e rat i o the of pea k TR velocity to the RVOT TVI (>0 . 1 2) is associ ated with a n e l evated p u l m o n a ry vasc u l a r resi sta nce on ca rd i ac catheterization . 2

CAS E 9 8

l I

Exertional Dyspnea fol l owing M itra l and Aortic Va lve Replacements

Atissue prostheses with exertional dyspnea. She undergoes transthoracic echocardio­

5 5-year-old woman presents 3 years post mitral and aortic valve replacements with

graphy (Videos 98- 1 to 98-8 and Figs. 98- 1 to 98-6) .

F i g u re 98-1

F i g u re 98-3

F i g u re 98-2

F i g u re 98-4

323

3 2 4 I E C H O C A R D I O G RA P H Y: A C A S E - B A S E D R E V I E W

QUESTION 3. Based on the transthoracic echocardio­

gram, the mitral valve prosthesis likely has: A. Normal function B. Stenosis/obstruction C. Regurgitation QUESTION 4. The patient is referred for transesoph­ ageal echocardiography (Videos 98-9 to 98- 1 6 and Figs. 98-7 to 98-9) . Doppler imaging of left and right pulmonary veins are compatible with severe mitral valve regurgitation. F i g u re 98-5

F i g u re 98-6

F i g u re 98-7

QUESTION I. Calculate the aortic valve prosthesis orifice area, assuming a sewing ring inner diameter of 25 mm and an outer diameter of 28 mm.

A. B. C. D. E.

1 . 0 cm2 1 .25 cm2 2.2 cm2 3 . 0 cm2 3 . 7 cm2

QUESTION 2. The best equation for calculating the

mitral prosthesis effect orifice area is: A. B. C. D.

220/pressure halftime (PHT) 7 5 8/deceleration time (DT) 1 5 0/PHT PHT and/or DT are not helpful in the assess­ ment of mitral valve prosthetic area

F i g u re 98-8

C A S E 98 I 3 2 5

QUESTION 6. Before consideration of periprosthetic

leak closure, the patient developed fever and chills. Blood cultures grew out Strep tococcus bovis. With which of the following medical conditions is S. bovis associated? A. B. C. D.

F i g u re 98-9

A. True B. False QUESTION 5. The mechanism of the tricuspid valve

Rheumatoid arthritis Lung cancer Colon cancer Recent dental extraction

QUESTION 7. Repeat transesophageal echocardiogra­ phy was performed (Videos 98- 17 to 98-24) . Based on the echocardiographic evidence, infective endocar­ ditis is:

A. Unlikely B. Possible c. Likely

regurgitation is: A. Tricuspid valve leaflet prolapse B. Tricupsid annular dilatation C. Tricuspid leaflet perforation

QUESTION 8. The mitral prosthetic surface seen in

Video 98-25 is: A. Atrial B . Ventricular

3 2 6 I E C H O C A R D I O G RA P H Y: A C A S E - B A S E D R E V I E W

ANS WER 1: B. The a rea of a n aortic p rosthesis may be ca l c u l ated by the conti n u ity method s i m i l a r to the way aortic va lve a rea is ca lcu lated i n a n aortic ste n osis case . Ao rtic p rosthesis a rea = LVOT a rea x LVOT TVl/AP TVI = S RO D 2 x 0 . 7 8 5 x LVOT TVI/ AP TVI = 2 . 82 x 0 . 7 8 5 x 2 0/1 00 = 1 . 2 5 cm2 where S R O D , sewi ng ring outer d i a m eter. I n this case, two d i fferent s i g n a l s a re obta i ned of the aortic va lve g ra d i ents. The fi rst from the a p i ca l long axis is clea rly s u bopti m a l with the pea k s i g n a l s obta i ned from the r i g h t s u p raclavicu l a r locati on . As with aortic stenosis cases, it i s critica l to perfo rm a com p l ete Dop­ pler i nte rrogation, with D o p p l e r sa m p les ta ken from all locati ons n ot j ust from the a pex, i n c l u d i n g with the n o n i m a g i n g p robe. ANS WER 2: D. A native m itra l va lve a rea may be esti mated by using either the eq uation 2 2 0/p ressu re ha lft i m e or 7 5 8/decel e ration t i m e . H owever, the PHT (or DT) should not be used to calcu late p rosthetic a reas as th ese equations a re i n accu rate . ANS WER 3: C. The early m itra l i nflow (E) ve locity is very h i g h . Alth o u g h t h i s may s i m p l y rep resent very h i g h l eft ve ntri c u l a r fi l l i n g p ressu res ( i n the a bsence of va lve dysf u n ction), a ve l ocity of >2 m per secon d typ i ca l ly i n d i cates va lve dysfu n cti o n . H oweve r, t h i s m a y refl ect stenosis, reg u rg itati o n , o r both . Little, i f a n y, reg u rg itation is seen on 2 D co lor D o p p l e r i m ag­ ing; h oweve r, there i s s i g n ifica nt acoustic shadowi n g o f the l eft atri u m . A h e l pf u l i n d icator i s the p ressu re ha lfti m e . Here the p ressu re h a lftime is short (60 m i l l i ­ secon d s), a n d th is s u g g ests that t h e e l evated E velocity is l i kely rel ated to reg u rg itati on rather than obstruction (associ ated with a pro l o n g ed p ressu re h a lft i m e) if the va lve is dysfu ncti on a l . F i n a l ly, a high output state may

Reference 1. Alazmi W, Bustamante M, O'Loughlin C, et al. The association of Streptococcus bovis bacteremia and gastrointestinal diseases: a retrospective analysis. Dig Dis Sci. 2006; 5 1 (4) :732-736.

g ive rise to a h i g h E ve locity eve n in the a bsence of p rosthesis dysf u n ction . H oweve r, the norm a l LVOT TVI (20 cm) i n the sett i n g of a normal heart rate (77 beats per m i n ute) exc l udes a h i g h output state . ANS WER 4: A. Both p u l m o n a ry ve i n s i maged dem­ onstrate the p resence of systol i c flow reve rsa ls, w h i c h a re a s p e c i f i c (a lth o u g h not sensitive) s i g n o f severe m itra l valve reg u rg itati o n . See The Echo Manual, 3rd Edition, d i scussion of signs that suggest severe MR on page 2 1 8.

ANS WER 5: B. As is i n d icated on both the tra nstho­ racic (Videos 98-8 and 98-25) a n d tra nseso p h a g e a l (Videos 98-1 5 and 98-1 6) ech oca rd i o g ra m s, there is severe centra l tricusp i d va lve reg u rg itati o n seco n d a ry to lack of leafl et coa ptation with u n d e rlyi n g a n n u l a r d i l atation . ANS WER 6: C. S. bovis is a g ra m positive baci l l us bacteria that is fou n d typ ica l ly i n the lower gastroi ntes­ t i n a l tract . There is a corre l ation between col o n i c ab­ normal ities, particu l a rly colon ca rci noma a n d 5. bovis infecti o n . The identification of S. bovis i nfection s h o u l d p ro m pt a colon oscopy t o eva l u ate f o r poss i b l e colon ca n cer. 1 ANS WER 7: C. I n the sett i n g o f p rosthetic va lvu l a r dysfu nction, fever c h i l ls, a n d positive bactere m i a (with a n org a n i s m com m o n ly associated with i n fective endo­ ca rd itis), the p retest p roba b i l ity for i n fective e n doca r­ d itis i s very h i g h . M oreover, on T E E i m a g i n g , there is a new m o b i l e mass (pres u m ed vegetation) attached to the a ntero latera l m itra l a n n u l us i n the location of the peri p rosthetic reg u rg itatio n . ANS WER B: B . Th ree-d i m e n s i o n a l i m a g i n g (Video 98-2 5) i l l u strates the ventri c u l a r aspect of the m itra l prosthesis. The pati ent we nt on to redo s u rgery with m itra l va lve re- re p lacement.

CAS E 9 9

Exertional Shortn ess of Breath and Chest Pai n i n a 30-Year-Old Woman

A99- 12 and Figs. 99- 1 to 99-9) with exertional shortness of breath and chest pain.

30-year-old woman presents for a transthoracic echocardiogram (Videos 99- 1 to

F i g u re 99- 1

F i g u re 99-3

F i g u re 99-2

F i g u re 99-4

327

3 2 8 I E C H O C A R D I O G RA P H Y: A C A S E - B A S E D R E V I E W

F i g u re 99-5

F i g u re 99-8

F i g u re 99-6

F i g u re 99-9

QUESTION 1. Which of the following dimensions indicates right ventricular (RV) enlargement?

A. Mid RV diameter > 3 5 mm from the apical four-chamber view B. Basal RV diameter > 3 5 mm from the apical four-chamber view C. RV long-axis length > 75 mm from the apical four-chamber view D. RV outflow tract diameter from the parasternal short axis (at the aortic valve level) >23 mm F i g u re 99-7

C A S E 99 I 3 2 9

QUESTION 2. Which of the following clinical sce­

QUESTION 6. The estimated pulmonary arterial pres­

narios best explains the echocardiographic findings?

sures in this case are:

A. Acute saddle pulmonary embolism B. Pulmonary venous hypertension secondary to left-sided valvular disease C. Chronic thromboembolic disease leading to pulmonary hypertension and right heart failure D. Extrinsic pulmonary arterial compression lead­ ing to secondary RV dysfunction E. Primary severe tricuspid valve regurgitation (TR) leading to secondary RV dysfunction QUESTION 3. The hepatic vein Doppler pattern is consistent with all of the following excep t.

A. Marked V waves on examination of the j ugular venous pressure B . Severe TR C. Kussmaul sign D. Marked RV dysfunction QUESTION 4. The severity and mechanism of the TR

is: A. Moderate regurgitation secondary to bileaflet prolapse B . Severe regurgitation secondary to bileaflet pro­ lapse C. Tricuspid annulus dilation with secondary (functional) moderate regurgitation D. Tricuspid annulus dilation with secondary (functional) severe regurgitation QUESTION 5. The calculated cardiac output equals:

A. B. C. D.

3.8 4.4 5.6 6.4

L L L L

per per per per

minute minute minute minute

A. B. C. D.

36/24 64/ 1 6 74126 84/36

mm mm mm mm

Hg Hg Hg Hg

QUESTION 7. Calculate the pulmonary artery (PA)

capacitance. A. B. c. D.

0.6 0.8 1 .0 1 .2

QUESTION 8. Which of the following measures is normal in this patient?

A. Tricuspid annular systolic plane excursion (TAPSE) of 16 mm B . Peak systolic tricuspid annular tissue velocity of 1 1 cm per second C. A right index of myocardial performance by pulsed wave Doppler of 0.7 D . A RV fractional area change of 42% QUESTION 9. Based on the findings, the appropriate

management step should include all of the following

excep t. A. Lifelong warfarin anticoagulation B. Consideration of surgical thromboendarterec­ tomy C. Thrombolytic therapy D. Placement of a vena caval filter

3 3 0 I E C H O C A R D I O G RA P H Y: A C A S E - B A S E D R E V I E W

The reg u rg ita nt flow can b e ca lcu lated a s fo l l ows : ANS WER I: A. Recently p u b l ished g u i d e l i nes o n the echoca rd iogra p h i c assessment of the RV i n d i cate that the " RV d i m e n s i o n is best est i m ated at e n d -d i asto l e from a r i g h t ve ntricle-focused a p i c a l fo u r-c h a m be r vi ew. E i t h e r the d ia m eter >42 m m at the base a n d [or] > 35 mm at the m i d l evel i n d icates RV d i l atation . S i m i ­ l a rly, l o n g i tu d i n a l d i mension > 8 6 m m or t h e d i a meter of the RV outflow tract from the pa ra-sternal short-axis view >27 m m i n d i cates RV e n l a rgement . " 1 ANS WER 2: C. The ech oca rd i o g ra p h i c fi n d i n g s a re of m a rked RV e n l a rg e m e n t a n d dysf u n ct i o n i n the setti n g of severe p u l m o n a ry hyperte n s i o n . I m a g i n g o f t h e p u l m o n a ry a rte r i a l b i f u rcation i n deed d e m o n ­ strates evi d e n ce of t h rom b u s , the l i kely mech a n i sm of the p u l m o n a ry hypertension . Howeve r, the degree of p u l m o n a ry hypertension s u g gests s i g n ifica nt c h ro­ n i city to the p rocess . The RV is exq u i sitely sensitive to acute c h a n ges i n RV afterload and in the acute sett i n g is u n a b l e t o s u p port a right ve ntri c u l a r systo l i c p ressu re (RVS P) h i g h e r t h a n 50 to 60 m m H g . A RVS P h i g h e r than t h i s ra n g e suggests a c h ro n i c p rocess . See The Echo Manual, 3rd Edition, d iscuss io n of chron ic thro m boembolic p u l mo n a ry hypertension on page 1 5 1 .

ANS WER 3: C. The p resence of ma rked systol i c flow reve rsa l s may occ u r i n the sett i n g of seve re TR a n d/ or RV dysfu nction (the earlier pea k i n g of the reversa l i n t h i s case s u g g ests the mech a n ism is more the lat­ ter, as reversa ls due to severe TR te n d to be more l ate pea k i n g ) . The ve n o u s reversa l of flow i n systo l e ca n be a p p reci ated on exa m i nation as p ro m i nent V waves on c l i n ica l exa m i nation . Kussm a u l s i g n (an i ncrease i n venous p ress u re on i n s p i ration) is s i g n with constrictive pericarditis, wh i c h i s associ ated with p ro m i nent dia­ stolic flow reversa ls with expi rati on . ANS WER 4: D. The tricuspid va lve l eafl ets a re nor­ m a l , but a re p u l led a p a rt by d i lation of the tricusp i d va lve a n n u l us seco n d a ry t o R V e n l a rgement. T h i s leads to poor leaflet coa ptati on and functi o n a l reg u rg itation . A n u m ber of factors p resent here suggest the p resence of severe TR. Th ese i n c l u d e ( 1 ) color flow reg u rg ita nt jet a rea �3 0 % of the right atri a l (RA) a rea, (2) dense conti n uous wave Doppler s i g n a l , (3) a n n u l us d i l atation (�40 m m ) a n d i n a d e q u ate cusp coaptation, (4) a n ef­ fective reg u rg itant orifice ( E RO) �0 . 4 cm2, a n d (5) a reg u rg ita nt vol u m e �45 m l .

Flow rate = (r)2 x 6 . 2 8 x a l i a s i n g ve locity x a n g l e correction factor* = (0 . 8cm)2 x 6 . 2 8 x 3 8 cm per seco n d x 2 2 0/1 80 = 1 87 cc per secon d * U n l i ke the f l a t leafl ets o f m itra l va lve, the tricuspid va lve l eafl ets a re fu n n e l s h a ped . T h i s needs to be ac­ cou nted for when ca lcu lati n g PISA. The assu m ption of a h e m i s p h e r i c s h a p e of the flow converg e n ce n eeds to be corrected for by measu r i n g the a n g l e of the leaf­ lets . Typical ly, the a n g l e of the tricuspid l eafl ets is 2 2 0° rather t h a n the 1 80° of the m itra l va lve . Without t h i s adj ustment, the a d d it i o n a l a rea o f flow convergence wo u l d n ot be acco u n ted for if the a n g l e of the leafl ets were n ot meas u red, which wou l d lead to u n derestima­ tion of the reg u rg itation severity. ERO = F l ow rate/pea k TR velocity = 1 87 cc per secon d/400 cm per seco n d = 0 .47 cm2 Reg u rg ita nt vol u m e = E R O x re g u rg i ta n t t i m e ve l o c ity i nteg ra l (TVI) = 0 . 47 cm2 x 1 2 0 cm = 5 6 cc See The Echo Manual, 3rd Edition, d iscussion of PISA method on page 2 1 5, and d i scussion of tricuspid valve reg u rg itation on pages 219 and 220.

ANS WER 5: B. Stroke vol u m e (SV) is ca l c u l ated from the l eft ventri c u l a r outflow tract (LVOT) as a p rod uct of the LVOT a rea (LVOT d i a meter x 0 . 7 8 5) and the LVOT TVI . Here SV = (2 . 0 cm)2 x 0 . 7 8 5 x 1 8 cm = 5 7 m l at a heart rate of 78 beats per m i n ute = 4 . 4 L per m i n ute . See The Echo Manual, 3rd Edition, F i g u re 4- 1 6 on page 7 1 .

ANS WER 6: D . There a re th ree key p a ra m eters i n t h e assessment o f PA p ress u res. I n t h e a bsence o f RV outflow tract obstructi o n , RVS P is e q u iva l ent to the p u l m o n a ry a rtery systo l i c p ressu re (PASP). This can be est i m ated based o n the modified Bernou l l i e q u ati o n : 4v2 + a n esti mate o f R A p ress u re (wh e re t h e v = pea k TR systo l i c velocity) . S i m i la rly, the g rad ient at e n d d i as­ tol e across the p u l m o n a ry va lve (end-d i asto l i c p u l mo­ n a ry reg u rg ita nt ve locity) ca n be used to est i m ate the

CASE 99 I 3 3 1

p u l mo n a ry a rtery d i asto l i c pressu re (PA D P) . I n this case, the d i l ated i nferior ve na cava with l ittle, if any, col l a ps­ i b i l ity with res p i ration s u g gests a ma rked e l evation i n RA p ressu re (conventiona l ly est i mated a t 2 0 m m H g ) . The h e patic ve i n D o p p l e r d e m o n strates systol i c flow reversa ls with o n l y d i asto l i c forwa rd flow (a lso fi n d i ngs that suggest a high RA p ress u re) . PAS P = RVS P = 4 (4)2 = 4 (2)2

+

+

2 0 = 84 m m H g

a n n u l a r ve locities a re both red u ced p a rti c u l a rly for a yo u n g patient where they a re typ i ca l ly m u ch h i g h e r (TAPSE > 1 8 t o 2 0 a n d systol i c tissue D o p p l e r i m a g i n g (TD I) > 1 5) . Not meas u red h e re, the r i g ht i n dex o f myo­ ca rd i a l performa nce (a n i nteg rative ma rker of systol i c a n d d i asto l i c f u n ction b u t p ro n e t o e rror i n t h e sett i n g o f ma rked R A hypertension) g reate r than 0 . 4 is a b n o r­ m a l . The RV fract i o n a l a rea c h a n g e is n o r m a l l y g reater than that of 3 5 % .

20 = 36 mm Hg

S e e The Echo Manual, 3rd Edition, d iscuss io n of tricuspid reg u rg itant a n d p u l monary reg u rg itant velocities on pages 1 44 to 1 47 .

ANS WER 7: D. Est i mated by either ech oca rd iog ra­ phy or catheterizatio n , PA ca pacita n ce (PA CAP) is l i kely a better meas u re of the severity of d i sease i n p u l mo­ n a ry hypertension than p u l m o n a ry vasc u l a r resista nce, as it i n corporates the p u lsati l e com ponent of c h a n ges in p u l m o n a ry a rteria l p ressu re and fl ow, which is u p to o n e h a l f o f t h e powe r tra n sfe rred from t h e RV to the p u l m o n a ry vasc u l a r bed . PA CAP is defi ned as the ratio of ca rd i a c stro ke vo l u m e (SV) to PA p u lse p ress u re wh ich can be derived from the pea k tricusp i d reg u rg i ­ ta nt (TR) a n d t h e e n d d i asto l i c p u l m o n a ry reg u rg ita nt (PRend ) ve locities (Vma J SV was ca l c u l ated i n the a n swer to Q u estion 2 . PV capacita nce = SV/PA p u lse p ressu re = ( LVOT a rea x TVl )/4 (TR V m a x2 PRend vma/) = 57/4(42 - 22) = 5 7/48 = 1 .2

ANS WER 9: C. O n the basis of t h e c h ro n i city of the eve nt, there wi l l be l ittl e g a i ned by th ro m b o lytic therapy as the venous clot wi l l h ave beco m e sca rred and fi b roti c . Anticoa g u lation and fi lter p l acement a re i n d i cated, a l t h o u g h thei r ro le is p revention of fu rther e m b o l i c events . C o n s i d erati o n of th ro m boe n d a rter­ ectomy by a n experienced s u rgeon i n a n experienced center is critica l as outco mes for the patients may be d ra m atica l ly i m p roved . H e re t h i s pati ent did p roceed with s u ccessf u l t h ro m boe n d a rte rectomy ( F i g . 99- 1 O) with resol ution of sym ptoms a n d p u l m o n a ry hyperten­ sion o n fol l ow- u p .

-

ANS WER 8: D. M easu res of RV l o n g i tu d i n a l systo l i c f u n ction a re i m porta nt para m eters t o obta i n i n t h e as­ sessment of RV systo l i c fu nction in patients with p u l ­ m o n a ry hypertension . Measures o f 1 6 m m f o r TAPS E a n d 1 1 cm per secon d for peak systo l i c tricuspid va lve

F i g u re 99- 1 0

References Rudski LG, Lai WW, Afilalo J, et al. Guidelines for the echocar­ diographic assessment of the right heart in adults: a report from the American Society of Echocardiography. J Am Soc Echocardiogr. 20 1 0;23:685-7 1 3 .

Mahapatra S, Nishimura RA, Oh JK, et al. The prognostic value of pulmonary vascular capacitance determined by Doppler echocar­ diography in patients with pulmonary arterial hypertension. JAm Soc Echocardiogr. 2006; 1 9 : 1 045- 1 050.

CASE

1 00

Exertional Dyspnea with System i c Hypertension

A cal history of systemic hypertension treated with lisinopril and hydrochlorothiazide.

n 80-year-old man is referred for an evaluation of exertional dyspnea. He has a medi­

Over the past 2 years, he describes progressive limiting exertional shortness of breath. Work-up to date includes a normal chest X-ray, pulmonary function tests, coronary an­ giogram, and right heart catheterization.

QUESTION 1. There is echocardiographic evidence of

(Videos 100- 1 to 100-8 and Figs. 1 00- 1 to 1 00- 1 0) :

F i g u re 1 00-3

F i g u re 1 00-1

F i g u re 1 00-4

F i g u re 1 00-2

332

CASE 1 00 I 333

F i g u re 1 00-5

F i g u re 1 00-8

F i g u re 1 00-6

F i g u re 1 00-9

F i g u re 1 00-7

F i g u re 1 00-1 0

3 3 4 I E C H O C A R D I O G RA P H Y: A C A S E - B A S E D R E V I E W

A. Pulmonary hypertension B. Normal ej ection fraction low output cardiomyopathy C. Constrictive pericarditis D. Aortic stenosis E. Hypertensive heart disease QUESTION2. The pulmonary vein profile (Fig. 1 00-4)

suggests: A. B. C. D.

The absence ofleft atrial (LA) mechanical function The presence of normal LA pressure The presence of pulmonary arterial hypertension The presence of pulmonary vein stenosis

QUESTION 3. If the systemic blood pressure is 1 00/60 mm Hg, then the LA pressure is approximately:

A. B. C. D.

9 to 1 1 mm Hg 1 4 to 1 6 mm Hg 1 9 to 2 1 mm Hg 25 to 28 mm Hg

QUESTION 4. Which of the following investigations

is most likely to give the diagnosis? A. B. C. D. E. F.

Cardiac MRI Transesophageal echocardiography Dobutamine stress echocardiography Exercise echocardiography Repeat coronary angiography Pulmonary consultation

QUESTION5. Based on your review of the images eos

100-9

to

(Vid­ 100-12 and Figs. 1 00- 1 1 and 1 00- 1 2) ,

what would b e the expected clinical findings?

F i g ure 1 00-1 1

F i g u re 1 00-1 2

A. Harsh systolic murmur B. Systemic hypertension (systolic blood pressure [SBP] > 1 60 mm Hg) C. No audible murmurs D. Irregularly irregular pulse QUESTION 6. What is the recommended manage­

ment strategy? A. B. C. D.

Refer to cardiac surgeon Refer to interventional cardiologist Change in medical therapy No change, routine follow-up

CASE 1 00 I 3 3 5

ANS WER I: E. The i nferior ve na cava (IVC) is of nor­ m a l size a n d co l l a pses norma l ly with i ns p i ration s u g ­ gesti n g n o rm a l r i g h t atr i a l p ress u re (e . g . , 5 m m H g ) a n d m a k i n g constriction q u ite u n l i kely. C o u p led with a pea k tricu s p i d reg u rg ita nt ve locity of 2 . 5 m per sec­ o n d , the esti mated r i g ht ventric u l a r (RV) systol i c p res­ s u re is eq u a l to 4(2 . 5)2 + 5 30 mm Hg (i . e . , normal p u l m o n a ry p ressu res) . The ca l c u l ated ca rd i a c output is e q u a l to 0 . 78 5 x (2)2 x 35 x 88 9.7 L per m i n ute (i . e . , if a nyth i n g , h i g h output n ot low output) . C o n stricti on is u n l i kely with a normal IVC a n d a " no n restrictive " mi­ tra l i nflow pattern that does n ot c h a n g e with res p i ra­ tio n . The aortic va lve seen i n short axis (Video 1 00-2) o p e n s n o r m a l ly without restrict i o n , a n d the m i l d ly e levated g rad ient o bta i ned by conti n uous wave Dop­ pler t h ro u g h the l eft ventri c u l a r outflow tract (LVOT) is l ate pea k i n g , consistent with dyn a m i c outflow tract obstruction rather than fixed va lvu l a r obstruction . The u pper normal wa l l thicknesses and basa l septa l hyper­ trophy a re f i n d i n g s consistent with a n e l d e rly patient with a h i sto ry of system i c hypertensio n . =

=

p u l m o n a ry ve n o u s flow rese m b l es early m itra l fl ow. The pea k ve locity a n d dece l e ration t i m e corre l ate we l l with th ose of m itra l E ve l ocity beca use t h e LA f u n c­ tions m a i n ly as a passive con d u it for flow d u ri n g early d i asto l e . The a n a lysis of p u l m o n a ry ve i n flow velocities com p l e m e nts the assessment of the m itra l flow ve l oc­ ity pattern . Th is i s especia l ly true if the m itra l E a n d A waves fuse. I n t h i s situat i o n , the ratio between p u l ­ m o n a ry ve i n systo l i c a n d d i asto l i c flow ve locities ca n be h e l pf u l i n cha racterizi n g d i asto l i c fi l l i n g i n patie nts with s i n u s rhyt h m (PVS2 >> PVd in i m p a i red rel axation a n d PVS2 < < PVd i n restrictive fi l l i n g ) . ANS WER 3: A. The L A p ress u re may be est i m ated based o n a ca lcu lation of LV p ressu re (LAP) u s i n g the pea k m itra l reg u rg ita nt (M R) velocity to estimate LV sys­ tol i c p ress u re (LVS P) j u st as one uses the pea k tricuspid reg u rg itant velocity to ca l c u l ate RV systol i c press u re . LVS P

=

LVS P a lso

=

=

ANS WER 2 : B . Pu l mo n a ry ve i n Doppler record i n g s show fou r d i sti nct velocity co m ponents (F i g . 1 00- 1 1 ) : two systo l i c velocities (PVS 1 a n d PVS2), d i astolic veloc­ ity (PVd), a n d atria l flow reversa l velocity (PVa). The fi rst systol i c forwa rd flow, PVS 1 , occu rs early in systole a n d is related t o atri a l relaxation, w h i c h decreases L A pres­ s u re a n d fosters p u l monary venous flow i nto the LA. The second systol i c forwa rd flow, PVS2 , occu rs i n m i d t o late systol e a n d is prod uced b y t h e increase i n p u l mo­ n a ry venous pressu re . At normal LA pressu re, the late systol i c increase in p u l monary venous p ressu re is larger a n d more ra pid than LA press u re . However, at elevated fi l l i n g pressu res, the late systol i c pressu re i ncrease in the LA is equal to or more ra pid than that i n the p u l monary vei n , resu lti ng i n ea rlier pea k velocity of PVS2 . The re­ m a i n i n g p u l monary vei n flow velocity com ponents (PVd, PVa, a n d PVS 1 ) fol l ow phasic changes i n LA pressure . W i t h n o r m a l atrioventri c u l a r con d u cti o n , the systo l i c co m po n e nts a re c l osely co n n ecte d , a n d a d i st i n ct PVS 1 peak velocity may not be i d e ntified i n 7 0 % of patie nts (as i n t h i s case) . D u ri n g d i astole, forwa rd flow ve locity (PVd) occu rs afte r o pe n i n g of the m itra l va lve a n d in conj u nction with the decrease in LA p ress u re . W i t h atri a l contract i o n , t h e i n crease i n L A p ress u re may res u l t i n flow reve rsa l i nto the p u l m o n a ry ve i n . The extent a n d d u ration of the flow reve rsa l a re re­ l ated to l eft ve ntri c u l a r (LV) d i a sto l i c p ressu re, LA co m p l i a n ce, a n d h e a rt rate . T h e d i a sto l i c p h a se of

=

117

=

117 117

=

LA p ressu re

=

=

4(Peak MR vel)2 + LAP Syste m i c S B P + LVOT pea k g rad ient 1 00 + 4(2 . 0 5)2 1 00 + 1 6 . 78 1 1 7 mm H g 4 (Pea k M R vel . )2 + LA p ressu re 4(5 . 1 4)2 + LA p ressu re 1 06 + LA p ressu re =

1 1 mm H g

ANS WER 4 : D . Echoca rd i o g ra p hy d e m o n strates a sma l l LV cavity with hyperdyna m i c LV systo l i c f u n ction (high LV ejection fraction and high ca rd i a c output) . The LV global thickn ess is norm a l , but there is hypertrophy of the basa l septu m . There is associated LVOT obstruc­ tion with systo l i c a nte rior motion of the m itra l a p p a ra­ tus with m i l d to moderate M R. At rest there is a pea k g rad ient of 1 8 m m H g , which i n c reases to 2 5 m m H g with Va lsa lva . It i s i m porta nt t o assess whether there is evi dence of i ncreasi n g o utfl ow tract obstructi on with exercise. ANS WER 5: A. Post stress, there was s i g n ifi ca nt dy­ n a m i c LVOT obstru ction with severe M R. I n recove ry (with patient sta n d i ng), the a m o u nt of m itra l a p pa ­ ratu s systo l i c a nterior motion i n creased even f u rther and t h e a m o u nt of MR beco m es very severe; t h i s was associ ated with a sym ptomatic d rop i n S B P from 1 2 7 mm H g to 94 mm H g .

3 3 6 I E C H O C A R D I O G RA P H Y: A C A S E - B A S E D R E V I E W

On the basis of the ava i l a b l e Doppler data , the pea k LVOT ve locity of 5 m per secon d at a t i m e when the correspon d i n g peak MR ve locity was 8 m per seco nd suggests the system i c b lood p ressu re is n ot h i g h .

I n the sett i n g of seve re LVOT obstructi on a n d severe acute M R, the LA p ressu re wo u l d be expected to be ve ry h i g h . I n d eed, the meas u red S B P was 1 2 7 mm H g . i . e . a ca lcu lated LA p ressu re o f 1 5 6- 1 2 7 = 2 9 m m H g .

S i m i l a r t o t h e method i n t h e a n swer t o 3 , LVS P = 4 (Pea k M R vel)2 LVS P a lso = System i c S B P g ra d i ent 4 (Pea k MR vel)2 + LA p ress u re = S B P

+

+

+

LA p ressu re

LVOT pea k

LVOT pea k g rad ient

4 (Peak M R vel)2 LVOT pea k g rad ient = S B P - LA p ressu re 4 (8)2 - 4(5)2 = S B P - LA p ressu re 2 5 6 - 1 00 = 1 5 6 = S B P - LA p ressu re

ANS WER 6: C. Altho u g h a septa l red uction p ro­ ced u re either myecto my or se pta l a b l ation is a con­ s i d erati o n , this s h o u l d be rese rved for patients who fa i l medical thera py. C u rrently the patient is o n two vasod i l ators . These were stopped a n d i n t h e i r place a negative i n otro p i c a nti hyperte nsive (,8-blocker) sta rted . With i n weeks, the pati ent's sym ptoms h a d e n t i re l y resolved .

Index Note: Page number followed by 'f' indicate figures. A velocity, value for, 207, 209 Abasacal echocardiographic score, 267, 268 Abdominal and lower extremity swelling, 1 39- 1 4 1 , 1 3 9/ Abdominal aorta, 9, 9/, 1 0-1 1 Doppler assessment of, 1 0 pulsed wave Doppler signal, 1 9, 1 9/, 2 1 Abdominal aortic velocity, 5, 6 ACE. See Angiotensin-converting enzyme (ACE) inhibition Acoustic shadowing artifact, 296 Acute aortic regurgitation, 38 Acute infarction left anterior descending coronary artery (LAD) , in distribution of, 2 1 3, 2 1 6 Acute myocardial infarction, 232 Acute pleuritic chest pain and lightheadedness, 99- 1 02, 99- 1 00/, 1 0 1/ Acute pulmonary embolism, 46, 47 Acute severe dyspnea, 25/ diagnosis, 25, 27 diagnostic coronary angiography, 25, 27 emergent mitral valve repair, with long-term outcomes, 26, 27 intravenous phenylephrine and increasing intravascular volume, 26, 27 mitral valve, flail scallop of, 25, 27 surgical manipulation of heart, 26, 27 transesophageal echocardiography and, 25, 27 Acute severe retrosternal chest discomfort, 1 03-1 05, 1 03/ agitated saline contrast study, 1 04, 1 0 5 cardiac index, 1 04, 1 05 management, 1 04, 1 0 5 pathology, 1 04 Agitated saline, 1 05, 2 1 3, 238 administration of, 2 1 1 , 303, 305, 320, 322 contrast study, 1 04, 1 05 AIDS, 299 Akinesis, 2 1 5, 2 1 6, 254

ALCAPA. See Anomalous origin of the left main coronary artery from the pulmonary artery (ALCAPA) Aliasing velocity, 78, 78/, 80 a-galactosidase A gene, 1 8 8 Amniotic fluid embolus, 1 1 7 Amyl nitrite, 1 56, 1 56-1 57/, 1 59 Anemia, iron-deficiency, 2 1 9, 220 Angina, severe, 1 1 9- 1 20 Angiotensin-converting enzyme (ACE) inhibition, 1 1 5, 1 1 7 Anomalous origin of the left main coronary artery from the pulmonary artery (ALCAPA) , 238 in infancy, 237, 238 Anterior ischemia, 1 44, 145, 1 45/ Anterior leaflet prolapse, with severe mitral valve regurgitation (MR), 40, 4 1 Anterior mitral valve leaflet prolapse, with flail middle scallop, 39/, 40, 4 1 Antibiotic prophylaxis, fo r dental procedures, 220 Antimicrobial therapy, reinstitution of, 1 98, 1 99 Antineutrophilic cytoplasmic antibodies, 49 Antithrombotic therapy, 1 73, 1 74 Aortic coarctation, 5, 6, 263, 265 Aortic dissection, 9, 1 1 Aortic stenosis, 89, 90 peak velocity of, 1 46, 1 48 Aortic systolic blood pressures, 1 2, 1 4 Aortic valve (AV) effective regurgitant orifice (ERO) of, 227, 229 factors with, 1 9, 2 1 findings, potential explanations for, 1 76, 1 77 mitral valves and, 222, 224 opening status, 1 50, 1 5 0/, 1 5 1 pathology of, 227, 228, 227-228/ Aortic valve area (AVA) , 1 7, 1 8, 43, 45, 60, 6 1 , 1 77 calculation of, 1 7, 1 8, 66-67/, 67, 69, 88, 90, 1 46-1 47, 1 46/, 148 maximal velocity from, 88, 90 mean reduction in, 89, 90 replacement of, 89, 90

Aortic valve cusp, 3 1 , 3 1/, 33 Aortic valve diastolic pressure halftime, 43, 45 Aortic valve endocarditis, 60, 6 1 Aortic valve pressure gradients, Doppler-derived and catheter­ derived, 1 7, 1 8 Aortic valve prosthesis orifice area, calculation of, 324, 326 Aortic valve regurgitation, 1 3 1 , 1 32, 203, 204 degree of, 228, 229 moderate-severe, 1 39, 1 40 severe, 222, 224, 262-263/, 262, 265, 270, 270/, 27 1 , 272 Aortic valve replacement, 60, 6 1 after diagnostic coronary angiography, 68, 68/, 69, 69/ urgent, 60, 6 1 Aortic valve sclerosis with trivial stenosis, 1 82, 1 82/, 1 83 Aortic valve stenosis, symptoms and signs of, 1 7, 1 8 Aortic valve vegetation, 1 99 Apical flow, 1 8 1 Apical hypertrophic cardiomyopathy, 1 1 0, 1 1 0-1 1 1/, 1 1 3 Apical impulse, laterally displaced, 29 1-293, 29 1/ Apical three- or five-chamber view inferobasal septum, 5, 6 Apical variant of hypertrophic cardiomyopathy, 1 79, 1 80, 1 8 1 Ascending thoracic aorta, 252-253, 252/, 254, 254-255/ Ascites, cause for, 52, 54 ASDs. See Atrial septa! defects (ASDs) Aspirin, 275, 283, 284 antithrombotic therapy, 1 73, 17 4 Asthma, 48, 49 Atherosclerotic plaque in thoracic aorta, 2 1 2, 2 1 3 Atrial fibrillation, 1 80, 1 8 1 , 282, 284. See also Symptomatic paroxysmal atrial fibrillation Atrial flow reversal velocity, 75 Atrial flutter, 207, 209 exertional shortness of breath with, 3 1 6-3 1 8, 3 1 6-3 1 7/ 337

3 3 8 I I N D EX

Atrial myxoma, 1 9 1 , 1 92, 1 93, 1 92-1 93/ Atrial septal aneurysms, 2 1 3, 278, 279 Atrial septal defects (ASDs) , 28-29, 30, 292, 293 caval flow verses transseptal flow, 30 sinus venosus defects in, 30 Atropine administration of, 261 use in dobutamine stress echocardiography, 230, 232 ventricular dysrhythmia, 232 Atypical chest discomfort, 1 39- 1 4 1 , 1 3 9/ See also Chest pain, atypical Austin Flint murmur, 87 AV. See Aortic valve (AV) AVA. See Aortic valve area (AVA) Back pain, fever and, 1 98-1 99, 1 99/ Balloon valvuloplasty, 224-225 Benign primary cardiac tumor, 294295, 295/, 296 Bernoulli equation, 4, 6, 1 4, 5 8 ,8-adrenoceptor antagonism, 1 1 5, 1 1 7 ,8-blockers, 1 20, 1 59, 230, 232, 336 ,8-blocker therapy, 302, 304 ,8-lactam and aminoglycoside, combination antibiotic therapy with, 60, 6 1 ,8-myosin heavy chain defects in gene encoding, 302, 304 Bicuspid aortic valve disease, 9, 1 1 , 292, 293 coarctation of aorta, 20, 2 1 configuration of, 2 1 endocarditis prophylaxis and, 1 9, 2 1 Bilateral lower extremity edema, 78-80 Bioprosthesis, thrombotic obstruction of, 1 72-1 73, 1 72/, 1 74 Bioprosthetic thrombosis, prevention of, 283, 284 Biplane area-length method, 1 1 5 , 1 1 7 Blood cyst, 1 6 1 , 1 6 1/, 1 62 BrockenbroughBraunwald-Morrow sign, 1 4 Bromocriptine, 1 1 5, 1 1 7 Bronchogenic cysts, 292, 293 Brugada syndrome, 304 Cabergoline, 140 Carbonated beverage, 73, 74 Cardiac amyloidosis, 128 Cardiac arrest, 1 42-1 43, 143/ Cardiac defibrillator, implantation with, 2 1 5, 2 1 6 Cardiac fibromas, 1 42, 143 Cardiac hemangiomas, 296 Cardiac hemodynamic catheterization with dobutamine infusion, 68, 69 Cardiac index, 1 04, 1 05 , 1 1 5 , 1 1 7, 320, 322

Cardiac murmur etiology of, 46, 47 during pregnancy, 46-47 Cardiac output (CO) , 1 46, 148, 1 83, 2 1 6, 329, 330 calculation of, 4, 6, 227, 228, 294, 294/, 296 Doppler-derived, 23, 23f, 24, 1 2 1 , 1 2 l f, 1 22, 1 3 5 , 1 36, 1 50, 1 5 1 , 208, 209, 246, 247, 27 1 , 272 limiting factor in, 1 50, 1 5 1 stroke volume (SV) and, 6 Cardiac sarcoidosis, 1 88 Cardiac tamponade hepatic veins of, 302/, 302, 304 rwo-dimensional (2D) echocardiographic finding for, 1 63, 1 64 Cardiac troponin T, 1 24, 1 26 genetic defects in, 302, 304 Cardiac vegetation, 1 89, 1 90 Cardiomyopathy, dilated, 244-247, 244-245/, 246/ Carney syndrome, 1 92, 1 94 Carotid disease, 1 82-1 83 Catecholamines, 200, 20 1 , 232 Catheter-derived aortic valve pressure gradients, 1 7, 1 8 Catheterization, 1 06, 1 07 Chamber compression, 263, 265 Chest pain, 297-299, 297-298/ atypical, 240-242, 240/, 242/ and ECG changes, 2 1 4-2 1 6 cardiac defibrillator, implantation with, 2 1 5, 2 1 6 diastolic function, 2 1 5, 2 1 6 Doppler derived stroke volume, 2 1 5, 2 1 6 echocardiogram, 214-2 1 5, 214-2 1 5/ echocardiographic parameters, 2 1 5, 2 1 6, 2 1 5/ further testing, 2 1 5, 2 1 6 myocardium proportion, 2 1 5, 2 1 6 exertional, 1 44-145, 145/ retrosternal. See Retrosternal chest pain severe. See Severe chest pain sharp, 1 9 1- 1 94, 1 9 1-1 92/, 1 92-1 93/ Chest tightness and dyspnea, 1 1 0-1 1 3 , 1 1 0-1 1 2/, 1 1 3/ Chronic cardiomyopathy, 1 1 7 Chronic obstructive lung disease, 54 Chronic thromboembolic pulmonary hypertension, 1 0 1 , 329, 330 Churg-Strauss syndrome, 48, 49, 1 40 Class III exertional shortness of breath without angina, 94-97, 94-96/, 97/ Class III shortness of breath, 22 1-225, 22 1-222/, 225/ CO. See Cardiac output (CO)

Coarctation of aorta, 1 6 1 , 203, 204 Colchicine and indomethacin, combination of, 287, 288 Cold left foot, 2 1 2-2 1 3 , 2 1 2/ Colon cancer, 325, 326 Color Doppler imaging, 1 86 Color map, zero baseline for, 266, 268 Computed tomography (CT) scan, 237, 238, 238/, 286-287, 287/, 288 of chest, 8 1 , 8 1/, 83 Congenital absence of pericardium, 29 1/, 292, 293 Congenital aortic valve stenosis pathology, 1 76, 1 77 status post replacement, 1 75-1 78, 1 75-1 76/ Congenital pulmonary valve stenosis, 64, 65 Constriction, 52, 54 diagnosis, Doppler profiles with right heart failure, 52-53, 5 5 pericardial thickening in, 52, 55 Constrictive pericarditis, 54, 5 5 , 56-58, 56-57/ diagnosis, 57, 5 8 finding in, 8 5-86, 86/, 8 7 physical examination findings for, 86, 87 Continuous wave Doppler echocardiography, 56/, 57 Continuous wave Doppler interrogation, 1 83 across ventricular septal defect, 1 07, 1 32 Continuous wave Doppler signal, 14, 1 8, 1 48 evidence of, 1 86 right ventricular outflow tract, 1 86 Contractile reserve, definition of, 68, 70 Coronary aneurysm, complications of, 263, 265 Coronary angiography, 1 7, 1 8, 1 06, 1 07, 1 2 1 , 1 22, 1 47, 148, 1 54, 1 5 5 , 1 65, 1 66 Coronary artery, 94, 97, 230, 232 grafting of, 78-80, 78-79/ left main, 89, 89/, 90 Coronary artery atherosclerosis, 1 1 9, 1 20 Coronary artery spasm, 200, 20 1 Coronary sinus, 34, 36 dilatation of, 1 54, 1 5 5 Coronary-ventricular fistula, 1 3 Corticosteroid therapy, immunosuppression with, 1 99 Cor triatriatum, 9 1 , 93 Cross-sectional area (CSA), 1 83 CSA. See Cross-sectional area (CSA) CT scan. See Computed tomography (CT) scan Cusps, right and left, 1 9, 1 9/, 2 1

I N D EX I 3 3 9

DC cardioversion, 235, 235/, 236 DeBakey rype III aortic dissection, 1 98, 1 99 Descending thoracic aorta, 2 1 0, 2 1 0/, 2 1 1 , 2 1 1/, 2 1 2, 2 1 3 Desmin gene mutations, 302, 304 Diabetes mellitus, progressive exertional shortness of breath with, 1 87-1 88, 1 88/ Diabetic nephropathy, 273-275, 273/, 274/ Diastolic and systolic murmurs, 262, 262-263/ Diastolic dysfunction grade of, 8-9, 8/, 1 0 for heart failure, 84, 87 mitral inflow pattern, 260, 261 Diastolic filling, restrictive, 2 1 5, 2 1 6 Diastolic flow reversal, 5 1/, 5 5 abdominal aortic velociry, 6 consistent with constriction, 5 5 i n constrictive pericarditis, 8 5-86, 87 in descending aorta, 1 39, 1 40 in hepatic vein, 257, 258 severe aortic valve regurgitation, 272 Diastolic function, 73, 74 assessment of, 286, 288 chest pain and ECG changes, 2 1 5, 2 1 6 grade, 200, 200/, 20 1 myocardial infarction, 23, 23/, 24 mildly abnormal, 1 65, 1 66 moderate to severely abnormal, 1 27, 1 29 myocardial relaxation and, 207, 209 normal, 308, 309 progressive NYHA class III dyspnea, 67, 69 severely abnormal, 1 1 1 , 1 1 3 Diastolic mitral valve regurgitation, 1 76, 1 76/, 1 77-1 78 Diastolic retrograde flow, 140 Diastolic reversal flow time velociry integral (TVI) , 5, 6 Diffuse ST-segment depression, 200-20 1 , 200/ Dilated cardiomyopathy, 244-247, 244-245/, 246/ Dissection, concomitant risk of, 2 1 Dobutamine, 2 6 1 Dobutamine-atropine stress echocardiography, 259, 261 Dobutamine infusion cardiac hemodynamic catheterization with, 68, 69 indications for cessation of, 230, 232 Dobutamine stress echocardiography, 1 08-1 09, 1 09/, 230, 232, 273, 275 atropine use in, 230, 232 evidence, 230, 232

nonsustained ventricular tachycardia, 232 Doppler-derived aortic valve area, 43, 45, 66-67/, 67, 69 Doppler-derived aortic valve pressure gradients, 1 7, 1 8 Doppler-derived cardiac output (CO) , 23, 23/, 24, 1 2 1 , 1 2 1/, 1 22, 1 3 5 , 1 36, 1 50, 1 5 1 , 208, 209, 246, 247, 27 1 , 272 Doppler-derived left ventricular cardiac index, 1 1 5, 1 1 7 Doppler-derived left ventricular stroke volume (SV), 67, 69, 2 1 5, 2 1 6 Doppler findings constellation of, 258 and echo two-dimensional, 82, 82/, 83, 1 82, 1 82/, 1 83 moderate-severe aortic valve regurgitation, 1 39, 140 pulmonary artery end-diastolic pressure, 57, 5 8 Doppler flow, 1 79-1 80, 1 80/, 1 8 1 Doppler profiles, with right heart failure constriction diagnosis, 52-53, 5 5 Doppler stress echocardiography, 1 57, 1 59 Dyskinesis, 254 Dyspnea, 50, 1 2 1- 1 22. See also Progressive dyspnea and asymmetrical ground-glass pulmonary infiltrates, 1 70-1 7 4, 1 70- 1 7 1/, 1 72/ transesophageal echocardiograrn, 1 72, 1 74 and chest tightness, 1 1 0-1 1 3, 1 1 0-1 1 2/, 1 1 3/ and fever with history of severe pancreatitis, 289, 289/, 290 intermittent with modest exertion, 230-232 with modest exertion diastolic function, 1 65 , 1 66 etiology for, 1 65 , 1 66 and orthopnea, 34-36, 34-35/ palpitations and, 207-209, 207/, 208/ and presyncope, 1 46- 1 48, 1 46/, 1 47/ progressive exertional shortness of breath after, 1 84-1 86, 1 84-1 8 5/ Dysrhythmia etiology of, 1 42, 1 43 treatment of, 1 42, 1 43, 143/ Dyssynchrony of myocardial relaxation, 1 79-1 80, 1 80/, 1 8 1 Early diastolic medial annular peak tissue velociry (e') , 52, 54 Ebstein anomaly, 1 6 1 , 1 62, 1 62/

Echo contrast administration, 298, 299 Echo two-dimensional findings moderate-severe aortic valve regurgitation, 1 39, 140 ECM 0 catheter, 46 redrawing, 46, 47 EF. See Ejection fraction (EF) Effective regurgitant orifice (ERO) , 32, 33, 76, 77, 78, 80, 1 50, 1 5 1 , 1 52-1 54, 1 5 5, 1 68, 1 69, 1 86, 234, 236, 250, 25 1 , 265, 330 of aortic valve, 43, 45, 227, 229 calculation of, 40, 4 1 , 274, 275, 277, 279, 282, 284 formula for, 78, 80 of mitral valve, 1 95 , 1 97 Eisenmenger syndrome, 2 1 9, 220 Ejection fraction (EF), 4 Elevated right ventricular (RV) enddiastolic pressures, 1 85 , 1 85/, 1 86 Emergency cardiac surgery, 1 65 , 1 66 Emotional stress, 1 1 9, 1 20 End-diastolic velociry of pulmonary regurgitation (PR) , 5 8 Endocarditis, 272, 275 Endocarditis prophylaxis, 2 1 9, 220 Endomyocardial biopsy, 1 2 1 , 1 22 Endothelin receptor antagonists, 220 Enlarged coronary sinus, cause of, 1 54, 1 5 5 Enterococcus, 1 78 Epstein-Barr virus, 299 Ergotamine, for migraine headaches, 1 39, 140 ERO. See Effective regurgitant orifice (ERO) Eustachian valve, 2 1 0, 2 1 1 Exercise stress echocardiography anterior ischemia and, 1 44, 145, 145/ severe ischemia on, 1 44, 145 Exertional chest pain, 1 44-145, 145/ Exertional dyspnea, 1 14, 1 14-1 1 5/ Doppler-derived left ventricular cardiac index, 1 1 5, 1 1 7 etiology for clinical and echocardiographic findings, 1 1 5, 1 17 left atrial (LA) volume index, 1 1 5, 1 17 left ventricular ejection fraction (LVEF) , calculation of, 1 1 5 , 1 1 7 management, 1 1 5, 1 1 7 progressive, 248-25 1 , 248/, 249/, 250/ repeat transthoracic echocardiography, 1 1 6, 1 1 6/, 1 1 7 with systemic hypertension, 332, 332-334/ clinical findings, 334, 334/, 335-336

340 I I N D EX

Exertional dyspnea (continued) diagnosing, 334, 335 echocardiography evidence, 332334, 332-333/ left atrial (LA) pressure, 334, 335 management strategy, 334, 336 pulmonary vein profile, 332/, 334, 335 Exertional fatigue, 1 2 1- 1 22, 1 2 1/, 270-272 Exertional shortness of breath, 42-45, 42-43/, 78-80, 1 79-1 8 1 , 1 79-1 80/, 234-236, 234-235/, 2 8 1 -284, 28 1/, 282/, 283/ See also Progressive exertional shortness of breath and chest pain, 327-33 1 , 327-328/, 33 1/ with episode of atrial flutter, 3 1 6-3 1 8, 3 1 6-3 1 7/ with history of lymphoma, 256-258, 256-257/ and lower extremity edema, 28-30, 28/ atrial septal defects (ASDs) and, 28-29, 30 evidence, 28, 28/, 30 mean pulmonary artery pressure (MPAP) , 28, 30 with no medical history, 1 33, 1 33-1 3'if 1 3 5-1 36, 1 3 5/ after transesophageal echocardiography, 1 35 , 1 36 slowly progressive, 266-269, 266-267/ Eisenmenger syndrome and, 2 1 9, 220 endocarditis prophylaxis, 2 1 9, 220 examination, unexpected finding on, 2 1 8, 2 1 8/, 220 pulmonary artery (PA) end­ diastolic pressure, estimation of, 2 1 8, 220 right ventricular longitudinal contractility, 2 1 8/, 2 1 9, 220 right ventricular systolic pressure, estimation of, 2 1 9, 220 risk of, 2 1 9, 220 with systolic blood pressure, 2 1 8-220, 2 1 8/ Fabry disease, 1 88 Fainting episode, soft systolic murmur after, 300-305, 300-302/, 304/ Fatigue, 1 67-1 69 Fat, pericardial, 1 37, 1 37/, 1 38 Femoral pulses, 2 1 Fenfluramine and dexfenfluramine, 140 "Fen-Phen," 140 Fever, 1 67-1 69

and back pain, 1 98-1 99, 1 99/ on examination, 1 98-1 99 treatment undergone, 1 9 8 and dyspnea, with history o f severe pancreatitis, 289, 289/, 290 Fibrinolytic therapy, in pulmonary embolism, 1 0 1 Flow rate, 1 83 Foramen ovale patent, 2 1 2, 2 1 3 Functional class III exertional dyspnea, 202/ associated lesions, 203, 204 diagnosis, 202, 204 instantaneous maximum gradient, 202, 204 management, 203, 204 Functional class III shortness of breath, 282, 282-283/ Functional ventricular chamber, 1 1 3 Hammock valve. See Mitral valve (MV) arcade Harsh pansystolic murmur, 307, 309 Harsh systolic murmurs, 5 9-6 1 , 59/, 60/, 63-65, 63-64/, 334, 334/, 335 acute management, 64, 65 echocardiogram of, 5 9-60/ echocardiographic images, 60, 6 1 management, 6 0 , 6 1 obtaining transesophageal echocardiography, 60, 6 1 HCM. See Hypertrophic cardiomyopathy (HCM) Headache, 2 1 9, 220 Heart failure, 84-87, 84-86/, 1 80, 181 etiology for, 84, 87 Heart Mate II, 1 5 1 Heart murmur, 1 6- 1 8 , 1 6- 1 7/ Hematuria with large renal mass, 88-90, 88/ etiology for, 89, 89/, 90 Hemochromatosis, 1 22 testing for, 1 2 1 , 1 22 Hemodynamic cardiac catheterization, transesophageal echocardiogram of, 6 1 Hepatic carcinoid metastases with patent foramen ovale, 1 39, 1 40- 1 4 1 Hepatic vein of cardiac tamponade, 302-303, 303/, 304-305 diastolic flow reversal in, 257, 258 dilation of, 1 63, 1 64 Doppler pattern, 1 37, 1 37/, 1 38, 1 3 8/, 329, 330 Doppler with respirometer, 56/, 57, 5 8

flow patterns, explanation for, 249, 249/, 250/, 25 1 Hepatopulmonary syndrome, 322 Hereditary hemorrhagic telangiectasia, 50-5 1/, 52, 54 HIV infection, 1 22 "Hockey-stick'' appearance, 1 5 5, 268, 284, 3 1 4 Holodiastolic flow reversals, in descending thoracic aorta, 263, 265 Hydrochlorothiazide, 332 Hyperlipidemia, 1 44, 1 82-1 83 Hypertension, 1 82-1 83 treatment for, 37, 38 Hypertensive heart disease, 334, 335 Hypertrophic cardiomyopathy (HCM) , 1 3, 1 56, 1 59, 1 79, 1 8 1 , 304 complications associated with, 1 80, 1 8 1 with gene defect, 304 sudden cardiac death risk stratification, 1 4 Hypertrophic obstructive cardiomyopathy, 1 4- 1 5 Hypokinesis, 254 Hypotension, 1 09 with exercise, for severe coronary disease, 1 44, 1 45 treatment strategies for, 1 1 9, 120 Idiopathic dilated cardiomyopathy, 1 22 Immunosuppression, 1 1 7 Indeterminate left ventricular filling, 286, 288 Indomethacin and colchicine, combination of, 287, 288 Infective endocarditis, 1 76, 1 77, 2 1 9, 220, 325, 326 embolic complication of, 1 89, 1 90, 274, 275 predisposes to, 1 89, 1 90 Inferior and lateral ischemia, 1 08, 1 09, 1 09/ Inferobasal septum, 5, 6 Inflow cannula systolic velocity profile, 1 50, 1 50/, 1 5 1 Inhaled prostacyclin analogs, 220 Instantaneous maximum gradient, 202, 204 Interventricular dependence, 85, 8 5/, 87 Intra-aortic balloon pump, 27, 77 and surgical consultation, 1 06, 1 07 Intra-atrial septum, 1 5 1 Intracardiac right-left shunt, 1 04, 1 05 Intracardiac thrombus, 1 24, 1 26 Intraoperative transesophageal echocardiography, 1 77-1 78

I N D EX I 3 4 1

Intrapulmonary shunt, 320, 322 echocardiographic evidence of, 52, 54 in patients with cirrhosis, 32 1 , 322 Intravenous fluid and inotrope administration, 1 04, 1 05 Intravenous phenylephrine, 1 1 2, 1 1 3, 1 1 3/ and increasing intravascular volume, 26, 27 Intravenous thrombolytic therapy, 283, 284 Iron-deficiency anemia, 2 1 9, 220 Iron studies, 1 2 1 , 1 22 Ischemia, 263, 265 without echocardiographic evidence of, 273, 275 Ischemic cardiomyopathy, 237, 238 Isosorbide mononitrite, 208, 209 Kussmaul sign, 330 LAA .

See Left atrial appendage ( LAA) Lambl excrescences, 242 Lateral wall ischemia, 230, 232 Left anterior descending coronary artery acute infarction in, 2 1 4-2 1 5, 2 1 4-2 1 5/, 2 1 6 wall motion abnormalities in, 44, 45 Left atrial ablation procedure, 8 1 , 83 Left atrial appendage ( LAA) , 253, 253/, 255, 288 flow velocities, 235, 236 Left atrial hypertension, 1 28, 1 29 Left atrial index, 286, 288 Left atrial pressure, 334, 335 normal, 8 1 , 8 1/, 83 Left atrial radiofrequency ablation, 286, 286/, 288 Left atrial systolic strain assessment, 288 Left atrial thrombus, 267 Left atrial volume, 4, 6 calculation of, 286, 288 index, 1 1 5, 1 1 7 Left atrioventricular (AV) groove, round structure in, 34, 36 Left coronary cusp, 3 1 , 33 Left heart failure, recurrent episodes of, 276-280, 276/, 277/, 279/ Left inferior pulmonary vein, 1 35, 1 36 Left main coronary artery, 89, 89/, 90 Left pleural effusion, 276, 279 Left ventricular (LV) aneurysm, 23, 24 Left ventricular assist device, 1 50, 1 5 1

Left ventricular cardiac index, Doppler-derived, 1 1 5, 1 1 7 Left ventricular cardiomyopathy, 57, 58 Left ventricular diastolic filling pressure, 67, 68 on mitral inflow, 1 63, 1 64 Left ventricular diastolic function, 9 1-92, 9 1-92/, 93, 257, 258 Left ventricular dilation, 145, 308, 309 Left ventricular dimensions, measurement of, 7, 7f, 1 0 Left ventricular ejection fraction (LVEF) , 4, 6, 1 0, 24, 46, 47, 1 63, 1 64, 1 95, 1 97, 208, 209, 2 1 5, 2 1 6, 245, 247, 282, 284 calculation of, 1 1 5, 1 1 7, 24 1 , 242, 252, 252/, 254 estimation of, 2 1 0, 2 1 1 at rest, 273, 273/, 275 two-dimensional, M-mode, 275 Left ventricular end-diastolic dimensions (LVEDD) , 254 Left ventricular end-diastolic pressure (LVEDP) , 43, 45 Left ventricular end-systolic dimensions (LVESD), 254 Left ventricular filling pressures assessment of, 286, 288 increase of, 34-35, 34-35/, 36 normal, 1 50, 1 5 1 patient's dyspnea cause and, 95-96, 97, 97/ severely elevated, 1 23, 1 2 5 Left ventricular fractional shortening (FS), calculation of, 1 67-1 68, 1 67/, 1 69 Left ventricular function, 48, 49 wall motion abnormalities and, 48, 49 Left ventricular hypertrophy, 1 88, 203, 204 Left ventricular opacification, 1 44, 145 Left ventricular outflow tract (LVOT) to AoV TVI ratio (LVOT:AoV TVI) , 1 7, 1 8 cardiac output of, 227, 228 diameter, 294, 296 gradient, 13, 1 3/, 14 obstruction, 14, 1 5, 89, 90 signal, 1 4 time velocity integral (TVI) o f, 46, 47, 88, 90 Left ventricular stroke volume (SV), 1 82, 1 82/, 1 83 Left ventricular systolic function, 69, 208, 208/, 209 Left ventricular systolic pressure, 1 4

Left ventricular wall segment, 2 1 2, 2 1 2/, 2 1 3 Leiomyosarcoma, 298, 299 Libman-Sacks endocarditis, 1 97 Lightheadedness and severe fatigue, 1 06-1 07, 1 06/, 1 07/ Lipomatous tissue, 1 3 8 Lisinopril, 332 Lithium, 1 40 Liver function test abnormalities, 50-5 5, 50-5 1/, 52-53/ Longitudinal right ventricular contractility, 2 1 5, 2 1 6 Loud systolic murmur, 1 30-1 32, 1 30/, 1 3 1/, 2 1 8, 220 Lower extremity edema, 234-236, 234-235/, 270-272 progressive exertional fatigue and, 1 6 1- 1 62, 1 6 1/, 1 62/ Lupus anticoagulant, 1 96, 1 97 LVEDD. See Left ventricular end­ diastolic dimensions (LVEDD) LVEF. See Left ventricular ejection fraction (LVEF) Lymphoma, exertional shortness of breath with, 256-258, 256-257/ Malaise, 1 67-1 69 Malignant tumors, 298, 299 Maximal instantaneous intracavitatory gradient, calculation of, 12, 1 4 Maximum instantaneous gradient, 202, 204 Maximum instantaneous systolic gradient across pulmonary valve, 64, 65 Mean pulmonary artery pressure (mPAP), 28, 30, 1 72, 1 74 Mechanical index, 1 08, 1 09, 1 44, 145 Medial mitral annulus, tissue Doppler velocity of, 75, 75/, 77 Medications, for palpitations and dyspnea, 208, 209 Membranous ventricular septal defect (VSD), 1 3 1 , 1 3 1/, 1 32, 306, 309 Mesothelioma, 299 Metastatic carcinoma, 205, 205/, 206 Methicillin-sensi tive Staphylococcus aureus, 1 89, 1 90 Metoprolol succinate, 1 56 Microvascular dysfunction, 200, 20 1 Mid inferior wall left ventricular wall segment, 2 1 2, 2 1 2/, 2 1 3 Midsystole, 32, 33 Migraine headaches, ergotamine for, 1 39, 1 40 Mild aortic regurgitation, 43, 45 Mild-moderate stenosis, 15 5

342 I I N D EX

Mitral and aortic valves, pathology affecting, 222, 224 Mitral and tricuspid valve replacements, 1 54, 1 5 5 Mitral annular early diastolic velocities (e') , 52, 54 Mitral annulus diameter, 3 1 2, 3 1 4 Mitral annulus dilation, with secondary regurgitation, 235, 236 Mitral annulus early diastolic velocity, in constriction, 86, 87 Mitral annulus velocity, 57, 57/, 58 Mitral balloon valvuloplasty, outcome of, 267, 268, 269t Mitral bioprosthesis, pressure halftime of, 1 72, 1 74, 248, 248/, 25 1 Mitral effective regurgitant orifice (ERO) area, 234, 234/, 236 Mitral inflow, 56/, 57, 58, 75, 75/, 77 A wave duration for, 75, 76/, 77 color M-mode of, 97 deceleration time (OT) of, 84-85, 87 pattern of, 8/, 1 0, 74, 259, 259/, 261 respiratory variation in, pattern of, 5 1/, 52, 54, 1 63, 1 64 restrictive, 97 stroke volume, calculation of, 247 Mitral lateral annulus e' velocity, 67, 69 Mitral prosthesis effect orifice area, calculation of, 324, 326 flow velocities for, 1 72, 17 4 surgical re-replacement of, 250, 25 1 ventricular aspect of, 325, 326 Mitral regurgitant volume, 33, 247 Mitral regurgitation, 1 4 severity of, 34, 3 6 , 76, 77 Mitral stenosis assessing degree of, 266, 268 pulsed wave Doppler and, 39/, 40, 4 1 severe, 267, 269 Mitral tissue prosthesis, 282, 284 Mitral valve (MV) 30 image of, 266, 268 anterior leaflet prolapse with severe mitral valve regurgitation (MR) , 40, 4 1 effective regurgitant orifice (ERO) area of, 1 95, 1 97 Mitral valve arcade, 3 1 2, 3 1 4 Mitral valve area (MVA) , 266-267, 266-267/, 268, 269 by continuity equation on, 223, 224 by pressure half-time (PHT) method, 223, 224 Mitral valve disease, mechanism of, 282, 284 Mitral valve endocarditis, 1 96, 1 97

Mitral valve leaflets flail scallop of, 25, 27 scallops of, 24 1 , 242, 242f Mitral valve pathology, 1 76, 1 77 diagnosis of, 3 1 , 33 Mitral valve periprosthetic regurgitation, location of, 264, 265 Mitral valve pressure halftime (PHT) , 267, 268 Mitral valve prolapse, 3 1 , 33, 236 Mitral valve prosthesis, 324, 324-325/, 326 Doppler interrogation of, 282, 283/, 284 Mitral valve prosthesis area, 79, 79/, 80, 1 92, 1 94 Mitral valve prosthetic gradient, 1 72-1 73, 1 72/, 1 74, 277, 279 Mitral valve prosthetic regurgitation, 248, 25 1 Mitral valve regurgitant fraction, calculation of, 246, 247 Mitral valve regurgitation (MR) , 1 89, 1 90, 237, 238 degree of, severe, 1 96, 1 97, 249, 25 1 mechanism of, 234, 234-235/, 236 moderate-severe, 32, 33 quantification of, 3 1 2, 3 1 4 regurgitant volume of, 1 96, 1 97 severe, 34, 36, 262, 265, 3 1 4, 325, 326 volume, 1 67-1 68, 1 67-1 68/, 1 69, 3 1 3, 3 1 3/, 3 1 4-3 1 5 Mitral valve stenosis, severe, 223, 224 Mitral valve surgery, referral for, 40, 4 1 Mitral valve tenting area, 34, 36 M-mode echocardiogram, 85, 85/, 87 M-mode echocardiography, of left atrial myxoma, 1 9 1- 1 92, 1 92/, 1 93 M-mode recordings showing pulmonary hypertension, 1 3 5 , 1 36 M-mode tracing, of hypertrophic cardiomyopathy, 1 57-1 5 8, 1 57/, 1 59 Mobile structure in right atrium, 2 1 0, 2 1 1 Monomorphic ventricular tachycardia, after cardiac arrest, 1 42-1 43, 1 43f Montelukast, 1 40 Motor function transient loss of, in right upper extremity, 2 1 0-2 1 1 , 21 lf Murmur, 270-272, 306-309, 306-307f asmptomatic, 226-229, 226-227/, 228-228f intervention, 308, 309 in adulthood, 237, 238 computed tomography scan, 237, 238, 238f

in infancy, 237, 238 management, 237, 238 on routine physical examination, 237-238, 238! physical examination findings in, 307, 309 with regular exercise, 226-229, 226-227/, 227-228f with systemic hypertension, 205-206 etiology of, 205, 205/, 206 Muscular ventricular septal defect, 1 30, 1 32 Myocardial echo contrast intravenous infusion, 246, 247 Myocardial infarction, 1 80, 1 8 1 complications, 23, 24 diastolic function grade, 23, 23/, 24 Doppler-derived cardiac output (CO) , 23, 23/, 24 extent of, 2 1 5, 2 1 6 management, 23, 24 prognostic factors, 23, 24 Myocardial ischemia, 97 dobutamine stress echocardiography, 1 08, 1 09 Myocardial relaxation, 54, 207, 209 dyssynchrony of, 1 79-1 80, 1 80/, 1 8 1 Myocardial thickness, 2 1 5, 2 1 6 Myocardium, stunned, 2 1 6 Myxomas, 1 93, 206, 242 Carney syndrome with, 1 92, 1 94 symptoms of, 1 92, 1 94 Myxomatous mitral valve disease with flail segment, 25, 27 Nafcillin, 1 90 Near-field clutter artifact, 295, 295/, 296, 296f Nephropathy, diabetic, 273-275, 273/, 274f Noise artifact, 295, 295/, 296, 296f Noncoronary cusp to right atrium, 79, 79/, 80 Normal images and values, 1-6, 1-'if Normal left ventricular (LV) myocardial relaxation property, 4, 6 Normal mitral prosthetic function, 277, 279 NT-pro-BNP, 1 24, 1 26 NYHA functional class III, 1 2- 1 5 , 1 2- 1 3f Omeprazole, 1 40 Osler-Weber-Rendu syndrome, 1 94. See also Hereditary hemorrhagic telangiectasia Oxygen saturation samples, 1 06, 1 07

I N D EX I 343

Palpitations and dyspnea, 207-209, 207/, 208/ A velocity, value for, 207, 209 diastolic function, 207 E/e' ratio, 207, 209 left ventricular systolic function, 208, 208/ medications, 208, 209 Pancreatitis, severe, 289, 289/ Pansystolic murmur, harsh, 307, 309 Papillary fibroelastoma (PFE) , 89, 89/, 90, 206, 240, 242 Papillary muscle rupture, 76, 77, 23 1 , 232 Paraganglioma, 265 Parasternal long-axis imaging, 74, 1 37, 1 37/, 1 38, 1 56, 1 59 Patent ductus arteriosus (PDA), 63/, 64, 65, 1 39, 1 40, 1 6 1 Patent foramen ovale (PFO) , 5 , 6 , 1 28, 1 29, 2 1 2, 2 1 3 with bidirectional flow, 1 87, 1 88, 1 88/ and higher risk of stroke, 2 1 2, 2 1 3, 278, 279 with left-right shunt, 1 98, 1 99, 1 99/ PDE5 inhibitors, 220 Peak left ventricular systolic pressure, 1 1 9, 1 20 Peak transaortic pressure gradient, 1 7, 1 8 Peak tricuspid regurgitant (TR)/ right ventricular outflow tract time velocity integral (RVOT TVl) , in patients with cirrhosis, 32 1 , 322 Peak velocity of aortic stenosis, 1 46, 148 from continuous wave Doppler interrogation, 1 07 mitral early filling, 69 across pulmonary valve, 65 versus time velocity integral, 3 1 5 across ventricular septal defect, 1 32 Pergolide, 140 Pericardial cyst, 263-264, 265 Pericardial effusion, 1 64 Pericardial fat, 1 37, 1 37/, 1 3 8 Pericardial fluid, laboratory findings on, 303, 305 Pericardial thickening, in constriction, 52, 5 5 Pericardial tumors, 298, 299 Pericarditis, constrictive, 56-5 8, 56-57/ Pericardium, congenital absence of, 29 1/, 292, 293 Peripartum cardiomyopathy, 1 1 7

Peripheral edema, 1 52, 1 52-1 54/, 1 54-1 5 5 Peripheral neuropathy, 48-49 Periprosthetic leak, location of, 277, 279 Perivalvular leak, percutaneous repair of, 278, 280, 279/ Phenylephrine, 37, 38 infusion of, 1 1 3 Pheochromocytoma, 1 1 9, 1 20, 20 1 , 206 Pleural effusion, 265 Portopulmonary hypertension, 322 Posterior mitral leaflet perforation, 274, 275 Posterior mitral valve leaflet echocardiographic evidence of vegetation on, 274, 275 Post mitral and aortic valve replacements with tissue prostheses with exertional dyspnea, 323-326, 323-325/ Postmyocardial infarction rupture, of ventricular septum, 1 06, 1 07 Postviral myocarditis, 1 22 Precordial T wave inversion, 300-302, 304 Pregnancy, cardiac murmur during, 46-47 Pressure half-time method, 1 94 Presyncope and dyspnea, 1 46-148, 1 46/, 1 47/ Primary amyloidosis, 1 87, 1 88 versus senile cardiac amyloidosis, 1 28, 1 29 Primary biliary cirrhosis, 3 1 9-322, 3 1 9-320/ Primary pulmonary hypertension, 47 Primary ventricular tachydysrhythmia, 1 6 1 , 1 62 Progressive dyspnea, 48-49, 75-77, 75-76/ etiology for, 76, 77 treatment of, 76, 77 Progressive exertional dyspnea, 1 27-1 29, 1 27-128/, 1 29/, 1 39-1 4 1 , 1 39/, 1 52, 1 52-1 54/, 1 54-1 5 5, 248-25 1 , 248/, 249/, 250/ Progressive exertional fatigue and lower extremity edema examination, 1 6 1 , 1 62 pathology, 1 6 1 , 1 62, 1 62/ secondary finding, 1 6 1 , 1 6 1/, 1 62 Progressive exertional shortness of breath, 1 2-1 5, 1 2- 1 3/ with diabetes and systemic hypertension, 1 87-1 88, 1 88/ and lower extremity edema, 1 63-1 64

clinical findings in, 1 63, 1 64 after onset of dyspnea, 1 84-1 86, 1 84-1 85/ three month history of, 1 23-1 26, 1 23/, 1 24/ Progressive fatigue and marked lower extremity edema, 1 37- 1 3 8 , 1 37/, 1 3 8/ Progressive NYHA class III dyspnea, 66-70, 66-67/, 68/, 69/ Progressive NYHA class II-III exertional dyspnea and palpitations, 1 56-1 59, 1 56-1 57/, 1 59/ Prostaglandin infusion, 64, 65 Prosthetic function orifice area, 1 74, 1 77 Proximal isovelocity surface area (PISA) method, 4 1 , 78, 78-79/, 80 aliasing velocity, 40 ERO calculation, 40, 4 1 optimal timing fo r measurement of, 32, 33 radius of, 1 68, 1 69, 1 95 , 1 97 regurgitant volume calculation, 40, 4 1 Pseudonormalized diastolic dysfunction, diagnosis of, 95, 97 Pulmonary arterial flow, 48, 48/, 49 Pulmonary arterial pressures, 329, 330-33 1 Pulmonary artery (PA) capacitance, 73, 74, 329, 33 1 Pulmonary artery catheter, 25, 27 Pulmonary artery diastolic pressure (PADP) , 246, 247, 309, 330-33 1 estimation of, 8, 8/, 1 0, 73, 74 Pulmonary artery end-diastolic pressure (PAEDP) , 57, 58, 294, 294/, 296 estimation of, 2 1 8, 220 Pulmonary artery pressures, 1 3 5 , 1 36 hemodynamic measures of, 1 63, 1 64 Pulmonary artery systolic pressure (PASP) , 56/, 57, 58, 246, 247, 330-33 1 estimation of, 8, 8/, 1 0, 73, 74 Pulmonary balloon valvuloplasty, 64, 65 Pulmonary disease, 1 65, 1 66 Pulmonary embolism, transthoracic echocardiography of, 1 00, 1 0 1 Pulmonary hypertension, 1 3 1 , 1 32, 1 3 5 , 1 36, 309, 320, 322, 329, 330 Pulmonary regurgitation (PR) velocity, 56/, 57, 5 8 Pulmonary valve, maximum instantaneous systolic gradient across, 64, 65 Pulmonary valve regurgitation, severe, 1 85 , 1 86 Pulmonary valve stenosis, 205, 206 signs of severe, 64, 65

3 44 I I N D E X

Pulmonary vascular (PY) capacitance, 33 1 Pulmonary vasodilators, 2 1 9, 220 Pulmonary vein Doppler interrogation of, 8/, 1 0 Doppler recording, 5 , 6 pattern, 29 1-292, 29 1/, 293 profile, 332/, 334, 335 Pulmonary vein flow, 256/, 258 pulsed wave Doppler assessment 0 £ 39/, 40, 41 Pulmonary vein pulsed wave Doppler interrogation, 3 1 2, 3 1 2/, 3 1 4 Pulmonary vein stenosis Doppler findings with, 82, 82/, 83 symptoms of, 83 Pulmonary venous drainage, 3 1 7, 3 1 8 Pulsed wave Doppler signal from abdominal aorta, 1 9, 1 9/, 2 1 assessment o f pulmonary vein flow, 39/, 40, 4 1 Pulsus alternans, 87 Quinones method, 4 Radiation-induced cardiac disease, 258 Reduced second heart sound, 87 Regional wall motion assessment, 1 56/, 1 57, 1 57/, 1 59 Regurgitant flow, 45, 1 5 1 , 1 5 5 , 25 1 , 265 Regurgitant fraction, 33 Regurgitant jets, combined radius 0£ 249, 250/, 25 1 Regurgitant volume, 3 1 , 3 1/, 33, 40, 4 1 , 76, 76/, 77, 1 86, 236, 265, 330 of mitral valve regurgitation (MR) , 1 96, 1 97 Retrosternal chest pain, 1 08-1 09, 1 09/ Reverberation artifact, 296, 296/ "Reverse-dagger" shape, of tricuspid regurgitation (TR) velocity, 1 39, 1 3 9/, 1 4 1 Rheumatic disease, 222, 224, 282, 284 aortic and mitral valve replacements for, 276 valvular findings, etiology for, 266, 268 Rheumatic mitral stenosis, 266, 268 Rheumatic mitral valve disease, 282, 284 Rhinosinusitis, 48-49 Right atrial (RA) pressure, 5 8 based o n hepatic vein velocity, 257, 257/, 258 estimation of, 249, 249/, 25 1 Right atrium, mobile structure in, 2 1 0, 2 1 1

Right coronary artery (RCA), 34, 36 Right coronary sinus, 27 1 , 272 Right heart failure, 54, 5 5 , 56-58, 56-57/ Right index of myocardial performance (RIMP) , 73, 74 Right ventricle in cardiac output, 1 50, 1 5 1 Right ventricular (RV) diastolic collapse, 1 63, 1 64 Right ventricular dysfunction, severe, 249, 25 1 Right ventricular end-diastolic pressures, 1 85 , 1 85/, 1 86 Right ventricular end-systolic pressure, hemodynamic measures 0£ 1 63, 1 64 Right ventricular enlargement, 328, 330 cause for, 1 3 5 , 1 36 Right ventricular fractional area, 329, 33 1 Right ventricular free wall, 294-295, 294-295/, 296 Right ventricular function, parameters 0£ 1 54, 1 5 5 Right ventricular index of myocardial performance, 1 00, 1 0 1 , 1 0 if Right ventricular index of myocardial performance (RIMP) , 1 3 5 , 1 36, 1 54, 1 5 5 Right ventricular longitudinal contractility, 2 1 8/, 2 1 9, 220 Right ventricular longitudinal systolic function, 257, 257/, 258 explanation of, 1 05 measures 0£ 135, 1 36, 329, 33 1 Right ventricular outflow tract (RVOT) obstruction, 48, 48/, 49, 58, 1 3 1 , 1 32 Right ventricular size contractility of, 3 1 7, 3 1 8 moderate to severe enlargement in, 3 1 6-3 1 7, 3 1 6-3 1 7/, 3 1 8 Right ventricular systolic function, normal, 1 72, 17 4 Right ventricular systolic pressure (RVSP) , 58, 1 06, 1 07, 1 07/, 1 30, 1 32, 32 1 , 322 calculation 0£ 4, 6, 247, 308, 309 estimation 0£ 1 0, 2 1 9, 220 Right ventricular volume overload, 1 85 , 1 86 RIMP. See Right index of myocardial performance Roth spots, 1 90 Rupture, 263, 265 Ruptured sinus, ofValsalva aneurysm, 270, 270-271/, 272

Scallops, of mitral valve, 25, 27 Scimitar syndrome, 3 1 8 Sclerodactyly and esophageal dilatation, 1 63, 1 64 Scleroderma, 1 22 Sclerosis, systemic, 71-74, 7 1-73/ SCN5A sodium channel gene, defects in, 302, 304 Second heart sound and opening snap, interval between, 266, 268 Secundum atrial septal defect, 1 3 5 , 1 36 Senile cardiac amyloidosis versus primary (AL) amyloidosis, 1 28, 1 29 Septal hypertrophic cardiomyopathy, 1 1 0, 1 1 0/, 1 1 3 Septal hypertrophy, with risk of sudden cardiac death, 302, 304 Septal myectomy, 1 5 Septal rupture, 1 07 Serotonin 2B receptors, 1 40 Serum free light chains, 1 24, 1 26 immunoassay for, 1 23-1 24, 1 2 5 Serum uric acid, 1 24, 1 26 Serum white blood cell count test, 1 68, 1 69 Severe angina, 1 1 9- 1 20 Severe chest pain, 37-38 clinical evaluation for, 37, 38 management, 37, 38 for hypotension, 37, 38 risk factors for, 37, 38 Severe chronic obstructive pulmonary disease, 259-26 1 , 259/ use of echo contrast, 260 Severe ischemic cardiomyopathy and functional class IV left heart failure, 1 49-1 5 1 , 149- 1 5 0/, 1 5 if Severe mitral valve regurgitation, 1 1 1-1 1 2, 1 1 1- 1 1 2/, 1 1 3 Severe pancreatitis history, fever and dyspnea with, 289, 289/, 290 Severe tricuspid valve regurgitation, 1 65, 1 66 Shah, Pravin, 1 5 8, 1 59 Sharp chest pain, 1 9 1- 1 94, 1 9 1-1 92/, 1 92-1 93/ auscultatory finding, 1 92, 1 93 management, 1 92, 1 94 Shone syndrome, 1 94 Sinus ofValsalva aneurysm, 27 1 , 272 Soft systolic murmur after fainting episode, 300-305, 300-302/, 304/ Sphericity index, 246, 247 Stable functional Class II exertional dyspnea, 252-254, 252-25 5/ Staphylococcus aureus, 273, 275

I N D EX I 3 4 5

Staphylococcus bovis, 275 ST elevation myocardial infarction, 23, 24 Steroid therapy, 1 99 Streptococcus bovis, 325, 326 Streptococcus viridans, 6 1 Stress echocardiogram, 1 57, 1 57/, 1 5 9 Stress echocardiography dobutamine, peak doses of, 1 08-1 09, 1 09/ Stress-induced cardiomyopathy, 1 1 9, 1 20, 200, 20 1 pathogenesis for, 200, 20 1 Stroke, higher risk of and patent foramen ovale (PFO) , 2 1 2, 2 1 3, 278, 280 Stroke volume (SV) , 6, 24, 67, 69, 1 05, 1 1 7, 1 36, 1 5 1 , 209, 247, 272, 329, 330 Doppler derived, 2 1 5, 2 1 6 Subaortic valve membranes, 204 surgical resection of, 203, 204 Subarachnoid hemorrhage, 1 1 9, 1 20 Sudden cardiac death (SCD) , 1 2, 14, 1 80, 1 8 1 risk of, 302, 304 Sudden cardiac death risk stratification, in hypertrophic cardiomyopathy, 1 4 Sunitinib therapy, 206 Superior vena cava, 5 1/, 54 Supraventricular rhythm abnormalities, 1 62 Surgical myectomy, 1 3 , 1 3/, 1 5 Surgical septal myectomy, 1 57, 1 59, 1 59/ Suture material, 278, 279 Symptomatic paroxysmal atrial fibrillation, 8 1-83, 8 1-82/ history of, 285-288, 285-286/ Syncopal episode, 7-1 1 , 7-9/ Syncope, 2 1 9, 220 cardiovascular causes for, 9, 1 1 Syphilis, 272 Systemic anticoagulation, 1 2 1 , 1 22, 1 96, 1 97 Systemic blood pressure, 1 1 9, 1 20 Systemic hypertension, 1 9-2 1 , 1 9/, 2 1/ exertional dyspnea with, 332-336, 332-334/ murmur with, 205-206, 205/ progressive exertional shortness of breath with, 1 87-1 88, 1 88/ Systemic sclerosis, 7 1 -74, 7 1-73/ Systolic and diastolic murmurs, 262, 262-263/

Systolic anterior motion of anterior mitral valve leaflet, 1 5 7-1 5 8, 1 57/, 1 59 Systolic blood pressure, 1 4 exertional shortness o f breath with, 2 1 8-220, 2 1 8/ Systolic ejection murmur, 1 6- 1 8, 1 6/, 1 7/ with amyl nitrite inhalation, 1 56, 1 56-1 57/, 1 59 Systolic flow reversals, 53/, 55, 138 in hepatic veins, 58, 1 54, 1 5 5 pulmonary vein, 3 1 4, 326 Systolic longitudinal strain patterns, 1 24, 1 2 5 , 1 2'if Systolic murmur, 3 1-33, 3 1-32/, 42-45, 42-43/, 78-80, 234-236, 234-235/, 3 1 0-3 1 5, 3 1 0-3 1 2/, 3 1 3/, 322 asymptomatic, 39-4 1 , 39-40/ explanation for, 23 1 , 232 harsh, 334, 334/, 335 loud, 2 1 8, 220 Tamponade, characteristics of, 148 Tei index. See Right index of myocardial performance (RIMP) Tetralogy of Fallot, 1 6 1 Thoracic aorta, ultrasound examination of, 2 1 3 Thoracic radiation, 257, 258 Thromboendarterectomy, 329, 33 1 , 33 1/ Thrombolytic therapy, 1 65 , 1 66 Thrombotic obstruction, of bioprosthesis, 1 72-1 73, 1 72/, 1 74 Thrombus, 1 76, 1 77, 289, 290 incidence of, 289, 290 Transesophageal echocardiogram (TEE), 148, 1 89, 1 90, 289, 289/, 290 dyspnea and, 1 72, 17 4 progressive exertional dyspnea, 249, 25 1 recurrent episodes of left heart failure, 277, 279 Transesophageal echocardiography, 8 1 , 8 1/, 83 and acute severe dyspnea, 25, 27 anterior mitral valve leaflet prolapse with flail middle scallop, 39/, 40, 4 1 class III shortness of breath, 223, 224-225 congenital aortic valve stenosis, 1 76, 1 77 Doppler findings, 82, 82/, 83 Transient loss of motor function descending thoracic aorta, structure, 2 1 0, 2 1 0/, 2 1 1 , 2 1 1/

estimating left ventricular ejection fraction, 2 1 0, 2 1 1 management, 2 1 0, 2 1 1 mobile structure in right atrium, 2 1 0, 2 1 1 in right upper extremity, 2 1 0-2 1 1 , 2 1 1/ Transient loss of vision, 1 89-1 90 with modest exertional shortness of breath, 1 95-1 97, 195/ treatment of, 1 96, 1 97 Transmitral flow, time velocity integral (TVI) of, 3 1 2, 3 1 4 Transmitral prosthetic valve diastolic mean gradient, 276/, 276-277, 279 Transthoracic echocardiographic images apical hypertrophic cardiomyopathy and, 1 1 0, 1 1 0-1 1 1/, 1 1 3 Transthoracic echocardiography, 9, 1 1 , 63, 63-64f, 64 performed after valve surgery, 60, 6 1 fo r severe angina, 1 1 9- 1 20 soft systolic murmur after fainting episode, 302, 305 on sonography student, 9 1-93, 9 1-92/ Treadmill exercise stress test, 6 1 Treadmill stress echocardiogram, 1 57 Tricupsid annular dilatation, 325, 326 Tricuspid annular plane systolic excursion (TAPSE) , 1 04, 1 05, 1 74, 258, 295, 296 and RV TOI, 3 1 6-3 1 7/, 3 1 7, 3 1 8 Tricuspid annular tissue peak velocity, 295, 296 Tricuspid annulus dilation, 329, 330 Tricuspid effective regurgitant orifice (ERO) area, 1 52-1 54, 1 52-1 54/, 1 5 5 Tricuspid regurgitant jet, shape of, 1 39, 1 4 1 Tricuspid regurgitation (TR) severe, 57, 5 8 velocity o f, 6, 5 7 , 5 8 volume, 7 8 , 78-79/, 79, 80, 1 85 , 1 86 Tricuspid valve, 1 37, 1 38, 1 3 8/ Tricuspid valve regurgitation, 25 1 mechanism of, 325, 326, 329, 330 severe, 1 54, 1 5 5 , 1 85, 1 86, 234-235, 236, 277, 279, 329, 330 Tumor plop, 1 92, 1 93 Tumor resection, 1 42, 1 43, 143/ Turner syndrome, 37, 38, 1 94 T-wave inversion, 1 67, 1 8 1 in precordial leads, 302, 304

346 I I N D EX

Twelve-lead ECG, 1 47, 1 47/, 1 48, 1 56 with hypertrophic cardiomyopathy (HCM) , 1 79, 1 8 1 septal hypertrophic cardiomyopathy and, 1 1 0, 1 1 0/, 1 1 3 Two-dimensional (2D) echocardiographic finding, 94, 97, 1 24, 1 2 5 aortic valve (AoV) in, 1 82, 1 82/, 1 83 for cardiac tamponade, 1 63, 1 64 T wave inversions in precordial leads, 302, 304 Two pillow orthopnea, 1 2 1 - 1 22 Ultrasound examination, of thoracic aorta, 2 1 3 Ultrasound frequency, 1 08, 1 09 Valsalva aneurysm, ruptured sinus of, 270, 270/, 27 1 , 271/, 272 Valsalva maneuver, 5, 6, 206 Valvular findings, etiology for, 266, 268

Valvular vegetation, 6 1 Vascular radiology, 2 1 0, 2 1 1 , 2 1 1/ Vasodilator agents, 302, 304 Vasodilator therapy, 77 Vasopressor agents, 38 Vegetation aortic valve, 1 99 cardiac, 1 89, 1 90 valvular, 6 1 Vena caval filter, placement of, 329, 33 1 Vena contracta, 1 40 Venous cast, 1 00, 1 0 1 Ventricular dysrhythmias, 237, 238 Ventricular perspective, 266, 268 Ventricular septal defect complications of, 1 3 1 , 1 32 muscular type, 1 30, 1 32 Ventricular septal rupture, 1 07 Ventricular tachycardia with rapid palpitations, 294-296, 294-295/, 296/

Vision, transient loss of, 1 89-1 90 with modest exertional shortness of breath, 1 95-1 97, 1 95/ Visual disturbances, 2 1 9, 220 Wall motion abnormalities in distribution of left anterior descending coronary artery, 44, 45 left ventricular (LV) function and, 48, 49 Warfarin, 283, 284 anticoagulation of, 2 1 2, 2 1 3, 329, 33 1 anti thrombotic therapy, 1 73, 1 74 Weight gain, 1 52, 1 52-1 54/, 1 54-1 5 5 Weight loss, 48-49 Wolff-Parkinson-White syndrome, 1 62 Zero baseline, for color map, 266, 268